Site Loader

Π‘ΠΎΠ΄Π΅Ρ€ΠΆΠ°Π½ΠΈΠ΅

ΠŸΠΎΡΠ»Π΅Π΄ΠΎΠ²Π°Ρ‚Π΅Π»ΡŒΠ½ΠΎΠ΅ ΠΈ ΠΏΠ°Ρ€Π°Π»Π»Π΅Π»ΡŒΠ½ΠΎΠ΅ соСдинСниС

Π’ Π΄Π°Π½Π½ΠΎΠΉ ΡΡ‚Π°Ρ‚ΡŒΠ΅ Ρ€Π΅Ρ‡ΡŒ ΠΏΠΎΠΉΠ΄Π΅Ρ‚ ΠΎ ΠΏΠΎΡΠ»Π΅Π΄ΠΎΠ²Π°Ρ‚Π΅Π»ΡŒΠ½ΠΎΠΌ ΠΈ ΠΏΠ°Ρ€Π°Π»Π»Π΅Π»ΡŒΠ½ΠΎΠΌ соСдинСнии ΠΏΡ€ΠΎΠ²ΠΎΠ΄Π½ΠΈΠΊΠΎΠ². На ΠΏΡ€ΠΈΠΌΠ΅Ρ€Π°Ρ… Π±ΡƒΠ΄ΡƒΡ‚ рассмотрСны Π΄Π°Π½Π½Ρ‹Π΅ соСдинСния ΠΈ ΠΊΠ°ΠΊ ΠΏΡ€ΠΈ Ρ‚Π°ΠΊΠΈΡ… соСдинСниях Π±ΡƒΠ΄ΡƒΡ‚ ΠΈΠ·ΠΌΠ΅Π½ΡΡ‚ΡŒΡΡ Ρ‚Π°ΠΊΠΈΠ΅ Π²Π΅Π»ΠΈΡ‡ΠΈΠ½Ρ‹ ΠΊΠ°ΠΊ:

  • Ρ‚ΠΎΠΊ;
  • напряТСниС;
  • сопротивлСниС.

Π’ Ρ‚Π°Π±Π»ΠΈΡ†Π΅ 1.8 [Π›2, с.24] ΠΏΡ€ΠΈΠ²Π΅Π΄Π΅Π½Ρ‹ схСмы ΠΈ Ρ„ΠΎΡ€ΠΌΡƒΠ»Ρ‹ ΠΏΠΎ ΠΎΠΏΡ€Π΅Π΄Π΅Π»Π΅Π½ΠΈΡŽ сопротивлСний, Ρ‚ΠΎΠΊΠΎΠ² ΠΈ напряТСний ΠΏΡ€ΠΈ ΠΏΠ°Ρ€Π°Π»Π»Π΅Π»ΡŒΠ½ΠΎΠΌ ΠΈ ΠΏΠΎΡΠ»Π΅Π΄ΠΎΠ²Π°Ρ‚Π΅Π»ΡŒΠ½ΠΎΠΌ соСдинСнии.

ΠŸΠΎΡΠ»Π΅Π΄ΠΎΠ²Π°Ρ‚Π΅Π»ΡŒΠ½ΠΎΠ΅ соСдинСниС

ΠŸΠΎΡΠ»Π΅Π΄ΠΎΠ²Π°Ρ‚Π΅Π»ΡŒΠ½Ρ‹ΠΌ соСдинСниСм Π½Π°Π·Ρ‹Π²Π°ΡŽΡ‚ΡΡ Ρ‚Π΅ участки Ρ†Π΅ΠΏΠΈ, ΠΏΠΎ ΠΊΠΎΡ‚ΠΎΡ€Ρ‹ΠΌ всСгда проходят ΠΎΠ΄ΠΈΠ½Π°ΠΊΠΎΠ²Ρ‹Π΅ Ρ‚ΠΎΠΊΠΈ.

ΠŸΡ€ΠΈ ΠΏΠΎΡΠ»Π΅Π΄ΠΎΠ²Π°Ρ‚Π΅Π»ΡŒΠ½ΠΎΠΌ соСдинСнии:

  • сила Ρ‚ΠΎΠΊΠ° Π²ΠΎ всСх ΠΏΡ€ΠΎΠ²ΠΎΠ΄Π½ΠΈΠΊΠ°Ρ… ΠΎΠ΄ΠΈΠ½Π°ΠΊΠΎΠ²Π°;
  • напряТСниС Π½Π° всём соСдинСнии Ρ€Π°Π²Π½ΠΎ суммС напряТСний Π½Π° ΠΎΡ‚Π΄Π΅Π»ΡŒΠ½Ρ‹Ρ… ΠΏΡ€ΠΎΠ²ΠΎΠ΄Π½ΠΈΠΊΠ°Ρ…;
  • сопротивлСниС всСго соСдинСния Ρ€Π°Π²Π½ΠΎ суммС сопротивлСний ΠΎΡ‚Π΄Π΅Π»ΡŒΠ½Ρ‹Ρ… ΠΏΡ€ΠΎΠ²ΠΎΠ΄Π½ΠΈΠΊΠΎΠ².

ΠŸΡ€ΠΈΠΌΠ΅Ρ€ 1

ΠŸΠΎΡΠ»Π΅Π΄ΠΎΠ²Π°Ρ‚Π΅Π»ΡŒΠ½ΠΎ ΠΏΠΎΠ΄ΠΊΠ»ΡŽΡ‡Π΅Π½Ρ‹ Π΄Π²Π΅ Π»Π°ΠΌΠΏΡ‹ накаливания ΠΎΠ΄ΠΈΠ½Π°ΠΊΠΎΠ²ΠΎΠΉ мощности Π Π»1=Π Π»2=100 Π’Ρ‚ ΠΊ сСти с напряТСниСм UΠ½=220Π’. Π‘ΠΎΠΏΡ€ΠΎΡ‚ΠΈΠ²Π»Π΅Π½ΠΈΠ΅ Π½ΠΈΡ‚ΠΈ Π² Π»Π°ΠΌΠΏΠ°Ρ… составляСт RΠ»1=RΠ»2=122 Ом. НоминальноС напряТСниС для Π»Π°ΠΌΠΏ Ρ€Π°Π²Π½ΠΎ 220 B. На рис.1 ΠΏΠΎΠΊΠ°Π·Π°Π½ΠΎ ΠΏΠΎΡΠ»Π΅Π΄ΠΎΠ²Π°Ρ‚Π΅Π»ΡŒΠ½ΠΎΠ΅ Π²ΠΊΠ»ΡŽΡ‡Π΅Π½ΠΈΠ΅ Π»Π°ΠΌΠΏ.

РСшСниС

БоставляСм схСму замСщСния, выраТая ΠΊΠ°ΠΆΠ΄ΡƒΡŽ ΠΈΠ· входящих элСмСнтов Ρ†Π΅ΠΏΠΈ (Π² Π΄Π°Π½Π½ΠΎΠΌ случаС Π»Π°ΠΌΠΏΡ‹ накаливания) Π² Π²ΠΈΠ΄Π΅ сопротивлСний.

1. ΠžΠΏΡ€Π΅Π΄Π΅Π»ΡΠ΅ΠΌ Ρ‚ΠΎΠΊ ΠΏΡ€ΠΎΡ‚Π΅ΠΊΠ°ΡŽΡ‰Π΅ΠΉ ΠΏΠΎ участкам Ρ†Π΅ΠΏΠΈ:

IΠ½ = UΠ½/RΠ»1+ RΠ»2 = 220/122+122 = 0,9 A

2. ΠžΠΏΡ€Π΅Π΄Π΅Π»ΡΠ΅ΠΌ напряТСниС Π½Π° ΠΊΠ°ΠΆΠ΄ΠΎΠΉ ΠΈΠ· Π»Π°ΠΌΠΏ накаливания, Ρ‚Π°ΠΊ ΠΊΠ°ΠΊ ΠΌΠΎΡ‰Π½ΠΎΡΡ‚ΡŒ Π»Π°ΠΌΠΏ Ρƒ нас одинаковая, Ρ‚ΠΎ ΠΈ напряТСниС для ΠΊΠ°ΠΆΠ΄ΠΎΠΉ ΠΈΠ· Π»Π°ΠΌΠΏ Π±ΡƒΠ΄Π΅Ρ‚ ΠΎΠ΄ΠΈΠ½Π°ΠΊΠΎΠ²ΠΎ:

UΠ»1=UΠ»2 = IΠ½*R = 0,9*122 = 110 B

Как ΠΌΡ‹ Π²ΠΈΠ΄ΠΈΠΌ напряТСниС источника (Π² Π΄Π°Π½Π½ΠΎΠΌ ΠΏΡ€ΠΈΠΌΠ΅Ρ€Π΅ 220 Π’) Ρ€Π°Π·Π΄Π΅Π»ΠΈΡ‚ΡŒΡΡ ΠΏΠΎΡ€ΠΎΠ²Π½Ρƒ, ΠΌΠ΅ΠΆΠ΄Ρƒ ΠΎΠ±ΠΎΠΈΠΌΠΈ ΠΏΠΎΡΠ»Π΅Π΄ΠΎΠ²Π°Ρ‚Π΅Π»ΡŒΠ½ΠΎ Π²ΠΊΠ»ΡŽΡ‡Π΅Π½Π½Ρ‹ΠΌΠΈ Π»Π°ΠΌΠΏΠ°ΠΌΠΈ. ΠŸΡ€ΠΈ этом Π»Π°ΠΌΠΏΡ‹ Π±ΡƒΠ΄ΡƒΡ‚ Π΅Π»ΠΈ свСтит, ΠΈΡ… Π½Π°ΠΊΠ°Π» Π±ΡƒΠ΄Π΅Ρ‚ Π½Π΅ΠΏΠΎΠ»Π½Ρ‹ΠΌ.

Для Ρ‚ΠΎΠ³ΠΎ Ρ‡Ρ‚ΠΎΠ±Ρ‹ Π»Π°ΠΌΠΏΡ‹ Π³ΠΎΡ€Π΅Π»ΠΈ с ΠΏΠΎΠ»Π½Ρ‹ΠΌ Π½Π°ΠΊΠ°Π»ΠΎΠΌ, Π½ΡƒΠΆΠ½ΠΎ ΡƒΠ²Π΅Π»ΠΈΡ‡ΠΈΡ‚ΡŒ напряТСниС источника с 220Π’ Π΄ΠΎ 440Π’, ΠΏΡ€ΠΈ этом Π½Π° ΠΊΠ°ΠΆΠ΄ΠΎΠΉ ΠΈΠ· Π»Π°ΠΌΠΏ ΡƒΡΡ‚Π°Π½ΠΎΠ²ΠΈΡ‚ΡŒΡΡ номинальноС (Ρ€Π°Π±ΠΎΡ‡Π΅Π΅) напряТСниС Ρ€Π°Π²Π½ΠΎΠ΅ 220Π’.

ΠŸΡ€ΠΈΠΌΠ΅Ρ€ 2

ΠŸΠΎΡΠ»Π΅Π΄ΠΎΠ²Π°Ρ‚Π΅Π»ΡŒΠ½ΠΎ ΠΏΠΎΠ΄ΠΊΠ»ΡŽΡ‡Π΅Π½Ρ‹ Π΄Π²Π΅ Π»Π°ΠΌΠΏΡ‹ накаливания ΠΌΠΎΡ‰Π½ΠΎΡΡ‚ΡŒ Π Π»1 = 100 Π’Ρ‚ ΠΈ Π Π»2 = 75 Π’Ρ‚ ΠΊ сСти с напряТСниСм UΠ½=220Π’. Π‘ΠΎΠΏΡ€ΠΎΡ‚ΠΈΠ²Π»Π΅Π½ΠΈΠ΅ Π½ΠΈΡ‚ΠΈ Π² Π»Π°ΠΌΠΏΠ°Ρ… ΡΠΎΡΡ‚Π°Π²Π»ΡΡŽΡ‚ RΠ»1= 122 Ом для стоваттной Π»Π°ΠΌΠΏΡ‹ ΠΈ RΠ»2= 153 Ом для сСмидСсяти пяти Π²Π°Ρ‚Π½ΠΎΠΉ Π»Π°ΠΌΠΏΡ‹.

РСшСниС

1. ΠžΠΏΡ€Π΅Π΄Π΅Π»ΡΠ΅ΠΌ Ρ‚ΠΎΠΊ ΠΏΡ€ΠΎΡ‚Π΅ΠΊΠ°ΡŽΡ‰Π΅ΠΉ ΠΏΠΎ участкам Ρ†Π΅ΠΏΠΈ:

IΠ½ = UΠ½/RΠ»1+ RΠ»2 = 220/100+75 = 0,8 A

2. ΠžΠΏΡ€Π΅Π΄Π΅Π»ΡΠ΅ΠΌ напряТСниС Π½Π° ΠΊΠ°ΠΆΠ΄ΠΎΠΉ ΠΈΠ· Π»Π°ΠΌΠΏ накаливания:

UΠ»1= IΠ½*RΠ»1 = 0,8*122 = 98 B
UΠ»2= IΠ½*RΠ»2 = 0,8*153 = 122 B

Π˜ΡΡ…ΠΎΠ΄Ρ ΠΈΠ· Ρ€Π΅Π·ΡƒΠ»ΡŒΡ‚Π°Ρ‚ΠΎΠ² расчСтов, Π±ΠΎΠ»Π΅Π΅ мощная Π»Π°ΠΌΠΏΠ° 100 Π’Ρ‚ ΠΏΠΎΠ»ΡƒΡ‡Π°Π΅Ρ‚ ΠΏΡ€ΠΈ этом мСньшСС напряТСниС. Но Ρ‚ΠΎΠΊ Π² Π΄Π²ΡƒΡ… ΠΏΠΎΡΠ»Π΅Π΄ΠΎΠ²Π°Ρ‚Π΅Π»ΡŒΠ½ΠΎ Π²ΠΊΠ»ΡŽΡ‡Π΅Π½Π½Ρ‹Ρ… Π΄Π°ΠΆΠ΅ Ρ€Π°Π·Π½Ρ‹Ρ… Π»Π°ΠΌΠΏΠ°Ρ… остаСтся ΠΎΠ΄ΠΈΠ½Π°ΠΊΠΎΠ²Ρ‹ΠΌ. НапримСр, Ссли ΠΎΠ΄Π½Π° ΠΈΠ· Π»Π°ΠΌΠΏ ΠΏΠ΅Ρ€Π΅Π³ΠΎΡ€ΠΈΡ‚ (порвСтся Π΅Π΅ Π½ΠΈΡ‚ΡŒ накаливания), погаснут ΠΎΠ±Π΅ Π»Π°ΠΌΠΏΡ‹.

Π”Π°Π½Π½ΠΎΠ΅ соСдинСниС Π»Π°ΠΌΠΏΠΎΡ‡Π΅ΠΊ, Π½Π°ΠΏΡ€ΠΈΠΌΠ΅Ρ€, ΠΈΡΠΏΠΎΠ»ΡŒΠ·ΡƒΠ΅Ρ‚ΡΡ Π² Ρ‚Ρ€Π°ΠΌΠ²Π°ΠΉΠ½ΠΎΠΌ Π²Π°Π³ΠΎΠ½Π΅ для освСщСния салона.

ΠŸΠ°Ρ€Π°Π»Π»Π΅Π»ΡŒΠ½ΠΎΠ΅ соСдинСниС

ΠŸΠ°Ρ€Π°Π»Π»Π΅Π»ΡŒΠ½ΠΎΠ΅ соСдинСниС – это соСдинСниС, ΠΏΡ€ΠΈ ΠΊΠΎΡ‚ΠΎΡ€ΠΎΠΌ Π½Π°Ρ‡Π°Π»Π° всСх ΠΏΡ€ΠΎΠ²ΠΎΠ΄Π½ΠΈΠΊΠΎΠ² ΠΏΡ€ΠΈΡΠΎΠ΅Π΄ΠΈΠ½ΡΡŽΡ‚ΡΡ ΠΊ ΠΎΠ΄Π½ΠΎΠΉ Ρ‚ΠΎΡ‡ΠΊΠ΅ Ρ†Π΅ΠΏΠΈ, Π° ΠΈΡ… ΠΊΠΎΠ½Ρ†Ρ‹ ΠΊ Π΄Ρ€ΡƒΠ³ΠΎΠΉ.

Π’ΠΎΡ‡ΠΊΠΈ Ρ†Π΅ΠΏΠΈ, ΠΊ ΠΊΠΎΡ‚ΠΎΡ€Ρ‹ΠΌ сходится нСсколько ΠΏΡ€ΠΎΠ²ΠΎΠ΄ΠΎΠ², Π½Π°Π·Ρ‹Π²Π°ΡŽΡ‚ ΡƒΠ·Π»Π°ΠΌΠΈ. Участки Ρ†Π΅ΠΏΠΈ, ΡΠΎΠ΅Π΄ΠΈΠ½ΡΡŽΡ‰ΠΈΠ΅ ΠΌΠ΅ΠΆΠ΄Ρƒ собой ΡƒΠ·Π»Ρ‹, Π½Π°Π·Ρ‹Π²Π°ΡŽΡ‚ вСтвями.

ΠŸΡ€ΠΈ ΠΏΠ°Ρ€Π°Π»Π»Π΅Π»ΡŒΠ½ΠΎΠΌ соСдинСнии:

  • напряТСниС Π½Π° всСх ΠΏΡ€ΠΎΠ²ΠΎΠ΄Π½ΠΈΠΊΠ°Ρ… ΠΎΠ΄ΠΈΠ½Π°ΠΊΠΎΠ²ΠΎ;
  • сила Ρ‚ΠΎΠΊΠ° Π² мСстС соСдинСния ΠΏΡ€ΠΎΠ²ΠΎΠ΄Π½ΠΈΠΊΠΎΠ² Ρ€Π°Π²Π½Π° суммС Ρ‚ΠΎΠΊΠΎΠ² Π² ΠΎΡ‚Π΄Π΅Π»ΡŒΠ½Ρ‹Ρ… ΠΏΡ€ΠΎΠ²ΠΎΠ΄Π½ΠΈΠΊΠ°Ρ…;
  • Π²Π΅Π»ΠΈΡ‡ΠΈΠ½Π°, обратная ΡΠΎΠΏΡ€ΠΎΡ‚ΠΈΠ²Π»Π΅Π½ΠΈΡŽ всСго соСдинСния, Ρ€Π°Π²Π½Π° суммС Π²Π΅Π»ΠΈΡ‡ΠΈΠ½, ΠΎΠ±Ρ€Π°Ρ‚Π½Ρ‹Ρ… сопротивлСниям ΠΎΡ‚Π΄Π΅Π»ΡŒΠ½Ρ‹Ρ… ΠΏΡ€ΠΎΠ²ΠΎΠ΄Π½ΠΈΠΊΠΎΠ².

ΠŸΡ€ΠΈΠΌΠ΅Ρ€ 3

ΠžΠΏΡ€Π΅Π΄Π΅Π»ΠΈΡ‚ΡŒ Ρ‚ΠΎΠΊΠΈ ΠΈ напряТСния всСх участков Ρ†Π΅ΠΏΠΈ (рис.5), Ссли извСстно:

  • НоминальноС напряТСниС сСти UΠ½ = 220Π’;
  • Π‘ΠΎΠΏΡ€ΠΎΡ‚ΠΈΠ²Π»Π΅Π½ΠΈΠ΅ Π½ΠΈΡ‚ΠΈ Π² Π»Π°ΠΌΠΏΠ°Ρ… HL1 ΠΈ HL2 ΡΠΎΡΡ‚Π°Π²Π»ΡΡŽΡ‚ RΠ»1 = RΠ»2 = 122 Ом.
  • Π‘ΠΎΠΏΡ€ΠΎΡ‚ΠΈΠ²Π»Π΅Π½ΠΈΠ΅ Π½ΠΈΡ‚ΠΈ Π² Π»Π°ΠΌΠΏΠ΅ HL3 ΡΠΎΡΡ‚Π°Π²Π»ΡΡŽΡ‚ RΠ»3 = 153 Ом.

РСшСниС

БоставляСм схСму замСщСния для схСмы, прСдставлСнной Π½Π° рис.5.

1. ΠžΠΏΡ€Π΅Π΄Π΅Π»ΡΠ΅ΠΌ ΠΏΡ€ΠΎΠ²ΠΎΠ΄ΠΈΠΌΠΎΡΡ‚ΡŒ всСй Ρ†Π΅ΠΏΠΈ [Π›1, с.47] ΠΈ согласно Ρ‚Π°Π±Π»ΠΈΡ†Ρ‹ 1.8:

2. ΠžΠΏΡ€Π΅Π΄Π΅Π»ΡΠ΅ΠΌ сопротивлСниС всСй Ρ†Π΅ΠΏΠΈ [Π›1, с.47]:

3. ΠžΠΏΡ€Π΅Π΄Π΅Π»ΡΠ΅ΠΌ силу Ρ‚ΠΎΠΊΠ° Ρ†Π΅ΠΏΠΈ ΠΏΠΎ Π·Π°ΠΊΠΎΠ½Ρƒ Ома:

4. ΠžΠΏΡ€Π΅Π΄Π΅Π»ΡΠ΅ΠΌ Ρ‚ΠΎΠΊΠΈ для ΠΊΠ°ΠΆΠ΄ΠΎΠΉ Ρ†Π΅ΠΏΠΈ [Π›1, с.47]:

5. Π’Ρ‹ΠΏΠΎΠ»Π½ΠΈΠΌ ΠΏΡ€ΠΎΠ²Π΅Ρ€ΠΊΡƒ, согласно ΠΊΠΎΡ‚ΠΎΡ€ΠΎΠΉ, сила Ρ‚ΠΎΠΊΠ° Π² мСстС соСдинСния ΠΏΡ€ΠΎΠ²ΠΎΠ΄Π½ΠΈΠΊΠΎΠ² Ρ€Π°Π²Π½Π° суммС Ρ‚ΠΎΠΊΠΎΠ² Π² ΠΎΡ‚Π΄Π΅Π»ΡŒΠ½Ρ‹Ρ… ΠΏΡ€ΠΎΠ²ΠΎΠ΄Π½ΠΈΠΊΠ°Ρ…:

IΠ»1+ IΠ»2+ IΠ»3=IΠΎΠ±Ρ‰.=1,8+1,8+1,44=5,04=5,04 (условиС выполняСтся)

БмСшанноС соСдинСниС

Π‘ΠΌΠ΅ΡˆΠ°Π½Π½Ρ‹ΠΌ соСдинСниСм – называСтся ΠΏΠΎΡΠ»Π΅Π΄ΠΎΠ²Π°Ρ‚Π΅Π»ΡŒΠ½ΠΎ-ΠΏΠ°Ρ€Π°Π»Π»Π΅Π»ΡŒΠ½ΠΎΠ΅ соСдинСниС сопротивлСний ΠΈΠ»ΠΈ участков Ρ†Π΅ΠΏΠΈ.

ΠŸΡ€ΠΈΠΌΠ΅Ρ€ 4

ΠžΠΏΡ€Π΅Π΄Π΅Π»ΠΈΡ‚ΡŒ Ρ‚ΠΎΠΊΠΈ ΠΈ напряТСния всСх участков Ρ†Π΅ΠΏΠΈ (рис.7), Ссли извСстно:

  • НоминальноС напряТСниС сСти UΠ½ = 220Π’;
  • Π‘ΠΎΠΏΡ€ΠΎΡ‚ΠΈΠ²Π»Π΅Π½ΠΈΠ΅ Π½ΠΈΡ‚ΠΈ Π² Π»Π°ΠΌΠΏΠ°Ρ… HL1, HL2, HL3 ΡΠΎΡΡ‚Π°Π²Π»ΡΡŽΡ‚ RΠ»1 = RΠ»2 = RΠ»3 = 122 Ом.
  • Π‘ΠΎΠΏΡ€ΠΎΡ‚ΠΈΠ²Π»Π΅Π½ΠΈΠ΅ Π½ΠΈΡ‚ΠΈ Π² Π»Π°ΠΌΠΏΠ΅ HL4 ΡΠΎΡΡ‚Π°Π²Π»ΡΡŽΡ‚ RΠ»4 = 153 Ом.
  • Π Π΅Π·ΡƒΠ»ΡŒΡ‚Π°Ρ‚Ρ‹ расчСтов для участка Ρ†Π΅ΠΏΠΈ Π’Π‘ (ΠΏΠ°Ρ€Π°Π»Π»Π΅Π»ΡŒΠ½ΠΎΠ΅ соСдинСниС ΠΏΡ€ΠΎΠ²ΠΎΠ΄Π½ΠΈΠΊΠΎΠ²) ΠΏΡ€ΠΈΠΌΠ΅Π½ΠΈΠΌ ΠΈΠ· ΠΏΡ€ΠΈΠΌΠ΅Ρ€Π° 3:
    Π‘ΠΎΠΏΡ€ΠΎΡ‚ΠΈΠ²Π»Π΅Π½ΠΈΠ΅ Ρ†Π΅ΠΏΠΈ Π’Π‘ составляСт Rвс = 43,668 Ом.

РСшСниС

БоставляСм схСму замСщСния для схСмы, прСдставлСнной Π½Π° рис.7.

1. ΠžΠΏΡ€Π΅Π΄Π΅Π»ΡΠ΅ΠΌ сопротивлСниС всСй Ρ†Π΅ΠΏΠΈ:

RΠΎΠ±Ρ‰ = RΠ°Π²+Rвс = RΠ»1+Rвс = 122+43,688 = 165,688 Ом

2. ΠžΠΏΡ€Π΅Π΄Π΅Π»ΡΠ΅ΠΌ силу Ρ‚ΠΎΠΊΠ° Ρ†Π΅ΠΏΠΈ, согласно Π·Π°ΠΊΠΎΠ½Π° Ома:

3. ΠžΠΏΡ€Π΅Π΄Π΅Π»ΡΠ΅ΠΌ напряТСниС Π½Π° ΠΏΠ΅Ρ€Π²ΠΎΠΌ сопротивлСнии:

UΠ°Π²=UΠ»1= IΠΎΠ±Ρ‰*RΠ»1 = 1,33*122 = 162 B

4. ΠžΠΏΡ€Π΅Π΄Π΅Π»ΡΠ΅ΠΌ напряТСниС Π½Π° участкС Π’Π‘:

Uвс= IΠΎΠ±Ρ‰*Rвс = 1,33*43,688 = 58,1 B

5. ΠžΠΏΡ€Π΅Π΄Π΅Π»ΡΠ΅ΠΌ Ρ‚ΠΎΠΊΠΈ для ΠΊΠ°ΠΆΠ΄ΠΎΠΉ Ρ†Π΅ΠΏΠΈ участка Π’Π‘:

6. Π’Ρ‹ΠΏΠΎΠ»Π½ΠΈΠΌ ΠΏΡ€ΠΎΠ²Π΅Ρ€ΠΊΡƒ для участка Ρ†Π΅ΠΏΠΈ Π’Π‘:

IΠ»2+ IΠ»3+ IΠ»4= IΠΎΠ±Ρ‰.=0,48+0,48+0,38=1,33=1,33 (условиС выполняСтся)

Π›ΠΈΡ‚Π΅Ρ€Π°Ρ‚ΡƒΡ€Π°:

  1. ΠžΠ±Ρ‰Π°Ρ элСктротСхника с основами элСктроники, Π’.Π‘. Попов, 1972 Π³.
  2. Бправочная ΠΊΠ½ΠΈΠ³Π° элСктрика. Π’.И. Π“Ρ€ΠΈΠ³ΠΎΡ€ΡŒΠ΅Π²Π°. 2004 Π³.

ВсСго Π½Π°ΠΈΠ»ΡƒΡ‡ΡˆΠ΅Π³ΠΎ! Π”ΠΎ Π½ΠΎΠ²Ρ‹Ρ… встрСч Π½Π° сайтС Raschet.info.

ΠŸΠΎΡΠ»Π΅Π΄ΠΎΠ²Π°Ρ‚Π΅Π»ΡŒΠ½ΠΎΠ΅ ΠΈ ΠΏΠ°Ρ€Π°Π»Π»Π΅Π»ΡŒΠ½ΠΎΠ΅ соСдинСниС ΠΏΡ€ΠΎΠ²ΠΎΠ΄Π½ΠΈΠΊΠΎΠ² β€” ΡƒΡ€ΠΎΠΊ. Π€ΠΈΠ·ΠΈΠΊΠ°, 8 класс.

Π’ Π±Ρ‹Ρ‚Ρƒ ΠΈ Π² ΠΏΡ€ΠΎΠΌΡ‹ΡˆΠ»Π΅Π½Π½ΠΎΡΡ‚ΠΈ Π² ΡΠ»Π΅ΠΊΡ‚Ρ€ΠΈΡ‡Π΅ΡΠΊΡƒΡŽ Ρ†Π΅ΠΏΡŒ ΡΠΎΠ΅Π΄ΠΈΠ½ΡΡŽΡ‚ΡΡ сразу нСсколько ΠΏΠΎΡ‚Ρ€Π΅Π±ΠΈΡ‚Π΅Π»Π΅ΠΉ элСктричСской энСргии. Π Π°Π·Π»ΠΈΡ‡Π°ΡŽΡ‚ Ρ‚Ρ€ΠΈ Π²ΠΈΠ΄Π° соСдинСния сопротивлСний (рСзисторов):

  1. ΠΏΠΎΡΠ»Π΅Π΄ΠΎΠ²Π°Ρ‚Π΅Π»ΡŒΠ½ΠΎΠ΅ соСдинСниС ΠΏΡ€ΠΎΠ²ΠΎΠ΄Π½ΠΈΠΊΠΎΠ²
  2. ΠΏΠ°Ρ€Π°Π»Π»Π΅Π»ΡŒΠ½ΠΎΠ΅ соСдинСниС ΠΏΡ€ΠΎΠ²ΠΎΠ΄Π½ΠΈΠΊΠΎΠ²
  3. смСшанноС соСдинСниС ΠΏΡ€ΠΎΠ²ΠΎΠ΄Π½ΠΈΠΊΠΎΠ²

ΠŸΠΎΡΠ»Π΅Π΄ΠΎΠ²Π°Ρ‚Π΅Π»ΡŒΠ½ΠΎΠ΅ соСдинСниС ΠΏΡ€ΠΎΠ²ΠΎΠ΄Π½ΠΈΠΊΠΎΠ²

Π‘Ρ…Π΅ΠΌΠ° соСдинСния выглядит ΡΠ»Π΅Π΄ΡƒΡŽΡ‰ΠΈΠΌ ΠΎΠ±Ρ€Π°Π·ΠΎΠΌ:

Β 

Β 

ΠžΠ±Ρ€Π°Ρ‚ΠΈ Π²Π½ΠΈΠΌΠ°Π½ΠΈΠ΅!

ΠŸΡ€ΠΈ ΠΏΠΎΡΠ»Π΅Π΄ΠΎΠ²Π°Ρ‚Π΅Π»ΡŒΠ½ΠΎΠΌ соСдинСнии всС входящиС Π² Π½Π΅Π³ΠΎ ΠΏΡ€ΠΎΠ²ΠΎΠ΄Π½ΠΈΠΊΠΈ ΡΠΎΠ΅Π΄ΠΈΠ½ΡΡŽΡ‚ΡΡ Π΄Ρ€ΡƒΠ³ Π·Π° Π΄Ρ€ΡƒΠ³ΠΎΠΌ, Ρ‚.Π΅. ΠΊΠΎΠ½Π΅Ρ† ΠΏΠ΅Ρ€Π²ΠΎΠ³ΠΎ ΠΏΡ€ΠΎΠ²ΠΎΠ΄Π½ΠΈΠΊΠ° соСдиняСтся с Π½Π°Ρ‡Π°Π»ΠΎΠΌ Π²Ρ‚ΠΎΡ€ΠΎΠ³ΠΎ.

Β 

Β 

ΠžΠΏΡ‹Ρ‚ ΠΏΠΎΠΊΠ°Π·Ρ‹Π²Π°Π΅Ρ‚:

Π‘ΠΈΠ»Π° Ρ‚ΠΎΠΊΠ° Π² Π»ΡŽΠ±Ρ‹Ρ… частях Ρ†Π΅ΠΏΠΈ ΠΎΠ΄Π½Π° ΠΈ Ρ‚Π° ΠΆΠ΅ (ΠΎΠ± этом ΡΠ²ΠΈΠ΄Π΅Ρ‚Π΅Π»ΡŒΡΡ‚Π²ΡƒΡŽΡ‚ показания Π°ΠΌΠΏΠ΅Ρ€ΠΌΠ΅Ρ‚Ρ€ΠΎΠ²): I=I1=I2.

Если Π²Ρ‹ΠΊΡ€ΡƒΡ‚ΠΈΡ‚ΡŒ ΠΎΠ΄Π½Ρƒ Π»Π°ΠΌΠΏΡƒ, Ρ‚ΠΎ Ρ†Π΅ΠΏΡŒ разомкнётся,Β Π° другая Π»Π°ΠΌΠΏΠ° Ρ‚ΠΎΠΆΠ΅ погаснСт.

Β 

ΠžΠΏΡ‹Ρ‚ ΠΏΠΎΠΊΠ°Π·Ρ‹Π²Π°Π΅Ρ‚ ΡΠ»Π΅Π΄ΡƒΡŽΡ‰Π΅Π΅:Β 

ΠŸΡ€ΠΈ ΠΏΠΎΡΠ»Π΅Π΄ΠΎΠ²Π°Ρ‚Π΅Π»ΡŒΠ½ΠΎΠΌ соСдинСнии сопротивлСний Ρ€Π΅Π·ΡƒΠ»ΡŒΡ‚ΠΈΡ€ΡƒΡŽΡ‰Π΅Π΅ напряТСниС Ρ€Π°Π²Π½ΠΎ суммС напряТСний Π½Π° участках: U=U1+U2.

Β 

Β 

Π Π΅Π·ΡƒΠ»ΡŒΡ‚ΠΈΡ€ΡƒΡŽΡ‰Π΅Π΅ сопротивлСниС ΠΏΠΎΡΠ»Π΅Π΄ΠΎΠ²Π°Ρ‚Π΅Π»ΡŒΠ½ΠΎ соСдинённых ΠΏΠΎΡ‚Ρ€Π΅Π±ΠΈΡ‚Π΅Π»Π΅ΠΉ Ρ€Π°Π²Π½ΠΎ суммС сопротивлСний ΠΏΠΎΡ‚Ρ€Π΅Π±ΠΈΡ‚Π΅Π»Π΅ΠΉ: R=R1+R2.


Для ΠΏΡ€ΠΎΠ²Π΅Ρ€ΠΊΠΈ Π΄Π°Π½Π½ΠΎΠ³ΠΎ утвСрТдСния ΠΌΠΎΠΆΠ½ΠΎ ΠΈΡΠΏΠΎΠ»ΡŒΠ·ΠΎΠ²Π°Ρ‚ΡŒ ΠΎΠΌΠΌΠ΅Ρ‚Ρ€. ΠŸΡ€ΠΈ ΠΏΠΎΠ΄ΠΊΠ»ΡŽΡ‡Π΅Π½ΠΈΠΈ ΠΎΠΌΠΌΠ΅Ρ‚Ρ€Π° ΠΊΠ»ΡŽΡ‡ Π΄ΠΎΠ»ΠΆΠ΅Π½ Π±Ρ‹Ρ‚ΡŒ Ρ€Π°Π·ΠΎΠΌΠΊΠ½ΡƒΡ‚!

Β 

ΠžΠΌΠΌΠ΅Ρ‚Ρ€ ΠΏΠΎΠ΄ΠΊΠ»ΡŽΡ‡Π°ΡŽΡ‚ ΠΏΠΎ ΠΎΡ‡Π΅Ρ€Π΅Π΄ΠΈ ΠΊ ΠΊΠ°ΠΆΠ΄ΠΎΠΌΡƒ ΠΏΠΎΡ‚Ρ€Π΅Π±ΠΈΡ‚Π΅Π»ΡŽ, Π° ΠΏΠΎΡ‚ΠΎΠΌ ΠΊ ΠΎΠ±ΠΎΠΈΠΌ ΠΎΠ΄Π½ΠΎΠ²Ρ€Π΅ΠΌΠ΅Π½Π½ΠΎ.

Β 

Π‘ΠΎΠΏΡ€ΠΎΡ‚ΠΈΠ²Π»Π΅Π½ΠΈΠ΅ Ρ†Π΅ΠΏΠΈ \(R\), состоящСй ΠΈΠ·Β \(n\) ΠΎΠ΄ΠΈΠ½Π°ΠΊΠΎΠ²Ρ‹Ρ… Π»Π°ΠΌΠΏ, сопротивлСниСм R1 каТдая, Π² \(n\) Ρ€Π°Π· большС сопротивлСния ΠΎΠ΄Π½ΠΎΠΉ Π»Π°ΠΌΠΏΡ‹: \(R\) = R1* \(n\).

Β 

ΠŸΠ°Ρ€Π°Π»Π»Π΅Π»ΡŒΠ½ΠΎΠ΅ соСдинСниС ΠΏΡ€ΠΎΠ²ΠΎΠ΄Π½ΠΈΠΊΠΎΠ²

Π‘Ρ…Π΅ΠΌΠ° соСдинСния выглядит ΡΠ»Π΅Π΄ΡƒΡŽΡ‰ΠΈΠΌ ΠΎΠ±Ρ€Π°Π·ΠΎΠΌ:

Β 

Β 

ΠžΠ±Ρ€Π°Ρ‚ΠΈ Π²Π½ΠΈΠΌΠ°Π½ΠΈΠ΅!

ΠŸΡ€ΠΈ ΠΏΠ°Ρ€Π°Π»Π»Π΅Π»ΡŒΠ½ΠΎΠΌ соСдинСнии всС входящиС Π² Π½Π΅Π³ΠΎ ΠΏΡ€ΠΎΠ²ΠΎΠ΄Π½ΠΈΠΊΠΈ ΠΎΠ΄Π½ΠΈΠΌ своим ΠΊΠΎΠ½Ρ†ΠΎΠΌ ΠΏΡ€ΠΈΡΠΎΠ΅Π΄ΠΈΠ½ΡΡŽΡ‚ΡΡ ΠΊ ΠΎΠ΄Π½ΠΎΠΉ Ρ‚ΠΎΡ‡ΠΊΠ΅ Ρ†Π΅ΠΏΠΈ А,Β Π° Π²Ρ‚ΠΎΡ€Ρ‹ΠΌ ΠΊΠΎΠ½Ρ†ΠΎΠΌΒ β€” ΠΊ Π΄Ρ€ΡƒΠ³ΠΎΠΉ Ρ‚ΠΎΡ‡ΠΊΠ΅ Π’.

Β 

Β 

ΠžΠΏΡ‹Ρ‚ Π΄ΠΎΠΊΠ°Π·Ρ‹Π²Π°Π΅Ρ‚:

Π‘ΠΈΠ»Π° Ρ‚ΠΎΠΊΠ° Π² Π½Π΅Ρ€Π°Π·Π²Π΅Ρ‚Π²Π»Ρ‘Π½Π½ΠΎΠΉ части Ρ†Π΅ΠΏΠΈ Ρ€Π°Π²Π½Π° суммС сил Ρ‚ΠΎΠΊΠ° Π² ΠΎΡ‚Π΄Π΅Π»ΡŒΠ½Ρ‹Ρ… ΠΏΠ°Ρ€Π°Π»Π»Π΅Π»ΡŒΠ½ΠΎ соСдинённых ΠΏΡ€ΠΎΠ²ΠΎΠ΄Π½ΠΈΠΊΠ°Ρ….

Об этом ΡΠ²ΠΈΠ΄Π΅Ρ‚Π΅Π»ΡŒΡΡ‚Π²ΡƒΡŽΡ‚ показания Π°ΠΌΠΏΠ΅Ρ€ΠΌΠ΅Ρ‚Ρ€ΠΎΠ²: I=I1+I2.

Β 

Β 

Если Π²Ρ‹ΠΊΡ€ΡƒΡ‚ΠΈΡ‚ΡŒ ΠΎΠ΄Π½Ρƒ Π»Π°ΠΌΠΏΡƒ, Ρ‚ΠΎ другая Π»Π°ΠΌΠΏΠ° ΠΏΡ€ΠΎΠ΄ΠΎΠ»ΠΆΠ°Π΅Ρ‚ Π³ΠΎΡ€Π΅Ρ‚ΡŒ. Π­Ρ‚ΠΎ свойство ΠΈΡΠΏΠΎΠ»ΡŒΠ·ΡƒΡŽΡ‚ для ΠΏΠΎΠ΄ΠΊΠ»ΡŽΡ‡Π΅Π½ΠΈΡ Π±Ρ‹Ρ‚ΠΎΠ²Ρ‹Ρ… ΠΏΡ€ΠΈΠ±ΠΎΡ€ΠΎΠ² Π² ΠΏΠΎΠΌΠ΅Ρ‰Π΅Π½ΠΈΠΈ.

Β 

ΠžΠΏΡ‹Ρ‚ ΡΠ²ΠΈΠ΄Π΅Ρ‚Π΅Π»ΡŒΡΡ‚Π²ΡƒΠ΅Ρ‚, Ρ‡Ρ‚ΠΎ:

НапряТСниС Π½Π° участкС Ρ†Π΅ΠΏΠΈ АВ ΠΈ Π½Π° ΠΊΠΎΠ½Ρ†Π°Ρ… всСх ΠΏΠ°Ρ€Π°Π»Π»Π΅Π»ΡŒΠ½ΠΎ соСдинённых ΠΏΡ€ΠΎΠ²ΠΎΠ΄Π½ΠΈΠΊΠΎΠ² ΠΎΠ΄Π½ΠΎ ΠΈ Ρ‚ΠΎ ΠΆΠ΅.

Об этом ΡΠ²ΠΈΠ΄Π΅Ρ‚Π΅Π»ΡŒΡΡ‚Π²ΡƒΡŽΡ‚ показания Π²ΠΎΠ»ΡŒΡ‚ΠΌΠ΅Ρ‚Ρ€ΠΎΠ²:

U=U1=U2.

Β 

Β 

ΠžΠ±Ρ‰Π΅Π΅ сопротивлСниС Ρ†Π΅ΠΏΠΈ ΠΏΡ€ΠΈ ΠΏΠ°Ρ€Π°Π»Π»Π΅Π»ΡŒΠ½ΠΎΠΌ соСдинСнии ΠΏΡ€ΠΎΠ²ΠΎΠ΄Π½ΠΈΠΊΠΎΠ² опрСдСляСтся ΠΏΠΎ Ρ„ΠΎΡ€ΠΌΡƒΠ»Π΅:

Β 

1R=1R1+1R2.

Β 

ΠžΠ±Ρ€Π°Ρ‚Π½ΠΎΠ΅ Π·Π½Π°Ρ‡Π΅Π½ΠΈΠ΅ ΠΎΠ±Ρ‰Π΅Π³ΠΎ сопротивлСния Ρ€Π°Π²Π½ΠΎ суммС ΠΎΠ±Ρ€Π°Ρ‚Π½Ρ‹Ρ… Π·Π½Π°Ρ‡Π΅Π½ΠΈΠΉ сопротивлСний ΠΎΡ‚Π΄Π΅Π»ΡŒΠ½Ρ‹Ρ… ΠΏΡ€ΠΎΠ²ΠΎΠ΄Π½ΠΈΠΊΠΎΠ².

Для ΠΏΡ€ΠΎΠ²Π΅Ρ€ΠΊΠΈ Ρ„ΠΎΡ€ΠΌΡƒΠ»Ρ‹ ΠΌΠΎΠΆΠ½ΠΎ ΠΈΡΠΏΠΎΠ»ΡŒΠ·ΠΎΠ²Π°Ρ‚ΡŒ ΠΎΠΌΠΌΠ΅Ρ‚Ρ€. ΠŸΡ€ΠΈ ΠΏΠΎΠ΄ΠΊΠ»ΡŽΡ‡Π΅Π½ΠΈΠΈ ΠΎΠΌΠΌΠ΅Ρ‚Ρ€Π° ΠΊΠ»ΡŽΡ‡ Π΄ΠΎΠ»ΠΆΠ΅Π½ Π±Ρ‹Ρ‚ΡŒ Ρ€Π°Π·ΠΎΠΌΠΊΠ½ΡƒΡ‚!

Β 

Π‘ΠΎΠΏΡ€ΠΎΡ‚ΠΈΠ²Π»Π΅Π½ΠΈΠ΅ Ρ†Π΅ΠΏΠΈ \(R\), состоящСй ΠΈΠ· \(n\) ΠΎΠ΄ΠΈΠ½Π°ΠΊΠΎΠ²Ρ‹Ρ… Π»Π°ΠΌΠΏ, сопротивлСниСм R1 каТдая, Π² \(n\) Ρ€Π°Π· мСньшС сопротивлСния ΠΎΠ΄Π½ΠΎΠΉ Π»Π°ΠΌΠΏΡ‹: \(R\) = R1/ \(n\).

Π˜ΡΡ‚ΠΎΡ‡Π½ΠΈΠΊΠΈ:

http://files.school-collection.edu.ru/dlrstore/669ba073-e921-11dc-95ff-0800200c9a66/3_17.swf
http://files.school-collection.edu.ru/dlrstore/669ba074-e921-11dc-95ff-0800200c9a66/3_18.swf

http://class-fizika.narod.ru/8_33.htm

ΠŸΠΎΡΠ»Π΅Π΄ΠΎΠ²Π°Ρ‚Π΅Π»ΡŒΠ½ΠΎΠ΅ ΠΈ ΠΏΠ°Ρ€Π°Π»Π»Π΅Π»ΡŒΠ½ΠΎΠ΅ соСдинСниС кондСнсаторов

Π‘ΠΎΠ΅Π΄ΠΈΠ½Π΅Π½ΠΈΠ΅ кондСнсаторов Π² элСктричСской Ρ†Π΅ΠΏΠΈ ΠΌΠΎΠΆΠ΅Ρ‚ Π±Ρ‹Ρ‚ΡŒ ΠΏΠΎΡΠ»Π΅Π΄ΠΎΠ²Π°Ρ‚Π΅Π»ΡŒΠ½Ρ‹ΠΌ, ΠΏΠ°Ρ€Π°Π»Π»Π΅Π»ΡŒΠ½Ρ‹ΠΌ ΠΈ ΠΏΠΎΡΠ»Π΅Π΄ΠΎΠ²Π°Ρ‚Π΅Π»ΡŒΠ½ΠΎ-ΠΏΠ°Ρ€Π΅Π»Π»Π΅Π»ΡŒΠ½Ρ‹ΠΌ (ΡΠΌΠ΅ΡˆΠ°Π½Π½Ρ‹ΠΌ).

Если провСсти аналогию ΠΌΠ΅ΠΆΠ΄Ρƒ соСдинСниСм кондСнсаторов ΠΈ соСдинСниСм рСзисторов , Ρ‚ΠΎ стоит ΠΎΡ‚ΠΌΠ΅Ρ‚ΠΈΡ‚ΡŒ, Ρ‡Ρ‚ΠΎ Ρ„ΠΎΡ€ΠΌΡƒΠ»Ρ‹ расчСта ΠΎΠ±Ρ‰Π΅ΠΉ Смкости ΠΈ ΠΎΠ±Ρ‰Π΅Π³ΠΎ сопротивлСния ΠΈΠ΄Π΅Π½Ρ‚ΠΈΡ‡Π½Ρ‹, Ρ‚ΠΎΠ»ΡŒΠΊΠΎ ΠΌΠ΅ΠΆΠ΄Ρƒ Ρ€Π°Π·Π½Ρ‹ΠΌΠΈ Ρ‚ΠΈΠΏΠ°ΠΌΠΈ соСдинСний:

Π€ΠΎΡ€ΠΌΡƒΠ»Π° CΠΎΠ±Ρ‰ ΠΏΡ€ΠΈ ΠΏΠ°Ρ€Π°Π»Π»Π΅Π»ΡŒΠ½ΠΎΠΌ соСдинСнии кондСнсаторов = Ρ„ΠΎΡ€ΠΌΡƒΠ»Π° RΠΎΠ±Ρ‰ ΠΏΡ€ΠΈ ΠΏΠΎΡΠ»Π΅Π΄ΠΎΠ²Π°Ρ‚Π΅Π»ΡŒΠ½ΠΎΠΌ соСдинСнии рСзисторов.

Π€ΠΎΡ€ΠΌΡƒΠ»Π° CΠΎΠ±Ρ‰ ΠΏΡ€ΠΈ ΠΏΠΎΡΠ»Π΅Π΄ΠΎΠ²Π°Ρ‚Π΅Π»ΡŒΠ½ΠΎΠΌ соСдинСнии кондСнсаторов = Ρ„ΠΎΡ€ΠΌΡƒΠ»Π° RΠΎΠ±Ρ‰ ΠΏΡ€ΠΈ ΠΏΠ°Ρ€Π°Π»Π»Π΅Π»ΡŒΠ½ΠΎΠΌ соСдинСнии рСзисторов.

  • CΠΎΠ±Ρ‰ β€” общая Π΅ΠΌΠΊΠΎΡΡ‚ΡŒ
  • RΠΎΠ±Ρ‰ β€” ΠΎΠ±Ρ‰Π΅Π΅ сопротивлСниС

ΠŸΠ°Ρ€Π°Π»Π»Π΅Π»ΡŒΠ½ΠΎΠ΅ соСдинСниС кондСнсаторов

ΠŸΠ°Ρ€Π°Π»Π»Π΅Π»ΡŒΠ½ΠΎΠ΅ соСдинСниС кондСнсаторов β€” это соСдинСниС ΠΏΡ€ΠΈ ΠΊΠΎΡ‚ΠΎΡ€ΠΎΠΌ кондСнсаторы ΡΠΎΠ΅Π΄ΠΈΠ½ΡΡŽΡ‚ΡΡ собой ΠΎΠ±ΠΎΠΈΠΌΠΈ ΠΊΠΎΠ½Ρ‚Π°ΠΊΡ‚Π°ΠΌΠΈ. Π’ Ρ€Π΅Π·ΡƒΠ»ΡŒΡ‚Π°Ρ‚Π΅ ΠΊ ΠΎΠ΄Π½ΠΎΠΉ Ρ‚ΠΎΡ‡ΠΊΠ΅ ΠΌΠΎΠΆΠ΅Ρ‚ Π±Ρ‹Ρ‚ΡŒ присоСдинСно нСсколько кондСнсаторов.

ΠŸΡ€ΠΈ ΠΏΠ°Ρ€Π°Π»Π»Π΅Π»ΡŒΠ½ΠΎΠΌ соСдинСнии формируСтся ΠΎΠ΄ΠΈΠ½ большой кондСнсатор с ΠΏΠ»ΠΎΡ‰Π°Π΄ΡŒΡŽ ΠΎΠ±ΠΊΠ»Π°Π΄ΠΎΠΊ, Ρ€Π°Π²Π½ΠΎΠΉ суммС ΠΏΠ»ΠΎΡ‰Π°Π΄Π΅ΠΉ ΠΎΠ±ΠΊΠ»Π°Π΄ΠΎΠΊ всСх ΠΎΡ‚Π΄Π΅Π»ΡŒΠ½Ρ‹Ρ… ΠΊΠΎΠΌΠΏΠΎΠ½Π΅Π½Ρ‚ΠΎΠ². ΠŸΠΎΡΠΊΠΎΠ»ΡŒΠΊΡƒ Π΅ΠΌΠΊΠΎΡΡ‚ΡŒ кондСнсаторов прямо ΠΏΡ€ΠΎΠΏΠΎΡ€Ρ†ΠΈΠΎΠ½Π°Π»ΡŒΠ½Π° ΠΏΠ»ΠΎΡ‰Π°Π΄ΠΈ ΠΎΠ±ΠΊΠ»Π°Π΄ΠΎΠΊ, общая Π΅ΠΌΠΊΠΎΡΡ‚ΡŒ Π‘ΠΎΠ±Ρ‰ ΠΏΡ€ΠΈ ΠΏΠ°Ρ€Π°Π»Π»Π΅Π»ΡŒΠ½ΠΎΠΌ соСдинСнии равняСтся суммС СмкостСй всСх кондСнсаторов Π² Ρ†Π΅ΠΏΠΈ.

ΠŸΠ°Ρ€Π°Π»Π»Π΅Π»ΡŒΠ½ΠΎΠ΅ соСдинСниС кондСнсаторов

НапряТСниС ΠΏΡ€ΠΈ ΠΏΠ°Ρ€Π°Π»Π»Π΅Π»ΡŒΠ½ΠΎΠΌ соСдинСнии

На всС ΠΏΠ°Ρ€Π°Π»Π»Π΅Π»ΡŒΠ½ΠΎ соСдинСнныС кондСнсаторы ΠΏΠ°Π΄Π°Π΅Ρ‚ ΠΎΠ΄ΠΈΠ½Π°ΠΊΠΎΠ²ΠΎΠ΅ напряТСниС. Π’Π°ΠΊ происходит, ΠΏΠΎΡ‚ΠΎΠΌΡƒ Ρ‡Ρ‚ΠΎ сущСствуСт всСго лишь Π΄Π²Π΅ Ρ‚ΠΎΡ‡ΠΊΠΈ, ΠΌΠ΅ΠΆΠ΄Ρƒ ΠΊΠΎΡ‚ΠΎΡ€Ρ‹ΠΌΠΈ ΠΌΠΎΠΆΠ΅Ρ‚ Π±Ρ‹Ρ‚ΡŒ Ρ€Π°Π·Π½ΠΎΡΡ‚ΡŒ ΠΏΠΎΡ‚Π΅Π½Ρ†ΠΈΠ°Π»ΠΎΠ² (напряТСниС). Π”Ρ€ΡƒΠ³ΠΈΠΌΠΈ словами, ΠΌΠΎΠΆΠ½ΠΎ ΡΠΊΠ°Π·Π°Ρ‚ΡŒ Ρ‡Ρ‚ΠΎ ΠΏΡ€ΠΈ ΠΏΠ°Ρ€Π°Π»Π»Π΅Π»ΡŒΠ½ΠΎΠΌ соСдинСнии всС кондСнсаторы ΠΏΠΎΠ΄ΠΊΠ»ΡŽΡ‡Π΅Π½Ρ‹ ΠΊ ΠΎΠ΄Π½ΠΎΠΌΡƒ источнику напряТСния.

ПадСниС напряТСния ΠΏΡ€ΠΈ ΠΏΠ°Ρ€Π°Π»Π»Π΅Π»ΡŒΠ½ΠΎΠΌ соСдинСнии

Π’ΠΎΠΊ ΠΏΡ€ΠΈ ΠΏΠ°Ρ€Π°Π»Π»Π΅Π»ΡŒΠ½ΠΎΠΌ соСдинСнии

Π’ΠΎΠΊ кондСнсатора Π²ΠΎ врСмя ΠΏΠ΅Ρ€Π΅Ρ…ΠΎΠ΄Π½ΠΎΠ³ΠΎ ΠΏΠ΅Ρ€ΠΈΠΎΠ΄Π° зависит ΠΎΡ‚ Π΅Π³ΠΎ Смкости ΠΈ измСнСния напряТСния:

  • ic β€” Ρ‚ΠΎΠΊ кондСнсатора
  • C β€” Π•ΠΌΠΊΠΎΡΡ‚ΡŒ кондСнсатора
  • Ξ”VC/Ξ”t – Π‘ΠΊΠΎΡ€ΠΎΡΡ‚ΡŒ измСнСния напряТСния

ΠŸΡ€ΠΈ ΠΏΠ°Ρ€Π°Π»Π»Π΅Π»ΡŒΠ½ΠΎΠΌ соСдинСнии Ρ‡Π΅Ρ€Π΅Π· ΠΊΠ°ΠΆΠ΄Ρ‹ΠΉ кондСнсатор ΠΏΠΎΡ‚Π΅Ρ‡Π΅Ρ‚ ΠΎΠ΄Π΅Π»ΡŒΠ½Ρ‹ΠΉ Ρ‚ΠΎΠΊ, Π² зависимости ΠΎΡ‚ Смкости кондСнсатора:

Π’ΠΎΠΊ ΠΏΡ€ΠΈ ΠΏΠ°Ρ€Π°Π»Π»Π΅Π»ΡŒΠ½ΠΎΠΌ соСдинСнии

ΠŸΠΎΡΠ»Π΅Π΄ΠΎΠ²Π°Ρ‚Π΅Π»ΡŒΠ½ΠΎΠ΅ соСдинСниС кондСнсаторов

ΠŸΠΎΡΠ»Π΅Π΄ΠΎΠ²Π°Ρ‚Π΅Π»ΡŒΠ½ΠΎΠ΅ соСдинСниС кондСнсаторов – это соСдинСниС Π΄Π²ΡƒΡ… ΠΈΠ»ΠΈ Π±ΠΎΠ»Π΅Π΅ кондСнсаторов Π² Ρ„ΠΎΡ€ΠΌΠ΅ Ρ†Π΅ΠΏΠΈ, Π² ΠΊΠΎΡ‚ΠΎΡ€ΠΎΠΉ ΠΊΠ°ΠΆΠ΄Ρ‹ΠΉ ΠΎΡ‚Π΄Π΅Π»ΡŒΠ½Ρ‹ΠΉ кондСнсатор соСдиняСтся с Π΄Ρ€ΡƒΠ³ΠΈΠΌ ΠΎΡ‚Π΄Π΅Π»ΡŒΠ½Ρ‹ΠΌ кондСнсатором Ρ‚ΠΎΠ»ΡŒΠΊΠΎ Π² ΠΎΠ΄Π½ΠΎΠΉ Ρ‚ΠΎΡ‡ΠΊΠ΅.

ΠŸΠΎΡΠ»Π΅Π΄ΠΎΠ²Π°Ρ‚Π΅Π»ΡŒΠ½ΠΎΠ΅ соСдинСниС кондСнсаторов

Π’ΠΎΠΊ ΠΏΡ€ΠΈ ΠΏΠΎΡΠ»Π΅Π΄ΠΎΠ²Π°Ρ‚Π΅Π»ΡŒΠ½ΠΎΠΌ соСдинСнии

Π’ΠΎΠΊ (iC), Π·Π°Ρ€ΡΠΆΠ°ΡŽΡ‰ΠΈΠΉ ΠΏΠΎΡΠ»Π΅Π΄ΠΎΠ²Π°Ρ‚Π΅Π»ΡŒΠ½ΡƒΡŽ Ρ†Π΅ΠΏΡŒ кондСнсаторов, Π±ΡƒΠ΄Π΅Ρ‚ ΠΎΠ΄ΠΈΠ½Π°ΠΊΠΎΠ²Ρ‹ΠΌ для всСх кондСнсаторов, ΠΏΠΎΡΠΊΠΎΠ»ΡŒΠΊΡƒ Ρƒ Π½Π΅Π³ΠΎ Π΅ΡΡ‚ΡŒ Ρ‚ΠΎΠ»ΡŒΠΊΠΎ ΠΎΠ΄ΠΈΠ½ Π²ΠΎΠ·ΠΌΠΎΠΆΠ½Ρ‹ΠΉ ΠΏΡƒΡ‚ΡŒ прохоТдСния:

ВслСдствиС Ρ‚ΠΎΠ³ΠΎ Ρ‡Ρ‚ΠΎ Ρ‡Π΅Ρ€Π΅Π· всС ΠΏΠΎΡΠ»Π΅Π΄ΠΎΠ²Π°Ρ‚Π΅Π»ΡŒΠ½ΠΎ соСдинСнныС кондСнсаторы Ρ‚Π΅Ρ‡Π΅Ρ‚ ΠΎΠ΄ΠΈΠ½Π°ΠΊΠΎΠ²Ρ‹ΠΉ Ρ‚ΠΎΠΊ, количСство Π½Π°ΠΊΠΎΠΏΠ»Π΅Π½ΠΎΠ³ΠΎ элСктричСского заряда для ΠΊΠ°ΠΆΠ΄ΠΎΠ³ΠΎ кондСнсатора Π±ΡƒΠ΄Π΅Ρ‚ ΠΎΠ΄ΠΈΠ½Π°ΠΊΠΎΠ²Ρ‹ΠΌ, нСзависимо ΠΎΡ‚ Π΅Π³ΠΎ Смкости. Π’Π°ΠΊ происходит, ΠΏΠΎΡ‚ΠΎΠΌΡƒ Ρ‡Ρ‚ΠΎ элСктричСский заряд, Π½Π°ΠΊΠ°ΠΏΠ»ΠΈΠ²Π°Π΅ΠΌΡ‹ΠΉ Π½Π° ΠΎΠ±ΠΊΠ»Π°Π΄ΠΊΠ΅ любого кондСнсатора, Π΄ΠΎΠ»ΠΆΠ΅Π½ ΠΏΡ€ΠΈΠΉΡ‚ΠΈ с ΠΎΠ±ΠΊΠ»Π°Π΄ΠΊΠΈ ΠΏΡ€ΠΈΠΌΡ‹ΠΊΠ°ΡŽΡ‰Π΅Π³ΠΎ кондСнсатора.

Π’Π°ΠΊΠΈΠΌ ΠΎΠ±Ρ€Π°Π·ΠΎΠΌ, ΠΏΠΎΡΠ»Π΅Π΄ΠΎΠ²Π°Ρ‚Π΅Π»ΡŒΠ½ΠΎ соСдинСнныС кондСнсаторы ΠΈΠΌΠ΅ΡŽΡ‚ ΠΎΠ΄ΠΈΠ½Π°ΠΊΠΎΠ²Ρ‹ΠΉ элСктричСский заряд:

ΠŸΠΎΡΠΌΠΎΡ‚Ρ€ΠΈΠΌ Π½Π° ΠΏΠΎΡΠ»Π΅Π΄ΠΎΠ²Π°Ρ‚Π΅Π»ΡŒΠ½ΡƒΡŽ Ρ†Π΅ΠΏΡŒ ΠΈΠ· Ρ‚Ρ€Π΅Ρ… кондСнсаторов Π½Π° рисункС Π²Ρ‹ΡˆΠ΅. ΠŸΡ€Π°Π²Π°Ρ ΠΎΠ±ΠΊΠ»Π°Π΄ΠΊΠ° ΠΏΠ΅Ρ€Π²ΠΎΠ³ΠΎ кондСнсатора Π‘1 соСдиняСтся с Π»Π΅Π²ΠΎΠΉ Π²Ρ‚ΠΎΡ€ΠΎΠ³ΠΎ кондСнсатора Π‘2, Ρƒ ΠΊΠΎΡ‚ΠΎΡ€ΠΎΠ³ΠΎ правая ΠΎΠ±ΠΊΠ»Π°Π΄ΠΊΠ° соСдиняСтся с Π»Π΅Π²ΠΎΠΉ Ρ‚Ρ€Π΅Ρ‚ΡŒΠ΅Π³ΠΎ кондСнсатора Π‘3. Π­Ρ‚ΠΎ ΠΎΠ·Π½Π°Ρ‡Π°Π΅Ρ‚, Ρ‡Ρ‚ΠΎ Π² Ρ€Π΅ΠΆΠΈΠΌΠ΅ постоянного Ρ‚ΠΎΠΊΠ° кондСнсатор Π‘2 элСктричСски ΠΈΠ·ΠΎΠ»ΠΈΡ€ΠΎΠ²Π°Π½ ΠΎΡ‚ ΠΎΠ±Ρ‰Π΅ΠΉ Ρ†Π΅ΠΏΠΈ.

Π’ ΠΈΡ‚ΠΎΠ³ΠΎΠ΅ эффСктивная ΠΏΠ»ΠΎΡ‰Π°Π΄ΡŒ ΠΎΠ±ΠΊΠ»Π°Π΄ΠΎΠΊ ΡƒΠΌΠ΅Π½ΡŒΡˆΠ°Π΅Ρ‚ΡΡ Π΄ΠΎ ΠΏΠ»ΠΎΡ‰Π°Π΄ΠΈ ΠΎΠ±ΠΊΠ»Π°Π΄ΠΎΠΊ самого малСнького кондСнсатора. Π­Ρ‚ΠΎ ΠΎΠ±ΡŠΡΡΠ½ΡΠ΅Ρ‚ΡΡ Ρ‚Π΅ΠΌ, Ρ‡Ρ‚ΠΎ ΠΊΠ°ΠΊ Ρ‚ΠΎΠ»ΡŒΠΊΠΎ ΠΎΠ±ΠΊΠ»Π°Π΄ΠΊΠΈ наимСншСй ΠΏΠ»ΠΎΡ‰Π°Π΄ΠΈ заполнятся элСктричСским зарядом, Π΄Π°Π½Π½Ρ‹ΠΉ кондСнсатор пСрСстанСт ΠΏΡ€ΠΎΠΏΡƒΡΠΊΠ°Ρ‚ΡŒ Ρ‚ΠΎΠΊ. Π’ Ρ€Π΅Π·ΡƒΠ»ΡŒΡ‚Π°Ρ‚Π΅ Ρ‚ΠΎΠΊ ΠΏΡ€Π΅ΠΊΡ€Π°Ρ‚ΠΈΡ‚ΡŒΡΡ Π²ΠΎ всСй Ρ†Π΅ΠΏΠΈ, ΠΈ процСсс зарядки ΠΎΡΡ‚Π°Π»ΡŒΠ½Ρ‹Ρ… кондСнсаторов Ρ‚Π°ΠΊΠΆΠ΅ прСкратится.

ΠŸΡ€ΠΈ ΠΏΠΎΡΠ»Π΅Π΄ΠΎΠ²Π°Ρ‚Π΅Π»ΡŒΠ½ΠΎΠΌ соСдинСнии ΠΎΠ±Ρ‰Π΅Π΅ расстояниС ΠΌΠ΅ΠΆΠ΄Ρƒ ΠΎΠ±ΠΊΠ»Π°Π΄ΠΊΠ°ΠΌΠΈ увСличиваСтся Π΄ΠΎ суммы расстояний ΠΌΠ΅ΠΆΠ΄Ρƒ ΠΎΠ±ΠΊΠ»Π°Π΄ΠΊΠ°ΠΌΠΈ всСх кондСнсаторов.

Π’Π°ΠΊΠΈΠΌ ΠΎΠ±Ρ€Π°Π·ΠΎΠΌ, ΠΏΠΎΡΠ»Π΅Π΄ΠΎΠ²Π°Ρ‚Π΅Π»ΡŒΠ½Π°Ρ Ρ†Π΅ΠΏΡŒ Ρ„ΠΎΡ€ΠΌΠΈΡ€ΡƒΠ΅Ρ‚ ΠΎΠ΄ΠΈΠ½ большой кондСнсатор с ΠΏΠ»ΠΎΡ‰Π°Π΄ΡŒΡŽ ΠΎΠ±ΠΊΠ»Π°Π΄ΠΎΠΊ элСмСнта с наимСньшСй Π΅ΠΌΠΊΠΎΡΡ‚ΡŒΡŽ, ΠΈ расстояниСм ΠΌΠ΅ΠΆΠ΄Ρƒ ΠΎΠ±ΠΊΠ»Π°Π΄ΠΊΠ°ΠΌΠΈ, Ρ€Π°Π²Π½ΠΎΠΌΡƒ суммС всСх расстояний Π² Ρ†Π΅ΠΏΠΈ.

ΠŸΠ»ΠΎΡ‰Π°Π΄ΡŒ ΠΈ расстояниС ΠΌΠ΅ΠΆΠ΄Ρƒ ΠΎΠ±ΠΊΠ»Π°Π΄ΠΊΠ°ΠΌΠΈ ΠΏΡ€ΠΈ ΠΏΠΎΡΠ»Π΅Π΄ΠΎΠ²Π°Ρ‚Π΅Π»ΡŒΠ½ΠΎΠΌ соСдинСнии

ПадСниС напряТСния ΠΈ общая Π΅ΠΌΠΊΠΎΡΡ‚ΡŒ ΠΏΡ€ΠΈ ΠΏΠΎΡΠ»Π΅Π΄ΠΎΠ²Π°Ρ‚Π΅Π»ΡŒΠ½ΠΎΠΌ соСдинСнии

На ΠΊΠ°ΠΆΠ΄Ρ‹ΠΉ ΠΎΡ‚Π΄Π΅Π»ΡŒΠ½Ρ‹ΠΉ кондСнсатор Π² ΠΏΠΎΡΠ»Π΅Π΄ΠΎΠ²Π°Ρ‚Π΅Π»ΡŒΠ½ΠΎΠΉ Ρ†Π΅ΠΏΠΈ ΠΏΠ°Π΄Π°Π΅Ρ‚ Ρ€Π°Π·Π½ΠΎΠ΅ напряТСниС. ΠŸΠΎΡΠΊΠΎΠ»ΡŒΠΊΡƒ Π΅ΠΌΠΊΠΎΡΡ‚ΡŒ ΠΎΠ±Ρ€Π°Ρ‚Π½ΠΎ ΠΏΡ€ΠΎΠΏΡ€Ρ†ΠΈΠΎΠ½Π°Π»ΡŒΠ½Π° Π½Π°ΠΏΡ€ΡΠΆΠ΅Π½ΠΈΡŽ (Π‘ = Q/V), Ρ‚ΠΎ Ρ‡Π΅ΠΌ мСньшС Π΅ΠΌΠΊΠΎΡΡ‚ΡŒ кондСнсатора, Ρ‚Π΅ΠΌ большСС напряТСниС Π½Π° Π½Π΅Π³ΠΎ ΡƒΠΏΠ°Π΄Π΅Ρ‚.

ΠŸΡ€ΠΈΠΌΠ΅Π½ΠΈΠΌ Π·Π°ΠΊΠΎΠ½ ΠšΠΈΡ€Ρ…Π³ΠΎΡ„Π° для напряТСния Π² ΠΏΠΎΡΠ»Π΅Π΄ΠΎΠ²Π°Ρ‚Π΅Π»ΡŒΠ½ΠΎΠΉ Ρ†Π΅ΠΏΠΈ ΠΈΠ· Ρ‚Ρ€Π΅Ρ… кондСнсаторов:

ПадСниС напряТСния ΠΏΡ€ΠΈ ΠΏΠΎΡΠ»Π΅Π΄ΠΎΠ²Π°Ρ‚Π΅Π»ΡŒΠ½ΠΎΠΌ соСдинСнии

Π•ΠΌΠΊΠΎΡΡ‚ΡŒ кондСнсатора прямо ΠΏΡ€ΠΎΠΏΠΎΡ€Ρ†ΠΈΠΎΠ½Π°Π»ΡŒΠ½Π° Π΅Π³ΠΎ заряду ΠΈ ΠΎΠ±Ρ€Π°Ρ‚Π½ΠΎ ΠΏΡ€ΠΎΠΏΠΎΡ€Ρ†ΠΈΠΎΠ½Π°Π»ΡŒΠ½Π° Π΅Π³ΠΎ Π½Π°ΠΏΡ€ΡΠΆΠ΅Π½ΠΈΡŽ β€” C = Q/V. Как ΡƒΠΆΠ΅ ΡƒΠΏΠΎΠΌΠΈΠ½Π°Π»ΠΎΡΡŒ Π²Ρ‹ΡˆΠ΅, ΠΏΠΎΡΠ»Π΅Π΄ΠΎΠ²Π°Ρ‚Π΅Π»ΡŒΠ½ΠΎ соСдинСнныС кондСнсаторы ΠΈΠΌΠ΅ΡŽΡ‚ ΠΎΠ΄ΠΈΠ½Π°ΠΊΠΎΠ²Ρ‹ΠΉ элСктричСский заряд β€” QΠΎΠ±Ρ‰ = Q1 = Q2 = Q3.

Π‘Π»Π΅Π΄ΠΎΠ²Π°Ρ‚Π΅Π»ΡŒΠ½ΠΎ:

Π Π°Π·Π΄Π΅Π»ΠΈΠ² всС Π²Ρ‹Ρ€Π°ΠΆΠ΅Π½ΠΈΠ΅ Π½Π° QΠΎΠ±Ρ‰ ΠΌΡ‹ ΠΏΠΎΠ»ΡƒΡ‡ΠΈΠΌ ΡƒΡ€Π°Π²Π½Π΅Π½ΠΈΠ΅ для ΠΎΠ±Ρ‰Π΅ΠΉ Смкости ΠΏΡ€ΠΈ ΠΏΠΎΡΠ»Π΅Π΄ΠΎΠ²Π°Ρ‚Π΅Π»ΡŒΠ½ΠΎΠΌ соСдинСнии:

Из Π΄Π°Π½Π½ΠΎΠ³ΠΎ уравнСния ΠΌΠΎΠΆΠ½ΠΎ Π»Π΅Π³ΠΊΠΎ вывСсти Ρ„ΠΎΡ€ΠΌΡƒΠ»Ρƒ ΠΎΠ±Ρ‰Π΅ΠΉ Смкости для любого частного случая ΠΏΠΎΡΠ»Π΅Π΄ΠΎΠ²Π°Ρ‚Π΅Π»ΡŒΠ½ΠΎΠ³ΠΎ соСдинСния.

НапримСр, общая Π΅ΠΌΠΊΠΎΡΡ‚ΡŒ для Ρ‚Ρ€Π΅Ρ… кондСнсаторов:

ΠžΠ±Ρ‰Π°Ρ Π΅ΠΌΠΊΠΎΡΡ‚ΡŒ для Π΄Π²ΡƒΡ… кондСнсаторов:

БмСшанноС соСдинСниС кондСнсаторов

Если Π² Ρ†Π΅ΠΏΠΈ Π΅ΡΡ‚ΡŒ ΠΈ ΠΏΠΎΡΠ»Π΅Π΄ΠΎΠ²Π°Ρ‚Π΅Π»ΡŒΠ½ΠΎΠ΅ ΠΈ ΠΏΠ°Ρ€Π°Π»Π»Π΅Π»ΡŒΠ½ΠΎΠ΅ соСдинСниС, Ρ‚ΠΎ Ρ‚Π°ΠΊΡƒΡŽ Ρ†Π΅ΠΏΡŒ Π½Π°Π·Ρ‹Π²Π°ΡŽΡ‚ смСшанной ΠΈΠ»ΠΈ ΠΏΠΎΡΠ»Π΅Π΄ΠΎΠ²Π°Ρ‚Π΅Π»ΡŒΠ½ΠΎ-ΠΏΠ°Ρ€Π°Π»Π»Π΅Π»ΡŒΠ½ΠΎΠΉ. Π’Π΅ΠΌ Π½Π΅ ΠΌΠ΅Π½Π΅Π΅, смСшанноС соСдинСниС ΠΌΠΎΠΆΠ΅Ρ‚ ΠΈΠΌΠ΅Ρ‚ΡŒ ΠΊΠ°ΠΊ ΠΏΠΎΡΠ»Π΅Π΄ΠΎΠ²Π°Ρ‚Π΅Π»ΡŒΠ½Ρ‹ΠΉ, Ρ‚Π°ΠΊ ΠΈ ΠΏΠ°Ρ€Π°Π»Π»Π΅Π»ΡŒΠ½Ρ‹ΠΉ Ρ…Π°Ρ€Π°ΠΊΡ‚Π΅Ρ€.

БмСшанноС соСдинСниС кондСнсаторов

ΠžΠ±Ρ‰Π°Ρ Π΅ΠΌΠΊΠΎΡΡ‚ΡŒ смСшанного соСдинСния кондСнсаторов

Π§Ρ‚ΠΎΠ±Ρ‹ ΠΏΠΎΡΡ‡ΠΈΡ‚Π°Ρ‚ΡŒ ΠΎΠ±Ρ‰ΡƒΡŽ Π΅ΠΌΠΊΠΎΡΡ‚ΡŒ смСшанного соСдинСния кондСнсаторов, ΡΠ»Π΅Π΄ΡƒΡŽΡ‚ Ρ‚Π°ΠΊΠΎΠΌΡƒ ΠΆΠ΅ Π°Π»Π³ΠΎΡ€ΠΈΡ‚ΠΌΡƒ, ΠΊΠ°ΠΊ ΠΈ ΠΏΡ€ΠΈ расчСтС ΠΎΠ±Ρ‰Π΅Π³ΠΎ сопротивлСния смСшанного соСдинСния рСзисторов.

  • ЦСпь Ρ€Π°Π·Π±ΠΈΠ²Π°ΡŽΡ‚ Π½Π° участки с Ρ‚ΠΎΠ»ΡŒΠΊΠΎ ΠΏΠ°Ρ€Π΅Π»Π»Π΅Π»ΡŒΠ½Ρ‹ΠΌ ΠΈΠ»ΠΈ Ρ‚ΠΎΠ»ΡŒΠΊΠΎ ΠΏΠΎΡΠ»Π΅Π΄ΠΎΠ²Π°Ρ‚Π΅Π»ΡŒΠ½Ρ‹ΠΌ соСдинСниСм
  • Π’Ρ‹Ρ‡ΠΈΡΠ»ΡΡŽΡ‚ ΠΎΠ±Ρ‰ΡƒΡŽ Π΅ΠΌΠΊΠΎΡΡ‚ΡŒ для ΠΊΠ°ΠΆΠ΄ΠΎΠ³ΠΎ ΠΎΡ‚Π΄Π΅Π»ΡŒΠ½ΠΎΠ³ΠΎ участка.
  • Π’Ρ‹Ρ‡ΠΈΡΠ»ΡΡŽΡ‚ ΠΎΠ±Ρ‰ΡƒΡŽ Π΅ΠΌΠΊΠΎΡΡ‚ΡŒ для всСй Ρ†Π΅ΠΏΠΈ смСшанного соСдинСния.

Π’Π°ΠΊ это Π±ΡƒΠ΄Π΅Ρ‚ Π²Ρ‹Π³Π»ΡΠ΄Π΅Ρ‚ΡŒ для схСмы 2:

ΠŸΡ€Π΅ΠΎΠ±Ρ€Π°Π·ΠΎΠ²Π°Π½ΠΈΠ΅ смСшанного соСдинСния Π² ΠΏΠ°Ρ€Π°Π»Π»Π΅Π»ΡŒΠ½ΠΎΠ΅

Π—Π°Ρ‡Π΅ΠΌ всС это Π½ΡƒΠΆΠ½ΠΎ?

Π’ΠΏΠΎΠ»Π½Π΅ справСдливым ΠΌΠΎΠΆΠ΅Ρ‚ ΠΎΠΊΠ°Π·Π°Ρ‚ΡŒΡΡ вопрос, для Ρ‡Π΅Π³ΠΎ Π½Π°Π΄ΠΎ ΡΠΎΠ΅Π΄ΠΈΠ½ΡΡ‚ΡŒ кондСнсаторы ΠΏΠΎΡΠ»Π΅Π΄ΠΎΠ²Π°Ρ‚Π΅Π»ΡŒΠ½ΠΎ, Ссли общая Π΅ΠΌΠΊΠΎΡΡ‚ΡŒ Π±ΡƒΠ΄Π΅Ρ‚ мСньшС? Π‘ΠΊΠΎΡ€Π΅Π΅ всСго, ΠΏΠ΅Ρ€Π²Ρ‹ΠΌ Ρ‡Ρ‚ΠΎ ΠΏΡ€ΠΈΡ…ΠΎΠ΄ΠΈΡ‚ Π² Π³ΠΎΠ»ΠΎΠ²Ρƒ β€” это Ρ‡Ρ‚ΠΎΠ±Ρ‹ ΠΏΠΎΠ»ΡƒΡ‡ΠΈΡ‚ΡŒ Π½ΠΎΠ²Ρ‹ΠΉ эквивалСнтный кондСнсатор с мСньшСй Π΅ΠΌΠΊΠΎΡΡ‚ΡŒΡŽ. Но Π² производствС микросхСм вряд Π»ΠΈ Π±ΡƒΠ΄ΡƒΡ‚ Π΄Π΅Π»Π°Ρ‚ΡŒ ΠΏΠΎΠ΄ΠΎΠ±Π½ΠΎΠ΅, ΠΏΠΎΡΠΊΠΎΠ»ΡŒΠΊΡƒ, Π²ΠΎ -ΠΏΠ΅Ρ€Π²Ρ‹Ρ…, ΠΎΠ±Ρ‹Ρ‡Π½ΠΎ Π½ΡƒΠΆΠ½ΠΎ ΡΠΊΠΎΠ½ΠΎΠΌΠΈΡ‚ΡŒ мСсто Π½Π° ΠΏΠ΅Ρ‡Π°Ρ‚Π½ΠΎΠΉ ΠΏΠ»Π°Ρ‚Π΅, Π° Π²ΠΎ-Π²Ρ‚ΠΎΡ€Ρ‹Ρ…, Π½Π΅Ρ‚ смысла Ρ‚Ρ€Π°Ρ‚ΠΈΡ‚ΡŒ дСньги Π½Π° Π΄Π²Π° ΠΊΠΎΠΌΠΏΠΎΠ½Π΅Π½Ρ‚Π° ΠΈΠ»ΠΈ большС, Ссли ΠΌΠΎΠΆΠ½ΠΎ ΠΊΡƒΠΏΠΈΡ‚ΡŒ ΠΎΠ΄ΠΈΠ½ с Ρ‚Ρ€Π΅Π±ΡƒΠ΅ΠΌΠΎΠΉ Π΅ΠΌΠΊΠΎΡΡ‚ΡŒΡŽ.

Но Ссли Π² ΠΏΠ°Ρ€Π°Π»Π»Π΅Π»ΡŒΠ½ΠΎΠΌ ΠΈΠ»ΠΈ ΠΏΠΎΡΠ»Π΅Π΄ΠΎΠ²Π°Ρ‚Π΅Π»ΡŒΠ½ΠΎΠΌ соСдинСнии кондСнсаторов Π΅Ρ‰Π΅ Π΅ΡΡ‚ΡŒ Ρ…ΠΎΡ‚ΡŒ какая-Ρ‚ΠΎ Π»ΠΎΠ³ΠΈΠΊΠ°, Ρ‚ΠΎ ΠΊΠΎΠΌΡƒ Π²ΠΎΠΎΠ±Ρ‰Π΅ Π½ΡƒΠΆΠ½ΠΎ смСшанноС?

Π”Π΅Π»ΠΎ Π² Ρ‚ΠΎΠΌ, Ρ‡Ρ‚ΠΎ Π΅ΠΌΠΊΠΎΡΡ‚ΡŒΡŽ, Ρ‚ΠΎ Π΅ΡΡ‚ΡŒ ΡΠΏΠΎΡΠΎΠ±Π½ΠΎΡΡ‚ΡŒΡŽ Π½Π°ΠΊΠ°ΠΏΠ»ΠΈΠ²Π°Ρ‚ΡŒ элСктричСский заряд, ΠΎΠ±Π»Π°Π΄Π°Π΅Ρ‚ любоС Ρ‚Π΅Π»ΠΎ Π² ΠΏΡ€ΠΈΡ€ΠΎΠ΄Π΅, Π΄Π°ΠΆΠ΅ чСловСчСскоС. Если ΠΌΡ‹ Π³ΠΎΠ²ΠΎΡ€ΠΈΠΌ ΠΎ элСктричСской Ρ†Π΅ΠΏΠΈ, Ρ‚ΠΎ всС Π΅Π΅ элСмСнты Π½Π° ΠΏΡ€Π°ΠΊΡ‚ΠΈΠΊΠ΅ ΠΎΠ±Π»Π°Π΄Π°ΡŽΡ‚ Π΅ΠΌΠΊΠΎΡΡ‚ΡŒΡŽ, ΠΈ ΠΈΡ… ΠΌΠΎΠΆΠ½ΠΎ ΠΏΡ€Π΅Π΄ΡΡ‚Π°Π²ΠΈΡ‚ΡŒ ΠΊΠ°ΠΊ кондСнсаторы. Часто Ρ‚Π°ΠΊΡƒΡŽ Π΅ΠΌΠΊΠΎΡΡ‚ΡŒ Π΅Ρ‰Π΅ Π½Π°Π·Ρ‹Π²Π°ΡŽΡ‚ паразитичСской, ΠΏΠΎΡ‚ΠΎΠΌΡƒ ΠΊΠ°ΠΊ ΠΎΠ½Π° создаСт Ρ€Π°Π·Π½ΠΎΠ³ΠΎ Ρ€ΠΎΠ΄Π° ΠΏΠΎΠΌΠ΅Ρ…ΠΈ.

НапримСр, Ρƒ нас Π΅ΡΡ‚ΡŒ какая-Ρ‚ΠΎ элСктронная Ρ†Π΅ΠΏΡŒ с мноТСством Ρ€Π°Π·Π»ΠΈΡ‡Π½Ρ‹Ρ… ΠΊΠΎΠΌΠΏΠΎΠ½Π΅Π½Ρ‚ΠΎΠ², которая ΠΏΡ€ΠΈΠ½ΠΈΠΌΠ°Π΅Ρ‚ сигнал, ΠΎΠ±Ρ€Π°Π±Π°Ρ‚Ρ‹Π²Π°Π΅Ρ‚ Π΅Π³ΠΎ ΠΎΠΏΡ€Π΅Π΄Π΅Π»Π΅Π½Π½Ρ‹ΠΌ ΠΎΠ±Ρ€Π°Π·ΠΎΠΌ ΠΈ Π²Ρ‹Π΄Π°Π΅Ρ‚ Π½Π° Π²Ρ‹Ρ…ΠΎΠ΄ Ρ€Π΅Π·ΡƒΠ»ΡŒΡ‚Π°Ρ‚. Π˜Π·Π²Π΅ΡΡ‚Π½ΠΎ, Ρ‡Ρ‚ΠΎ врСмя Π·Π°Π΄Π΅Ρ€ΠΆΠΊΠΈ сигнала, Π² основном, зависит ΠΎΡ‚ паразитичСской Смкости элСктронных ΠΊΠΎΠΌΠΏΠΎΠ½Π΅Π½Ρ‚ΠΎΠ² схСмы. ΠŸΠΎΡΠΊΠΎΠ»ΡŒΠΊΡƒ Π΄ΠΎΠ»ΠΆΠ½ΠΎ ΠΏΡ€ΠΎΠΉΡ‚ΠΈ врСмя зарядки паразитичСской Смкости, ΠΏΡ€Π΅ΠΆΠ΄Π΅ Ρ‡Π΅ΠΌ ΠΎΠ½Π° Π½Π°Ρ‡Π½Π΅Ρ‚ ΠΏΡ€ΠΎΠΏΡƒΡΠΊΠ°Ρ‚ΡŒ сигнал. Если ΠΌΡ‹ Ρ…ΠΎΡ‚ΠΈΠΌ ΡƒΠ·Π½Π°Ρ‚ΡŒ врСмя Π·Π°Π΄Π΅Ρ€ΠΆΠΊΠΈ, Π½ΡƒΠΆΠ½ΠΎ ΠΏΠΎΡΡ‡ΠΈΡ‚Π°Ρ‚ΡŒ ΠΎΠ±Ρ‰ΡƒΡŽ Π΅ΠΌΠΊΠΎΡΡ‚ΡŒ всСх ΠΊΠΎΠΌΠΏΠΎΠ½Π΅Π½Ρ‚ΠΎΠ², ΠΊΠΎΠ½Π²Π΅Ρ€Ρ‚ΠΈΡ€ΠΎΠ²Π°Π² ΠΈΡ… Π² Ρ†Π΅ΠΏΡŒ ΠΈΠ· кондСнсаторов.

НапряТСниС ΠΈ сопротивлСниС ΠΏΡ€ΠΈ ΠΏΠΎΡΠ»Π΅Π΄ΠΎΠ²Π°Ρ‚Π΅Π»ΡŒΠ½ΠΎΠΌ соСдинСнии

Π‘ΠΎΠΏΡ€ΠΎΡ‚ΠΈΠ²Π»Π΅Π½ΠΈΠ΅ ΠΏΡ€ΠΎΠ²ΠΎΠ΄Π½ΠΈΠΊΠΎΠ². ΠŸΠ°Ρ€Π°Π»Π»Π΅Π»ΡŒΠ½ΠΎΠ΅ ΠΈ ΠΏΠΎΡΠ»Π΅Π΄ΠΎΠ²Π°Ρ‚Π΅Π»ΡŒΠ½ΠΎΠ΅ соСдинСниС ΠΏΡ€ΠΎΠ²ΠΎΠ΄Π½ΠΈΠΊΠΎΠ².

ЭлСктри́чСскоС сопротивлС́ниС β€” физичСская Π²Π΅Π»ΠΈΡ‡ΠΈΠ½Π°, Ρ…Π°Ρ€Π°ΠΊΡ‚Π΅Ρ€ΠΈΠ·ΡƒΡŽΡ‰Π°Ρ свойства ΠΏΡ€ΠΎΠ²ΠΎΠ΄Π½ΠΈΠΊΠ° ΠΏΡ€Π΅ΠΏΡΡ‚ΡΡ‚Π²ΠΎΠ²Π°Ρ‚ΡŒ ΠΏΡ€ΠΎΡ…ΠΎΠΆΠ΄Π΅Π½ΠΈΡŽ элСктричСского Ρ‚ΠΎΠΊΠ° ΠΈ равная ΠΎΡ‚Π½ΠΎΡˆΠ΅Π½ΠΈΡŽΠ½Π°ΠΏΡ€ΡΠΆΠ΅Π½ΠΈΡ Π½Π° ΠΊΠΎΠ½Ρ†Π°Ρ… ΠΏΡ€ΠΎΠ²ΠΎΠ΄Π½ΠΈΠΊΠ° ΠΊ силС Ρ‚ΠΎΠΊΠ°, ΠΏΡ€ΠΎΡ‚Π΅ΠΊΠ°ΡŽΡ‰Π΅Π³ΠΎ ΠΏΠΎ Π½Π΅ΠΌΡƒ [1] . Π‘ΠΎΠΏΡ€ΠΎΡ‚ΠΈΠ²Π»Π΅Π½ΠΈΠ΅ для Ρ†Π΅ΠΏΠ΅ΠΉ ΠΏΠ΅Ρ€Π΅ΠΌΠ΅Π½Π½ΠΎΠ³ΠΎ Ρ‚ΠΎΠΊΠ° ΠΈ для ΠΏΠ΅Ρ€Π΅ΠΌΠ΅Π½Π½Ρ‹Ρ… элСктромагнитных ΠΏΠΎΠ»Π΅ΠΉ описываСтся понятиями импСданса ΠΈ Π²ΠΎΠ»Π½ΠΎΠ²ΠΎΠ³ΠΎ сопротивлСния. Π‘ΠΎΠΏΡ€ΠΎΡ‚ΠΈΠ²Π»Π΅Π½ΠΈΠ΅ΠΌ (рСзистором) Ρ‚Π°ΠΊΠΆΠ΅ Π½Π°Π·Ρ‹Π²Π°ΡŽΡ‚ Ρ€Π°Π΄ΠΈΠΎΠ΄Π΅Ρ‚Π°Π»ΡŒ, ΠΏΡ€Π΅Π΄Π½Π°Π·Π½Π°Ρ‡Π΅Π½Π½ΡƒΡŽ для ввСдСния Π² элСктричСскиС Ρ†Π΅ΠΏΠΈ Π°ΠΊΡ‚ΠΈΠ²Π½ΠΎΠ³ΠΎ сопротивлСния.

Π‘ΠΎΠΏΡ€ΠΎΡ‚ΠΈΠ²Π»Π΅Π½ΠΈΠ΅ (часто обозначаСтся Π±ΡƒΠΊΠ²ΠΎΠΉ R ΠΈΠ»ΠΈ r) считаСтся, Π² ΠΎΠΏΡ€Π΅Π΄Π΅Π»Ρ‘Π½Π½Ρ‹Ρ… ΠΏΡ€Π΅Π΄Π΅Π»Π°Ρ…, постоянной Π²Π΅Π»ΠΈΡ‡ΠΈΠ½ΠΎΠΉ для Π΄Π°Π½Π½ΠΎΠ³ΠΎ ΠΏΡ€ΠΎΠ²ΠΎΠ΄Π½ΠΈΠΊΠ°; Π΅Ρ‘ ΠΌΠΎΠΆΠ½ΠΎ Ρ€Π°ΡΡΡ‡ΠΈΡ‚Π°Ρ‚ΡŒ ΠΊΠ°ΠΊ

U β€” Ρ€Π°Π·Π½ΠΎΡΡ‚ΡŒ элСктричСских ΠΏΠΎΡ‚Π΅Π½Ρ†ΠΈΠ°Π»ΠΎΠ² (напряТСниС) Π½Π° ΠΊΠΎΠ½Ρ†Π°Ρ… ΠΏΡ€ΠΎΠ²ΠΎΠ΄Π½ΠΈΠΊΠ°;

I β€” сила Ρ‚ΠΎΠΊΠ°, ΠΏΡ€ΠΎΡ‚Π΅ΠΊΠ°ΡŽΡ‰Π΅Π³ΠΎ ΠΌΠ΅ΠΆΠ΄Ρƒ ΠΊΠΎΠ½Ρ†Π°ΠΌΠΈ ΠΏΡ€ΠΎΠ²ΠΎΠ΄Π½ΠΈΠΊΠ° ΠΏΠΎΠ΄ дСйствиСм разности ΠΏΠΎΡ‚Π΅Π½Ρ†ΠΈΠ°Π»ΠΎΠ².

ΠŸΡ€ΠΈ ΠΏΠΎΡΠ»Π΅Π΄ΠΎΠ²Π°Ρ‚Π΅Π»ΡŒΠ½ΠΎΠΌ соСдинСнии ΠΏΡ€ΠΎΠ²ΠΎΠ΄Π½ΠΈΠΊΠΎΠ² (рис. 1.9.1) сила Ρ‚ΠΎΠΊΠ° Π²ΠΎ всСх ΠΏΡ€ΠΎΠ²ΠΎΠ΄Π½ΠΈΠΊΠ°Ρ… ΠΎΠ΄ΠΈΠ½Π°ΠΊΠΎΠ²Π°:

ΠŸΠΎΡΠ»Π΅Π΄ΠΎΠ²Π°Ρ‚Π΅Π»ΡŒΠ½ΠΎΠ΅ соСдинСниС ΠΏΡ€ΠΎΠ²ΠΎΠ΄Π½ΠΈΠΊΠΎΠ²

По Π·Π°ΠΊΠΎΠ½Ρƒ Ома, напряТСния U1 ΠΈ U2 Π½Π° ΠΏΡ€ΠΎΠ²ΠΎΠ΄Π½ΠΈΠΊΠ°Ρ… Ρ€Π°Π²Π½Ρ‹

ΠžΠ±Ρ‰Π΅Π΅ напряТСниС U Π½Π° ΠΎΠ±ΠΎΠΈΡ… ΠΏΡ€ΠΎΠ²ΠΎΠ΄Π½ΠΈΠΊΠ°Ρ… Ρ€Π°Π²Π½ΠΎ суммС напряТСний U1 ΠΈ U2:

Π³Π΄Π΅ R – элСктричСскоС сопротивлСниС всСй Ρ†Π΅ΠΏΠΈ. ΠžΡ‚ΡΡŽΠ΄Π° слСдуСт:

ΠŸΡ€ΠΈ ΠΏΠΎΡΠ»Π΅Π΄ΠΎΠ²Π°Ρ‚Π΅Π»ΡŒΠ½ΠΎΠΌ соСдинСнии ΠΏΠΎΠ»Π½ΠΎΠ΅ сопротивлСниС Ρ†Π΅ΠΏΠΈ Ρ€Π°Π²Π½ΠΎ суммС сопротивлСний ΠΎΡ‚Π΄Π΅Π»ΡŒΠ½Ρ‹Ρ… ΠΏΡ€ΠΎΠ²ΠΎΠ΄Π½ΠΈΠΊΠΎΠ².

Π­Ρ‚ΠΎΡ‚ Ρ€Π΅Π·ΡƒΠ»ΡŒΡ‚Π°Ρ‚ справСдлив для любого числа ΠΏΠΎΡΠ»Π΅Π΄ΠΎΠ²Π°Ρ‚Π΅Π»ΡŒΠ½ΠΎ соСдинСнных ΠΏΡ€ΠΎΠ²ΠΎΠ΄Π½ΠΈΠΊΠΎΠ².

ΠŸΡ€ΠΈ ΠΏΠ°Ρ€Π°Π»Π»Π΅Π»ΡŒΠ½ΠΎΠΌ соСдинСнии (рис. 1.9.2) напряТСния U1 ΠΈ U2 Π½Π° ΠΎΠ±ΠΎΠΈΡ… ΠΏΡ€ΠΎΠ²ΠΎΠ΄Π½ΠΈΠΊΠ°Ρ… ΠΎΠ΄ΠΈΠ½Π°ΠΊΠΎΠ²Ρ‹:

Π‘ΡƒΠΌΠΌΠ° Ρ‚ΠΎΠΊΠΎΠ² I1 + I2, ΠΏΡ€ΠΎΡ‚Π΅ΠΊΠ°ΡŽΡ‰ΠΈΡ… ΠΏΠΎ ΠΎΠ±ΠΎΠΈΠΌ ΠΏΡ€ΠΎΠ²ΠΎΠ΄Π½ΠΈΠΊΠ°ΠΌ, Ρ€Π°Π²Π½Π° Ρ‚ΠΎΠΊΡƒ Π² Π½Π΅Ρ€Π°Π·Π²Π΅Ρ‚Π²Π»Π΅Π½Π½ΠΎΠΉ Ρ†Π΅ΠΏΠΈ:

Π­Ρ‚ΠΎΡ‚ Ρ€Π΅Π·ΡƒΠ»ΡŒΡ‚Π°Ρ‚ слСдуСт ΠΈΠ· Ρ‚ΠΎΠ³ΠΎ, Ρ‡Ρ‚ΠΎ Π² Ρ‚ΠΎΡ‡ΠΊΠ°Ρ… развСтвлСния Ρ‚ΠΎΠΊΠΎΠ² (ΡƒΠ·Π»Ρ‹ A ΠΈ B) Π² Ρ†Π΅ΠΏΠΈ постоянного Ρ‚ΠΎΠΊΠ° Π½Π΅ ΠΌΠΎΠ³ΡƒΡ‚ Π½Π°ΠΊΠ°ΠΏΠ»ΠΈΠ²Π°Ρ‚ΡŒΡΡ заряды. НапримСр, ΠΊ ΡƒΠ·Π»Ρƒ A Π·Π° врСмя Ξ”t ΠΏΠΎΠ΄Ρ‚Π΅ΠΊΠ°Π΅Ρ‚ заряд IΞ”t, Π° ΡƒΡ‚Π΅ΠΊΠ°Π΅Ρ‚ ΠΎΡ‚ ΡƒΠ·Π»Π° Π·Π° Ρ‚ΠΎ ΠΆΠ΅ врСмя заряд I1Ξ”t + I2Ξ”t. Π‘Π»Π΅Π΄ΠΎΠ²Π°Ρ‚Π΅Π»ΡŒΠ½ΠΎ,I = I1 + I2.

ΠŸΠ°Ρ€Π°Π»Π»Π΅Π»ΡŒΠ½ΠΎΠ΅ соСдинСниС ΠΏΡ€ΠΎΠ²ΠΎΠ΄Π½ΠΈΠΊΠΎΠ²

Записывая Π½Π° основании Π·Π°ΠΊΠΎΠ½Π° Ома

Π³Π΄Π΅ R – элСктричСскоС сопротивлСниС всСй Ρ†Π΅ΠΏΠΈ, ΠΏΠΎΠ»ΡƒΡ‡ΠΈΠΌ

ΠŸΡ€ΠΈ ΠΏΠ°Ρ€Π°Π»Π»Π΅Π»ΡŒΠ½ΠΎΠΌ соСдинСнии ΠΏΡ€ΠΎΠ²ΠΎΠ΄Π½ΠΈΠΊΠΎΠ² Π²Π΅Π»ΠΈΡ‡ΠΈΠ½Π°, обратная ΠΎΠ±Ρ‰Π΅ΠΌΡƒ ΡΠΎΠΏΡ€ΠΎΡ‚ΠΈΠ²Π»Π΅Π½ΠΈΡŽ Ρ†Π΅ΠΏΠΈ, Ρ€Π°Π²Π½Π° суммС Π²Π΅Π»ΠΈΡ‡ΠΈΠ½, ΠΎΠ±Ρ€Π°Ρ‚Π½Ρ‹Ρ… сопротивлСниям ΠΏΠ°Ρ€Π°Π»Π»Π΅Π»ΡŒΠ½ΠΎ Π²ΠΊΠ»ΡŽΡ‡Π΅Π½Π½Ρ‹Ρ… ΠΏΡ€ΠΎΠ²ΠΎΠ΄Π½ΠΈΠΊΠΎΠ².

Π­Ρ‚ΠΎΡ‚ Ρ€Π΅Π·ΡƒΠ»ΡŒΡ‚Π°Ρ‚ справСдлив для любого числа ΠΏΠ°Ρ€Π°Π»Π»Π΅Π»ΡŒΠ½ΠΎ Π²ΠΊΠ»ΡŽΡ‡Π΅Π½Π½Ρ‹Ρ… ΠΏΡ€ΠΎΠ²ΠΎΠ΄Π½ΠΈΠΊΠΎΠ².

Π€ΠΎΡ€ΠΌΡƒΠ»Ρ‹ для ΠΏΠΎΡΠ»Π΅Π΄ΠΎΠ²Π°Ρ‚Π΅Π»ΡŒΠ½ΠΎΠ³ΠΎ ΠΈ ΠΏΠ°Ρ€Π°Π»Π»Π΅Π»ΡŒΠ½ΠΎΠ³ΠΎ соСдинСния ΠΏΡ€ΠΎΠ²ΠΎΠ΄Π½ΠΈΠΊΠΎΠ² ΠΏΠΎΠ·Π²ΠΎΠ»ΡΡŽΡ‚ Π²ΠΎ ΠΌΠ½ΠΎΠ³ΠΈΡ… случаях Ρ€Π°ΡΡΡ‡ΠΈΡ‚Ρ‹Π²Π°Ρ‚ΡŒ сопротивлСниС слоТной Ρ†Π΅ΠΏΠΈ, состоящСй ΠΈΠ· ΠΌΠ½ΠΎΠ³ΠΈΡ… рСзисторов. На рис. 1.9.3 ΠΏΡ€ΠΈΠ²Π΅Π΄Π΅Π½ ΠΏΡ€ΠΈΠΌΠ΅Ρ€ Ρ‚Π°ΠΊΠΎΠΉ слоТной Ρ†Π΅ΠΏΠΈ ΠΈ ΡƒΠΊΠ°Π·Π°Π½Π° ΠΏΠΎΡΠ»Π΅Π΄ΠΎΠ²Π°Ρ‚Π΅Π»ΡŒΠ½ΠΎΡΡ‚ΡŒ вычислСний.

РасчСт сопротивлСния слоТной Ρ†Π΅ΠΏΠΈ. БопротивлСния всСх ΠΏΡ€ΠΎΠ²ΠΎΠ΄Π½ΠΈΠΊΠΎΠ² ΡƒΠΊΠ°Π·Π°Π½Ρ‹ Π²ΠΎΠΌΠ°Ρ… (Ом)

Π‘Π»Π΅Π΄ΡƒΠ΅Ρ‚ ΠΎΡ‚ΠΌΠ΅Ρ‚ΠΈΡ‚ΡŒ, Ρ‡Ρ‚ΠΎ Π΄Π°Π»Π΅ΠΊΠΎ Π½Π΅ всС слоТныС Ρ†Π΅ΠΏΠΈ, состоящиС ΠΈΠ· ΠΏΡ€ΠΎΠ²ΠΎΠ΄Π½ΠΈΠΊΠΎΠ² с Ρ€Π°Π·Π»ΠΈΡ‡Π½Ρ‹ΠΌΠΈ сопротивлСниями, ΠΌΠΎΠ³ΡƒΡ‚ Π±Ρ‹Ρ‚ΡŒ рассчитаны с ΠΏΠΎΠΌΠΎΡ‰ΡŒΡŽ Ρ„ΠΎΡ€ΠΌΡƒΠ» для ΠΏΠΎΡΠ»Π΅Π΄ΠΎΠ²Π°Ρ‚Π΅Π»ΡŒΠ½ΠΎΠ³ΠΎ ΠΈ ΠΏΠ°Ρ€Π°Π»Π»Π΅Π»ΡŒΠ½ΠΎΠ³ΠΎ соСдинСния. На рис. 1.9.4 ΠΏΡ€ΠΈΠ²Π΅Π΄Π΅Π½ ΠΏΡ€ΠΈΠΌΠ΅Ρ€ элСктричСской Ρ†Π΅ΠΏΠΈ, ΠΊΠΎΡ‚ΠΎΡ€ΡƒΡŽ нСльзя Ρ€Π°ΡΡΡ‡ΠΈΡ‚Π°Ρ‚ΡŒ ΡƒΠΊΠ°Π·Π°Π½Π½Ρ‹ΠΌ Π²Ρ‹ΡˆΠ΅ ΠΌΠ΅Ρ‚ΠΎΠ΄ΠΎΠΌ.

ΠŸΡ€ΠΈΠΌΠ΅Ρ€ элСктричСской Ρ†Π΅ΠΏΠΈ, которая Π½Π΅ сводится ΠΊ ΠΊΠΎΠΌΠ±ΠΈΠ½Π°Ρ†ΠΈΠΈ ΠΏΠΎΡΠ»Π΅Π΄ΠΎΠ²Π°Ρ‚Π΅Π»ΡŒΠ½ΠΎ ΠΈ ΠΏΠ°Ρ€Π°Π»Π»Π΅Π»ΡŒΠ½ΠΎ соСдинСнных ΠΏΡ€ΠΎΠ²ΠΎΠ΄Π½ΠΈΠΊΠΎΠ²

Π’ΠΎ всСх элСктричСских схСмах ΠΈΡΠΏΠΎΠ»ΡŒΠ·ΡƒΡŽΡ‚ΡΡ рСзисторы, ΠΏΡ€Π΅Π΄ΡΡ‚Π°Π²Π»ΡΡŽΡ‰ΠΈΠ΅ собой элСмСнты, с Ρ‚ΠΎΡ‡Π½ΠΎ установлСнным Π·Π½Π°Ρ‡Π΅Π½ΠΈΠ΅ΠΌ сопротивлСния. Благодаря спСцифичСским качСствам этих устройств, становится Π²ΠΎΠ·ΠΌΠΎΠΆΠ½ΠΎΠΉ Ρ€Π΅Π³ΡƒΠ»ΠΈΡ€ΠΎΠ²ΠΊΠ° напряТСния ΠΈ силы Ρ‚ΠΎΠΊΠ° Π½Π° Π»ΡŽΠ±Ρ‹Ρ… участках схСмы. Π”Π°Π½Π½Ρ‹Π΅ свойства Π»Π΅ΠΆΠ°Ρ‚ Π² основС Ρ€Π°Π±ΠΎΡ‚Ρ‹ практичСски всСх элСктронных ΠΏΡ€ΠΈΠ±ΠΎΡ€ΠΎΠ² ΠΈ оборудования. Π’Π°ΠΊ, напряТСниС ΠΏΡ€ΠΈ ΠΏΠ°Ρ€Π°Π»Π»Π΅Π»ΡŒΠ½ΠΎΠΌ ΠΈ ΠΏΠΎΡΠ»Π΅Π΄ΠΎΠ²Π°Ρ‚Π΅Π»ΡŒΠ½ΠΎΠΌ соСдинСнии рСзисторов Π±ΡƒΠ΄Π΅Ρ‚ ΠΎΡ‚Π»ΠΈΡ‡Π°Ρ‚ΡŒΡΡ. ΠŸΠΎΡΡ‚ΠΎΠΌΡƒ ΠΊΠ°ΠΆΠ΄Ρ‹ΠΉ Π²ΠΈΠ΄ соСдинСния ΠΌΠΎΠΆΠ΅Ρ‚ ΠΏΡ€ΠΈΠΌΠ΅Π½ΡΡ‚ΡŒΡΡ Ρ‚ΠΎΠ»ΡŒΠΊΠΎ Π² ΠΎΠΏΡ€Π΅Π΄Π΅Π»Π΅Π½Π½Ρ‹Ρ… условиях, Ρ‡Ρ‚ΠΎΠ±Ρ‹ Ρ‚Π° ΠΈΠ»ΠΈ иная элСктричСская схСма ΠΌΠΎΠ³Π»Π° Π² ΠΏΠΎΠ»Π½ΠΎΠΌ объСмС Π²Ρ‹ΠΏΠΎΠ»Π½ΡΡ‚ΡŒ свои Ρ„ΡƒΠ½ΠΊΡ†ΠΈΠΈ.

НапряТСниС ΠΏΡ€ΠΈ ΠΏΠΎΡΠ»Π΅Π΄ΠΎΠ²Π°Ρ‚Π΅Π»ΡŒΠ½ΠΎΠΌ соСдинСнии

ΠŸΡ€ΠΈ ΠΏΠΎΡΠ»Π΅Π΄ΠΎΠ²Π°Ρ‚Π΅Π»ΡŒΠ½ΠΎΠΌ соСдинСнии Π΄Π²Π° рСзистора ΠΈ Π±ΠΎΠ»Π΅Π΅ ΡΠΎΠ΅Π΄ΠΈΠ½ΡΡŽΡ‚ΡΡ Π² ΠΎΠ±Ρ‰ΡƒΡŽ Ρ†Π΅ΠΏΡŒ Ρ‚Π°ΠΊΠΈΠΌ ΠΎΠ±Ρ€Π°Π·ΠΎΠΌ, Ρ‡Ρ‚ΠΎ ΠΊΠ°ΠΆΠ΄Ρ‹ΠΉ ΠΈΠ· Π½ΠΈΡ… ΠΈΠΌΠ΅Π΅Ρ‚ ΠΊΠΎΠ½Ρ‚Π°ΠΊΡ‚ с Π΄Ρ€ΡƒΠ³ΠΈΠΌ устройством Ρ‚ΠΎΠ»ΡŒΠΊΠΎ Π² ΠΎΠ΄Π½ΠΎΠΉ Ρ‚ΠΎΡ‡ΠΊΠ΅. Π˜Π½Π°Ρ‡Π΅ говоря, ΠΊΠΎΠ½Π΅Ρ† ΠΏΠ΅Ρ€Π²ΠΎΠ³ΠΎ рСзистора соСдиняСтся с Π½Π°Ρ‡Π°Π»ΠΎΠΌ Π²Ρ‚ΠΎΡ€ΠΎΠ³ΠΎ, Π° ΠΊΠΎΠ½Π΅Ρ† Π²Ρ‚ΠΎΡ€ΠΎΠ³ΠΎ – с Π½Π°Ρ‡Π°Π»ΠΎΠΌ Ρ‚Ρ€Π΅Ρ‚ΡŒΠ΅Π³ΠΎ ΠΈ Ρ‚.Π΄.

ΠžΡΠΎΠ±Π΅Π½Π½ΠΎΡΡ‚ΡŒΡŽ Π΄Π°Π½Π½ΠΎΠΉ схСмы являСтся ΠΏΡ€ΠΎΡ…ΠΎΠΆΠ΄Π΅Π½ΠΈΠ΅ Ρ‡Π΅Ρ€Π΅Π· всС ΠΏΠΎΠ΄ΠΊΠ»ΡŽΡ‡Π΅Π½Π½Ρ‹Π΅ рСзисторы ΠΎΠ΄Π½ΠΎΠ³ΠΎ ΠΈ Ρ‚ΠΎΠ³ΠΎ ΠΆΠ΅ значСния элСктричСского Ρ‚ΠΎΠΊΠ°. Π‘ возрастаниСм количСства элСмСнтов Π½Π° рассматриваСмом участкС Ρ†Π΅ΠΏΠΈ, Ρ‚Π΅Ρ‡Π΅Π½ΠΈΠ΅ элСктричСского Ρ‚ΠΎΠΊΠ° становится всС Π±ΠΎΠ»Π΅Π΅ Π·Π°Ρ‚Ρ€ΡƒΠ΄Π½Π΅Π½Π½Ρ‹ΠΌ. Π­Ρ‚ΠΎ происходит ΠΈΠ·-Π·Π° увСличСния ΠΎΠ±Ρ‰Π΅Π³ΠΎ сопротивлСния рСзисторов ΠΏΡ€ΠΈ ΠΈΡ… ΠΏΠΎΡΠ»Π΅Π΄ΠΎΠ²Π°Ρ‚Π΅Π»ΡŒΠ½ΠΎΠΌ соСдинСнии. Π”Π°Π½Π½ΠΎΠ΅ свойство отраТаСтся Ρ„ΠΎΡ€ΠΌΡƒΠ»ΠΎΠΉ: RΠΎΠ±Ρ‰ = R1 + R2.

РаспрСдСлСниС напряТСния, Π² соотвСтствии с Π·Π°ΠΊΠΎΠ½ΠΎΠΌ Ома, осущСствляСтся Π½Π° ΠΊΠ°ΠΆΠ΄Ρ‹ΠΉ рСзистор ΠΏΠΎ Ρ„ΠΎΡ€ΠΌΡƒΠ»Π΅: VRn = IRn x Rn. Π’Π°ΠΊΠΈΠΌ ΠΎΠ±Ρ€Π°Π·ΠΎΠΌ, ΠΏΡ€ΠΈ ΡƒΠ²Π΅Π»ΠΈΡ‡Π΅Π½ΠΈΠΈ сопротивлСния рСзистора, возрастаСт ΠΈ ΠΏΠ°Π΄Π°ΡŽΡ‰Π΅Π΅ Π½Π° Π½Π΅Π³ΠΎ напряТСниС.

НапряТСниС ΠΏΡ€ΠΈ ΠΏΠ°Ρ€Π°Π»Π»Π΅Π»ΡŒΠ½ΠΎΠΌ соСдинСнии

ΠŸΡ€ΠΈ ΠΏΠ°Ρ€Π°Π»Π»Π΅Π»ΡŒΠ½ΠΎΠΌ соСдинСнии, Π²ΠΊΠ»ΡŽΡ‡Π΅Π½ΠΈΠ΅ рСзисторов Π² ΡΠ»Π΅ΠΊΡ‚Ρ€ΠΈΡ‡Π΅ΡΠΊΡƒΡŽ Ρ†Π΅ΠΏΡŒ выполняСтся Ρ‚Π°ΠΊΠΈΠΌ ΠΎΠ±Ρ€Π°Π·ΠΎΠΌ, Ρ‡Ρ‚ΠΎ всС элСмСнты сопротивлСний ΠΏΠΎΠ΄ΠΊΠ»ΡŽΡ‡Π°ΡŽΡ‚ΡΡ Π΄Ρ€ΡƒΠ³ ΠΊ Π΄Ρ€ΡƒΠ³Ρƒ сразу ΠΎΠ±ΠΎΠΈΠΌΠΈ ΠΊΠΎΠ½Ρ‚Π°ΠΊΡ‚Π°ΠΌΠΈ. Одна Ρ‚ΠΎΡ‡ΠΊΠ°, ΠΏΡ€Π΅Π΄ΡΡ‚Π°Π²Π»ΡΡŽΡ‰Π°Ρ собой элСктричСский ΡƒΠ·Π΅Π», ΠΌΠΎΠΆΠ΅Ρ‚ ΡΠΎΠ΅Π΄ΠΈΠ½ΡΡ‚ΡŒ ΠΎΠ΄Π½ΠΎΠ²Ρ€Π΅ΠΌΠ΅Π½Π½ΠΎ нСсколько рСзисторов.

Π’Π°ΠΊΠΎΠ΅ соСдинСниС ΠΏΡ€Π΅Π΄ΠΏΠΎΠ»Π°Π³Π°Π΅Ρ‚ Ρ‚Π΅Ρ‡Π΅Π½ΠΈΠ΅ ΠΎΡ‚Π΄Π΅Π»ΡŒΠ½ΠΎΠ³ΠΎ Ρ‚ΠΎΠΊΠ° Π² ΠΊΠ°ΠΆΠ΄ΠΎΠΌ рСзисторС. Π‘ΠΈΠ»Π° этого Ρ‚ΠΎΠΊΠ° находится Π² ΠΎΠ±Ρ€Π°Ρ‚Π½ΠΎ ΠΏΡ€ΠΎΠΏΠΎΡ€Ρ†ΠΈΠΎΠ½Π°Π»ΡŒΠ½ΠΎΠΉ зависимости с сопротивлСниСм рСзистора. Π’ Ρ€Π΅Π·ΡƒΠ»ΡŒΡ‚Π°Ρ‚Π΅, происходит ΡƒΠ²Π΅Π»ΠΈΡ‡Π΅Π½ΠΈΠ΅ ΠΎΠ±Ρ‰Π΅ΠΉ проводимости Π΄Π°Π½Π½ΠΎΠ³ΠΎ участка Ρ†Π΅ΠΏΠΈ, ΠΏΡ€ΠΈ ΠΎΠ±Ρ‰Π΅ΠΌ ΡƒΠΌΠ΅Π½ΡŒΡˆΠ΅Π½ΠΈΠΈ сопротивлСния. Π’ случаС ΠΏΠ°Ρ€Π°Π»Π»Π΅Π»ΡŒΠ½ΠΎΠ³ΠΎ соСдинСния рСзисторов с Ρ€Π°Π·Π»ΠΈΡ‡Π½Ρ‹ΠΌ сопротивлСниСм, Π·Π½Π°Ρ‡Π΅Π½ΠΈΠ΅ ΠΎΠ±Ρ‰Π΅Π³ΠΎ сопротивлСния Π½Π° этом участкС всСгда Π±ΡƒΠ΄Π΅Ρ‚ Π½ΠΈΠΆΠ΅ самого малСнького сопротивлСния ΠΎΡ‚Π΄Π΅Π»ΡŒΠ½ΠΎ взятого рСзистора.

На прСдставлСнной схСмС, напряТСниС ΠΌΠ΅ΠΆΠ΄Ρƒ Ρ‚ΠΎΡ‡ΠΊΠ°ΠΌΠΈ А ΠΈ Π’ прСдставляСт собой Π½Π΅ Ρ‚ΠΎΠ»ΡŒΠΊΠΎ ΠΎΠ±Ρ‰Π΅Π΅ напряТСниС для всСго участка, Π½ΠΎ ΠΈ напряТСниС, ΠΏΠΎΡΡ‚ΡƒΠΏΠ°ΡŽΡ‰Π΅Π΅ ΠΊ ΠΊΠ°ΠΆΠ΄ΠΎΠΌΡƒ ΠΎΡ‚Π΄Π΅Π»ΡŒΠ½ΠΎ взятому рСзистору. Π’Π°ΠΊΠΈΠΌ ΠΎΠ±Ρ€Π°Π·ΠΎΠΌ, Π² случаС ΠΏΠ°Ρ€Π°Π»Π»Π΅Π»ΡŒΠ½ΠΎΠ³ΠΎ соСдинСния, напряТСниС, ΠΏΠΎΠ΄Π°Π²Π°Π΅ΠΌΠΎΠ΅ ΠΊΠΎ всСм рСзисторам, Π±ΡƒΠ΄Π΅Ρ‚ ΠΎΠ΄ΠΈΠ½Π°ΠΊΠΎΠ²Ρ‹ΠΌ.

Π’ Ρ€Π΅Π·ΡƒΠ»ΡŒΡ‚Π°Ρ‚Π΅, напряТСниС ΠΏΡ€ΠΈ ΠΏΠ°Ρ€Π°Π»Π»Π΅Π»ΡŒΠ½ΠΎΠΌ ΠΈ ΠΏΠΎΡΠ»Π΅Π΄ΠΎΠ²Π°Ρ‚Π΅Π»ΡŒΠ½ΠΎΠΌ соСдинСнии Π±ΡƒΠ΄Π΅Ρ‚ ΠΎΡ‚Π»ΠΈΡ‡Π°Ρ‚ΡŒΡΡ Π² ΠΊΠ°ΠΆΠ΄ΠΎΠΌ случаС. Благодаря этому свойству, имССтся Ρ€Π΅Π°Π»ΡŒΠ½Π°Ρ Π²ΠΎΠ·ΠΌΠΎΠΆΠ½ΠΎΡΡ‚ΡŒ ΠΎΡ‚Ρ€Π΅Π³ΡƒΠ»ΠΈΡ€ΠΎΠ²Π°Ρ‚ΡŒ Π΄Π°Π½Π½ΡƒΡŽ Π²Π΅Π»ΠΈΡ‡ΠΈΠ½Ρƒ Π½Π° любом участкС Ρ†Π΅ΠΏΠΈ.

ΠŸΠΎΡΠ»Π΅Π΄ΠΎΠ²Π°Ρ‚Π΅Π»ΡŒΠ½ΠΎΠ΅ соСдинСниС рСзисторов

ΠŸΠΎΡΠ»Π΅Π΄ΠΎΠ²Π°Ρ‚Π΅Π»ΡŒΠ½ΠΎΠ΅ соСдинСниС – это соСдинСниС Π΄Π²ΡƒΡ… ΠΈΠ»ΠΈ Π±ΠΎΠ»Π΅Π΅ рСзисторов Π² Ρ„ΠΎΡ€ΠΌΠ΅ Ρ†Π΅ΠΏΠΈ, Π² ΠΊΠΎΡ‚ΠΎΡ€ΠΎΠΉ ΠΊΠ°ΠΆΠ΄Ρ‹ΠΉ ΠΎΡ‚Π΄Π΅Π»ΡŒΠ½Ρ‹ΠΉ рСзистор соСдиняСтся с Π΄Ρ€ΡƒΠ³ΠΈΠΌ ΠΎΡ‚Π΄Π΅Π»ΡŒΠ½Ρ‹ΠΌ рСзистором Ρ‚ΠΎΠ»ΡŒΠΊΠΎ Π² ΠΎΠ΄Π½ΠΎΠΉ Ρ‚ΠΎΡ‡ΠΊΠ΅.

ΠžΠ±Ρ‰Π΅Π΅ сопротивлСниС R

ΠΎΠ±Ρ‰

ΠŸΡ€ΠΈ Ρ‚Π°ΠΊΠΎΠΌ соСдинСнии, Ρ‡Π΅Ρ€Π΅Π· всС рСзисторы ΠΏΡ€ΠΎΡ…ΠΎΠ΄ΠΈΡ‚ ΠΎΠ΄ΠΈΠ½ ΠΈ Ρ‚ΠΎΡ‚ ΠΆΠ΅ элСктричСский Ρ‚ΠΎΠΊ. Π§Π΅ΠΌ большС элСмСнтов Π½Π° Π΄Π°Π½Π½ΠΎΠΌ участкС элСктричСской Ρ†Π΅ΠΏΠΈ, Ρ‚Π΅ΠΌ Β«Ρ‚Ρ€ΡƒΠ΄Π½Π΅Π΅Β» Ρ‚ΠΎΠΊΡƒ ΠΏΡ€ΠΎΡ‚Π΅ΠΊΠ°Ρ‚ΡŒ Ρ‡Π΅Ρ€Π΅Π· Π½Π΅Π³ΠΎ. Π‘Π»Π΅Π΄ΠΎΠ²Π°Ρ‚Π΅Π»ΡŒΠ½ΠΎ, ΠΏΡ€ΠΈ ΠΏΠΎΡΠ»Π΅Π΄ΠΎΠ²Π°Ρ‚Π΅Π»ΡŒΠ½ΠΎΠΌ соСдинСнии рСзисторов ΠΈΡ… ΠΎΠ±Ρ‰Π΅Π΅ сопротивлСниС увСличиваСтся, ΠΈ ΠΎΠ½ΠΎ Ρ€Π°Π²Π½ΠΎ суммС всСх сопротивлСний.

НапряТСниС ΠΏΡ€ΠΈ ΠΏΠΎΡΠ»Π΅Π΄ΠΎΠ²Π°Ρ‚Π΅Π»ΡŒΠ½ΠΎΠΌ соСдинСнии

НапряТСниС ΠΏΡ€ΠΈ ΠΏΠΎΡΠ»Π΅Π΄ΠΎΠ²Π°Ρ‚Π΅Π»ΡŒΠ½ΠΎΠΌ соСдинСнии распрСдСляСтся Π½Π° ΠΊΠ°ΠΆΠ΄Ρ‹ΠΉ рСзистор согласно Π·Π°ΠΊΠΎΠ½Ρƒ Ома:

Π’.Π΅ Ρ‡Π΅ΠΌ большСС сопротивлСниС рСзистора, Ρ‚Π΅ΠΌ большСС напряТСниС Π½Π° Π½Π΅Π³ΠΎ ΠΏΠ°Π΄Π°Π΅Ρ‚.

ΠŸΠ°Ρ€Π°Π»Π»Π΅Π»ΡŒΠ½ΠΎΠ΅ соСдинСниС рСзисторов

ΠŸΠ°Ρ€Π°Π»Π»Π΅Π»ΡŒΠ½ΠΎΠ΅ соСдинСниС – это соСдинСниС, ΠΏΡ€ΠΈ ΠΊΠΎΡ‚ΠΎΡ€ΠΎΠΌ рСзисторы ΡΠΎΠ΅Π΄ΠΈΠ½ΡΡŽΡ‚ΡΡ ΠΌΠ΅ΠΆΠ΄Ρƒ собой ΠΎΠ±ΠΎΠΈΠΌΠΈ ΠΊΠΎΠ½Ρ‚Π°ΠΊΡ‚Π°ΠΌΠΈ. Π’ Ρ€Π΅Π·ΡƒΠ»ΡŒΡ‚Π°Ρ‚Π΅ ΠΊ ΠΎΠ΄Π½ΠΎΠΉ Ρ‚ΠΎΡ‡ΠΊΠ΅ (элСктричСскому ΡƒΠ·Π»Ρƒ) ΠΌΠΎΠΆΠ΅Ρ‚ Π±Ρ‹Ρ‚ΡŒ присоСдинСно нСсколько рСзисторов.

ΠžΠ±Ρ‰Π΅Π΅ сопротивлСниС R

ΠΎΠ±Ρ‰

ΠŸΡ€ΠΈ Ρ‚Π°ΠΊΠΎΠΌ соСдинСнии, Ρ‡Π΅Ρ€Π΅Π· ΠΊΠ°ΠΆΠ΄Ρ‹ΠΉ рСзистор ΠΏΠΎΡ‚Π΅Ρ‡Π΅Ρ‚ ΠΎΡ‚Π΄Π΅Π»ΡŒΠ½Ρ‹ΠΉ Ρ‚ΠΎΠΊ. Π‘ΠΈΠ»Π° Π΄Π°Π½Π½ΠΎΠ³ΠΎ Ρ‚ΠΎΠΊΠ° Π±ΡƒΠ΄Π΅Ρ‚ ΠΎΠ±Ρ€Π°Ρ‚Π½ΠΎ ΠΏΡ€ΠΎΠΏΠΎΡ€Ρ†ΠΈΠΎΠ½Π°Π»ΡŒΠ½Π° ΡΠΎΠΏΡ€ΠΎΡ‚ΠΈΠ²Π»Π΅Π½ΠΈΡŽ рСзистора. Π’ Ρ€Π΅Π·ΡƒΠ»ΡŒΡ‚Π°Ρ‚Π΅ общая ΠΏΡ€ΠΎΠ²ΠΎΠ΄ΠΈΠΌΠΎΡΡ‚ΡŒ Ρ‚Π°ΠΊΠΎΠ³ΠΎ участка элСктричСской Ρ†Π΅ΠΏΠΈ увСличиваСтся, Π° ΠΎΠ±Ρ‰Π΅Π΅ сопротивлСниС Π² свою ΠΎΡ‡Π΅Ρ€Π΅Π΄ΡŒ ΡƒΠΌΠ΅Π½ΡŒΡˆΠ°Π΅Ρ‚ΡΡ.

Π’Π°ΠΊΠΈΠΌ ΠΎΠ±Ρ€Π°Π·ΠΎΠΌ, ΠΏΡ€ΠΈ ΠΏΠ°Ρ€Π°Π»Π»Π΅Π»ΡŒΠ½ΠΎΠΌ подсоСдинСнии рСзисторов с Ρ€Π°Π·Π½Ρ‹ΠΌ сопротивлСниСм, ΠΎΠ±Ρ‰Π΅Π΅ сопротивлСниС Π±ΡƒΠ΄Π΅Ρ‚ всСгда мСньшС значСния самого малСнького ΠΎΡ‚Π΄Π΅Π»ΡŒΠ½ΠΎΠ³ΠΎ рСзистора.

Π€ΠΎΡ€ΠΌΡƒΠ»Π° ΠΎΠ±Ρ‰Π΅ΠΉ проводимости ΠΏΡ€ΠΈ ΠΏΠ°Ρ€Π°Π»Π»Π΅Π»ΡŒΠ½ΠΎΠΌ соСдинСнии рСзисторов:

Π€ΠΎΡ€ΠΌΡƒΠ»Π° эквивалСнтного ΠΎΠ±Ρ‰Π΅Π³ΠΎ сопротивлСния ΠΏΡ€ΠΈ ΠΏΠ°Ρ€Π°Π»Π»Π΅Π»ΡŒΠ½ΠΎΠΌ соСдинСнии рСзисторов:

Для Π΄Π²ΡƒΡ… ΠΎΠ΄ΠΈΠ½Π°ΠΊΠΎΠ²Ρ‹Ρ… рСзисторов ΠΎΠ±Ρ‰Π΅Π΅ сопротивлСниС Π±ΡƒΠ΄Π΅Ρ‚ Ρ€Π°Π²Π½ΠΎ ΠΏΠΎΠ»ΠΎΠ²ΠΈΠ½Π΅ ΠΎΠ΄Π½ΠΎΠ³ΠΎ ΠΎΡ‚Π΄Π΅Π»ΡŒΠ½ΠΎΠ³ΠΎ рСзистора:

БоотвСтствСнно, для n ΠΎΠ΄ΠΈΠ½Π°ΠΊΠΎΠ²Ρ‹Ρ… рСзисторов ΠΎΠ±Ρ‰Π΅Π΅ сопротивлСниС Π±ΡƒΠ΄Π΅Ρ‚ Ρ€Π°Π²Π½ΠΎ Π·Π½Π°Ρ‡Π΅Π½ΠΈΡŽ ΠΎΠ΄Π½ΠΎΠ³ΠΎ рСзистора, Ρ€Π°Π·Π΄Π΅Π»Π΅Π½Π½ΠΎΠ³ΠΎ Π½Π° n.

НапряТСниС ΠΏΡ€ΠΈ ΠΏΠ°Ρ€Π°Π»Π»Π΅Π»ΡŒΠ½ΠΎΠΌ соСдинСнии

НапряТСниС ΠΌΠ΅ΠΆΠ΄Ρƒ Ρ‚ΠΎΡ‡ΠΊΠ°ΠΌΠΈ A ΠΈ B являСтся ΠΊΠ°ΠΊ ΠΎΠ±Ρ‰ΠΈΠΌ напряТСниСм для всСго участка Ρ†Π΅ΠΏΠΈ, Ρ‚Π°ΠΊ ΠΈ напряТСниСм, ΠΏΠ°Π΄Π°ΡŽΡ‰ΠΈΠΌ Π½Π° ΠΊΠ°ΠΆΠ΄Ρ‹ΠΉ рСзистор Π² ΠΎΡ‚Π΄Π΅Π»ΡŒΠ½ΠΎΡΡ‚ΠΈ. ΠŸΠΎΡΡ‚ΠΎΠΌΡƒ ΠΏΡ€ΠΈ ΠΏΠ°Ρ€Π°Π»Π»Π΅Π»ΡŒΠ½ΠΎΠΌ соСдинСнии Π½Π° всС рСзисторы ΡƒΠΏΠ°Π΄Π΅Ρ‚ ΠΎΠ΄ΠΈΠ½Π°ΠΊΠΎΠ²ΠΎΠ΅ напряТСниС.

ЭлСктричСский Ρ‚ΠΎΠΊ ΠΏΡ€ΠΈ ΠΏΠ°Ρ€Π°Π»Π»Π΅Π»ΡŒΠ½ΠΎΠΌ соСдинСнии

Π§Π΅Ρ€Π΅Π· ΠΊΠ°ΠΆΠ΄Ρ‹ΠΉ рСзистор Ρ‚Π΅Ρ‡Π΅Ρ‚ Ρ‚ΠΎΠΊ, сила ΠΊΠΎΡ‚ΠΎΡ€ΠΎΠ³ΠΎ ΠΎΠ±Ρ€Π°Ρ‚Π½ΠΎ ΠΏΡ€ΠΎΠΏΠΎΡ€Ρ†ΠΈΠΎΠ½Π°Π»ΡŒΠ½Π° ΡΠΎΠΏΡ€ΠΎΡ‚ΠΈΠ²Π»Π΅Π½ΠΈΡŽ рСзистора. Для Ρ‚ΠΎΠ³ΠΎ Ρ‡Ρ‚ΠΎΠ±Ρ‹ ΡƒΠ·Π½Π°Ρ‚ΡŒ ΠΊΠ°ΠΊΠΎΠΉ Ρ‚ΠΎΠΊ Ρ‚Π΅Ρ‡Π΅Ρ‚ Ρ‡Π΅Ρ€Π΅Π· ΠΎΠΏΡ€Π΅Π΄Π΅Π»Π΅Π½Π½Ρ‹ΠΉ рСзистор, ΠΌΠΎΠΆΠ½ΠΎ Π²ΠΎΡΠΏΠΎΠ»ΡŒΠ·ΠΎΠ²Π°Ρ‚ΡŒΡΡ Π·Π°ΠΊΠΎΠ½ΠΎΠΌ Ома:

БмСшанноС соСдинСниС рСзисторов

Π‘ΠΌΠ΅ΡˆΠ°Π½Π½Ρ‹ΠΌ соСдинСниСм Π½Π°Π·Ρ‹Π²Π°ΡŽΡ‚ участок Ρ†Π΅ΠΏΠΈ, Π³Π΄Π΅ Ρ‡Π°ΡΡ‚ΡŒ рСзисторов ΡΠΎΠ΅Π΄ΠΈΠ½ΡΡŽΡ‚ΡΡ ΠΌΠ΅ΠΆΠ΄Ρƒ собой ΠΏΠΎΡΠ»Π΅Π΄ΠΎΠ²Π°Ρ‚Π΅Π»ΡŒΠ½ΠΎ, Π° Ρ‡Π°ΡΡ‚ΡŒ ΠΏΠ°Ρ€Π°Π»Π»Π΅Π»ΡŒΠ½ΠΎ. Π’ свою ΠΎΡ‡Π΅Ρ€Π΅Π΄ΡŒ, смСшанноС соСдинСниС Π±Ρ‹Π²Π°Π΅Ρ‚ ΠΏΠΎΡΠ»Π΅Π΄ΠΎΠ²Π°Ρ‚Π΅Π»ΡŒΠ½ΠΎΠ³ΠΎ ΠΈ ΠΏΠ°Ρ€Π°Π»Π»Π΅Π»ΡŒΠ½ΠΎΠ³ΠΎ Ρ‚ΠΈΠΏΠΎΠ².

ΠžΠ±Ρ‰Π΅Π΅ сопротивлСниС R

ΠΎΠ±Ρ‰

Для Ρ‚ΠΎΠ³ΠΎ Ρ‡Ρ‚ΠΎΠ±Ρ‹ ΠΏΠΎΡΡ‡ΠΈΡ‚Π°Ρ‚ΡŒ ΠΎΠ±Ρ‰Π΅Π΅ сопротивлСниС смСшанного соСдинСния:

  • ЦСпь Ρ€Π°Π·Π±ΠΈΠ²Π°ΡŽΡ‚ Π½Π° участки с Ρ‚ΠΎΠ»ΡŒΠΊΠΎ ΠΏΠ°Ρ€Π΅Π»Π»Π΅Π»ΡŒΠ½Ρ‹ΠΌ ΠΈΠ»ΠΈ Ρ‚ΠΎΠ»ΡŒΠΊΠΎ ΠΏΠΎΡΠ»Π΅Π΄ΠΎΠ²Π°Ρ‚Π΅Π»ΡŒΠ½Ρ‹ΠΌ соСдинСниСм.
  • Π’Ρ‹Ρ‡ΠΈΡΠ»ΡΡŽΡ‚ ΠΎΠ±Ρ‰Π΅Π΅ сопротивлСниС для ΠΊΠ°ΠΆΠ΄ΠΎΠ³ΠΎ ΠΎΡ‚Π΄Π΅Π»ΡŒΠ½ΠΎΠ³ΠΎ участка.
  • Π’Ρ‹Ρ‡ΠΈΡΠ»ΡΡŽΡ‚ ΠΎΠ±Ρ‰Π΅Π΅ сопротивлСниС для всСй Ρ†Π΅ΠΏΠΈ смСшанного соСдинСния.

Π’Π°ΠΊ это Π±ΡƒΠ΄Π΅Ρ‚ Π²Ρ‹Π³Π»ΡΠ΄Π΅Ρ‚ΡŒ для схСмы 1:

Π’Π°ΠΊΠΆΠ΅ сущСствуСт Π±ΠΎΠ»Π΅Π΅ быстрый способ расчСта ΠΎΠ±Ρ‰Π΅Π³ΠΎ сопротивлСния для смСшанного соСдинСния. МоТно, Π² соотвСтствии схСмС, сразу Π·Π°ΠΏΠΈΡΡ‹Π²Π°Ρ‚ΡŒ Ρ„ΠΎΡ€ΠΌΡƒΠ»Ρƒ ΡΠ»Π΅Π΄ΡƒΡŽΡ‰ΠΈΠΌ ΠΎΠ±Ρ€Π°Π·ΠΎΠΌ:

  • Если рСзисторы ΡΠΎΠ΅Π΄ΠΈΠ½ΡΡŽΡ‚ΡΡ ΠΏΠΎΡΠ»Π΅Π΄ΠΎΠ²Π°Ρ‚Π΅ΡŒΠ½ΠΎ β€” ΡΠΊΠ»Π°Π΄Ρ‹Π²Π°Ρ‚ΡŒ.
  • Если рСзисторы ΡΠΎΠ΅Π΄ΠΈΠ½ΡΡŽΡ‚ΡΡ ΠΏΠ°Ρ€Π°Π»Π»Π΅Π»ΡŒΠ½ΠΎ β€” ΠΈΡΠΏΠΎΠ»ΡŒΠ·ΠΎΠ²Π°Ρ‚ΡŒ условноС ΠΎΠ±ΠΎΠ·Π½Π°Ρ‡Π΅Π½ΠΈΠ΅ Β«||Β».
  • ΠŸΠΎΠ΄ΡΡ‚Π°Π²Π»ΡΡ‚ΡŒ Ρ„ΠΎΡ€ΠΌΡƒΠ»Ρƒ для ΠΏΠ°Ρ€Π°Π»Π»Π΅Π»ΡŒΠ½ΠΎΠ³ΠΎ соСдинСния Π³Π΄Π΅ стоит символ Β«||Β».

Π’Π°ΠΊ это Π±ΡƒΠ΄Π΅Ρ‚ Π²Ρ‹Π³Π»ΡΠ΄Π΅Ρ‚ΡŒ для схСмы 1:

ПослС подстановки Ρ„ΠΎΡ€ΠΌΡƒΠ»Ρ‹ ΠΏΠ°Ρ€Π°Π»Π»Π΅Π»ΡŒΠ½ΠΎΠ³ΠΎ соСдинСния вмСсто Β«||Β»:

Как мСняСтся напряТСниС ΠΏΡ€ΠΈ ΠΏΠΎΡΠ»Π΅Π΄ΠΎΠ²Π°Ρ‚Π΅Π»ΡŒΠ½ΠΎΠΌ соСдинСнии. Π‘ΠΎΠ΅Π΄ΠΈΠ½Π΅Π½ΠΈΠ΅ рСзисторов

Π’ΠΎΠΊ Π² элСктроцСпи ΠΏΡ€ΠΎΡ…ΠΎΠ΄ΠΈΡ‚ ΠΏΠΎ ΠΏΡ€ΠΎΠ²ΠΎΠ΄Π½ΠΈΠΊΠ°ΠΌ ΠΎΡ‚ источника напряТСния ΠΊ Π½Π°Π³Ρ€ΡƒΠ·ΠΊΠ΅, Ρ‚ΠΎ Π΅ΡΡ‚ΡŒ ΠΊ Π»Π°ΠΌΠΏΠ°ΠΌ, ΠΏΡ€ΠΈΠ±ΠΎΡ€Π°ΠΌ. Π’ Π±ΠΎΠ»ΡŒΡˆΠΈΠ½ΡΡ‚Π²Π΅ случаСв Π² качСствС ΠΏΡ€ΠΎΠ²ΠΎΠ΄Π½ΠΈΠΊΠ° ΠΈΡΠΏΠΎΠ»ΡŒΠ·ΡƒΡŽΡ‚ΡΡ ΠΌΠ΅Π΄Π½Ρ‹Π΅ ΠΏΡ€ΠΎΠ²ΠΎΠ΄Π°. Π’ Ρ†Π΅ΠΏΠΈ ΠΌΠΎΠΆΠ΅Ρ‚ Π±Ρ‹Ρ‚ΡŒ прСдусмотрСно нСсколько элСмСнтов с Ρ€Π°Π·Π½Ρ‹ΠΌΠΈ сопротивлСниями. Π’ схСмС ΠΏΡ€ΠΈΠ±ΠΎΡ€ΠΎΠ² ΠΏΡ€ΠΎΠ²ΠΎΠ΄Π½ΠΈΠΊΠΈ ΠΌΠΎΠ³ΡƒΡ‚ Π±Ρ‹Ρ‚ΡŒ соСдинСны ΠΏΠ°Ρ€Π°Π»Π»Π΅Π»ΡŒΠ½ΠΎ ΠΈΠ»ΠΈ ΠΏΠΎΡΠ»Π΅Π΄ΠΎΠ²Π°Ρ‚Π΅Π»ΡŒΠ½ΠΎ, Ρ‚Π°ΠΊΠΆΠ΅ ΠΌΠΎΠ³ΡƒΡ‚ Π±Ρ‹Ρ‚ΡŒ ΡΠΌΠ΅ΡˆΠ°Π½Π½Ρ‹Π΅ Ρ‚ΠΈΠΏΡ‹.

Π­Π»Π΅ΠΌΠ΅Π½Ρ‚ схСмы с сопротивлСниСм называСтся рСзистором, напряТСниС Π΄Π°Π½Π½ΠΎΠ³ΠΎ элСмСнта являСтся Ρ€Π°Π·Π½ΠΈΡ†Π΅ΠΉ ΠΏΠΎΡ‚Π΅Π½Ρ†ΠΈΠ°Π»ΠΎΠ² ΠΌΠ΅ΠΆΠ΄Ρƒ ΠΊΠΎΠ½Ρ†Π°ΠΌΠΈ рСзистора. ΠŸΠ°Ρ€Π°Π»Π»Π΅Π»ΡŒΠ½ΠΎΠ΅ ΠΈ ΠΏΠΎΡΠ»Π΅Π΄ΠΎΠ²Π°Ρ‚Π΅Π»ΡŒΠ½ΠΎΠ΅ элСктричСскоС соСдинСниС ΠΏΡ€ΠΎΠ²ΠΎΠ΄Π½ΠΈΠΊΠΎΠ² характСризуСтся Π΅Π΄ΠΈΠ½Ρ‹ΠΌ ΠΏΡ€ΠΈΠ½Ρ†ΠΈΠΏΠΎΠΌ функционирования, согласно ΠΊΠΎΡ‚ΠΎΡ€ΠΎΠΌΡƒ Ρ‚ΠΎΠΊ ΠΏΡ€ΠΎΡ‚Π΅ΠΊΠ°Π΅Ρ‚ ΠΎΡ‚ плюса ΠΊ минусу, соотвСтствСнно ΠΏΠΎΡ‚Π΅Π½Ρ†ΠΈΠ°Π» ΡƒΠΌΠ΅Π½ΡŒΡˆΠ°Π΅Ρ‚ΡΡ. На элСктросхСмах сопротивлСниС ΠΏΡ€ΠΎΠ²ΠΎΠ΄ΠΊΠΈ бСрСтся Π·Π° 0, ΠΏΠΎΡΠΊΠΎΠ»ΡŒΠΊΡƒ ΠΎΠ½ΠΎ Π½ΠΈΡ‡Ρ‚ΠΎΠΆΠ½ΠΎ Π½ΠΈΠ·ΠΊΠΎΠ΅.

ΠŸΠ°Ρ€Π°Π»Π»Π΅Π»ΡŒΠ½ΠΎΠ΅ соСдинСниС ΠΏΡ€Π΅Π΄ΠΏΠΎΠ»Π°Π³Π°Π΅Ρ‚, Ρ‡Ρ‚ΠΎ элСмСнты Ρ†Π΅ΠΏΡ‹ подсоСдинСны ΠΊ источнику ΠΏΠ°Ρ€Π°Π»Π»Π΅Π»ΡŒΠ½ΠΎ ΠΈ Π²ΠΊΠ»ΡŽΡ‡Π°ΡŽΡ‚ΡΡ ΠΎΠ΄Π½ΠΎΠ²Ρ€Π΅ΠΌΠ΅Π½Π½ΠΎ. ΠŸΠΎΡΠ»Π΅Π΄ΠΎΠ²Π°Ρ‚Π΅Π»ΡŒΠ½ΠΎΠ΅ соСдинСниС ΠΎΠ·Π½Π°Ρ‡Π°Π΅Ρ‚, Ρ‡Ρ‚ΠΎ ΠΏΡ€ΠΎΠ²ΠΎΠ΄Π½ΠΈΠΊΠΈ сопротивлСния ΠΏΠΎΠ΄ΠΊΠ»ΡŽΡ‡Π°ΡŽΡ‚ΡΡ Π² строгой ΠΏΠΎΡΠ»Π΅Π΄ΠΎΠ²Π°Ρ‚Π΅Π»ΡŒΠ½ΠΎΡΡ‚ΠΈ Π΄Ρ€ΡƒΠ³ Π·Π° Π΄Ρ€ΡƒΠ³ΠΎΠΌ.

ΠŸΡ€ΠΈ просчСтС ΠΈΡΠΏΠΎΠ»ΡŒΠ·ΡƒΠ΅Ρ‚ΡΡ ΠΌΠ΅Ρ‚ΠΎΠ΄ ΠΈΠ΄Π΅Π°Π»ΠΈΠ·Π°Ρ†ΠΈΠΈ, Ρ‡Ρ‚ΠΎ сущСствСнно ΡƒΠΏΡ€ΠΎΡ‰Π°Π΅Ρ‚ ΠΏΠΎΠ½ΠΈΠΌΠ°Π½ΠΈΠ΅. ЀактичСски Π² элСктричСских цСпях ΠΏΠΎΡ‚Π΅Π½Ρ†ΠΈΠ°Π» постСпСнно сниТаСтся Π² процСссС пСрСмСщСния ΠΏΠΎ ΠΏΡ€ΠΎΠ²ΠΎΠ΄ΠΊΠ΅ ΠΈ элСмСнтам, ΠΊΠΎΡ‚ΠΎΡ€Ρ‹Π΅ входят Π² ΠΏΠ°Ρ€Π°Π»Π»Π΅Π»ΡŒΠ½ΠΎΠ΅ ΠΈΠ»ΠΈ ΠΏΠΎΡΠ»Π΅Π΄ΠΎΠ²Π°Ρ‚Π΅Π»ΡŒΠ½ΠΎΠ΅ соСдинСниС.

ΠŸΠΎΡΠ»Π΅Π΄ΠΎΠ²Π°Ρ‚Π΅Π»ΡŒΠ½ΠΎΠ΅ соСдинСниС ΠΏΡ€ΠΎΠ²ΠΎΠ΄Π½ΠΈΠΊΠΎΠ²

Π‘Ρ…Π΅ΠΌΠ° ΠΏΠΎΡΠ»Π΅Π΄ΠΎΠ²Π°Ρ‚Π΅Π»ΡŒΠ½ΠΎΠ³ΠΎ соСдинСния ΠΏΠΎΠ΄Ρ€Π°Π·ΡƒΠΌΠ΅Π²Π°Π΅Ρ‚, Ρ‡Ρ‚ΠΎ ΠΎΠ½ΠΈ Π²ΠΊΠ»ΡŽΡ‡Π°ΡŽΡ‚ΡΡ Π² ΠΎΠΏΡ€Π΅Π΄Π΅Π»Π΅Π½Π½ΠΎΠΉ ΠΏΠΎΡΠ»Π΅Π΄ΠΎΠ²Π°Ρ‚Π΅Π»ΡŒΠ½ΠΎΡΡ‚ΠΈ ΠΎΠ΄ΠΈΠ½ Π·Π° Π΄Ρ€ΡƒΠ³ΠΈΠΌ. ΠŸΡ€ΠΈΡ‡Π΅ΠΌ сила Ρ‚ΠΎΠΊΠ° Π²ΠΎ всСх ΠΈΠ· Π½ΠΈΡ… Ρ€Π°Π²Π½Π°. Π”Π°Π½Π½Ρ‹Π΅ элСмСнты ΡΠΎΠ·Π΄Π°ΡŽΡ‚ Π½Π° участкС суммарноС напряТСниС. Заряды Π½Π΅ Π½Π°ΠΊΠ°ΠΏΠ»ΠΈΠ²Π°ΡŽΡ‚ΡΡ Π² ΡƒΠ·Π»Π°Ρ… элСктроцСпи, ΠΏΠΎΡΠΊΠΎΠ»ΡŒΠΊΡƒ Π² ΠΏΡ€ΠΎΡ‚ΠΈΠ²Π½ΠΎΠΌ случаС наблюдалось Π±Ρ‹ ΠΈΠ·ΠΌΠ΅Π½Π΅Π½ΠΈΠ΅ напряТСния ΠΈ силы Ρ‚ΠΎΠΊΠ°. ΠŸΡ€ΠΈ постоянном напряТСнии Ρ‚ΠΎΠΊ опрСдСляСтся Π·Π½Π°Ρ‡Π΅Π½ΠΈΠ΅ΠΌ сопротивлСния Ρ†Π΅ΠΏΠΈ, поэтому ΠΏΡ€ΠΈ ΠΏΠΎΡΠ»Π΅Π΄ΠΎΠ²Π°Ρ‚Π΅Π»ΡŒΠ½ΠΎΠΉ схСмС сопротивлСниС мСняСтся Π² случаС измСнСния ΠΎΠ΄Π½ΠΎΠΉ Π½Π°Π³Ρ€ΡƒΠ·ΠΊΠΈ.

НСдостатком Ρ‚Π°ΠΊΠΎΠΉ схСмы являСтся Ρ‚ΠΎΡ‚ Ρ„Π°ΠΊΡ‚, Ρ‡Ρ‚ΠΎ Π² случаС Π²Ρ‹Ρ…ΠΎΠ΄Π° ΠΈΠ· строя ΠΎΠ΄Π½ΠΎΠ³ΠΎ элСмСнта ΠΎΡΡ‚Π°Π»ΡŒΠ½Ρ‹Π΅ Ρ‚Π°ΠΊΠΆΠ΅ ΡƒΡ‚Ρ€Π°Ρ‡ΠΈΠ²Π°ΡŽΡ‚ Π²ΠΎΠ·ΠΌΠΎΠΆΠ½ΠΎΡΡ‚ΡŒ Ρ„ΡƒΠ½ΠΊΡ†ΠΈΠΎΠ½ΠΈΡ€ΠΎΠ²Π°Ρ‚ΡŒ, ΠΏΠΎΡΠΊΠΎΠ»ΡŒΠΊΡƒ Ρ†Π΅ΠΏΡŒ разрываСтся. ΠŸΡ€ΠΈΠΌΠ΅Ρ€ΠΎΠΌ ΠΌΠΎΠΆΠ΅Ρ‚ ΡΠ»ΡƒΠΆΠΈΡ‚ΡŒ гирлянда, которая Π½Π΅ Ρ€Π°Π±ΠΎΡ‚Π°Π΅Ρ‚ Π² случаС пСрСгорания ΠΎΠ΄Π½ΠΎΠΉ Π»Π°ΠΌΠΏΠΎΡ‡ΠΊΠΈ. Π­Ρ‚ΠΎ являСтся ΠΊΠ»ΡŽΡ‡Π΅Π²Ρ‹ΠΌ ΠΎΡ‚Π»ΠΈΡ‡ΠΈΠ΅ΠΌ ΠΎΡ‚ ΠΏΠ°Ρ€Π°Π»Π»Π΅Π»ΡŒΠ½ΠΎΠ³ΠΎ соСдинСния, Π² ΠΊΠΎΡ‚ΠΎΡ€ΠΎΠΌ элСмСнты ΠΌΠΎΠ³ΡƒΡ‚ Ρ„ΡƒΠ½ΠΊΡ†ΠΈΠΎΠ½ΠΈΡ€ΠΎΠ²Π°Ρ‚ΡŒ ΠΏΠΎ ΠΎΡ‚Π΄Π΅Π»ΡŒΠ½ΠΎΡΡ‚ΠΈ.

ΠŸΠΎΡΠ»Π΅Π΄ΠΎΠ²Π°Ρ‚Π΅Π»ΡŒΠ½Π°Ρ схСма ΠΏΡ€Π΅Π΄ΠΏΠΎΠ»Π°Π³Π°Π΅Ρ‚, Ρ‡Ρ‚ΠΎ ΠΏΠΎ ΠΏΡ€ΠΈΡ‡ΠΈΠ½Π΅ ΠΎΠ΄Π½ΠΎΡƒΡ€ΠΎΠ²Π½Π΅Π²ΠΎΠ³ΠΎ ΠΏΠΎΠ΄ΠΊΠ»ΡŽΡ‡Π΅Π½ΠΈΡ ΠΏΡ€ΠΎΠ²ΠΎΠ΄Π½ΠΈΠΊΠΎΠ² ΠΈΡ… сопротивлСниС Π² любой Ρ‚ΠΎΡ‡ΠΊΠΈ сСти Ρ€Π°Π²Π½ΠΎ. ΠžΠ±Ρ‰Π΅Π΅ сопротивлСниС равняСтся суммС ΡƒΠΌΠ΅Π½ΡŒΡˆΠ΅Π½ΠΈΡ напряТСний ΠΎΡ‚Π΄Π΅Π»ΡŒΠ½Ρ‹Ρ… элСмСнтов сСти.

ΠŸΡ€ΠΈ Π΄Π°Π½Π½ΠΎΠΌ Ρ‚ΠΈΠΏΠ΅ соСдинСния Π½Π°Ρ‡Π°Π»ΠΎ ΠΎΠ΄Π½ΠΎΠ³ΠΎ ΠΏΡ€ΠΎΠ²ΠΎΠ΄Π½ΠΈΠΊΠ° подсоСдиняСтся ΠΊ ΠΊΠΎΠ½Ρ†Ρƒ Π΄Ρ€ΡƒΠ³ΠΎΠ³ΠΎ. ΠšΠ»ΡŽΡ‡Π΅Π²Π°Ρ ΠΎΡΠΎΠ±Π΅Π½Π½ΠΎΡΡ‚ΡŒ соСдинСния состоит Π² Ρ‚ΠΎΠΌ, Ρ‡Ρ‚ΠΎ всС ΠΏΡ€ΠΎΠ²ΠΎΠ΄Π½ΠΈΠΊΠΈ находятся Π½Π° ΠΎΠ΄Π½ΠΎΠΌ ΠΏΡ€ΠΎΠ²ΠΎΠ΄Π΅ Π±Π΅Π· Ρ€Π°Π·Π²Π΅Ρ‚Π²Π»Π΅Π½ΠΈΠΉ, ΠΈ Ρ‡Π΅Ρ€Π΅Π· ΠΊΠ°ΠΆΠ΄Ρ‹ΠΉ ΠΈΠ· Π½ΠΈΡ… ΠΏΡ€ΠΎΡ‚Π΅ΠΊΠ°Π΅Ρ‚ ΠΎΠ΄ΠΈΠ½ элСктроток. Однако ΠΎΠ±Ρ‰Π΅Π΅ напряТСниС Ρ€Π°Π²Π½ΠΎ суммС напряТСний Π½Π° ΠΊΠ°ΠΆΠ΄ΠΎΠΌ. Π’Π°ΠΊΠΆΠ΅ ΠΌΠΎΠΆΠ½ΠΎ Ρ€Π°ΡΡΠΌΠΎΡ‚Ρ€Π΅Ρ‚ΡŒ соСдинСниС с Π΄Ρ€ΡƒΠ³ΠΎΠΉ Ρ‚ΠΎΡ‡ΠΊΠΈ зрСния – всС ΠΏΡ€ΠΎΠ²ΠΎΠ΄Π½ΠΈΠΊΠΈ Π·Π°ΠΌΠ΅Π½ΡΡŽΡ‚ΡΡ ΠΎΠ΄Π½ΠΈΠΌ эквивалСнтным рСзистором, ΠΈ Ρ‚ΠΎΠΊ Π½Π° Π½Π΅ΠΌ совпадаСт с ΠΎΠ±Ρ‰ΠΈΠΌ Ρ‚ΠΎΠΊΠΎΠΌ, ΠΊΠΎΡ‚ΠΎΡ€Ρ‹ΠΉ ΠΏΡ€ΠΎΡ…ΠΎΠ΄ΠΈΡ‚ Ρ‡Π΅Ρ€Π΅Π· всС рСзисторы. Π­ΠΊΠ²ΠΈΠ²Π°Π»Π΅Π½Ρ‚Π½ΠΎΠ΅ совокупноС напряТСниС являСтся суммой Π·Π½Π°Ρ‡Π΅Π½ΠΈΠΉ напряТСния ΠΏΠΎ ΠΊΠ°ΠΆΠ΄ΠΎΠΌΡƒ рСзистору. Π’Π°ΠΊ проявляСтся Ρ€Π°Π·Π½ΠΎΡΡ‚ΡŒ ΠΏΠΎΡ‚Π΅Π½Ρ†ΠΈΠ°Π»ΠΎΠ² Π½Π° рСзисторС.

ИспользованиС ΠΏΠΎΡΠ»Π΅Π΄ΠΎΠ²Π°Ρ‚Π΅Π»ΡŒΠ½ΠΎΠ³ΠΎ ΠΏΠΎΠ΄ΠΊΠ»ΡŽΡ‡Π΅Π½ΠΈΡ цСлСсообразно, ΠΊΠΎΠ³Π΄Π° трСбуСтся ΡΠΏΠ΅Ρ†ΠΈΠ°Π»ΡŒΠ½ΠΎ Π²ΠΊΠ»ΡŽΡ‡Π°Ρ‚ΡŒ ΠΈ Π²Ρ‹ΠΊΠ»ΡŽΡ‡Π°Ρ‚ΡŒ ΠΎΠΏΡ€Π΅Π΄Π΅Π»Π΅Π½Π½ΠΎΠ΅ устройство. К ΠΏΡ€ΠΈΠΌΠ΅Ρ€Ρƒ, элСктрозвонок ΠΌΠΎΠΆΠ΅Ρ‚ Π·Π²Π΅Π½Π΅Ρ‚ΡŒ Ρ‚ΠΎΠ»ΡŒΠΊΠΎ Π² ΠΌΠΎΠΌΠ΅Π½Ρ‚, ΠΊΠΎΠ³Π΄Π° присутствуСт соСдинСниС с источником напряТСния ΠΈ ΠΊΠ½ΠΎΠΏΠΊΠΎΠΉ. ΠŸΠ΅Ρ€Π²ΠΎΠ΅ ΠΏΡ€Π°Π²ΠΈΠ»ΠΎ гласит, Ρ‡Ρ‚ΠΎ Ссли Ρ‚ΠΎΠΊΠ° Π½Π΅Ρ‚ хотя Π±Ρ‹ Π½Π° ΠΎΠ΄Π½ΠΎΠΌ ΠΈΠ· элСмСнтов Ρ†Π΅ΠΏΠΈ, Ρ‚ΠΎ ΠΈ Π½Π° ΠΎΡΡ‚Π°Π»ΡŒΠ½Ρ‹Ρ… Π΅Π³ΠΎ Π½Π΅ Π±ΡƒΠ΄Π΅Ρ‚. БоотвСтствСнно ΠΏΡ€ΠΈ Π½Π°Π»ΠΈΡ‡ΠΈΠΈ Ρ‚ΠΎΠΊΠ° Π² ΠΎΠ΄Π½ΠΎΠΌ ΠΏΡ€ΠΎΠ²ΠΎΠ΄Π½ΠΈΠΊΠ΅ ΠΎΠ½ Π΅ΡΡ‚ΡŒ ΠΈ Π² ΠΎΡΡ‚Π°Π»ΡŒΠ½Ρ‹Ρ…. Π”Ρ€ΡƒΠ³ΠΈΠΌ ΠΏΡ€ΠΈΠΌΠ΅Ρ€ΠΎΠΌ ΠΌΠΎΠΆΠ΅Ρ‚ ΡΠ»ΡƒΠΆΠΈΡ‚ΡŒ Ρ„ΠΎΠ½Π°Ρ€ΠΈΠΊ Π½Π° Π±Π°Ρ‚Π°Ρ€Π΅ΠΉΠΊΠ°Ρ…, ΠΊΠΎΡ‚ΠΎΡ€Ρ‹ΠΉ свСтит Ρ‚ΠΎΠ»ΡŒΠΊΠΎ ΠΏΡ€ΠΈ Π½Π°Π»ΠΈΡ‡ΠΈΠΈ Π±Π°Ρ‚Π°Ρ€Π΅ΠΉΠΊΠΈ, исправной Π»Π°ΠΌΠΏΠΎΡ‡ΠΊΠΈ ΠΈ Π½Π°ΠΆΠ°Ρ‚ΠΎΠΉ ΠΊΠ½ΠΎΠΏΠΊΠΈ.

Π’ Π½Π΅ΠΊΠΎΡ‚ΠΎΡ€Ρ‹Ρ… случаях ΠΏΠΎΡΠ»Π΅Π΄ΠΎΠ²Π°Ρ‚Π΅Π»ΡŒΠ½Π°Ρ схСма нСцСлСсообразна. Π’ ΠΊΠ²Π°Ρ€Ρ‚ΠΈΡ€Π΅, Π³Π΄Π΅ систСма освСщСния состоит ΠΈΠ· мноТСства ΡΠ²Π΅Ρ‚ΠΈΠ»ΡŒΠ½ΠΈΠΊΠΎΠ², Π±Ρ€Π°, Π»ΡŽΡΡ‚Ρ€, Π½Π΅ стоит ΠΎΡ€Π³Π°Π½ΠΈΠ·ΠΎΠ²Ρ‹Π²Π°Ρ‚ΡŒ схСму Ρ‚Π°ΠΊΠΎΠ³ΠΎ Ρ‚ΠΈΠΏΠ°, ΠΏΠΎΡΠΊΠΎΠ»ΡŒΠΊΡƒ Π½Π΅Ρ‚ нСобходимости Π²ΠΊΠ»ΡŽΡ‡Π°Ρ‚ΡŒ ΠΈ Π²Ρ‹ΠΊΠ»ΡŽΡ‡Π°Ρ‚ΡŒ освСщСниС Π²ΠΎ всСх ΠΊΠΎΠΌΠ½Π°Ρ‚Π°Ρ… ΠΎΠ΄Π½ΠΎΠ²Ρ€Π΅ΠΌΠ΅Π½Π½ΠΎ. Π‘ этой Ρ†Π΅Π»ΡŒΡŽ Π»ΡƒΡ‡ΡˆΠ΅ ΠΈΡΠΏΠΎΠ»ΡŒΠ·ΠΎΠ²Π°Ρ‚ΡŒ ΠΏΠ°Ρ€Π°Π»Π»Π΅Π»ΡŒΠ½ΠΎΠ΅ соСдинСниС, Ρ‡Ρ‚ΠΎΠ±Ρ‹ ΠΈΠΌΠ΅Ρ‚ΡŒ Π²ΠΎΠ·ΠΌΠΎΠΆΠ½ΠΎΡΡ‚ΡŒ Π²ΠΊΠ»ΡŽΡ‡Π΅Π½ΠΈΡ свСта Π² ΠΎΡ‚Π΄Π΅Π»ΡŒΠ½ΠΎ взятых ΠΊΠΎΠΌΠ½Π°Ρ‚Π°Ρ….

ΠŸΠ°Ρ€Π°Π»Π»Π΅Π»ΡŒΠ½ΠΎΠ΅ соСдинСниС ΠΏΡ€ΠΎΠ²ΠΎΠ΄Π½ΠΈΠΊΠΎΠ²

Π’ ΠΏΠ°Ρ€Π°Π»Π»Π΅Π»ΡŒΠ½ΠΎΠΉ схСмС ΠΏΡ€ΠΎΠ²ΠΎΠ΄Π½ΠΈΠΊΠΈ ΠΏΡ€Π΅Π΄ΡΡ‚Π°Π²Π»ΡΡŽΡ‚ собой Π½Π°Π±ΠΎΡ€ рСзисторов, ΠΎΠ΄Π½ΠΈ ΠΊΠΎΠ½Ρ†Ρ‹ ΠΊΠΎΡ‚ΠΎΡ€Ρ‹Ρ… ΡΠΎΠ±ΠΈΡ€Π°ΡŽΡ‚ΡΡ Π² ΠΎΠ΄ΠΈΠ½ ΡƒΠ·Π΅Π», Π° Π΄Ρ€ΡƒΠ³ΠΈΠ΅ – Π²ΠΎ Π²Ρ‚ΠΎΡ€ΠΎΠΉ ΡƒΠ·Π΅Π». ΠŸΡ€Π΅Π΄ΠΏΠΎΠ»Π°Π³Π°Π΅Ρ‚ΡΡ, Ρ‡Ρ‚ΠΎ напряТСниС Π² ΠΏΠ°Ρ€Π°Π»Π»Π΅Π»ΡŒΠ½ΠΎΠΌ Ρ‚ΠΈΠΏΠ΅ соСдинСния ΠΎΠ΄ΠΈΠ½Π°ΠΊΠΎΠ²ΠΎΠ΅ Π½Π° всСх участках Ρ†Π΅ΠΏΠΈ. ΠŸΠ°Ρ€Π°Π»Π»Π΅Π»ΡŒΠ½Ρ‹Π΅ участки элСктроцСпи носят Π½Π°Π·Π²Π°Π½ΠΈΠ΅ Π²Π΅Ρ‚Π²Π΅ΠΉ ΠΈ проходят ΠΌΠ΅ΠΆΠ΄Ρƒ двумя ΡΠΎΠ΅Π΄ΠΈΠ½ΠΈΡ‚Π΅Π»ΡŒΠ½Ρ‹ΠΌΠΈ ΡƒΠ·Π»Π°ΠΌΠΈ, Π½Π° Π½ΠΈΡ… имССтся ΠΎΠ΄ΠΈΠ½Π°ΠΊΠΎΠ²ΠΎΠ΅ напряТСниС. Π’Π°ΠΊΠΎΠ΅ напряТСниС Ρ€Π°Π²Π½ΠΎ Π·Π½Π°Ρ‡Π΅Π½ΠΈΡŽ Π½Π° ΠΊΠ°ΠΆΠ΄ΠΎΠΌ ΠΏΡ€ΠΎΠ²ΠΎΠ΄Π½ΠΈΠΊΠ΅. Π‘ΡƒΠΌΠΌΠ° ΠΏΠΎΠΊΠ°Π·Π°Ρ‚Π΅Π»Π΅ΠΉ, ΠΎΠ±Ρ€Π°Ρ‚Π½Ρ‹Ρ… сопротивлСниям Π²Π΅Ρ‚Π²Π΅ΠΉ, являСтся ΠΎΠ±Ρ€Π°Ρ‚Π½ΠΎΠΉ ΠΈ ΠΏΠΎ ΠΎΡ‚Π½ΠΎΡˆΠ΅Π½ΠΈΡŽ ΠΊ ΡΠΎΠΏΡ€ΠΎΡ‚ΠΈΠ²Π»Π΅Π½ΠΈΡŽ ΠΎΡ‚Π΄Π΅Π»ΡŒΠ½ΠΎΠ³ΠΎ участка Ρ†Π΅ΠΏΠΈ ΠΏΠ°Ρ€Π°Π»Π»Π΅Π»ΡŒΠ½ΠΎΠΉ схСмы.

ΠŸΡ€ΠΈ ΠΏΠ°Ρ€Π°Π»Π»Π΅Π»ΡŒΠ½ΠΎΠΌ ΠΈ ΠΏΠΎΡΠ»Π΅Π΄ΠΎΠ²Π°Ρ‚Π΅Π»ΡŒΠ½ΠΎΠΌ соСдинСниях отличаСтся систСма расчСта сопротивлСний ΠΎΡ‚Π΄Π΅Π»ΡŒΠ½Ρ‹Ρ… ΠΏΡ€ΠΎΠ²ΠΎΠ΄Π½ΠΈΠΊΠΎΠ². Π’ случаС ΠΏΠ°Ρ€Π°Π»Π»Π΅Π»ΡŒΠ½ΠΎΠΉ схСмы Ρ‚ΠΎΠΊ ΡƒΡ…ΠΎΠ΄ΠΈΡ‚ ΠΏΠΎ вСтвям, Ρ‡Ρ‚ΠΎ способствуСт ΠΏΠΎΠ²Ρ‹ΡˆΠ΅Π½ΠΈΡŽ проводимости Ρ†Π΅ΠΏΠΈ ΠΈ ΡƒΠΌΠ΅Π½ΡŒΡˆΠ°Π΅Ρ‚ совокупноС сопротивлСниС. ΠŸΡ€ΠΈ ΠΏΠ°Ρ€Π°Π»Π»Π΅Π»ΡŒΠ½ΠΎΠΌ ΠΏΠΎΠ΄ΠΊΠ»ΡŽΡ‡Π΅Π½ΠΈΠΈ Π½Π΅ΡΠΊΠΎΠ»ΡŒΠΊΠΈΡ… рСзисторов с Π°Π½Π°Π»ΠΎΠ³ΠΈΡ‡Π½Ρ‹ΠΌΠΈ значСниями совокупноС сопротивлСниС Ρ‚Π°ΠΊΠΎΠΉ элСктроцСпи Π±ΡƒΠ΄Π΅Ρ‚ мСньшС ΠΎΠ΄Π½ΠΎΠ³ΠΎ рСзистора число Ρ€Π°Π·, Ρ€Π°Π²Π½ΠΎΠ΅ числу .

Π’ ΠΊΠ°ΠΆΠ΄ΠΎΠΉ Π²Π΅Ρ‚Π²ΠΈ прСдусмотрСно ΠΏΠΎ ΠΎΠ΄Π½ΠΎΠΌΡƒ рСзистору, ΠΈ элСктроток ΠΏΡ€ΠΈ достиТСнии Ρ‚ΠΎΡ‡ΠΊΠΈ развСтвлСния дСлится ΠΈ расходится ΠΊ ΠΊΠ°ΠΆΠ΄ΠΎΠΌΡƒ рСзистору, Π΅Π³ΠΎ ΠΈΡ‚ΠΎΠ³ΠΎΠ²ΠΎΠ΅ Π·Π½Π°Ρ‡Π΅Π½ΠΈΠ΅ Ρ€Π°Π²Π½ΠΎ суммС Ρ‚ΠΎΠΊΠΎΠ² Π½Π° всСх сопротивлСниях. ВсС рСзисторы Π·Π°ΠΌΠ΅Π½ΡΡŽΡ‚ΡΡ ΠΎΠ΄Π½ΠΈΠΌ эквивалСнтным рСзистором. ΠŸΡ€ΠΈΠΌΠ΅Π½ΡΡ Π·Π°ΠΊΠΎΠ½ Ома, становится понятным Π·Π½Π°Ρ‡Π΅Π½ΠΈΠ΅ сопротивлСния – ΠΏΡ€ΠΈ ΠΏΠ°Ρ€Π°Π»Π»Π΅Π»ΡŒΠ½ΠΎΠΉ схСмС ΡΡƒΠΌΠΌΠΈΡ€ΡƒΡŽΡ‚ΡΡ значСния, ΠΎΠ±Ρ€Π°Ρ‚Π½Ρ‹Π΅ сопротивлСниям Π½Π° рСзисторах.

ΠŸΡ€ΠΈ Π΄Π°Π½Π½ΠΎΠΉ схСмС Π·Π½Π°Ρ‡Π΅Π½ΠΈΠ΅ Ρ‚ΠΎΠΊΠ° ΠΎΠ±Ρ€Π°Ρ‚Π½ΠΎ ΠΏΡ€ΠΎΠΏΠΎΡ€Ρ†ΠΈΠΎΠ½Π°Π»ΡŒΠ½ΠΎ Π·Π½Π°Ρ‡Π΅Π½ΠΈΡŽ сопротивлСния. Π’ΠΎΠΊΠΈ Π² рСзисторах Π½Π΅ взаимосвязаны, поэтому ΠΏΡ€ΠΈ ΠΎΡ‚ΠΊΠ»ΡŽΡ‡Π΅Π½ΠΈΠΈ ΠΎΠ΄Π½ΠΎΠ³ΠΎ ΠΈΠ· Π½ΠΈΡ… это Π½ΠΈΠΊΠΎΠΈΠΌ ΠΎΠ±Ρ€Π°Π·ΠΎΠΌ Π½Π΅ отразится Π½Π° ΠΎΡΡ‚Π°Π»ΡŒΠ½Ρ‹Ρ…. По этой ΠΏΡ€ΠΈΡ‡ΠΈΠ½Π΅ такая схСма ΠΈΡΠΏΠΎΠ»ΡŒΠ·ΡƒΠ΅Ρ‚ΡΡ Π²ΠΎ мноТСствС устройств.

Рассматривая возмоТности примСнСния ΠΏΠ°Ρ€Π°Π»Π»Π΅Π»ΡŒΠ½ΠΎΠΉ схСмы Π² Π±Ρ‹Ρ‚Ρƒ, цСлСсообразно ΠΎΡ‚ΠΌΠ΅Ρ‚ΠΈΡ‚ΡŒ систСму освСщСния ΠΊΠ²Π°Ρ€Ρ‚ΠΈΡ€Ρ‹. ВсС Π»Π°ΠΌΠΏΡ‹ ΠΈ Π»ΡŽΡΡ‚Ρ€Ρ‹ Π΄ΠΎΠ»ΠΆΠ½Ρ‹ Π±Ρ‹Ρ‚ΡŒ соСдинСны ΠΏΠ°Ρ€Π°Π»Π»Π΅Π»ΡŒΠ½ΠΎ, Π² Ρ‚Π°ΠΊΠΎΠΌ случаС Π²ΠΊΠ»ΡŽΡ‡Π΅Π½ΠΈΠ΅ ΠΈ ΠΎΡ‚ΠΊΠ»ΡŽΡ‡Π΅Π½ΠΈΠ΅ ΠΎΠ΄Π½ΠΎΠ³ΠΎ ΠΈΠ· Π½ΠΈΡ… Π½ΠΈΠΊΠ°ΠΊ Π½Π΅ влияСт Π½Π° Ρ€Π°Π±ΠΎΡ‚Ρƒ ΠΎΡΡ‚Π°Π»ΡŒΠ½Ρ‹Ρ… Π»Π°ΠΌΠΏ. Π’Π°ΠΊΠΈΠΌ ΠΎΠ±Ρ€Π°Π·ΠΎΠΌ, добавляя Π²Ρ‹ΠΊΠ»ΡŽΡ‡Π°Ρ‚Π΅Π»ΡŒ ΠΊΠ°ΠΆΠ΄ΠΎΠΉ Π»Π°ΠΌΠΏΠΎΡ‡ΠΊΠΈ Π² Π²Π΅Ρ‚Π²ΡŒ Ρ†Π΅ΠΏΠΈ, ΠΌΠΎΠΆΠ½ΠΎ Π²ΠΊΠ»ΡŽΡ‡Π°Ρ‚ΡŒ ΠΈ ΠΎΡ‚ΠΊΠ»ΡŽΡ‡Π°Ρ‚ΡŒ ΡΠΎΠΎΡ‚Π²Π΅Ρ‚ΡΡ‚Π²ΡƒΡŽΡ‰ΠΈΠΉ ΡΠ²Π΅Ρ‚ΠΈΠ»ΡŒΠ½ΠΈΠΊ ΠΏΠΎ нСобходимости. ВсС ΠΎΡΡ‚Π°Π»ΡŒΠ½Ρ‹Π΅ Π»Π°ΠΌΠΏΡ‹ Ρ€Π°Π±ΠΎΡ‚Π°ΡŽΡ‚ нСзависимо.

ВсС элСктроприборы ΠΎΠ±ΡŠΠ΅Π΄ΠΈΠ½ΡΡŽΡ‚ΡΡ ΠΏΠ°Ρ€Π°Π»Π»Π΅Π»ΡŒΠ½ΠΎ Π² ΡΠ»Π΅ΠΊΡ‚Ρ€ΠΎΡΠ΅Ρ‚ΡŒ с напряТСниСм 220 Π’, Π·Π°Ρ‚Π΅ΠΌ ΠΎΠ½ΠΈ ΠΏΠΎΠ΄ΠΊΠ»ΡŽΡ‡Π°ΡŽΡ‚ΡΡ ΠΊ . Π’ΠΎ Π΅ΡΡ‚ΡŒ всС ΠΏΡ€ΠΈΠ±ΠΎΡ€Ρ‹ ΠΏΠΎΠ΄ΠΊΠ»ΡŽΡ‡Π°ΡŽΡ‚ΡΡ нСзависимо ΠΎΡ‚ ΠΏΠΎΠ΄ΠΊΠ»ΡŽΡ‡Π΅Π½ΠΈΡ ΠΏΡ€ΠΎΡ‡ΠΈΡ… устройств.

Π—Π°ΠΊΠΎΠ½Ρ‹ ΠΏΠΎΡΠ»Π΅Π΄ΠΎΠ²Π°Ρ‚Π΅Π»ΡŒΠ½ΠΎΠ³ΠΎ ΠΈ ΠΏΠ°Ρ€Π°Π»Π»Π΅Π»ΡŒΠ½ΠΎΠ³ΠΎ соСдинСния ΠΏΡ€ΠΎΠ²ΠΎΠ΄Π½ΠΈΠΊΠΎΠ²

Для Π΄Π΅Ρ‚Π°Π»ΡŒΠ½ΠΎΠ³ΠΎ понимания Π½Π° ΠΏΡ€Π°ΠΊΡ‚ΠΈΠΊΠ΅ ΠΎΠ±ΠΎΠΈΡ… Ρ‚ΠΈΠΏΠΎΠ² соСдинСний, ΠΏΡ€ΠΈΠ²Π΅Π΄Π΅ΠΌ Ρ„ΠΎΡ€ΠΌΡƒΠ»Ρ‹, ΠΎΠ±ΡŠΡΡΠ½ΡΡŽΡ‰ΠΈΠ΅ Π·Π°ΠΊΠΎΠ½Ρ‹ Π΄Π°Π½Π½Ρ‹Ρ… Ρ‚ΠΈΠΏΠΎΠ² соСдинСний. РасчСт мощности ΠΏΡ€ΠΈ ΠΏΠ°Ρ€Π°Π»Π»Π΅Π»ΡŒΠ½ΠΎΠΌ ΠΈ ΠΏΠΎΡΠ»Π΅Π΄ΠΎΠ²Π°Ρ‚Π΅Π»ΡŒΠ½ΠΎΠΌ Ρ‚ΠΈΠΏΠ΅ соСдинСния отличаСтся.

ΠŸΡ€ΠΈ ΠΏΠΎΡΠ»Π΅Π΄ΠΎΠ²Π°Ρ‚Π΅Π»ΡŒΠ½ΠΎΠΉ схСмС имССтся одинаковая сила Ρ‚ΠΎΠΊΠ° Π²ΠΎ всСх ΠΏΡ€ΠΎΠ²ΠΎΠ΄Π½ΠΈΠΊΠ°Ρ…:

Богласно Π·Π°ΠΊΠΎΠ½Ρƒ Ома, Π΄Π°Π½Π½Ρ‹Π΅ Ρ‚ΠΈΠΏΡ‹ соСдинСний ΠΏΡ€ΠΎΠ²ΠΎΠ΄Π½ΠΈΠΊΠΎΠ² Π² Ρ€Π°Π·Π½Ρ‹Ρ… случаях ΠΎΠ±ΡŠΡΡΠ½ΡΡŽΡ‚ΡΡ ΠΈΠ½Π°Ρ‡Π΅. Π’Π°ΠΊ, Π² случаС ΠΏΠΎΡΠ»Π΅Π΄ΠΎΠ²Π°Ρ‚Π΅Π»ΡŒΠ½ΠΎΠΉ схСмы, напряТСния Ρ€Π°Π²Π½Ρ‹ Π΄Ρ€ΡƒΠ³ Π΄Ρ€ΡƒΠ³Ρƒ:

U1 = IR1, U2 = IR2.

Помимо этого, ΠΎΠ±Ρ‰Π΅Π΅ напряТСниС Ρ€Π°Π²Π½ΠΎ суммС напряТСний ΠΎΡ‚Π΄Π΅Π»ΡŒΠ½ΠΎ взятых ΠΏΡ€ΠΎΠ²ΠΎΠ΄Π½ΠΈΠΊΠΎΠ²:

U = U1 + U2 = I(R1 + R2) = IR.

ПолноС сопротивлСниС элСктроцСпи рассчитываСтся ΠΊΠ°ΠΊ сумма Π°ΠΊΡ‚ΠΈΠ²Π½Ρ‹Ρ… сопротивлСний всСх ΠΏΡ€ΠΎΠ²ΠΎΠ΄Π½ΠΈΠΊΠΎΠ², Π²Π½Π΅ зависимости ΠΎΡ‚ ΠΈΡ… числа.

Π’ случаС ΠΏΠ°Ρ€Π°Π»Π»Π΅Π»ΡŒΠ½ΠΎΠΉ схСмы совокупноС напряТСниС Ρ†Π΅ΠΏΠΈ Π°Π½Π°Π»ΠΎΠ³ΠΈΡ‡Π½ΠΎ Π½Π°ΠΏΡ€ΡΠΆΠ΅Π½ΠΈΡŽ ΠΎΡ‚Π΄Π΅Π»ΡŒΠ½Ρ‹Ρ… элСмСнтов:

А совокупная сила элСктротока рассчитываСтся ΠΊΠ°ΠΊ сумма Ρ‚ΠΎΠΊΠΎΠ², ΠΊΠΎΡ‚ΠΎΡ€Ρ‹Π΅ ΠΈΠΌΠ΅ΡŽΡ‚ΡΡ ΠΏΠΎ всСм ΠΏΡ€ΠΎΠ²ΠΎΠ΄Π½ΠΈΠΊΠ°ΠΌ, располоТСнным ΠΏΠ°Ρ€Π°Π»Π»Π΅Π»ΡŒΠ½ΠΎ:

Π§Ρ‚ΠΎΠ±Ρ‹ ΠΎΠ±Π΅ΡΠΏΠ΅Ρ‡ΠΈΡ‚ΡŒ ΠΌΠ°ΠΊΡΠΈΠΌΠ°Π»ΡŒΠ½ΡƒΡŽ ΡΡ„Ρ„Π΅ΠΊΡ‚ΠΈΠ²Π½ΠΎΡΡ‚ΡŒ элСктричСских сСтСй, Π½Π΅ΠΎΠ±Ρ…ΠΎΠ΄ΠΈΠΌΠΎ ΠΏΠΎΠ½ΠΈΠΌΠ°Ρ‚ΡŒ ΡΡƒΡ‚ΡŒ ΠΎΠ±ΠΎΠΈΡ… Ρ‚ΠΈΠΏΠΎΠ² соСдинСний ΠΈ ΠΏΡ€ΠΈΠΌΠ΅Π½ΡΡ‚ΡŒ ΠΈΡ… цСлСсообразно, ΠΈΡΠΏΠΎΠ»ΡŒΠ·ΡƒΡ Π·Π°ΠΊΠΎΠ½Ρ‹ ΠΈ рассчитывая Ρ€Π°Ρ†ΠΈΠΎΠ½Π°Π»ΡŒΠ½ΠΎΡΡ‚ΡŒ практичСской Ρ€Π΅Π°Π»ΠΈΠ·Π°Ρ†ΠΈΠΈ.

БмСшанноС соСдинСниС ΠΏΡ€ΠΎΠ²ΠΎΠ΄Π½ΠΈΠΊΠΎΠ²

ΠŸΠΎΡΠ»Π΅Π΄ΠΎΠ²Π°Ρ‚Π΅Π»ΡŒΠ½Π°Ρ ΠΈ ΠΏΠ°Ρ€Π°Π»Π»Π΅Π»ΡŒΠ½Π°Ρ схСма соСдинСния сопротивлСния ΠΌΠΎΠ³ΡƒΡ‚ ΡΠΎΡ‡Π΅Ρ‚Π°Ρ‚ΡŒΡΡ Π² ΠΎΠ΄Π½ΠΎΠΉ элСктросхСмС ΠΏΡ€ΠΈ нСобходимости. К ΠΏΡ€ΠΈΠΌΠ΅Ρ€Ρƒ, допускаСтся ΠΏΠΎΠ΄ΠΊΠ»ΡŽΡ‡Π΅Π½ΠΈΠ΅ ΠΏΠ°Ρ€Π°Π»Π»Π΅Π»ΡŒΠ½Ρ‹Ρ… рСзисторов ΠΏΠΎ ΠΏΠΎΡΠ»Π΅Π΄ΠΎΠ²Π°Ρ‚Π΅Π»ΡŒΠ½ΠΎΠΉ ΠΈΠ»ΠΈ ΠΈΡ… Π³Ρ€ΡƒΠΏΠΏΠ΅, Ρ‚Π°ΠΊΠΎΠ΅ Ρ‚ΠΈΠΏ считаСтся ΠΊΠΎΠΌΠ±ΠΈΠ½ΠΈΡ€ΠΎΠ²Π°Π½Π½Ρ‹ΠΌ ΠΈΠ»ΠΈ ΡΠΌΠ΅ΡˆΠ°Π½Π½Ρ‹ΠΌ.

Π’ Ρ‚Π°ΠΊΠΎΠΌ случаС совокупноС сопротивлСниС рассчитываСтся посрСдством получСния сумм Π·Π½Π°Ρ‡Π΅Π½ΠΈΠΉ для ΠΏΠ°Ρ€Π°Π»Π»Π΅Π»ΡŒΠ½ΠΎΠ³ΠΎ соСдинСния Π² систСмС ΠΈ для ΠΏΠΎΡΠ»Π΅Π΄ΠΎΠ²Π°Ρ‚Π΅Π»ΡŒΠ½ΠΎΠ³ΠΎ. Π‘Π½Π°Ρ‡Π°Π»Π° Π½Π΅ΠΎΠ±Ρ…ΠΎΠ΄ΠΈΠΌΠΎ Ρ€Π°ΡΡΡ‡ΠΈΡ‚Ρ‹Π²Π°Ρ‚ΡŒ эквивалСнтныС сопротивлСния рСзисторов Π² ΠΏΠΎΡΠ»Π΅Π΄ΠΎΠ²Π°Ρ‚Π΅Π»ΡŒΠ½ΠΎΠΉ схСмС, Π° Π·Π°Ρ‚Π΅ΠΌ элСмСнтов ΠΏΠ°Ρ€Π°Π»Π»Π΅Π»ΡŒΠ½ΠΎΠ³ΠΎ. ΠŸΠΎΡΠ»Π΅Π΄ΠΎΠ²Π°Ρ‚Π΅Π»ΡŒΠ½ΠΎΠ΅ соСдинСниС считаСтся ΠΏΡ€ΠΈΠΎΡ€ΠΈΡ‚Π΅Ρ‚Π½Ρ‹ΠΌ, ΠΏΡ€ΠΈΡ‡Π΅ΠΌ схСмы Ρ‚Π°ΠΊΠΎΠ³ΠΎ ΠΊΠΎΠΌΠ±ΠΈΠ½ΠΈΡ€ΠΎΠ²Π°Π½Π½ΠΎΠ³ΠΎ Ρ‚ΠΈΠΏΠ° часто ΠΈΡΠΏΠΎΠ»ΡŒΠ·ΡƒΡŽΡ‚ΡΡ Π² Π±Ρ‹Ρ‚ΠΎΠ²ΠΎΠΉ Ρ‚Π΅Ρ…Π½ΠΈΠΊΠ΅ ΠΈ ΠΏΡ€ΠΈΠ±ΠΎΡ€Π°Ρ….

Π˜Ρ‚Π°ΠΊ, рассматривая Ρ‚ΠΈΠΏΡ‹ ΠΏΠΎΠ΄ΠΊΠ»ΡŽΡ‡Π΅Π½ΠΈΠΉ ΠΏΡ€ΠΎΠ²ΠΎΠ΄Π½ΠΈΠΊΠΎΠ² Π² элСктроцСпях ΠΈ ΠΎΡΠ½ΠΎΠ²Ρ‹Π²Π°ΡΡΡŒ Π½Π° Π·Π°ΠΊΠΎΠ½Π°Ρ… ΠΈΡ… функционирования, ΠΌΠΎΠΆΠ½ΠΎ ΠΏΠΎΠ»Π½ΠΎΡΡ‚ΡŒΡŽ ΠΏΠΎΠ½ΡΡ‚ΡŒ ΡΡƒΡ‚ΡŒ ΠΎΡ€Π³Π°Π½ΠΈΠ·Π°Ρ†ΠΈΠΈ схСм Π±ΠΎΠ»ΡŒΡˆΠΈΠ½ΡΡ‚Π²Π° Π±Ρ‹Ρ‚ΠΎΠ²Ρ‹Ρ… элСктроприборов. ΠŸΡ€ΠΈ ΠΏΠ°Ρ€Π°Π»Π»Π΅Π»ΡŒΠ½ΠΎΠΌ ΠΈ ΠΏΠΎΡΠ»Π΅Π΄ΠΎΠ²Π°Ρ‚Π΅Π»ΡŒΠ½ΠΎΠΌ соСдинСниях расчСт ΠΏΠΎΠΊΠ°Π·Π°Ρ‚Π΅Π»Π΅ΠΉ сопротивлСния ΠΈ силы Ρ‚ΠΎΠΊΠ° отличаСтся. Зная ΠΏΡ€ΠΈΠ½Ρ†ΠΈΠΏΡ‹ расчСта ΠΈ Ρ„ΠΎΡ€ΠΌΡƒΠ»Ρ‹, ΠΌΠΎΠΆΠ½ΠΎ Π³Ρ€Π°ΠΌΠΎΡ‚Π½ΠΎ ΠΈΡΠΏΠΎΠ»ΡŒΠ·ΠΎΠ²Π°Ρ‚ΡŒ ΠΊΠ°ΠΆΠ΄Ρ‹ΠΉ Ρ‚ΠΈΠΏ ΠΎΡ€Π³Π°Π½ΠΈΠ·Π°Ρ†ΠΈΠΈ Ρ†Π΅ΠΏΠ΅ΠΉ для ΠΏΠΎΠ΄ΠΊΠ»ΡŽΡ‡Π΅Π½ΠΈΡ элСмСнтов ΠΎΠΏΡ‚ΠΈΠΌΠ°Π»ΡŒΠ½Ρ‹ΠΌ способом ΠΈ с максимальной ΡΡ„Ρ„Π΅ΠΊΡ‚ΠΈΠ²Π½ΠΎΡΡ‚ΡŒΡŽ.

ΠŸΡ€Π°ΠΊΡ‚ΠΈΡ‡Π΅ΡΠΊΠΈ ΠΊΠ°ΠΆΠ΄ΠΎΠΌΡƒ, ΠΊΡ‚ΠΎ занимался элСктрикой, ΠΏΡ€ΠΈΡ…ΠΎΠ΄ΠΈΠ»ΠΎΡΡŒ Ρ€Π΅ΡˆΠ°Ρ‚ΡŒ вопрос ΠΏΠ°Ρ€Π°Π»Π»Π΅Π»ΡŒΠ½ΠΎΠ³ΠΎ ΠΈ ΠΏΠΎΡΠ»Π΅Π΄ΠΎΠ²Π°Ρ‚Π΅Π»ΡŒΠ½ΠΎΠ³ΠΎ соСдинСния элСмСнтов схСмы. НСкоторыС Ρ€Π΅ΡˆΠ°ΡŽΡ‚ ΠΏΡ€ΠΎΠ±Π»Π΅ΠΌΡ‹ ΠΏΠ°Ρ€Π°Π»Π»Π΅Π»ΡŒΠ½ΠΎΠ³ΠΎ ΠΈ ΠΏΠΎΡΠ»Π΅Π΄ΠΎΠ²Π°Ρ‚Π΅Π»ΡŒΠ½ΠΎΠ³ΠΎ соСдинСния ΠΏΡ€ΠΎΠ²ΠΎΠ΄Π½ΠΈΠΊΠΎΠ² ΠΌΠ΅Ρ‚ΠΎΠ΄ΠΎΠΌ Β«Ρ‚Ρ‹ΠΊΠ°Β», для ΠΌΠ½ΠΎΠ³ΠΈΡ… «нСсгораСмая» гирлянда являСтся нСобъяснимой, Π½ΠΎ ΠΏΡ€ΠΈΠ²Ρ‹Ρ‡Π½ΠΎΠΉ аксиомой. Π’Π΅ΠΌ Π½Π΅ ΠΌΠ΅Π½Π΅Π΅, всС эти ΠΈ ΠΌΠ½ΠΎΠ³ΠΈΠ΅ Π΄Ρ€ΡƒΠ³ΠΈΠ΅ ΠΏΠΎΠ΄ΠΎΠ±Π½Ρ‹Π΅ вопросы Π»Π΅Π³ΠΊΠΎ Ρ€Π΅ΡˆΠ°ΡŽΡ‚ΡΡ ΠΌΠ΅Ρ‚ΠΎΠ΄ΠΎΠΌ, ΠΏΡ€Π΅Π΄Π»ΠΎΠΆΠ΅Π½Π½Ρ‹ΠΌ Π΅Ρ‰Π΅ Π² самом Π½Π°Ρ‡Π°Π»Π΅ XIX Π²Π΅ΠΊΠ° Π½Π΅ΠΌΠ΅Ρ†ΠΊΠΈΠΌ Ρ„ΠΈΠ·ΠΈΠΊΠΎΠΌ Π“Π΅ΠΎΡ€Π³ΠΎΠΌ Омом. Π—Π°ΠΊΠΎΠ½Ρ‹, ΠΎΡ‚ΠΊΡ€Ρ‹Ρ‚Ρ‹Π΅ ΠΈΠΌ, Π΄Π΅ΠΉΡΡ‚Π²ΡƒΡŽΡ‚ ΠΈ ΠΏΠΎΠ½Ρ‹Π½Π΅, Π° ΠΏΠΎΠ½ΡΡ‚ΡŒ ΠΈΡ… смоТСт практичСски ΠΊΠ°ΠΆΠ΄Ρ‹ΠΉ.

ΠžΡΠ½ΠΎΠ²Π½Ρ‹Π΅ элСктричСскиС Π²Π΅Π»ΠΈΡ‡ΠΈΠ½Ρ‹ Ρ†Π΅ΠΏΠΈ

Для Ρ‚ΠΎΠ³ΠΎ Ρ‡Ρ‚ΠΎΠ±Ρ‹ Π²Ρ‹ΡΡΠ½ΠΈΡ‚ΡŒ, ΠΊΠ°ΠΊ Ρ‚ΠΎ ΠΈΠ»ΠΈ ΠΈΠ½ΠΎΠ΅ соСдинСниС ΠΏΡ€ΠΎΠ²ΠΎΠ΄Π½ΠΈΠΊΠΎΠ² повлияСт Π½Π° характСристики схСмы, Π½Π΅ΠΎΠ±Ρ…ΠΎΠ΄ΠΈΠΌΠΎ ΠΎΠΏΡ€Π΅Π΄Π΅Π»ΠΈΡ‚ΡŒΡΡ с Π²Π΅Π»ΠΈΡ‡ΠΈΠ½Π°ΠΌΠΈ, ΠΊΠΎΡ‚ΠΎΡ€Ρ‹Π΅ Ρ…Π°Ρ€Π°ΠΊΡ‚Π΅Ρ€ΠΈΠ·ΡƒΡŽΡ‚ Π»ΡŽΠ±ΡƒΡŽ ΡΠ»Π΅ΠΊΡ‚Ρ€ΠΈΡ‡Π΅ΡΠΊΡƒΡŽ Ρ†Π΅ΠΏΡŒ. Π’ΠΎΡ‚ основныС ΠΈΠ· Π½ΠΈΡ…:

Взаимная Π·Π°Π²ΠΈΡΠΈΠΌΠΎΡΡ‚ΡŒ элСктричСских Π²Π΅Π»ΠΈΡ‡ΠΈΠ½

Π’Π΅ΠΏΠ΅Ρ€ΡŒ Π½Π΅ΠΎΠ±Ρ…ΠΎΠ΄ΠΈΠΌΠΎ ΠΎΠΏΡ€Π΅Π΄Π΅Π»ΠΈΡ‚ΡŒΡΡ , ΠΊΠ°ΠΊ всС Π²Ρ‹ΡˆΠ΅ΠΏΠ΅Ρ€Π΅Ρ‡ΠΈΡΠ»Π΅Π½Π½Ρ‹Π΅ Π²Π΅Π»ΠΈΡ‡ΠΈΠ½Ρ‹ зависят ΠΎΠ΄Π½Π° ΠΎΡ‚ Π΄Ρ€ΡƒΠ³ΠΎΠΉ. ΠŸΡ€Π°Π²ΠΈΠ»Π° зависимости нСслоТны ΠΈ сводятся ΠΊ Π΄Π²ΡƒΠΌ основным Ρ„ΠΎΡ€ΠΌΡƒΠ»Π°ΠΌ:


Π—Π΄Π΅ΡΡŒ I – Ρ‚ΠΎΠΊ Π² Ρ†Π΅ΠΏΠΈ Π² Π°ΠΌΠΏΠ΅Ρ€Π°Ρ…, U – напряТСниС, ΠΏΠΎΠ΄Π²ΠΎΠ΄ΠΈΠΌΠΎΠ΅ ΠΊ Ρ†Π΅ΠΏΠΈ Π² Π²ΠΎΠ»ΡŒΡ‚Π°Ρ…, R – сопротивлСниС Ρ†Π΅ΠΏΠΈ Π² ΠΎΠΌΠ°Ρ…, P – элСктричСская ΠΌΠΎΡ‰Π½ΠΎΡΡ‚ΡŒ Ρ†Π΅ΠΏΠΈ Π² Π²Π°Ρ‚Ρ‚Π°Ρ….

ΠŸΡ€Π΅Π΄ΠΏΠΎΠ»ΠΎΠΆΠΈΠΌ, ΠΏΠ΅Ρ€Π΅Π΄ Π½Π°ΠΌΠΈ ΠΏΡ€ΠΎΡΡ‚Π΅ΠΉΡˆΠ°Ρ элСктричСская Ρ†Π΅ΠΏΡŒ, состоящая ΠΈΠ· источника питания с напряТСниСм U ΠΈ ΠΏΡ€ΠΎΠ²ΠΎΠ΄Π½ΠΈΠΊΠ° с сопротивлСниСм R (Π½Π°Π³Ρ€ΡƒΠ·ΠΊΠΈ).

ΠŸΠΎΡΠΊΠΎΠ»ΡŒΠΊΡƒ Ρ†Π΅ΠΏΡŒ Π·Π°ΠΌΠΊΠ½ΡƒΡ‚Π°, Ρ‡Π΅Ρ€Π΅Π· Π½Π΅Π΅ Ρ‚Π΅Ρ‡Π΅Ρ‚ Ρ‚ΠΎΠΊ I. Какой Π²Π΅Π»ΠΈΡ‡ΠΈΠ½Ρ‹ ΠΎΠ½ Π±ΡƒΠ΄Π΅Ρ‚? Π˜ΡΡ…ΠΎΠ΄Ρ ΠΈΠ· Π²Ρ‹ΡˆΠ΅ΠΏΡ€ΠΈΠ²Π΅Π΄Π΅Π½Π½ΠΎΠΉ Ρ„ΠΎΡ€ΠΌΡƒΠ»Ρ‹ 1, для Π΅Π³ΠΎ вычислСния Π½Π°ΠΌ Π½ΡƒΠΆΠ½ΠΎ Π·Π½Π°Ρ‚ΡŒ напряТСниС, Ρ€Π°Π·Π²ΠΈΠ²Π°Π΅ΠΌΠΎΠ΅ источником питания, ΠΈ сопротивлСниС Π½Π°Π³Ρ€ΡƒΠ·ΠΊΠΈ. Если ΠΌΡ‹ возьмСм, ΠΊ ΠΏΡ€ΠΈΠΌΠ΅Ρ€Ρƒ, паяльник с сопротивлСниСм спирали 100 Ом ΠΈ ΠΏΠΎΠ΄ΠΊΠ»ΡŽΡ‡ΠΈΠΌ Π΅Π³ΠΎ ΠΊ ΠΎΡΠ²Π΅Ρ‚ΠΈΡ‚Π΅Π»ΡŒΠ½ΠΎΠΉ Ρ€ΠΎΠ·Π΅Ρ‚ΠΊΠ΅ с напряТСниСм 220 Π’, Ρ‚ΠΎ Ρ‚ΠΎΠΊ Ρ‡Π΅Ρ€Π΅Π· паяльник Π±ΡƒΠ΄Π΅Ρ‚ ΡΠΎΡΡ‚Π°Π²Π»ΡΡ‚ΡŒ:

220 / 100 = 2,2 А.

Какова ΠΌΠΎΡ‰Π½ΠΎΡΡ‚ΡŒ этого паяльника ? Π’ΠΎΡΠΏΠΎΠ»ΡŒΠ·ΡƒΠ΅ΠΌΡΡ Ρ„ΠΎΡ€ΠΌΡƒΠ»ΠΎΠΉ 2:

2,2 * 220 = 484 Π’Ρ‚.

Π₯ΠΎΡ€ΠΎΡˆΠΈΠΉ получился паяльник, ΠΌΠΎΡ‰Π½Ρ‹ΠΉ, скорСС всСго, Π΄Π²ΡƒΡ€ΡƒΡ‡Π½Ρ‹ΠΉ. Π’ΠΎΡ‡Π½ΠΎ Ρ‚Π°ΠΊ ΠΆΠ΅, опСрируя этими двумя Ρ„ΠΎΡ€ΠΌΡƒΠ»Π°ΠΌΠΈ ΠΈ прСобразуя ΠΈΡ…, ΠΌΠΎΠΆΠ½ΠΎ ΡƒΠ·Π½Π°Ρ‚ΡŒ Ρ‚ΠΎΠΊ Ρ‡Π΅Ρ€Π΅Π· ΠΌΠΎΡ‰Π½ΠΎΡΡ‚ΡŒ ΠΈ напряТСниС, напряТСниС Ρ‡Π΅Ρ€Π΅Π· Ρ‚ΠΎΠΊ ΠΈ сопротивлСниС ΠΈ Ρ‚.Π΄. Бколько, ΠΊ ΠΏΡ€ΠΈΠΌΠ΅Ρ€Ρƒ, потрСбляСт Π»Π°ΠΌΠΏΠΎΡ‡ΠΊΠ° ΠΌΠΎΡ‰Π½ΠΎΡΡ‚ΡŒΡŽ 60 Π’Ρ‚ Π² вашСй Π½Π°ΡΡ‚ΠΎΠ»ΡŒΠ½ΠΎΠΉ Π»Π°ΠΌΠΏΠ΅:

60 / 220 = 0,27 А или 270 мА.

Π‘ΠΎΠΏΡ€ΠΎΡ‚ΠΈΠ²Π»Π΅Π½ΠΈΠ΅ спирали Π»Π°ΠΌΠΏΡ‹ Π² Ρ€Π°Π±ΠΎΡ‡Π΅ΠΌ Ρ€Π΅ΠΆΠΈΠΌΠ΅:

220 / 0,27 = 815 Ом.

Π‘Ρ…Π΅ΠΌΡ‹ с нСсколькими ΠΏΡ€ΠΎΠ²ΠΎΠ΄Π½ΠΈΠΊΠ°ΠΌΠΈ

ВсС рассмотрСнныС Π²Ρ‹ΡˆΠ΅ случаи ΡΠ²Π»ΡΡŽΡ‚ΡΡ простыми – ΠΎΠ΄ΠΈΠ½ источник, ΠΎΠ΄Π½Π° Π½Π°Π³Ρ€ΡƒΠ·ΠΊΠ°. Но Π½Π° ΠΏΡ€Π°ΠΊΡ‚ΠΈΠΊΠ΅ Π½Π°Π³Ρ€ΡƒΠ·ΠΎΠΊ ΠΌΠΎΠΆΠ΅Ρ‚ Π±Ρ‹Ρ‚ΡŒ нСсколько, ΠΈ соСдинСны ΠΎΠ½ΠΈ Π±Ρ‹Π²Π°ΡŽΡ‚ Ρ‚ΠΎΠΆΠ΅ ΠΏΠΎ-Ρ€Π°Π·Π½ΠΎΠΌΡƒ. БущСствуСт Ρ‚Ρ€ΠΈ Ρ‚ΠΈΠΏΠ° соСдинСния Π½Π°Π³Ρ€ΡƒΠ·ΠΊΠΈ:

  1. ΠŸΠ°Ρ€Π°Π»Π»Π΅Π»ΡŒΠ½ΠΎΠ΅.
  2. ΠŸΠΎΡΠ»Π΅Π΄ΠΎΠ²Π°Ρ‚Π΅Π»ΡŒΠ½ΠΎΠ΅.
  3. БмСшанноС.

ΠŸΠ°Ρ€Π°Π»Π»Π΅Π»ΡŒΠ½ΠΎΠ΅ соСдинСниС ΠΏΡ€ΠΎΠ²ΠΎΠ΄Π½ΠΈΠΊΠΎΠ²

Π’ Π»ΡŽΡΡ‚Ρ€Π΅ 3 Π»Π°ΠΌΠΏΡ‹, каТдая ΠΏΠΎ 60 Π’Ρ‚. Бколько потрСбляСт Π»ΡŽΡΡ‚Ρ€Π°? Π’Π΅Ρ€Π½ΠΎ, 180 Π’Ρ‚. Π‘Ρ‹ΡΡ‚Ρ€Π΅Π½ΡŒΠΊΠΎ подсчитываСм сначала Ρ‚ΠΎΠΊ Ρ‡Π΅Ρ€Π΅Π· Π»ΡŽΡΡ‚Ρ€Ρƒ:

180 / 220 = 0,818 А.

А Π·Π°Ρ‚Π΅ΠΌ ΠΈ Π΅Π΅ сопротивлСниС:

220 / 0,818 = 269 Ом.

ΠŸΠ΅Ρ€Π΅Π΄ этим ΠΌΡ‹ вычисляли сопротивлСниС ΠΎΠ΄Π½ΠΎΠΉ Π»Π°ΠΌΠΏΡ‹ (815 Ом) ΠΈ Ρ‚ΠΎΠΊ Ρ‡Π΅Ρ€Π΅Π· Π½Π΅Π΅ (270 мА). Π‘ΠΎΠΏΡ€ΠΎΡ‚ΠΈΠ²Π»Π΅Π½ΠΈΠ΅ ΠΆΠ΅ Π»ΡŽΡΡ‚Ρ€Ρ‹ оказалось Π²Ρ‚Ρ€ΠΎΠ΅ Π½ΠΈΠΆΠ΅, Π° Ρ‚ΠΎΠΊ — Π²Ρ‚Ρ€ΠΎΠ΅ Π²Ρ‹ΡˆΠ΅. А Ρ‚Π΅ΠΏΠ΅Ρ€ΡŒ ΠΏΠΎΡ€Π° Π²Π·Π³Π»ΡΠ½ΡƒΡ‚ΡŒ Π½Π° схСму Ρ‚Ρ€Π΅Ρ…Ρ€ΠΎΠΆΠΊΠΎΠ²ΠΎΠ³ΠΎ ΡΠ²Π΅Ρ‚ΠΈΠ»ΡŒΠ½ΠΈΠΊΠ°.

ВсС Π»Π°ΠΌΠΏΡ‹ Π² Π½Π΅ΠΌ соСдинСны ΠΏΠ°Ρ€Π°Π»Π»Π΅Π»ΡŒΠ½ΠΎ ΠΈ ΠΏΠΎΠ΄ΠΊΠ»ΡŽΡ‡Π΅Π½Ρ‹ ΠΊ сСти. ΠŸΠΎΠ»ΡƒΡ‡Π°Π΅Ρ‚ΡΡ, ΠΏΡ€ΠΈ ΠΏΠ°Ρ€Π°Π»Π»Π΅Π»ΡŒΠ½ΠΎΠΌ соСдинСнии Ρ‚Ρ€Π΅Ρ… Π»Π°ΠΌΠΏ ΠΎΠ±Ρ‰Π΅Π΅ сопротивлСниС Π½Π°Π³Ρ€ΡƒΠ·ΠΊΠΈ ΡƒΠΌΠ΅Π½ΡŒΡˆΠΈΠ»ΠΎΡΡŒ Π²Ρ‚Ρ€ΠΎΠ΅? Π’ нашСм случаС — Π΄Π°, Π½ΠΎ ΠΎΠ½ частный – всС Π»Π°ΠΌΠΏΡ‹ ΠΈΠΌΠ΅ΡŽΡ‚ ΠΎΠ΄ΠΈΠ½Π°ΠΊΠΎΠ²Ρ‹Π΅ сопротивлСниС ΠΈ ΠΌΠΎΡ‰Π½ΠΎΡΡ‚ΡŒ. Если каТдая ΠΈΠ· Π½Π°Π³Ρ€ΡƒΠ·ΠΎΠΊ Π±ΡƒΠ΄Π΅Ρ‚ ΠΈΠΌΠ΅Ρ‚ΡŒ своС сопротивлСниС, Ρ‚ΠΎ для подсчСта ΠΎΠ±Ρ‰Π΅Π³ΠΎ значСния простого дСлСния Π½Π° количСство Π½Π°Π³Ρ€ΡƒΠ·ΠΎΠΊ ΠΌΠ°Π»ΠΎ. Но ΠΈ Ρ‚ΡƒΡ‚ Π΅ΡΡ‚ΡŒ Π²Ρ‹Ρ…ΠΎΠ΄ ΠΈΠ· полоТСния – достаточно Π²ΠΎΡΠΏΠΎΠ»ΡŒΠ·ΠΎΠ²Π°Ρ‚ΡŒΡΡ Π²ΠΎΡ‚ этой Ρ„ΠΎΡ€ΠΌΡƒΠ»ΠΎΠΉ:

1/RΠΎΠ±Ρ‰. = 1/R1 + 1/R2 + … 1/Rn.

Для удобства использования Ρ„ΠΎΡ€ΠΌΡƒΠ»Ρƒ ΠΌΠΎΠΆΠ½ΠΎ Π»Π΅Π³ΠΊΠΎ ΠΏΡ€Π΅ΠΎΠ±Ρ€Π°Π·ΠΎΠ²Π°Ρ‚ΡŒ:

RΠΎΠ±Ρ‰. = (R1*R2*… Rn) / (R1+R2+ … Rn).

Π—Π΄Π΅ΡΡŒ RΠΎΠ±Ρ‰ . – ΠΎΠ±Ρ‰Π΅Π΅ сопротивлСниС Ρ†Π΅ΠΏΠΈ ΠΏΡ€ΠΈ ΠΏΠ°Ρ€Π°Π»Π»Π΅Π»ΡŒΠ½ΠΎΠΌ Π²ΠΊΠ»ΡŽΡ‡Π΅Π½ΠΈΠΈ Π½Π°Π³Ρ€ΡƒΠ·ΠΊΠΈ. R1 … Rn – сопротивлСния ΠΊΠ°ΠΆΠ΄ΠΎΠΉ Π½Π°Π³Ρ€ΡƒΠ·ΠΊΠΈ.

ΠŸΠΎΡ‡Π΅ΠΌΡƒ увСличился Ρ‚ΠΎΠΊ, ΠΊΠΎΠ³Π΄Π° Π²Ρ‹ Π²ΠΊΠ»ΡŽΡ‡ΠΈΠ»ΠΈ ΠΏΠ°Ρ€Π°Π»Π»Π΅Π»ΡŒΠ½ΠΎ Ρ‚Ρ€ΠΈ Π»Π°ΠΌΠΏΡ‹ вмСсто ΠΎΠ΄Π½ΠΎΠΉ, ΠΏΠΎΠ½ΡΡ‚ΡŒ нСслоТно – вСдь ΠΎΠ½ зависит ΠΎΡ‚ напряТСния (ΠΎΠ½ΠΎ ΠΎΡΡ‚Π°Π»ΠΎΡΡŒ Π½Π΅ΠΈΠ·ΠΌΠ΅Π½Π½Ρ‹ΠΌ), Π΄Π΅Π»Π΅Π½Π½ΠΎΠ³ΠΎ Π½Π° сопротивлСниС (ΠΎΠ½ΠΎ ΡƒΠΌΠ΅Π½ΡŒΡˆΠΈΠ»ΠΎΡΡŒ). ΠžΡ‡Π΅Π²ΠΈΠ΄Π½ΠΎ, Ρ‡Ρ‚ΠΎ ΠΈ ΠΌΠΎΡ‰Π½ΠΎΡΡ‚ΡŒ ΠΏΡ€ΠΈ ΠΏΠ°Ρ€Π°Π»Π»Π΅Π»ΡŒΠ½ΠΎΠΌ соСдинСнии увСличится ΠΏΡ€ΠΎΠΏΠΎΡ€Ρ†ΠΈΠΎΠ½Π°Π»ΡŒΠ½ΠΎ ΡƒΠ²Π΅Π»ΠΈΡ‡Π΅Π½ΠΈΡŽ Ρ‚ΠΎΠΊΠ°.

ΠŸΠΎΡΠ»Π΅Π΄ΠΎΠ²Π°Ρ‚Π΅Π»ΡŒΠ½ΠΎΠ΅ соСдинСниС

Π’Π΅ΠΏΠ΅Ρ€ΡŒ настала ΠΏΠΎΡ€Π° Π²Ρ‹ΡΡΠ½ΠΈΡ‚ΡŒ, ΠΊΠ°ΠΊ измСнятся ΠΏΠ°Ρ€Π°ΠΌΠ΅Ρ‚Ρ€Ρ‹ Ρ†Π΅ΠΏΠΈ, Ссли ΠΏΡ€ΠΎΠ²ΠΎΠ΄Π½ΠΈΠΊΠΈ (Π² нашСм случаС Π»Π°ΠΌΠΏΡ‹) ΡΠΎΠ΅Π΄ΠΈΠ½ΠΈΡ‚ΡŒ ΠΏΠΎΡΠ»Π΅Π΄ΠΎΠ²Π°Ρ‚Π΅Π»ΡŒΠ½ΠΎ.

РасчСт сопротивлСния ΠΏΡ€ΠΈ ΠΏΠΎΡΠ»Π΅Π΄ΠΎΠ²Π°Ρ‚Π΅Π»ΡŒΠ½ΠΎΠΌ соСдинСнии ΠΏΡ€ΠΎΠ²ΠΎΠ΄Π½ΠΈΠΊΠΎΠ² ΠΈΡΠΊΠ»ΡŽΡ‡ΠΈΡ‚Π΅Π»ΡŒΠ½ΠΎ прост:

RΠΎΠ±Ρ‰. = R1 + R2.

Π’Π΅ ΠΆΠ΅ Ρ‚Ρ€ΠΈ ΡˆΠ΅ΡΡ‚ΠΈΠ΄Π΅ΡΡΡ‚ΠΈΠ²Π°Ρ‚Ρ‚Π½Ρ‹Π΅ Π»Π°ΠΌΠΏΡ‹, соСдинСнныС ΠΏΠΎΡΠ»Π΅Π΄ΠΎΠ²Π°Ρ‚Π΅Π»ΡŒΠ½ΠΎ, составят ΡƒΠΆΠ΅ 2445 Ом (см. расчСты Π²Ρ‹ΡˆΠ΅). Какими Π±ΡƒΠ΄ΡƒΡ‚ послСдствия увСличСния сопротивлСния Ρ†Π΅ΠΏΠΈ? Богласно Ρ„ΠΎΡ€ΠΌΡƒΠ»Π°ΠΌ 1 ΠΈ 2 становится Π²ΠΏΠΎΠ»Π½Π΅ понятно, Ρ‡Ρ‚ΠΎ ΠΌΠΎΡ‰Π½ΠΎΡΡ‚ΡŒ ΠΈ сила Ρ‚ΠΎΠΊΠ° ΠΏΡ€ΠΈ ΠΏΠΎΡΠ»Π΅Π΄ΠΎΠ²Π°Ρ‚Π΅Π»ΡŒΠ½ΠΎΠΌ соСдинСнии ΠΏΡ€ΠΎΠ²ΠΎΠ΄Π½ΠΈΠΊΠΎΠ² ΡƒΠΏΠ°Π΄Π΅Ρ‚. Но ΠΏΠΎΡ‡Π΅ΠΌΡƒ Ρ‚Π΅ΠΏΠ΅Ρ€ΡŒ всС Π»Π°ΠΌΠΏΡ‹ горят тускло? Π­Ρ‚ΠΎ ΠΎΠ΄Π½ΠΎ ΠΈΠ· самых интСрСсных свойств ΠΏΠΎΡΠ»Π΅Π΄ΠΎΠ²Π°Ρ‚Π΅Π»ΡŒΠ½ΠΎΠ³ΠΎ ΠΏΠΎΠ΄ΠΊΠ»ΡŽΡ‡Π΅Π½ΠΈΡ ΠΏΡ€ΠΎΠ²ΠΎΠ΄Π½ΠΈΠΊΠΎΠ², ΠΊΠΎΡ‚ΠΎΡ€ΠΎΠ΅ ΠΎΡ‡Π΅Π½ΡŒ ΡˆΠΈΡ€ΠΎΠΊΠΎ ΠΈΡΠΏΠΎΠ»ΡŒΠ·ΡƒΠ΅Ρ‚ΡΡ. ВзглянСм Π½Π° гирлянду ΠΈΠ· Ρ‚Ρ€Π΅Ρ… Π·Π½Π°ΠΊΠΎΠΌΡ‹Ρ… Π½Π°ΠΌ, Π½ΠΎ ΠΏΠΎΡΠ»Π΅Π΄ΠΎΠ²Π°Ρ‚Π΅Π»ΡŒΠ½ΠΎ соСдинСнных Π»Π°ΠΌΠΏ.

ΠžΠ±Ρ‰Π΅Π΅ напряТСниС, ΠΏΡ€ΠΈΠ»ΠΎΠΆΠ΅Π½Π½ΠΎΠ΅ ΠΊΠΎ всСй Ρ†Π΅ΠΏΠΈ, Ρ‚Π°ΠΊ ΠΈ ΠΎΡΡ‚Π°Π»ΠΎΡΡŒ 220 Π’. Но ΠΎΠ½ΠΎ подСлилось ΠΌΠ΅ΠΆΠ΄Ρƒ ΠΊΠ°ΠΆΠ΄ΠΎΠΉ ΠΈΠ· Π»Π°ΠΌΠΏ ΠΏΡ€ΠΎΠΏΠΎΡ€Ρ†ΠΈΠΎΠ½Π°Π»ΡŒΠ½ΠΎ ΠΈΡ… ΡΠΎΠΏΡ€ΠΎΡ‚ΠΈΠ²Π»Π΅Π½ΠΈΡŽ! ΠŸΠΎΡΠΊΠΎΠ»ΡŒΠΊΡƒ Π»Π°ΠΌΠΏΡ‹ Ρƒ нас ΠΎΠ΄ΠΈΠ½Π°ΠΊΠΎΠ²ΠΎΠΉ мощности ΠΈ сопротивлСния, Ρ‚ΠΎ напряТСниС подСлилось ΠΏΠΎΡ€ΠΎΠ²Π½Ρƒ: U1 = U2 = U3 = U/3. Π’ΠΎ Π΅ΡΡ‚ΡŒ Π½Π° ΠΊΠ°ΠΆΠ΄ΡƒΡŽ ΠΈΠ· Π»Π°ΠΌΠΏ подаСтся Ρ‚Π΅ΠΏΠ΅Ρ€ΡŒ Π²Ρ‚Ρ€ΠΎΠ΅ мСньшСС напряТСниС, Π²ΠΎΡ‚ ΠΏΠΎΡ‡Π΅ΠΌΡƒ ΠΎΠ½ΠΈ свСтятся Ρ‚Π°ΠΊ тускло. Π’ΠΎΠ·ΡŒΠΌΠ΅Ρ‚Π΅ большС Π»Π°ΠΌΠΏ – ΡΡ€ΠΊΠΎΡΡ‚ΡŒ ΠΈΡ… ΡƒΠΏΠ°Π΄Π΅Ρ‚ Π΅Ρ‰Π΅ большС. Как Ρ€Π°ΡΡΡ‡ΠΈΡ‚Π°Ρ‚ΡŒ ΠΏΠ°Π΄Π΅Π½ΠΈΠ΅ напряТСния Π½Π° ΠΊΠ°ΠΆΠ΄ΠΎΠΉ ΠΈΠ· Π»Π°ΠΌΠΏ, Ссли всС ΠΎΠ½ΠΈ ΠΈΠΌΠ΅ΡŽΡ‚ Ρ€Π°Π·Π»ΠΈΡ‡Π½Ρ‹Π΅ сопротивлСния? Для этого достаточно Ρ‡Π΅Ρ‚Ρ‹Ρ€Π΅Ρ… Ρ„ΠΎΡ€ΠΌΡƒΠ», ΠΏΡ€ΠΈΠ²Π΅Π΄Π΅Π½Π½Ρ‹Ρ… Π²Ρ‹ΡˆΠ΅. Алгоритм расчСта Π±ΡƒΠ΄Π΅Ρ‚ ΡΠ»Π΅Π΄ΡƒΡŽΡ‰ΠΈΠΌ:

  1. Π˜Π·ΠΌΠ΅Ρ€ΡΠ΅Ρ‚Π΅ сопротивлСниС ΠΊΠ°ΠΆΠ΄ΠΎΠΉ ΠΈΠ· Π»Π°ΠΌΠΏ.
  2. РассчитываСтС ΠΎΠ±Ρ‰Π΅Π΅ сопротивлСниС Ρ†Π΅ΠΏΠΈ.
  3. По ΠΎΠ±Ρ‰ΠΈΠΌ Π½Π°ΠΏΡ€ΡΠΆΠ΅Π½ΠΈΡŽ ΠΈ ΡΠΎΠΏΡ€ΠΎΡ‚ΠΈΠ²Π»Π΅Π½ΠΈΡŽ рассчитываСтС Ρ‚ΠΎΠΊ Π² Ρ†Π΅ΠΏΠΈ.
  4. По ΠΎΠ±Ρ‰Π΅ΠΌΡƒ Ρ‚ΠΎΠΊΡƒ ΠΈ ΡΠΎΠΏΡ€ΠΎΡ‚ΠΈΠ²Π»Π΅Π½ΠΈΡŽ Π»Π°ΠΌΠΏ вычисляСтС ΠΏΠ°Π΄Π΅Π½ΠΈΠ΅ напряТСния Π½Π° ΠΊΠ°ΠΆΠ΄ΠΎΠΉ ΠΈΠ· Π½ΠΈΡ….

Π₯ΠΎΡ‚ΠΈΡ‚Π΅ Π·Π°ΠΊΡ€Π΅ΠΏΠΈΡ‚ΡŒ ΠΏΠΎΠ»ΡƒΡ‡Π΅Π½Π½Ρ‹Π΅ знания ? Π Π΅ΡˆΠΈΡ‚Π΅ ΠΏΡ€ΠΎΡΡ‚ΡƒΡŽ Π·Π°Π΄Π°Ρ‡Ρƒ, Π½Π΅ заглядывая Π² ΠΎΡ‚Π²Π΅Ρ‚ Π² ΠΊΠΎΠ½Ρ†Π΅:

Π’ вашСм распоряТСнии Π΅ΡΡ‚ΡŒ 15 ΠΎΠ΄Π½ΠΎΡ‚ΠΈΠΏΠ½Ρ‹Ρ… ΠΌΠΈΠ½ΠΈΠ°Ρ‚ΡŽΡ€Π½Ρ‹Ρ… Π»Π°ΠΌΠΏΠΎΡ‡Π΅ΠΊ, рассчитанных Π½Π° напряТСниС 13,5 Π’. МоТно Π»ΠΈ ΠΈΠ· Π½ΠΈΡ… ΡΠ΄Π΅Π»Π°Ρ‚ΡŒ Π΅Π»ΠΎΡ‡Π½ΡƒΡŽ гирлянду, ΠΏΠΎΠ΄ΠΊΠ»ΡŽΡ‡Π°Π΅ΠΌΡƒΡŽ ΠΊ ΠΎΠ±Ρ‹Ρ‡Π½ΠΎΠΉ Ρ€ΠΎΠ·Π΅Ρ‚ΠΊΠ΅, ΠΈ Ссли ΠΌΠΎΠΆΠ½ΠΎ, Ρ‚ΠΎ ΠΊΠ°ΠΊ?

БмСшанноС соСдинСниС

Π‘ ΠΏΠ°Ρ€Π°Π»Π»Π΅Π»ΡŒΠ½Ρ‹ΠΌ ΠΈ ΠΏΠΎΡΠ»Π΅Π΄ΠΎΠ²Π°Ρ‚Π΅Π»ΡŒΠ½Ρ‹ΠΌ соСдинСниСм ΠΏΡ€ΠΎΠ²ΠΎΠ΄Π½ΠΈΠΊΠΎΠ² Π²Ρ‹, ΠΊΠΎΠ½Π΅Ρ‡Π½ΠΎ, Π±Π΅Π· Ρ‚Ρ€ΡƒΠ΄Π° Ρ€Π°Π·ΠΎΠ±Ρ€Π°Π»ΠΈΡΡŒ. Но ΠΊΠ°ΠΊ Π±Ρ‹Ρ‚ΡŒ, Ссли ΠΏΠ΅Ρ€Π΅Π΄ Π²Π°ΠΌΠΈ оказалась ΠΏΡ€ΠΈΠΌΠ΅Ρ€Π½ΠΎ такая схСма?

БмСшанноС соСдинСниС ΠΏΡ€ΠΎΠ²ΠΎΠ΄Π½ΠΈΠΊΠΎΠ²

Как ΠΎΠΏΡ€Π΅Π΄Π΅Π»ΠΈΡ‚ΡŒ ΠΎΠ±Ρ‰Π΅Π΅ сопротивлСниС Ρ†Π΅ΠΏΠΈ? Для этого Π²Π°ΠΌ понадобится Ρ€Π°Π·Π±ΠΈΡ‚ΡŒ схСму Π½Π° нСсколько участков. Π’Ρ‹ΡˆΠ΅ΠΏΡ€ΠΈΠ²Π΅Π΄Π΅Π½Π½Π°Ρ конструкция достаточно проста ΠΈ участков Π±ΡƒΠ΄Π΅Ρ‚ Π΄Π²Π° — R1 ΠΈ R2,R3. Π‘Π½Π°Ρ‡Π°Π»Π° Π²Ρ‹ рассчитываСтС ΠΎΠ±Ρ‰Π΅Π΅ сопротивлСниС ΠΏΠ°Ρ€Π°Π»Π»Π΅Π»ΡŒΠ½ΠΎ соСдинСнных элСмСнтов R2,R3 ΠΈ Π½Π°Ρ…ΠΎΠ΄ΠΈΡ‚Π΅ RΠΎΠ±Ρ‰.23. Π—Π°Ρ‚Π΅ΠΌ вычисляСтС ΠΎΠ±Ρ‰Π΅Π΅ сопротивлСниС всСй Ρ†Π΅ΠΏΠΈ, состоящСй ΠΈΠ· R1 ΠΈ RΠΎΠ±Ρ‰.23, соСдинСнных ΠΏΠΎΡΠ»Π΅Π΄ΠΎΠ²Π°Ρ‚Π΅Π»ΡŒΠ½ΠΎ:

  • RΠΎΠ±Ρ‰.23 = (R2*R3) / (R2+R3).
  • RΡ†Π΅ΠΏΠΈ = R1 + RΠΎΠ±Ρ‰.23.

Π—Π°Π΄Π°Ρ‡Π° Ρ€Π΅ΡˆΠ΅Π½Π°, всС ΠΎΡ‡Π΅Π½ΡŒ просто. А Ρ‚Π΅ΠΏΠ΅Ρ€ΡŒ вопрос нСсколько слоТнСС.

Π‘Π»ΠΎΠΆΠ½ΠΎΠ΅ смСшанноС соСдинСниС сопротивлСний

Как Π±Ρ‹Ρ‚ΡŒ Ρ‚ΡƒΡ‚? Π’ΠΎΡ‡Π½ΠΎ Ρ‚Π°ΠΊ ΠΆΠ΅, просто Π½ΡƒΠΆΠ½ΠΎ ΠΏΡ€ΠΎΡΠ²ΠΈΡ‚ΡŒ Π½Π΅ΠΊΠΎΡ‚ΠΎΡ€ΡƒΡŽ Ρ„Π°Π½Ρ‚Π°Π·ΠΈΡŽ. РСзисторы R2, R4, R5 соСдинСны ΠΏΠΎΡΠ»Π΅Π΄ΠΎΠ²Π°Ρ‚Π΅Π»ΡŒΠ½ΠΎ. РассчитываСм ΠΈΡ… ΠΎΠ±Ρ‰Π΅Π΅ сопротивлСниС:

RΠΎΠ±Ρ‰.245 = R2+R4+R5.

Π’Π΅ΠΏΠ΅Ρ€ΡŒ ΠΏΠ°Ρ€Π°Π»Π»Π΅Π»ΡŒΠ½ΠΎ ΠΊ RΠΎΠ±Ρ‰.245 ΠΏΠΎΠ΄ΠΊΠ»ΡŽΡ‡Π°Π΅ΠΌ R3:

RΠΎΠ±Ρ‰.2345 = (R3* RΠΎΠ±Ρ‰.245) / (R3+ RΠΎΠ±Ρ‰.245).

RцСпи = R1+ Rобщ.2345+R6.

Π’ΠΎΡ‚ ΠΈ всС!

ΠžΡ‚Π²Π΅Ρ‚ Π½Π° Π·Π°Π΄Π°Ρ‡Ρƒ ΠΎ Π΅Π»ΠΎΡ‡Π½ΠΎΠΉ гирляндС

Π›Π°ΠΌΠΏΡ‹ ΠΈΠΌΠ΅ΡŽΡ‚ Ρ€Π°Π±ΠΎΡ‡Π΅Π΅ напряТСниС всСго 13.5 Π’, Π° Π² Ρ€ΠΎΠ·Π΅Ρ‚ΠΊΠ΅ 220 Π’, поэтому ΠΈΡ… Π½ΡƒΠΆΠ½ΠΎ Π²ΠΊΠ»ΡŽΡ‡Π°Ρ‚ΡŒ ΠΏΠΎΡΠ»Π΅Π΄ΠΎΠ²Π°Ρ‚Π΅Π»ΡŒΠ½ΠΎ.

ΠŸΠΎΡΠΊΠΎΠ»ΡŒΠΊΡƒ Π»Π°ΠΌΠΏΡ‹ ΠΎΠ΄Π½ΠΎΡ‚ΠΈΠΏΠ½Ρ‹Π΅, напряТСниС сСти раздСлится ΠΌΠ΅ΠΆΠ΄Ρƒ Π½ΠΈΠΌΠΈ ΠΏΠΎΡ€ΠΎΠ²Π½Ρƒ ΠΈ Π½Π° ΠΊΠ°ΠΆΠ΄ΠΎΠΉ Π»Π°ΠΌΠΏΠΎΡ‡ΠΊΠ΅ окаТСтся 220 / 15 = 14,6 Π’. Π›Π°ΠΌΠΏΡ‹ рассчитаны Π½Π° напряТСниС 13,5 Π’, поэтому такая гирлянда Ρ…ΠΎΡ‚ΡŒ ΠΈ Π·Π°Ρ€Π°Π±ΠΎΡ‚Π°Π΅Ρ‚, Π½ΠΎ ΠΎΡ‡Π΅Π½ΡŒ быстро ΠΏΠ΅Ρ€Π΅Π³ΠΎΡ€ΠΈΡ‚. Π§Ρ‚ΠΎΠ±Ρ‹ Ρ€Π΅Π°Π»ΠΈΠ·ΠΎΠ²Π°Ρ‚ΡŒ Π·Π°Π΄ΡƒΠΌΠΊΡƒ, Π²Π°ΠΌ понадобится ΠΌΠΈΠ½ΠΈΠΌΡƒΠΌ 220 / 13,5 = 17, Π° Π»ΡƒΡ‡ΡˆΠ΅ 18-19 Π»Π°ΠΌΠΏΠΎΡ‡Π΅ΠΊ.

ΠŸΠΎΠ΄Ρ€ΠΎΠ±Π½ΠΎΡΡ‚ΠΈ ΠšΠ°Ρ‚Π΅Π³ΠΎΡ€ΠΈΡ: Π‘Ρ‚Π°Ρ‚ΡŒΠΈ Π‘ΠΎΠ·Π΄Π°Π½ΠΎ: 06.09.2017 19:48

Как ΠΏΠΎΠ΄ΠΊΠ»ΡŽΡ‡ΠΈΡ‚ΡŒ Π² ΠΊΡƒΠΊΠΎΠ»ΡŒΠ½ΠΎΠΌ Π΄ΠΎΠΌΠΈΠΊΠ΅ нСсколько ΡΠ²Π΅Ρ‚ΠΈΠ»ΡŒΠ½ΠΈΠΊΠΎΠ²

Когда Π²Ρ‹ Π·Π°Π΄ΡƒΠΌΡ‹Π²Π°Π΅Ρ‚Π΅ΡΡŒ ΠΎ Ρ‚ΠΎΠΌ ΠΊΠ°ΠΊ ΡΠ΄Π΅Π»Π°Ρ‚ΡŒ освСщСниС Π² ΠΊΡƒΠΊΠΎΠ»ΡŒΠ½ΠΎΠΌ Π΄ΠΎΠΌΠΈΠΊΠ΅ ΠΈΠ»ΠΈ румбоксС, Π³Π΄Π΅ Π½Π΅ ΠΎΠ΄ΠΈΠ½, Π° нСсколько ΡΠ²Π΅Ρ‚ΠΈΠ»ΡŒΠ½ΠΈΠΊΠΎΠ², Ρ‚ΠΎ встаСт вопрос ΠΎ Ρ‚ΠΎΠΌ, ΠΊΠ°ΠΊ ΠΈΡ… ΠΏΠΎΠ΄ΠΊΠ»ΡŽΡ‡ΠΈΡ‚ΡŒ, ΠΎΠ±ΡŠΠ΅Π΄ΠΈΠ½ΠΈΡ‚ΡŒ Π² ΡΠ΅Ρ‚ΡŒ. БущСствуСт Π΄Π²Π° Ρ‚ΠΈΠΏΠ° ΠΏΠΎΠ΄ΠΊΠ»ΡŽΡ‡Π΅Π½ΠΈΡ: ΠΏΠΎΡΠ»Π΅Π΄ΠΎΠ²Π°Ρ‚Π΅Π»ΡŒΠ½ΠΎΠ΅ ΠΈ ΠΏΠ°Ρ€Π°Π»Π»Π΅Π»ΡŒΠ½ΠΎΠ΅, ΠΎ ΠΊΠΎΡ‚ΠΎΡ€Ρ‹Ρ… ΠΌΡ‹ ΡΠ»Ρ‹ΡˆΠ°Π»ΠΈ со школьной скамьи. Π˜Ρ… ΠΈ рассмотрим Π² этой ΡΡ‚Π°Ρ‚ΡŒΠ΅.

Π― ΠΏΠΎΡΡ‚Π°Ρ€Π°ΡŽΡΡŒ ΠΎΠΏΠΈΡΠ°Ρ‚ΡŒ всё простым доступным языком, Ρ‡Ρ‚ΠΎΠ±Ρ‹ всё Π±Ρ‹Π»ΠΎ понятно Π΄Π°ΠΆΠ΅ самым-самым гуманитариям, Π½Π΅ Π·Π½Π°ΠΊΠΎΠΌΡ‹ΠΌ с элСктричСскими прСмудростями.

ΠŸΡ€ΠΈΠΌΠ΅Ρ‡Π°Π½ΠΈΠ΅ : Π² этой ΡΡ‚Π°Ρ‚ΡŒΠ΅ рассмотрим Ρ‚ΠΎΠ»ΡŒΠΊΠΎ Ρ†Π΅ΠΏΡŒ с Π»Π°ΠΌΠΏΠΎΡ‡ΠΊΠ°ΠΌΠΈ накаливания. ΠžΡΠ²Π΅Ρ‰Π΅Π½ΠΈΠ΅ Π΄ΠΈΠΎΠ΄Π°ΠΌΠΈ Π±ΠΎΠ»Π΅Π΅ слоТноС ΠΈ Π±ΡƒΠ΄Π΅Ρ‚ рассмотрСно Π² Π΄Ρ€ΡƒΠ³ΠΎΠΉ ΡΡ‚Π°Ρ‚ΡŒΠ΅.

Для понимания каТдая схСма Π±ΡƒΠ΄Π΅Ρ‚ сопровоТдСна рисунком ΠΈ рядом с Ρ‡Π΅Ρ€Ρ‚Π΅ΠΆΠΎΠΌ элСктричСской ΠΌΠΎΠ½Ρ‚Π°ΠΆΠ½ΠΎΠΉ схСмой.
Π‘Π½Π°Ρ‡Π°Π»Π° рассмотрим условныС обозначСния Π½Π° элСктричСских схСмах.

НазваниС элСмСнта Π‘ΠΈΠΌΠ²ΠΎΠ» Π½Π° схСмС Π˜Π·ΠΎΠ±Ρ€Π°ΠΆΠ΅Π½ΠΈΠ΅
Π±Π°Ρ‚Π°Ρ€Π΅ΠΉΠΊΠ°/ элСмСнт питания
Π²Ρ‹ΠΊΠ»ΡŽΡ‡Π°Ρ‚Π΅Π»ΡŒ
ΠΏΡ€ΠΎΠ²ΠΎΠ΄
пСрСсСчСниС ΠΏΡ€ΠΎΠ²ΠΎΠ΄ΠΎΠ² (Π±Π΅Π· соСдинСния)
соСдинСниС ΠΏΡ€ΠΎΠ²ΠΎΠ΄ΠΎΠ² (ΠΏΠ°ΠΉΠΊΠΎΠΉ, скруткой)
лампа накаливания
нСисправная Π»Π°ΠΌΠΏΠ°
Π½Π΅Ρ€Π°Π±ΠΎΡ‚Π°ΡŽΡ‰Π°Ρ Π»Π°ΠΌΠΏΠ°
горящая Π»Π°ΠΌΠΏΠ°

Как ΡƒΠΆΠ΅ Π±Ρ‹Π»ΠΎ сказано, ΡΡƒΡ‰Π΅ΡΡ‚Π²ΡƒΡŽΡ‚ Π΄Π²Π° основных Ρ‚ΠΈΠΏΠ° ΠΏΠΎΠ΄ΠΊΠ»ΡŽΡ‡Π΅Π½ΠΈΡ: ΠΏΠΎΡΠ»Π΅Π΄ΠΎΠ²Π°Ρ‚Π΅Π»ΡŒΠ½ΠΎΠ΅ ΠΈ ΠΏΠ°Ρ€Π°Π»Π»Π΅Π»ΡŒΠ½ΠΎΠ΅. Π•ΡΡ‚ΡŒ Π΅Ρ‰Ρ‘ Ρ‚Ρ€Π΅Ρ‚ΡŒΠ΅, смСшанноС: ΠΏΠΎΡΠ»Π΅Π΄ΠΎΠ²Π°Ρ‚Π΅Π»ΡŒΠ½ΠΎ-ΠΏΠ°Ρ€Π°Π»Π»Π΅Π»ΡŒΠ½ΠΎΠ΅, ΠΎΠ±ΡŠΠ΅Π΄ΠΈΠ½ΡΡŽΡ‰Π΅Π΅ Ρ‚ΠΎ ΠΈ Π΄Ρ€ΡƒΠ³ΠΎΠ΅. НачнСм с ΠΏΠΎΡΠ»Π΅Π΄ΠΎΠ²Π°Ρ‚Π΅Π»ΡŒΠ½ΠΎΠ³ΠΎ, ΠΊΠ°ΠΊ Π±ΠΎΠ»Π΅Π΅ простого.

ΠŸΠΎΡΠ»Π΅Π΄ΠΎΠ²Π°Ρ‚Π΅Π»ΡŒΠ½ΠΎΠ΅ ΠΏΠΎΠ΄ΠΊΠ»ΡŽΡ‡Π΅Π½ΠΈΠ΅

Выглядит ΠΎΠ½ΠΎ Π²ΠΎΡ‚ Ρ‚Π°ΠΊ.

Π›Π°ΠΌΠΏΠΎΡ‡ΠΊΠΈ Ρ€Π°ΡΠΏΠΎΠ»Π°Π³Π°ΡŽΡ‚ΡΡ ΠΎΠ΄Π½Π° Π·Π° Π΄Ρ€ΡƒΠ³ΠΎΠΉ, ΠΊΠ°ΠΊ Π² Ρ…ΠΎΡ€ΠΎΠ²ΠΎΠ΄Π΅ Π΄Π΅Ρ€ΠΆΠ°ΡΡŒ Π·Π° Ρ€ΡƒΠΊΠΈ. По этому ΠΏΡ€ΠΈΠ½Ρ†ΠΈΠΏΡƒ Π±Ρ‹Π»ΠΈ сдСланы старыС совСтскиС гирлянды.

Достоинства — простота соСдинСния.
НСдостатки — Ссли ΠΏΠ΅Ρ€Π΅Π³ΠΎΡ€Π΅Π»Π° Ρ…ΠΎΡ‚ΡŒ ΠΎΠ΄Π½Π° Π»Π°ΠΌΠΏΠΎΡ‡ΠΊΠ°, Ρ‚ΠΎ Π½Π΅ Π±ΡƒΠ΄Π΅Ρ‚ Ρ€Π°Π±ΠΎΡ‚Π°Ρ‚ΡŒ вся Ρ†Π΅ΠΏΡŒ.

Надо Π±ΡƒΠ΄Π΅Ρ‚ ΠΏΠ΅Ρ€Π΅Π±ΠΈΡ€Π°Ρ‚ΡŒ, ΠΏΡ€ΠΎΠ²Π΅Ρ€ΡΡ‚ΡŒ ΠΊΠ°ΠΆΠ΄ΡƒΡŽ Π»Π°ΠΌΠΏΠΎΡ‡ΠΊΡƒ, Ρ‡Ρ‚ΠΎΠ±Ρ‹ Π½Π°ΠΉΡ‚ΠΈ Π½Π΅ΠΈΡΠΏΡ€Π°Π²Π½ΡƒΡŽ. Π­Ρ‚ΠΎ ΠΌΠΎΠΆΠ΅Ρ‚ Π±Ρ‹Ρ‚ΡŒ ΡƒΡ‚ΠΎΠΌΠΈΡ‚Π΅Π»ΡŒΠ½Ρ‹ΠΌ ΠΏΡ€ΠΈ большом количСствС Π»Π°ΠΌΠΏΠΎΡ‡Π΅ΠΊ. Π’Π°ΠΊ ΠΆΠ΅ Π»Π°ΠΌΠΏΠΎΡ‡ΠΊΠΈ Π΄ΠΎΠ»ΠΆΠ½Ρ‹ Π±Ρ‹Ρ‚ΡŒ ΠΎΠ΄Π½ΠΎΠ³ΠΎ Ρ‚ΠΈΠΏΠ°: напряТСниС, ΠΌΠΎΡ‰Π½ΠΎΡΡ‚ΡŒ.

ΠŸΡ€ΠΈ этом Ρ‚ΠΈΠΏΠ΅ ΠΏΠΎΠ΄ΠΊΠ»ΡŽΡ‡Π΅Π½ΠΈΡ напряТСния Π»Π°ΠΌΠΏΠΎΡ‡Π΅ΠΊ ΡΠΊΠ»Π°Π΄Ρ‹Π²Π°ΡŽΡ‚ΡΡ. НапряТСниС обозначаСтся Π±ΡƒΠΊΠ²ΠΎΠΉ U , измСряСтся Π² Π²ΠΎΠ»ΡŒΡ‚Π°Ρ… V . НапряТСниС источника питания Π΄ΠΎΠ»ΠΆΠ½ΠΎ Π±Ρ‹Ρ‚ΡŒ Ρ€Π°Π²Π½ΠΎ суммС напряТСний всСх Π»Π°ΠΌΠΏΠΎΡ‡Π΅ΠΊ Π² Ρ†Π΅ΠΏΠΈ.

ΠŸΡ€ΠΈΠΌΠ΅Ρ€ β„–1 : Π²Ρ‹ Ρ…ΠΎΡ‚ΠΈΡ‚Π΅ ΠΏΠΎΠ΄ΠΊΠ»ΡŽΡ‡ΠΈΡ‚ΡŒ Π² ΠΏΠΎΡΠ»Π΅Π΄ΠΎΠ²Π°Ρ‚Π΅Π»ΡŒΠ½ΡƒΡŽ Ρ†Π΅ΠΏΡŒ 3 Π»Π°ΠΌΠΏΠΎΡ‡ΠΊΠΈ напряТСниСм 1,5V. НапряТСниС источника питания, Π½Π΅ΠΎΠ±Ρ…ΠΎΠ΄ΠΈΠΌΠΎΠ΅ для Ρ€Π°Π±ΠΎΡ‚Ρ‹ Ρ‚Π°ΠΊΠΎΠΉ Ρ†Π΅ΠΏΠΈ 1,5+1,5+1,5=4,5V.

Π£ ΠΎΠ±Ρ‹Ρ‡Π½Ρ‹Ρ… ΠΏΠ°Π»ΡŒΡ‡ΠΈΠΊΠΎΠ²Ρ‹Ρ… Π±Π°Ρ‚Π°Ρ€Π΅Π΅ΠΊ напряТСниС 1,5V. Π§Ρ‚ΠΎΠ±Ρ‹ ΠΈΠ· Π½ΠΈΡ… ΠΏΠΎΠ»ΡƒΡ‡ΠΈΡ‚ΡŒ напряТСниС 4,5V ΠΈΡ… Ρ‚ΠΎΠΆΠ΅ Π½ΡƒΠΆΠ½ΠΎ ΡΠΎΠ΅Π΄ΠΈΠ½ΠΈΡ‚ΡŒ Π² ΠΏΠΎΡΠ»Π΅Π΄ΠΎΠ²Π°Ρ‚Π΅Π»ΡŒΠ½ΡƒΡŽ Ρ†Π΅ΠΏΡŒ, ΠΈΡ… напряТСния слоТатся.
ΠŸΠΎΠ΄Ρ€ΠΎΠ±Π½Π΅Π΅ ΠΎ Ρ‚ΠΎΠΌ, ΠΊΠ°ΠΊ Π²Ρ‹Π±Ρ€Π°Ρ‚ΡŒ источник питания написано Π² этой ΡΡ‚Π°Ρ‚ΡŒΠ΅

ΠŸΡ€ΠΈΠΌΠ΅Ρ€ β„–2: Π²Ρ‹ Ρ…ΠΎΡ‚ΠΈΡ‚Π΅ ΠΏΠΎΠ΄ΠΊΠ»ΡŽΡ‡ΠΈΡ‚ΡŒ ΠΊ источнику питания 12V Π»Π°ΠΌΠΏΠΎΡ‡ΠΊΠΈ ΠΏΠΎ 6V. 6+6=12v. МоТно ΠΏΠΎΠ΄ΠΊΠ»ΡŽΡ‡ΠΈΡ‚ΡŒ 2 Ρ‚Π°ΠΊΠΈΡ… Π»Π°ΠΌΠΏΠΎΡ‡ΠΊΠΈ.

ΠŸΡ€ΠΈΠΌΠ΅Ρ€ β„–3: Π²Ρ‹ Ρ…ΠΎΡ‚ΠΈΡ‚Π΅ ΡΠΎΠ΅Π΄ΠΈΠ½ΠΈΡ‚ΡŒ Π² Ρ†Π΅ΠΏΡŒ 2 Π»Π°ΠΌΠΏΠΎΡ‡ΠΊΠΈ ΠΏΠΎ 3V. 3+3=6V. НСобходим источник питания Π½Π° 6 V.

ПодвСдСм ΠΈΡ‚ΠΎΠ³: ΠΏΠΎΡΠ»Π΅Π΄ΠΎΠ²Π°Ρ‚Π΅Π»ΡŒΠ½ΠΎΠ΅ ΠΏΠΎΠ΄ΠΊΠ»ΡŽΡ‡Π΅Π½ΠΈΠ΅ просто Π² ΠΈΠ·Π³ΠΎΡ‚ΠΎΠ²Π»Π΅Π½ΠΈΠΈ, Π½ΡƒΠΆΠ½Ρ‹ Π»Π°ΠΌΠΏΠΎΡ‡ΠΊΠΈ ΠΎΠ΄Π½ΠΎΠ³ΠΎ Ρ‚ΠΈΠΏΠ°. НСдостатки: ΠΏΡ€ΠΈ Π²Ρ‹Ρ…ΠΎΠ΄Π΅ ΠΈΠ· строя ΠΎΠ΄Π½ΠΎΠΉ Π»Π°ΠΌΠΏΠΎΡ‡ΠΊΠΈ Π½Π΅ горят всС. Π’ΠΊΠ»ΡŽΡ‡ΠΈΡ‚ΡŒ ΠΈ Π²Ρ‹ΠΊΠ»ΡŽΡ‡ΠΈΡ‚ΡŒ Ρ†Π΅ΠΏΡŒ ΠΌΠΎΠΆΠ½ΠΎ Ρ‚ΠΎΠ»ΡŒΠΊΠΎ Ρ†Π΅Π»ΠΈΠΊΠΎΠΌ.

Π˜ΡΡ…ΠΎΠ΄Ρ ΠΈΠ· этого, для освСщСния ΠΊΡƒΠΊΠΎΠ»ΡŒΠ½ΠΎΠ³ΠΎ Π΄ΠΎΠΌΠΈΠΊΠ° цСлСсообразно ΡΠΎΠ΅Π΄ΠΈΠ½ΡΡ‚ΡŒ ΠΏΠΎΡΠ»Π΅Π΄ΠΎΠ²Π°Ρ‚Π΅Π»ΡŒΠ½ΠΎ Π½Π΅ Π±ΠΎΠ»Π΅Π΅ 2-3 Π»Π°ΠΌΠΏΠΎΡ‡Π΅ΠΊ. НапримСр, Π² Π±Ρ€Π°. Π§Ρ‚ΠΎΠ±Ρ‹ ΡΠΎΠ΅Π΄ΠΈΠ½ΠΈΡ‚ΡŒ большСС количСство Π»Π°ΠΌΠΏΠΎΡ‡Π΅ΠΊ, Π½Π΅ΠΎΠ±Ρ…ΠΎΠ΄ΠΈΠΌΠΎ ΠΈΡΠΏΠΎΠ»ΡŒΠ·ΠΎΠ²Π°Ρ‚ΡŒ Π΄Ρ€ΡƒΠ³ΠΎΠΉ Ρ‚ΠΈΠΏ ΠΏΠΎΠ΄ΠΊΠ»ΡŽΡ‡Π΅Π½ΠΈΡ — ΠΏΠ°Ρ€Π°Π»Π»Π΅Π»ΡŒΠ½ΠΎΠ΅.

Π§ΠΈΡ‚Π°ΠΉΡ‚Π΅ Ρ‚Π°ΠΊ ΠΆΠ΅ ΡΡ‚Π°Ρ‚ΡŒΠΈ ΠΏΠΎ Ρ‚Π΅ΠΌΠ΅:

  • ΠžΠ±Π·ΠΎΡ€ ΠΌΠΈΠ½ΠΈΠ°Ρ‚ΡŽΡ€Π½Ρ‹Ρ… Π»Π°ΠΌΠΏ накаливания
  • Π”ΠΈΠΎΠ΄Ρ‹ ΠΈΠ»ΠΈ Π»Π°ΠΌΠΏΡ‹ накаливания

ΠŸΠ°Ρ€Π°Π»Π»Π΅Π»ΡŒΠ½ΠΎΠ΅ ΠΏΠΎΠ΄ΠΊΠ»ΡŽΡ‡Π΅Π½ΠΈΠ΅ Π»Π°ΠΌΠΏΠΎΡ‡Π΅ΠΊ

Π’ΠΎΡ‚ Ρ‚Π°ΠΊ выглядит ΠΏΠ°Ρ€Π°Π»Π»Π΅Π»ΡŒΠ½ΠΎΠ΅ ΠΏΠΎΠ΄ΠΊΠ»ΡŽΡ‡Π΅Π½ΠΈΠ΅ Π»Π°ΠΌΠΏΠΎΡ‡Π΅ΠΊ.

Π’ этом Ρ‚ΠΈΠΏΠ΅ ΠΏΠΎΠ΄ΠΊΠ»ΡŽΡ‡Π΅Π½ΠΈΡ Ρƒ всСх Π»Π°ΠΌΠΏΠΎΡ‡Π΅ΠΊ ΠΈ источника питания ΠΎΠ΄ΠΈΠ½Π°ΠΊΠΎΠ²Ρ‹Π΅ напряТСния. Π’ΠΎ Π΅ΡΡ‚ΡŒ ΠΏΡ€ΠΈ источникС питания 12v каТдая ΠΈΠ· Π»Π°ΠΌΠΏΠΎΡ‡Π΅ΠΊ Π΄ΠΎΠ»ΠΆΠ½Π° ΠΈΠΌΠ΅Ρ‚ΡŒ Ρ‚ΠΎΠΆΠ΅ напряТСниС 12V. А количСство Π»Π°ΠΌΠΏΠΎΡ‡Π΅ΠΊ ΠΌΠΎΠΆΠ΅Ρ‚ Π±Ρ‹Ρ‚ΡŒ Ρ€Π°Π·Π»ΠΈΡ‡Π½Ρ‹ΠΌ. А Ссли Ρƒ вас, допустим, Π΅ΡΡ‚ΡŒ Π»Π°ΠΌΠΏΠΎΡ‡ΠΊΠΈ 6V, Ρ‚ΠΎ ΠΈ источник питания Π½ΡƒΠΆΠ½ΠΎ Π±Ρ€Π°Ρ‚ΡŒ 6V.

ΠŸΡ€ΠΈ Π²Ρ‹Ρ…ΠΎΠ΄Π΅ ΠΈΠ· строя ΠΎΠ΄Π½ΠΎΠΉ Π»Π°ΠΌΠΏΠΎΡ‡ΠΊΠΈ Π΄Ρ€ΡƒΠ³ΠΈΠ΅ ΠΏΡ€ΠΎΠ΄ΠΎΠ»ΠΆΠ°ΡŽΡ‚ Π³ΠΎΡ€Π΅Ρ‚ΡŒ.

Π›Π°ΠΌΠΏΠΎΡ‡ΠΊΠΈ ΠΌΠΎΠΆΠ½ΠΎ Π²ΠΊΠ»ΡŽΡ‡Π°Ρ‚ΡŒ нСзависимо Π΄Ρ€ΡƒΠ³ ΠΎΡ‚ Π΄Ρ€ΡƒΠ³Π°. Для этого ΠΊ ΠΊΠ°ΠΆΠ΄ΠΎΠΉ Π½ΡƒΠΆΠ½ΠΎ ΠΏΠΎΡΡ‚Π°Π²ΠΈΡ‚ΡŒ свой Π²Ρ‹ΠΊΠ»ΡŽΡ‡Π°Ρ‚Π΅Π»ΡŒ.

По этому ΠΏΡ€ΠΈΠ½Ρ†ΠΈΠΏΡƒ ΠΏΠΎΠ΄ΠΊΠ»ΡŽΡ‡Π΅Π½Ρ‹ элСктроприборы Π² Π½Π°ΡˆΠΈΡ… городских ΠΊΠ²Π°Ρ€Ρ‚ΠΈΡ€Π°Ρ…. Π£ всСх ΠΏΡ€ΠΈΠ±ΠΎΡ€ΠΎΠ² ΠΎΠ΄Π½ΠΎ напряТСниС 220V, Π²ΠΊΠ»ΡŽΡ‡Π°Ρ‚ΡŒ ΠΈ Π²Ρ‹ΠΊΠ»ΡŽΡ‡Π°Ρ‚ΡŒ ΠΈΡ… ΠΌΠΎΠΆΠ½ΠΎ нСзависимо Π΄Ρ€ΡƒΠ³ ΠΎΡ‚ Π΄Ρ€ΡƒΠ³Π°, ΠΌΠΎΡ‰Π½ΠΎΡΡ‚ΡŒ элСктроприборов ΠΌΠΎΠΆΠ΅Ρ‚ Π±Ρ‹Ρ‚ΡŒ Ρ€Π°Π·Π½ΠΎΠΉ.

Π’Ρ‹Π²ΠΎΠ΄ : ΠΏΡ€ΠΈ мноТСствС ΡΠ²Π΅Ρ‚ΠΈΠ»ΡŒΠ½ΠΈΠΊΠΎΠ² Π² ΠΊΡƒΠΊΠΎΠ»ΡŒΠ½ΠΎΠΌ Π΄ΠΎΠΌΠΈΠΊΠ΅ ΠΎΠΏΡ‚ΠΈΠΌΠ°Π»ΡŒΠ½ΠΎ ΠΏΠ°Ρ€Π°Π»Π»Π΅Π»ΡŒΠ½ΠΎΠ΅ ΠΏΠΎΠ΄ΠΊΠ»ΡŽΡ‡Π΅Π½ΠΈΠ΅, хотя ΠΎΠ½ΠΎ Ρ‡ΡƒΡ‚ΡŒ слоТнСС, Ρ‡Π΅ΠΌ ΠΏΠΎΡΠ»Π΅Π΄ΠΎΠ²Π°Ρ‚Π΅Π»ΡŒΠ½ΠΎΠ΅.

Рассмотрим Π΅Ρ‰Ρ‘ ΠΎΠ΄ΠΈΠ½ Π²ΠΈΠ΄ ΠΏΠΎΠ΄ΠΊΠ»ΡŽΡ‡Π΅Π½ΠΈΡ, ΡΠΎΠ΅Π΄ΠΈΠ½ΡΡŽΡ‰ΠΈΠΉ Π² сСбС ΠΏΠΎΡΠ»Π΅Π΄ΠΎΠ²Π°Ρ‚Π΅Π»ΡŒΠ½ΠΎΠ΅ ΠΈ ΠΏΠ°Ρ€Π°Π»Π»Π΅Π»ΡŒΠ½ΠΎΠ΅.

ΠšΠΎΠΌΠ±ΠΈΠ½ΠΈΡ€ΠΎΠ²Π°Π½Π½ΠΎΠ΅ ΠΏΠΎΠ΄ΠΊΠ»ΡŽΡ‡Π΅Π½ΠΈΠ΅

ΠŸΡ€ΠΈΠΌΠ΅Ρ€ ΠΊΠΎΠΌΠ±ΠΈΠ½ΠΈΡ€ΠΎΠ²Π°Π½Π½ΠΎΠ³ΠΎ ΠΏΠΎΠ΄ΠΊΠ»ΡŽΡ‡Π΅Π½ΠΈΡ.

Π’Ρ€ΠΈ ΠΏΠΎΡΠ»Π΅Π΄ΠΎΠ²Π°Ρ‚Π΅Π»ΡŒΠ½Ρ‹Π΅ Ρ†Π΅ΠΏΠΈ, соСдинСнныС ΠΏΠ°Ρ€Π°Π»Π»Π΅Π»ΡŒΠ½ΠΎ

А Π²ΠΎΡ‚ Π΄Ρ€ΡƒΠ³ΠΎΠΉ Π²Π°Ρ€ΠΈΠ°Π½Ρ‚:

Π’Ρ€ΠΈ ΠΏΠ°Ρ€Π°Π»Π»Π΅Π»ΡŒΠ½Ρ‹Π΅ Ρ†Π΅ΠΏΠΈ, соСдинСнныС ΠΏΠΎΡΠ»Π΅Π΄ΠΎΠ²Π°Ρ‚Π΅Π»ΡŒΠ½ΠΎ.

Участки Ρ‚Π°ΠΊΠΎΠΉ Ρ†Π΅ΠΏΠΈ, соСдинСнныС ΠΏΠΎΡΠ»Π΅Π΄ΠΎΠ²Π°Ρ‚Π΅Π»ΡŒΠ½ΠΎ, Π²Π΅Π΄ΡƒΡ‚ сСбя ΠΊΠ°ΠΊ ΠΏΠΎΡΠ»Π΅Π΄ΠΎΠ²Π°Ρ‚Π΅Π»ΡŒΠ½ΠΎΠ΅ соСдинСниС. А ΠΏΠ°Ρ€Π°Π»Π»Π΅Π»ΡŒΠ½Ρ‹Π΅ участки — ΠΊΠ°ΠΊ ΠΏΠ°Ρ€Π°Π»Π»Π΅Π»ΡŒΠ½ΠΎΠ΅ соСдинСниС.

ΠŸΡ€ΠΈΠΌΠ΅Ρ€

ΠŸΡ€ΠΈ Ρ‚Π°ΠΊΠΎΠΉ схСмС ΠΏΠ΅Ρ€Π΅Π³ΠΎΡ€Π°Π½ΠΈΠ΅ ΠΎΠ΄Π½ΠΎΠΉ Π»Π°ΠΌΠΏΠΎΡ‡ΠΊΠΈ Π²Ρ‹Π²Π΅Π΄Π΅Ρ‚ ΠΈΠ· строя вСсь участок, соСдинСнный ΠΏΠΎΡΠ»Π΅Π΄ΠΎΠ²Π°Ρ‚Π΅Π»ΡŒΠ½ΠΎ, Π° Π΄Π²Π΅ Π΄Ρ€ΡƒΠ³ΠΈΠ΅ ΠΏΠΎΡΠ»Π΅Π΄ΠΎΠ²Π°Ρ‚Π΅Π»ΡŒΠ½Ρ‹Π΅ цСписохранят Ρ€Π°Π±ΠΎΡ‚ΠΎΡΠΏΠΎΡΠΎΠ±Π½ΠΎΡΡ‚ΡŒ.

БоотвСтствСнно, ΠΈ Π²ΠΊΠ»ΡŽΡ‡Π°Ρ‚ΡŒ-Π²Ρ‹ΠΊΠ»ΡŽΡ‡Π°Ρ‚ΡŒ участки ΠΌΠΎΠΆΠ½ΠΎ нСзависимо Π΄Ρ€ΡƒΠ³ ΠΎΡ‚ Π΄Ρ€ΡƒΠ³Π°. Для этого ΠΊΠ°ΠΆΠ΄ΠΎΠΉ ΠΏΠΎΡΠ»Π΅Π΄ΠΎΠ²Π°Ρ‚Π΅Π»ΡŒΠ½ΠΎΠΉ Ρ†Π΅ΠΏΠΈ Π½ΡƒΠΆΠ½ΠΎ ΠΏΠΎΡΡ‚Π°Π²ΠΈΡ‚ΡŒ свой Π²Ρ‹ΠΊΠ»ΡŽΡ‡Π°Ρ‚Π΅Π»ΡŒ.

Но нСльзя Π²ΠΊΠ»ΡŽΡ‡ΠΈΡ‚ΡŒ ΠΎΠ΄Π½Ρƒ-Π΅Π΄ΠΈΠ½ΡΡ‚Π²Π΅Π½Π½ΡƒΡŽ Π»Π°ΠΌΠΏΠΎΡ‡ΠΊΡƒ.

ΠŸΡ€ΠΈ ΠΏΠ°Ρ€Π°Π»Π»Π΅Π»ΡŒΠ½ΠΎ-ΠΏΠΎΡΠ»Π΅Π΄ΠΎΠ²Π°Ρ‚Π΅Π»ΡŒΠ½ΠΎΠΌ ΠΏΠΎΠ΄ΠΊΠ»ΡŽΡ‡Π΅Π½ΠΈΠΈ ΠΏΡ€ΠΈ Π²Ρ‹Ρ…ΠΎΠ΄Π΅ ΠΈΠ· строя ΠΎΠ΄Π½ΠΎΠΉ Π»Π°ΠΌΠΏΠΎΡ‡ΠΊΠΈ Ρ†Π΅ΠΏΡŒ Π±ΡƒΠ΄Π΅Ρ‚ вСсти сСбя Ρ‚Π°ΠΊ:

А ΠΏΡ€ΠΈ Π½Π°Ρ€ΡƒΡˆΠ΅Π½ΠΈΠΈ Π½Π° ΠΏΠΎΡΠ»Π΅Π΄ΠΎΠ²Π°Ρ‚Π΅Π»ΡŒΠ½ΠΎΠΌ участкС Π²ΠΎΡ‚ Ρ‚Π°ΠΊ:

ΠŸΡ€ΠΈΠΌΠ΅Ρ€:

Π•ΡΡ‚ΡŒ 6 Π»Π°ΠΌΠΏΠΎΡ‡Π΅ΠΊ ΠΏΠΎ 3V, соСдинСнныС Π² 3 ΠΏΠΎΡΠ»Π΅Π΄ΠΎΠ²Π°Ρ‚Π΅Π»ΡŒΠ½Ρ‹Π΅ Ρ†Π΅ΠΏΠΈ ΠΏΠΎ 2 Π»Π°ΠΌΠΏΠΎΡ‡ΠΊΠΈ. Π¦Π΅ΠΏΠΈ Π² свою ΠΎΡ‡Π΅Ρ€Π΅Π΄ΡŒ соСдинСны ΠΏΠ°Ρ€Π°Π»Π»Π΅Π»ΡŒΠ½ΠΎ. Π Π°Π·Π±ΠΈΠ²Π°Π΅ΠΌ Π½Π° 3 ΠΏΠΎΡΠ»Π΅Π΄ΠΎΠ²Π°Ρ‚Π΅Π»ΡŒΠ½Ρ‹Ρ… участка ΠΈ просчитываСм этот участок.

На ΠΏΠΎΡΠ»Π΅Π΄ΠΎΠ²Π°Ρ‚Π΅Π»ΡŒΠ½ΠΎΠΌ участкС напряТСния Π»Π°ΠΌΠΏΠΎΡ‡Π΅ΠΊ ΡΠΊΠ»Π°Π΄Ρ‹Π²Π°ΡŽΡ‚ΡΡ, 3v+3V=6V. Π£ ΠΊΠ°ΠΆΠ΄ΠΎΠΉ ΠΏΠΎΡΠ»Π΅Π΄ΠΎΠ²Π°Ρ‚Π΅Π»ΡŒΠ½ΠΎΠΉ Ρ†Π΅ΠΏΠΈ напряТСниС 6V. ΠŸΠΎΡΠΊΠΎΠ»ΡŒΠΊΡƒ Ρ†Π΅ΠΏΠΈ соСдинСны ΠΏΠ°Ρ€Π°Π»Π»Π΅Π»ΡŒΠ½ΠΎ, Ρ‚ΠΎ ΠΈΡ… напряТСниС Π½Π΅ складываСтся, Π° Π·Π½Π°Ρ‡ΠΈΡ‚ Π½Π°ΠΌ Π½ΡƒΠΆΠ΅Π½ источник питания Π½Π° 6V.

ΠŸΡ€ΠΈΠΌΠ΅Ρ€

Π£ нас 6 Π»Π°ΠΌΠΏΠΎΡ‡Π΅ΠΊ ΠΏΠΎ 6V. Π›Π°ΠΌΠΏΠΎΡ‡ΠΊΠΈ соСдинСны ΠΏΠΎ 3 ΡˆΡ‚ΡƒΠΊΠΈ Π² ΠΏΠ°Ρ€Π°Π»Π»Π΅Π»ΡŒΠ½ΡƒΡŽ Ρ†Π΅ΠΏΡŒ, Π° Ρ†Π΅ΠΏΠΈ Π² свою ΠΎΡ‡Π΅Ρ€Π΅Π΄ΡŒ — ΠΏΠΎΡΠ»Π΅Π΄ΠΎΠ²Π°Ρ‚Π΅Π»ΡŒΠ½ΠΎ. Π Π°Π·Π±ΠΈΠ²Π°Π΅ΠΌ систСму Π½Π° Ρ‚Ρ€ΠΈ ΠΏΠ°Ρ€Π°Π»Π»Π΅Π»ΡŒΠ½Ρ‹Ρ… Ρ†Π΅ΠΏΠΈ.

Π’ ΠΎΠ΄Π½ΠΎΠΉ ΠΏΠ°Ρ€Π°Π»Π»Π΅Π»ΡŒΠ½ΠΎΠΉ Ρ†Π΅ΠΏΠΈ напряТСниС Ρƒ ΠΊΠ°ΠΆΠ΄ΠΎΠΉ Π»Π°ΠΌΠΏΠΎΡ‡ΠΊΠΈ 6V, ΠΏΠΎΡΠΊΠΎΠ»ΡŒΠΊΡƒ напряТСниС Π½Π΅ складываСтся, Ρ‚ΠΎ ΠΈ Ρƒ всСй Ρ†Π΅ΠΏΠΈ напряТСниС 6V. А сами Ρ†Π΅ΠΏΠΈ соСдинСны ΡƒΠΆΠ΅ ΠΏΠΎΡΠ»Π΅Π΄ΠΎΠ²Π°Ρ‚Π΅Π»ΡŒΠ½ΠΎ ΠΈ ΠΈΡ… напряТСния ΡƒΠΆΠ΅ ΡΠΊΠ»Π°Π΄Ρ‹Π²Π°ΡŽΡ‚ΡΡ. ΠŸΠΎΠ»ΡƒΡ‡Π°Π΅Ρ‚ΡΡ 6V+6V=12V. Π—Π½Π°Ρ‡ΠΈΡ‚, Π½ΡƒΠΆΠ΅Π½ источник питания 12V.

ΠŸΡ€ΠΈΠΌΠ΅Ρ€

Для ΠΊΡƒΠΊΠΎΠ»ΡŒΠ½Ρ‹Ρ… Π΄ΠΎΠΌΠΈΠΊΠΎΠ² ΠΌΠΎΠΆΠ½ΠΎ ΠΈΡΠΏΠΎΠ»ΡŒΠ·ΠΎΠ²Π°Ρ‚ΡŒ Ρ‚Π°ΠΊΠΎΠ΅ смСшанноС ΠΏΠΎΠ΄ΠΊΠ»ΡŽΡ‡Π΅Π½ΠΈΠ΅.

Допустим, Π² ΠΊΠ°ΠΆΠ΄ΠΎΠΉ ΠΊΠΎΠΌΠ½Π°Ρ‚Π΅ ΠΏΠΎ ΠΎΠ΄Π½ΠΎΠΌΡƒ ΡΠ²Π΅Ρ‚ΠΈΠ»ΡŒΠ½ΠΈΠΊΡƒ, всС ΡΠ²Π΅Ρ‚ΠΈΠ»ΡŒΠ½ΠΈΠΊΠΈ ΠΏΠΎΠ΄ΠΊΠ»ΡŽΡ‡Π΅Π½Ρ‹ ΠΏΠ°Ρ€Π°Π»Π»Π΅Π»ΡŒΠ½ΠΎ. Но Π² самих ΡΠ²Π΅Ρ‚ΠΈΠ»ΡŒΠ½ΠΈΠΊΠ°Ρ… Ρ€Π°Π·Π½ΠΎΠ΅ количСство Π»Π°ΠΌΠΏΠΎΡ‡Π΅ΠΊ: Π² Π΄Π²ΡƒΡ… — ΠΏΠΎ ΠΎΠ΄Π½ΠΎΠΉ Π»Π°ΠΌΠΏΠΎΡ‡ΠΊΠ΅, Π΅ΡΡ‚ΡŒ Π΄Π²ΡƒΡ…Ρ€ΠΎΠΆΠΊΠΎΠ²ΠΎΠ΅ Π±Ρ€Π° ΠΈΠ· Π΄Π²ΡƒΡ… Π»Π°ΠΌΠΏΠΎΡ‡Π΅ΠΊ ΠΈ трСхроТковая Π»ΡŽΡΡ‚Ρ€Π°. Π’ Π»ΡŽΡΡ‚Ρ€Π΅ ΠΈ Π±Ρ€Π° Π»Π°ΠΌΠΏΠΎΡ‡ΠΊΠΈ соСдинСны ΠΏΠΎΡΠ»Π΅Π΄ΠΎΠ²Π°Ρ‚Π΅Π»ΡŒΠ½ΠΎ.

Π£ ΠΊΠ°ΠΆΠ΄ΠΎΠ³ΠΎ ΡΠ²Π΅Ρ‚ΠΈΠ»ΡŒΠ½ΠΈΠΊΠ° свой Π²Ρ‹ΠΊΠ»ΡŽΡ‡Π°Ρ‚Π΅Π»ΡŒ. Π˜ΡΡ‚ΠΎΡ‡Π½ΠΈΠΊ питания 12V напряТСния. ΠžΠ΄ΠΈΠ½ΠΎΡ‡Π½Ρ‹Π΅ Π»Π°ΠΌΠΏΠΎΡ‡ΠΊΠΈ, соСдинСнныС ΠΏΠ°Ρ€Π°Π»Π»Π΅Π»ΡŒΠ½ΠΎ, Π΄ΠΎΠ»ΠΆΠ½Ρ‹ ΠΈΠΌΠ΅Ρ‚ΡŒ напряТСниС 12V. А Ρƒ Ρ‚Π΅Ρ…, Ρ‡Ρ‚ΠΎ соСдинСны ΠΏΠΎΡΠ»Π΅Π΄ΠΎΠ²Π°Ρ‚Π΅Π»ΡŒΠ½ΠΎ напряТСниС складываСтся Π½Π° участкС Ρ†Π΅ΠΏΠΈ
. БоотвСтствСнно, для участка Π±Ρ€Π° ΠΈΠ· Π΄Π²ΡƒΡ… Π»Π°ΠΌΠΏΠΎΡ‡Π΅ΠΊ 12V (ΠΎΠ±Ρ‰Π΅Π΅ напряТСниС)Π΄Π΅Π»ΠΈΠΌ Π½Π° 2 (количСство Π»Π°ΠΌΠΏΠΎΡ‡Π΅ΠΊ), ΠΏΠΎΠ»ΡƒΡ‡ΠΈΠΌ 6V (напряТСниС ΠΎΠ΄Π½ΠΎΠΉ Π»Π°ΠΌΠΏΠΎΡ‡ΠΊΠΈ).
Для участка Π»ΡŽΡΡ‚Ρ€Ρ‹ 12V:3=4V (напряТСниС ΠΎΠ΄Π½ΠΎΠΉ Π»Π°ΠΌΠΏΠΎΡ‡ΠΊΠΈ Π»ΡŽΡΡ‚Ρ€Ρ‹).
Π‘ΠΎΠ»ΡŒΡˆΠ΅ Ρ‚Ρ€Π΅Ρ… Π»Π°ΠΌΠΏΠΎΡ‡Π΅ΠΊ Π² ΠΎΠ΄Π½ΠΎΠΌ ΡΠ²Π΅Ρ‚ΠΈΠ»ΡŒΠ½ΠΈΠΊΠ΅ ΡΠΎΠ΅Π΄ΠΈΠ½ΡΡ‚ΡŒ ΠΏΠΎΡΠ»Π΅Π΄ΠΎΠ²Π°Ρ‚Π΅Π»ΡŒΠ½ΠΎ Π½Π΅ стоит.

Π’Π΅ΠΏΠ΅Ρ€ΡŒ Π²Ρ‹ ΠΈΠ·ΡƒΡ‡ΠΈΠ»ΠΈ всС хитрости ΠΏΠΎΠ΄ΠΊΠ»ΡŽΡ‡Π΅Π½ΠΈΡ Π»Π°ΠΌΠΏΠΎΡ‡Π΅ΠΊ накаливания Ρ€Π°Π·Π½Ρ‹ΠΌΠΈ способами. И, Π΄ΡƒΠΌΠ°ΡŽ, Ρ‡Ρ‚ΠΎ Π½Π΅ составит Ρ‚Ρ€ΡƒΠ΄Π° ΡΠ΄Π΅Π»Π°Ρ‚ΡŒ освСщСниС Π² ΠΊΡƒΠΊΠΎΠ»ΡŒΠ½ΠΎΠΌ Π΄ΠΎΠΌΠΈΠΊΠ΅ со ΠΌΠ½ΠΎΠ³ΠΈΠΌΠΈ Π»Π°ΠΌΠΏΠΎΡ‡ΠΊΠ°ΠΌΠΈ, любой слоТности. Если ΠΆΠ΅ Ρ‡Ρ‚ΠΎ-Ρ‚ΠΎ для вас Π΅Ρ‰Ρ‘ прСдставляСт слоТности, ΠΏΡ€ΠΎΡ‡ΠΈΡ‚Π°ΠΉΡ‚Π΅ ΡΡ‚Π°Ρ‚ΡŒΡŽ ΠΎ ΠΏΡ€ΠΎΡΡ‚Π΅ΠΉΡˆΠ΅ΠΌ способС ΡΠ΄Π΅Π»Π°Ρ‚ΡŒ свСт Π² ΠΊΡƒΠΊΠΎΠ»ΡŒΠ½ΠΎΠΌ Π΄ΠΎΠΌΠΈΠΊΠ΅, самыС Π±Π°Π·ΠΎΠ²Ρ‹Π΅ ΠΏΡ€ΠΈΠ½Ρ†ΠΈΠΏΡ‹. Π£Π΄Π°Ρ‡ΠΈ!

ΠŸΡ€ΠΈΡ‡Π΅ΠΌ это ΠΌΠΎΠ³ΡƒΡ‚ Π±Ρ‹Ρ‚ΡŒ Π½Π΅ Ρ‚ΠΎΠ»ΡŒΠΊΠΎ ΠΏΡ€ΠΎΠ²ΠΎΠ΄Π½ΠΈΠΊΠΈ, Π½ΠΎ ΠΈ кондСнсаторы. Π—Π΄Π΅ΡΡŒ Π²Π°ΠΆΠ½ΠΎ Π½Π΅ Π·Π°ΠΏΡƒΡ‚Π°Ρ‚ΡŒΡΡ Π² Ρ‚ΠΎΠΌ, ΠΊΠ°ΠΊ выглядит ΠΊΠ°ΠΆΠ΄ΠΎΠ΅ ΠΈΠ· Π½ΠΈΡ… Π½Π° схСмС. А ΡƒΠΆΠ΅ ΠΏΠΎΡ‚ΠΎΠΌ ΠΏΡ€ΠΈΠΌΠ΅Π½ΡΡ‚ΡŒ ΠΊΠΎΠ½ΠΊΡ€Π΅Ρ‚Π½Ρ‹Π΅ Ρ„ΠΎΡ€ΠΌΡƒΠ»Ρ‹. Π˜Ρ…, кстати, Π½ΡƒΠΆΠ½ΠΎ ΠΏΠΎΠΌΠ½ΠΈΡ‚ΡŒ Π½Π°ΠΈΠ·ΡƒΡΡ‚ΡŒ.

Как Ρ€Π°Π·Π»ΠΈΡ‡ΠΈΡ‚ΡŒ эти Π΄Π²Π° соСдинСния?

Π’Π½ΠΈΠΌΠ°Ρ‚Π΅Π»ΡŒΠ½ΠΎ посмотритС Π½Π° схСму. Если ΠΏΡ€ΠΎΠ²ΠΎΠ΄Π° ΠΏΡ€Π΅Π΄ΡΡ‚Π°Π²ΠΈΡ‚ΡŒ ΠΊΠ°ΠΊ Π΄ΠΎΡ€ΠΎΠ³Ρƒ, Ρ‚ΠΎ ΠΌΠ°ΡˆΠΈΠ½Ρ‹ Π½Π° Π½Π΅ΠΉ Π±ΡƒΠ΄ΡƒΡ‚ ΠΈΠ³Ρ€Π°Ρ‚ΡŒ Ρ€ΠΎΠ»ΡŒ рСзисторов. На прямой Π΄ΠΎΡ€ΠΎΠ³Π΅ Π±Π΅Π· ΠΊΠ°ΠΊΠΈΡ…-Π»ΠΈΠ±ΠΎ Ρ€Π°Π·Π²Π΅Ρ‚Π²Π»Π΅Π½ΠΈΠΉ ΠΌΠ°ΡˆΠΈΠ½Ρ‹ Π΅Π΄ΡƒΡ‚ ΠΎΠ΄Π½Π° Π·Π° Π΄Ρ€ΡƒΠ³ΠΎΠΉ, Π² Ρ†Π΅ΠΏΠΎΡ‡ΠΊΡƒ. Π’Π°ΠΊ ΠΆΠ΅ выглядит ΠΈ ΠΏΠΎΡΠ»Π΅Π΄ΠΎΠ²Π°Ρ‚Π΅Π»ΡŒΠ½ΠΎΠ΅ соСдинСниС ΠΏΡ€ΠΎΠ²ΠΎΠ΄Π½ΠΈΠΊΠΎΠ². Π”ΠΎΡ€ΠΎΠ³Π° Π² этом случаС ΠΌΠΎΠΆΠ΅Ρ‚ ΠΈΠΌΠ΅Ρ‚ΡŒ Π½Π΅ΠΎΠ³Ρ€Π°Π½ΠΈΡ‡Π΅Π½Π½ΠΎΠ΅ количСство ΠΏΠΎΠ²ΠΎΡ€ΠΎΡ‚ΠΎΠ², Π½ΠΎ Π½ΠΈ ΠΎΠ΄Π½ΠΎΠ³ΠΎ пСрСкрСстка. Как Π±Ρ‹ Π½ΠΈ виляла Π΄ΠΎΡ€ΠΎΠ³Π° (ΠΏΡ€ΠΎΠ²ΠΎΠ΄Π°), ΠΌΠ°ΡˆΠΈΠ½Ρ‹ (рСзисторы) всСгда Π±ΡƒΠ΄ΡƒΡ‚ располоТСны Π΄Ρ€ΡƒΠ³ Π·Π° Π΄Ρ€ΡƒΠ³ΠΎΠΌ, ΠΏΠΎ ΠΎΠ΄Π½ΠΎΠΉ Ρ†Π΅ΠΏΠΎΡ‡ΠΊΠ΅.

БовсСм Π΄Ρ€ΡƒΠ³ΠΎΠ΅ Π΄Π΅Π»ΠΎ, Ссли рассматриваСтся ΠΏΠ°Ρ€Π°Π»Π»Π΅Π»ΡŒΠ½ΠΎΠ΅ соСдинСниС. Π’ΠΎΠ³Π΄Π° рСзисторы ΠΌΠΎΠΆΠ½ΠΎ ΡΡ€Π°Π²Π½ΠΈΡ‚ΡŒ со спортсмСнами Π½Π° стартС. Они стоят ΠΊΠ°ΠΆΠ΄Ρ‹ΠΉ Π½Π° своСй Π΄ΠΎΡ€ΠΎΠΆΠΊΠ΅, Π½ΠΎ Π½Π°ΠΏΡ€Π°Π²Π»Π΅Π½ΠΈΠ΅ двиТСния Ρƒ Π½ΠΈΡ… ΠΎΠ΄ΠΈΠ½Π°ΠΊΠΎΠ²ΠΎΠ΅, ΠΈ Ρ„ΠΈΠ½ΠΈΡˆ Π² ΠΎΠ΄Π½ΠΎΠΌ мСстС. Π’Π°ΠΊ ΠΆΠ΅ ΠΈ рСзисторы β€” Ρƒ ΠΊΠ°ΠΆΠ΄ΠΎΠ³ΠΎ ΠΈΠ· Π½ΠΈΡ… свой ΠΏΡ€ΠΎΠ²ΠΎΠ΄, Π½ΠΎ всС ΠΎΠ½ΠΈ соСдинСны Π² Π½Π΅ΠΊΠΎΡ‚ΠΎΡ€ΠΎΠΉ Ρ‚ΠΎΡ‡ΠΊΠ΅.

Π€ΠΎΡ€ΠΌΡƒΠ»Ρ‹ для силы Ρ‚ΠΎΠΊΠ°

О Π½Π΅ΠΉ всСгда ΠΈΠ΄Π΅Ρ‚ Ρ€Π΅Ρ‡ΡŒ Π² Ρ‚Π΅ΠΌΠ΅ «ЭлСктричСство». ΠŸΠ°Ρ€Π°Π»Π»Π΅Π»ΡŒΠ½ΠΎΠ΅ ΠΈ ΠΏΠΎΡΠ»Π΅Π΄ΠΎΠ²Π°Ρ‚Π΅Π»ΡŒΠ½ΠΎΠ΅ соСдинСниС ΠΏΠΎ-Ρ€Π°Π·Π½ΠΎΠΌΡƒ Π²Π»ΠΈΡΡŽΡ‚ Π½Π° Π²Π΅Π»ΠΈΡ‡ΠΈΠ½Ρƒ Π² рСзисторах. Для Π½ΠΈΡ… Π²Ρ‹Π²Π΅Π΄Π΅Π½Ρ‹ Ρ„ΠΎΡ€ΠΌΡƒΠ»Ρ‹, ΠΊΠΎΡ‚ΠΎΡ€Ρ‹Π΅ ΠΌΠΎΠΆΠ½ΠΎ Π·Π°ΠΏΠΎΠΌΠ½ΠΈΡ‚ΡŒ. Но достаточно просто Π·Π°ΠΏΠΎΠΌΠ½ΠΈΡ‚ΡŒ смысл, ΠΊΠΎΡ‚ΠΎΡ€Ρ‹ΠΉ Π² Π½ΠΈΡ… вкладываСтся.

Π’Π°ΠΊ, Ρ‚ΠΎΠΊ ΠΏΡ€ΠΈ ΠΏΠΎΡΠ»Π΅Π΄ΠΎΠ²Π°Ρ‚Π΅Π»ΡŒΠ½ΠΎΠΌ соСдинСнии ΠΏΡ€ΠΎΠ²ΠΎΠ΄Π½ΠΈΠΊΠΎΠ² всСгда ΠΎΠ΄ΠΈΠ½Π°ΠΊΠΎΠ². Π’ΠΎ Π΅ΡΡ‚ΡŒ Π² ΠΊΠ°ΠΆΠ΄ΠΎΠΌ ΠΈΠ· Π½ΠΈΡ… Π·Π½Π°Ρ‡Π΅Π½ΠΈΠ΅ силы Ρ‚ΠΎΠΊΠ° Π½Π΅ отличаСтся. ΠŸΡ€ΠΎΠ²Π΅ΡΡ‚ΠΈ аналогию ΠΌΠΎΠΆΠ½ΠΎ, Ссли ΡΡ€Π°Π²Π½ΠΈΡ‚ΡŒ ΠΏΡ€ΠΎΠ²ΠΎΠ΄ с Ρ‚Ρ€ΡƒΠ±ΠΎΠΉ. Π’ Π½Π΅ΠΉ Π²ΠΎΠ΄Π° Ρ‚Π΅Ρ‡Π΅Ρ‚ всСгда ΠΎΠ΄ΠΈΠ½Π°ΠΊΠΎΠ²ΠΎ. И всС прСпятствия Π½Π° Π΅Π΅ ΠΏΡƒΡ‚ΠΈ Π±ΡƒΠ΄ΡƒΡ‚ ΡΠΌΠ΅Ρ‚Π°Ρ‚ΡŒΡΡ с ΠΎΠ΄Π½ΠΎΠΉ ΠΈ Ρ‚ΠΎΠΉ ΠΆΠ΅ силой. Π’Π°ΠΊ ΠΆΠ΅ с силой Ρ‚ΠΎΠΊΠ°. ΠŸΠΎΡΡ‚ΠΎΠΌΡƒ Ρ„ΠΎΡ€ΠΌΡƒΠ»Π° ΠΎΠ±Ρ‰Π΅ΠΉ силы Ρ‚ΠΎΠΊΠ° Π² Ρ†Π΅ΠΏΠΈ с ΠΏΠΎΡΠ»Π΅Π΄ΠΎΠ²Π°Ρ‚Π΅Π»ΡŒΠ½Ρ‹ΠΌ соСдинСниСм рСзисторов выглядит Ρ‚Π°ΠΊ:

I ΠΎΠ±Ρ‰ = I 1 = I 2

Π—Π΄Π΅ΡΡŒ Π±ΡƒΠΊΠ²ΠΎΠΉ I ΠΎΠ±ΠΎΠ·Π½Π°Ρ‡Π΅Π½Π° сила Ρ‚ΠΎΠΊΠ°. Π­Ρ‚ΠΎ общСпринятоС ΠΎΠ±ΠΎΠ·Π½Π°Ρ‡Π΅Π½ΠΈΠ΅, поэтому Π΅Π³ΠΎ Π½ΡƒΠΆΠ½ΠΎ Π·Π°ΠΏΠΎΠΌΠ½ΠΈΡ‚ΡŒ.

Π’ΠΎΠΊ ΠΏΡ€ΠΈ ΠΏΠ°Ρ€Π°Π»Π»Π΅Π»ΡŒΠ½ΠΎΠΌ соСдинСнии ΡƒΠΆΠ΅ Π½Π΅ Π±ΡƒΠ΄Π΅Ρ‚ постоянной Π²Π΅Π»ΠΈΡ‡ΠΈΠ½ΠΎΠΉ. ΠŸΡ€ΠΈ Ρ‚ΠΎΠΉ ΠΆΠ΅ Π°Π½Π°Π»ΠΎΠ³ΠΈΠΈ с Ρ‚Ρ€ΡƒΠ±ΠΎΠΉ получаСтся, Ρ‡Ρ‚ΠΎ Π²ΠΎΠ΄Π° раздСлится Π½Π° Π΄Π²Π° ΠΏΠΎΡ‚ΠΎΠΊΠ°, Ссли Ρƒ основной Ρ‚Ρ€ΡƒΠ±Ρ‹ Π±ΡƒΠ΄Π΅Ρ‚ ΠΎΡ‚Π²Π΅Ρ‚Π²Π»Π΅Π½ΠΈΠ΅. Π’ΠΎ ΠΆΠ΅ явлСниС Π½Π°Π±Π»ΡŽΠ΄Π°Π΅Ρ‚ΡΡ с Ρ‚ΠΎΠΊΠΎΠΌ, ΠΊΠΎΠ³Π΄Π° Π½Π° Π΅Π³ΠΎ ΠΏΡƒΡ‚ΠΈ появляСтся Ρ€Π°Π·Π²Π΅Ρ‚Π²Π»Π΅Π½ΠΈΠ΅ ΠΏΡ€ΠΎΠ²ΠΎΠ΄ΠΎΠ². Π€ΠΎΡ€ΠΌΡƒΠ»Π° ΠΎΠ±Ρ‰Π΅ΠΉ силы Ρ‚ΠΎΠΊΠ° ΠΏΡ€ΠΈ :

I ΠΎΠ±Ρ‰ = I 1 + I 2

Если Ρ€Π°Π·Π²Π΅Ρ‚Π²Π»Π΅Π½ΠΈΠ΅ составлСно ΠΈΠ· ΠΏΡ€ΠΎΠ²ΠΎΠ΄ΠΎΠ², ΠΊΠΎΡ‚ΠΎΡ€Ρ‹Ρ… большС Π΄Π²ΡƒΡ…, Ρ‚ΠΎ Π² ΠΏΡ€ΠΈΠ²Π΅Π΄Π΅Π½Π½ΠΎΠΉ Ρ„ΠΎΡ€ΠΌΡƒΠ»Π΅ Π½Π° Ρ‚Π°ΠΊΠΎΠ΅ ΠΆΠ΅ количСство станСт большС слагаСмых.

Π€ΠΎΡ€ΠΌΡƒΠ»Ρ‹ для напряТСния

Когда рассматриваСтся схСма, Π² ΠΊΠΎΡ‚ΠΎΡ€ΠΎΠΉ Π²Ρ‹ΠΏΠΎΠ»Π½Π΅Π½ΠΎ соСдинСниС ΠΏΡ€ΠΎΠ²ΠΎΠ΄Π½ΠΈΠΊΠΎΠ² ΠΏΠΎΡΠ»Π΅Π΄ΠΎΠ²Π°Ρ‚Π΅Π»ΡŒΠ½ΠΎ, Ρ‚ΠΎ напряТСниС Π½Π° всСм участкС опрСдСляСтся суммой этих Π²Π΅Π»ΠΈΡ‡ΠΈΠ½ Π½Π° ΠΊΠ°ΠΆΠ΄ΠΎΠΌ ΠΊΠΎΠ½ΠΊΡ€Π΅Ρ‚Π½ΠΎΠΌ рСзисторС. Π‘Ρ€Π°Π²Π½ΠΈΡ‚ΡŒ эту ΡΠΈΡ‚ΡƒΠ°Ρ†ΠΈΡŽ ΠΌΠΎΠΆΠ½ΠΎ с Ρ‚Π°Ρ€Π΅Π»ΠΊΠ°ΠΌΠΈ. Π£Π΄Π΅Ρ€ΠΆΠ°Ρ‚ΡŒ ΠΎΠ΄Π½Ρƒ ΠΈΠ· Π½ΠΈΡ… Π»Π΅Π³ΠΊΠΎ получится ΠΎΠ΄Π½ΠΎΠΌΡƒ Ρ‡Π΅Π»ΠΎΠ²Π΅ΠΊΡƒ, Π²Ρ‚ΠΎΡ€ΡƒΡŽ рядом ΠΎΠ½ Ρ‚ΠΎΠΆΠ΅ смоТСт Π²Π·ΡΡ‚ΡŒ, Π½ΠΎ ΡƒΠΆΠ΅ с Ρ‚Ρ€ΡƒΠ΄ΠΎΠΌ. Π”Π΅Ρ€ΠΆΠ°Ρ‚ΡŒ Π² Ρ€ΡƒΠΊΠ°Ρ… Ρ‚Ρ€ΠΈ Ρ‚Π°Ρ€Π΅Π»ΠΊΠΈ рядом Π΄Ρ€ΡƒΠ³ с Π΄Ρ€ΡƒΠ³ΠΎΠΌ ΠΎΠ΄Π½ΠΎΠΌΡƒ Ρ‡Π΅Π»ΠΎΠ²Π΅ΠΊΡƒ ΡƒΠΆΠ΅ Π½Π΅ удастся, потрСбуСтся ΠΏΠΎΠΌΠΎΡ‰ΡŒ Π²Ρ‚ΠΎΡ€ΠΎΠ³ΠΎ. И Ρ‚Π°ΠΊ Π΄Π°Π»Π΅Π΅. Усилия людСй ΡΠΊΠ»Π°Π΄Ρ‹Π²Π°ΡŽΡ‚ΡΡ.

Π€ΠΎΡ€ΠΌΡƒΠ»Π° для ΠΎΠ±Ρ‰Π΅Π³ΠΎ напряТСния участка Ρ†Π΅ΠΏΠΈ с ΠΏΠΎΡΠ»Π΅Π΄ΠΎΠ²Π°Ρ‚Π΅Π»ΡŒΠ½Ρ‹ΠΌ соСдинСниСм ΠΏΡ€ΠΎΠ²ΠΎΠ΄Π½ΠΈΠΊΠΎΠ² выглядит Ρ‚Π°ΠΊ:

U ΠΎΠ±Ρ‰ = U 1 + U 2 , Π³Π΄Π΅ U — ΠΎΠ±ΠΎΠ·Π½Π°Ρ‡Π΅Π½ΠΈΠ΅, принятоС для

Другая ситуация складываСтся, Ссли рассматриваСтся Когда Ρ‚Π°Ρ€Π΅Π»ΠΊΠΈ ставятся Π΄Ρ€ΡƒΠ³ Π½Π° Π΄Ρ€ΡƒΠ³Π°, ΠΈΡ… ΠΏΠΎ-ΠΏΡ€Π΅ΠΆΠ½Π΅ΠΌΡƒ ΠΌΠΎΠΆΠ΅Ρ‚ ΡƒΠ΄Π΅Ρ€ΠΆΠ°Ρ‚ΡŒ ΠΎΠ΄ΠΈΠ½ Ρ‡Π΅Π»ΠΎΠ²Π΅ΠΊ. ΠŸΠΎΡΡ‚ΠΎΠΌΡƒ ΡΠΊΠ»Π°Π΄Ρ‹Π²Π°Ρ‚ΡŒ Π½ΠΈΡ‡Π΅Π³ΠΎ Π½Π΅ приходится. Вакая ΠΆΠ΅ аналогия Π½Π°Π±Π»ΡŽΠ΄Π°Π΅Ρ‚ΡΡ ΠΏΡ€ΠΈ ΠΏΠ°Ρ€Π°Π»Π»Π΅Π»ΡŒΠ½ΠΎΠΌ соСдинСнии ΠΏΡ€ΠΎΠ²ΠΎΠ΄Π½ΠΈΠΊΠΎΠ². НапряТСниС Π½Π° ΠΊΠ°ΠΆΠ΄ΠΎΠΌ ΠΈΠ· Π½ΠΈΡ… ΠΎΠ΄ΠΈΠ½Π°ΠΊΠΎΠ²ΠΎΠ΅ ΠΈ Ρ€Π°Π²Π½ΠΎ Ρ‚ΠΎΠΌΡƒ, ΠΊΠΎΡ‚ΠΎΡ€ΠΎΠ΅ Π½Π° всСх Π½ΠΈΡ… сразу. Π€ΠΎΡ€ΠΌΡƒΠ»Π° ΠΎΠ±Ρ‰Π΅Π³ΠΎ напряТСния такая:

U ΠΎΠ±Ρ‰ = U 1 = U 2

Π€ΠΎΡ€ΠΌΡƒΠ»Ρ‹ для элСктричСского сопротивлСния

Π˜Ρ… ΡƒΠΆΠ΅ ΠΌΠΎΠΆΠ½ΠΎ Π½Π΅ Π·Π°ΠΏΠΎΠΌΠΈΠ½Π°Ρ‚ΡŒ, Π° Π·Π½Π°Ρ‚ΡŒ Ρ„ΠΎΡ€ΠΌΡƒΠ»Ρƒ Π·Π°ΠΊΠΎΠ½Π° Ома ΠΈ ΠΈΠ· Π½Π΅Π΅ Π²Ρ‹Π²ΠΎΠ΄ΠΈΡ‚ΡŒ Π½ΡƒΠΆΠ½ΡƒΡŽ. Из ΡƒΠΊΠ°Π·Π°Π½Π½ΠΎΠ³ΠΎ Π·Π°ΠΊΠΎΠ½Π° слСдуСт, Ρ‡Ρ‚ΠΎ напряТСниС Ρ€Π°Π²Π½ΠΎ ΠΏΡ€ΠΎΠΈΠ·Π²Π΅Π΄Π΅Π½ΠΈΡŽ силы Ρ‚ΠΎΠΊΠ° ΠΈ сопротивлСния. Π’ΠΎ Π΅ΡΡ‚ΡŒ U = I * R, Π³Π΄Π΅ R β€” сопротивлСниС.

Π’ΠΎΠ³Π΄Π° Ρ„ΠΎΡ€ΠΌΡƒΠ»Π°, с ΠΊΠΎΡ‚ΠΎΡ€ΠΎΠΉ Π½ΡƒΠΆΠ½ΠΎ Π±ΡƒΠ΄Π΅Ρ‚ Ρ€Π°Π±ΠΎΡ‚Π°Ρ‚ΡŒ, зависит ΠΎΡ‚ Ρ‚ΠΎΠ³ΠΎ, ΠΊΠ°ΠΊ Π²Ρ‹ΠΏΠΎΠ»Π½Π΅Π½ΠΎ соСдинСниС ΠΏΡ€ΠΎΠ²ΠΎΠ΄Π½ΠΈΠΊΠΎΠ²:

  • ΠΏΠΎΡΠ»Π΅Π΄ΠΎΠ²Π°Ρ‚Π΅Π»ΡŒΠ½ΠΎ, Π·Π½Π°Ρ‡ΠΈΡ‚, Π½ΡƒΠΆΠ½ΠΎ равСнство для напряТСния β€” I ΠΎΠ±Ρ‰ * R ΠΎΠ±Ρ‰ = I 1 * R 1 + I 2 * R 2;
  • ΠΏΠ°Ρ€Π°Π»Π»Π΅Π»ΡŒΠ½ΠΎ Π½Π΅ΠΎΠ±Ρ…ΠΎΠ΄ΠΈΠΌΠΎ ΠΏΠΎΠ»ΡŒΠ·ΠΎΠ²Π°Ρ‚ΡŒΡΡ Ρ„ΠΎΡ€ΠΌΡƒΠ»ΠΎΠΉ для силы Ρ‚ΠΎΠΊΠ° β€” U ΠΎΠ±Ρ‰ / R ΠΎΠ±Ρ‰ = U 1 / R 1 + U 2 / R 2 .

Π”Π°Π»Π΅Π΅ ΡΠ»Π΅Π΄ΡƒΡŽΡ‚ простыС прСобразования, ΠΊΠΎΡ‚ΠΎΡ€Ρ‹Π΅ ΠΎΡΠ½ΠΎΠ²Ρ‹Π²Π°ΡŽΡ‚ΡΡ Π½Π° Ρ‚ΠΎΠΌ, Ρ‡Ρ‚ΠΎ Π² ΠΏΠ΅Ρ€Π²ΠΎΠΌ равСнствС всС силы Ρ‚ΠΎΠΊΠ° ΠΈΠΌΠ΅ΡŽΡ‚ ΠΎΠ΄ΠΈΠ½Π°ΠΊΠΎΠ²ΠΎΠ΅ Π·Π½Π°Ρ‡Π΅Π½ΠΈΠ΅, Π° Π²ΠΎ Π²Ρ‚ΠΎΡ€ΠΎΠΌ β€” напряТСния Ρ€Π°Π²Π½Ρ‹. Π—Π½Π°Ρ‡ΠΈΡ‚, ΠΈΡ… ΠΌΠΎΠΆΠ½ΠΎ ΡΠΎΠΊΡ€Π°Ρ‚ΠΈΡ‚ΡŒ. Π’ΠΎ Π΅ΡΡ‚ΡŒ ΠΏΠΎΠ»ΡƒΡ‡Π°ΡŽΡ‚ΡΡ Ρ‚Π°ΠΊΠΈΠ΅ выраТСния:

  1. R ΠΎΠ±Ρ‰ = R 1 + R 2 (для ΠΏΠΎΡΠ»Π΅Π΄ΠΎΠ²Π°Ρ‚Π΅Π»ΡŒΠ½ΠΎΠ³ΠΎ соСдинСния ΠΏΡ€ΠΎΠ²ΠΎΠ΄Π½ΠΈΠΊΠΎΠ²).
  2. 1 / R ΠΎΠ±Ρ‰ = 1 / R 1 + 1 / R 2 (ΠΏΡ€ΠΈ ΠΏΠ°Ρ€Π°Π»Π»Π΅Π»ΡŒΠ½ΠΎΠΌ соСдинСнии).

ΠŸΡ€ΠΈ ΡƒΠ²Π΅Π»ΠΈΡ‡Π΅Π½ΠΈΠΈ числа рСзисторов, ΠΊΠΎΡ‚ΠΎΡ€Ρ‹Π΅ Π²ΠΊΠ»ΡŽΡ‡Π΅Π½Ρ‹ Π² ΡΠ΅Ρ‚ΡŒ, измСняСтся количСство слагаСмых Π² этих выраТСниях.

Π‘Ρ‚ΠΎΠΈΡ‚ ΠΎΡ‚ΠΌΠ΅Ρ‚ΠΈΡ‚ΡŒ, Ρ‡Ρ‚ΠΎ ΠΏΠ°Ρ€Π°Π»Π»Π΅Π»ΡŒΠ½ΠΎΠ΅ ΠΈ ΠΏΠΎΡΠ»Π΅Π΄ΠΎΠ²Π°Ρ‚Π΅Π»ΡŒΠ½ΠΎΠ΅ соСдинСниС ΠΏΡ€ΠΎΠ²ΠΎΠ΄Π½ΠΈΠΊΠΎΠ² ΠΏΠΎ-Ρ€Π°Π·Π½ΠΎΠΌΡƒ Π²Π»ΠΈΡΡŽΡ‚ Π½Π° ΠΎΠ±Ρ‰Π΅Π΅ сопротивлСниС. ΠŸΠ΅Ρ€Π²ΠΎΠ΅ ΠΈΠ· Π½ΠΈΡ… ΡƒΠΌΠ΅Π½ΡŒΡˆΠ°Π΅Ρ‚ сопротивлСниС участка Ρ†Π΅ΠΏΠΈ. ΠŸΡ€ΠΈΡ‡Π΅ΠΌ ΠΎΠ½ΠΎ оказываСтся мСньшС самого малСнького ΠΈΠ· ΠΈΡΠΏΠΎΠ»ΡŒΠ·ΠΎΠ²Π°Π½Π½Ρ‹Ρ… рСзисторов. ΠŸΡ€ΠΈ ΠΏΠΎΡΠ»Π΅Π΄ΠΎΠ²Π°Ρ‚Π΅Π»ΡŒΠ½ΠΎΠΌ соСдинСнии всС Π»ΠΎΠ³ΠΈΡ‡Π½ΠΎ: значСния ΡΠΊΠ»Π°Π΄Ρ‹Π²Π°ΡŽΡ‚ΡΡ, поэтому ΠΎΠ±Ρ‰Π΅Π΅ число всСгда Π±ΡƒΠ΄Π΅Ρ‚ самым большим.

Π Π°Π±ΠΎΡ‚Π° Ρ‚ΠΎΠΊΠ°

ΠŸΡ€Π΅Π΄Ρ‹Π΄ΡƒΡ‰ΠΈΠ΅ Ρ‚Ρ€ΠΈ Π²Π΅Π»ΠΈΡ‡ΠΈΠ½Ρ‹ ΡΠΎΡΡ‚Π°Π²Π»ΡΡŽΡ‚ Π·Π°ΠΊΠΎΠ½Ρ‹ ΠΏΠ°Ρ€Π°Π»Π»Π΅Π»ΡŒΠ½ΠΎΠ³ΠΎ соСдинСния ΠΈ ΠΏΠΎΡΠ»Π΅Π΄ΠΎΠ²Π°Ρ‚Π΅Π»ΡŒΠ½ΠΎΠ³ΠΎ располоТСния ΠΏΡ€ΠΎΠ²ΠΎΠ΄Π½ΠΈΠΊΠΎΠ² Π² Ρ†Π΅ΠΏΠΈ. ΠŸΠΎΡΡ‚ΠΎΠΌΡƒ ΠΈΡ… Π·Π½Π°Ρ‚ΡŒ Π½ΡƒΠΆΠ½ΠΎ ΠΎΠ±ΡΠ·Π°Ρ‚Π΅Π»ΡŒΠ½ΠΎ. ΠŸΡ€ΠΎ Ρ€Π°Π±ΠΎΡ‚Ρƒ ΠΈ ΠΌΠΎΡ‰Π½ΠΎΡΡ‚ΡŒ Π½Π΅ΠΎΠ±Ρ…ΠΎΠ΄ΠΈΠΌΠΎ просто Π·Π°ΠΏΠΎΠΌΠ½ΠΈΡ‚ΡŒ Π±Π°Π·ΠΎΠ²ΡƒΡŽ Ρ„ΠΎΡ€ΠΌΡƒΠ»Ρƒ. Она записываСтся Ρ‚Π°ΠΊ: А = I * U * t , Π³Π΄Π΅ А β€” Ρ€Π°Π±ΠΎΡ‚Π° Ρ‚ΠΎΠΊΠ°, t β€” врСмя Π΅Π³ΠΎ прохоТдСния ΠΏΠΎ ΠΏΡ€ΠΎΠ²ΠΎΠ΄Π½ΠΈΠΊΡƒ.

Для Ρ‚ΠΎΠ³ΠΎ Ρ‡Ρ‚ΠΎΠ±Ρ‹ ΠΎΠΏΡ€Π΅Π΄Π΅Π»ΠΈΡ‚ΡŒ ΠΎΠ±Ρ‰ΡƒΡŽ Ρ€Π°Π±ΠΎΡ‚Ρƒ ΠΏΡ€ΠΈ ΠΏΠΎΡΠ»Π΅Π΄ΠΎΠ²Π°Ρ‚Π΅Π»ΡŒΠ½ΠΎΠΌ соСдинСнии Π½ΡƒΠΆΠ½ΠΎ Π·Π°ΠΌΠ΅Π½ΠΈΡ‚ΡŒ Π² исходном Π²Ρ‹Ρ€Π°ΠΆΠ΅Π½ΠΈΠΈ напряТСниС. ΠŸΠΎΠ»ΡƒΡ‡ΠΈΡ‚ΡΡ равСнство: А = I * (U 1 + U 2) * t, раскрыв скобки Π² ΠΊΠΎΡ‚ΠΎΡ€ΠΎΠΌ получится, Ρ‡Ρ‚ΠΎ Ρ€Π°Π±ΠΎΡ‚Π° Π½Π° всСм участкС Ρ€Π°Π²Π½Π° ΠΈΡ… суммС Π½Π° ΠΊΠ°ΠΆΠ΄ΠΎΠΌ ΠΊΠΎΠ½ΠΊΡ€Π΅Ρ‚Π½ΠΎΠΌ ΠΏΠΎΡ‚Ρ€Π΅Π±ΠΈΡ‚Π΅Π»Π΅ Ρ‚ΠΎΠΊΠ°.

Аналогично ΠΈΠ΄Π΅Ρ‚ рассуТдСниС, Ссли рассматриваСтся схСма ΠΏΠ°Ρ€Π°Π»Π»Π΅Π»ΡŒΠ½ΠΎΠ³ΠΎ соСдинСния. Волько Π·Π°ΠΌΠ΅Π½ΡΡ‚ΡŒ полагаСтся силу Ρ‚ΠΎΠΊΠ°. Но Ρ€Π΅Π·ΡƒΠ»ΡŒΡ‚Π°Ρ‚ Π±ΡƒΠ΄Π΅Ρ‚ Ρ‚ΠΎΡ‚ ΠΆΠ΅: А = А 1 + А 2 .

ΠœΠΎΡ‰Π½ΠΎΡΡ‚ΡŒ Ρ‚ΠΎΠΊΠ°

ΠŸΡ€ΠΈ Π²Ρ‹Π²Π΅Π΄Π΅Π½ΠΈΠΈ Ρ„ΠΎΡ€ΠΌΡƒΠ»Ρ‹ для мощности (ΠΎΠ±ΠΎΠ·Π½Π°Ρ‡Π΅Π½ΠΈΠ΅ Β«Π Β») участка Ρ†Π΅ΠΏΠΈ ΠΎΠΏΡΡ‚ΡŒ Π½ΡƒΠΆΠ½ΠΎ ΠΏΠΎΠ»ΡŒΠ·ΠΎΠ²Π°Ρ‚ΡŒΡΡ ΠΎΠ΄Π½ΠΎΠΉ Ρ„ΠΎΡ€ΠΌΡƒΠ»ΠΎΠΉ: Π  = U * I. ПослС ΠΏΠΎΠ΄ΠΎΠ±Π½Ρ‹Ρ… рассуТдСний получаСтся, Ρ‡Ρ‚ΠΎ ΠΏΠ°Ρ€Π°Π»Π»Π΅Π»ΡŒΠ½ΠΎΠ΅ ΠΈ ΠΏΠΎΡΠ»Π΅Π΄ΠΎΠ²Π°Ρ‚Π΅Π»ΡŒΠ½ΠΎΠ΅ соСдинСниС ΠΎΠΏΠΈΡΡ‹Π²Π°ΡŽΡ‚ΡΡ Ρ‚Π°ΠΊΠΎΠΉ Ρ„ΠΎΡ€ΠΌΡƒΠ»ΠΎΠΉ для мощности: Π  = Π  1 + Π  2 .

Π’ΠΎ Π΅ΡΡ‚ΡŒ, ΠΊΠ°ΠΊ Π±Ρ‹ Π½ΠΈ Π±Ρ‹Π»ΠΈ составлСны схСмы, общая ΠΌΠΎΡ‰Π½ΠΎΡΡ‚ΡŒ Π±ΡƒΠ΄Π΅Ρ‚ ΡΠΊΠ»Π°Π΄Ρ‹Π²Π°Ρ‚ΡŒΡΡ ΠΈΠ· Ρ‚Π΅Ρ…, ΠΊΠΎΡ‚ΠΎΡ€Ρ‹Π΅ задСйствованы Π² Ρ€Π°Π±ΠΎΡ‚Π΅. ИмСнно этим ΠΎΠ±ΡŠΡΡΠ½ΡΠ΅Ρ‚ΡΡ Ρ‚ΠΎΡ‚ Ρ„Π°ΠΊΡ‚, Ρ‡Ρ‚ΠΎ нСльзя Π²ΠΊΠ»ΡŽΡ‡Π°Ρ‚ΡŒ Π² ΡΠ΅Ρ‚ΡŒ ΠΊΠ²Π°Ρ€Ρ‚ΠΈΡ€Ρ‹ ΠΎΠ΄Π½ΠΎΠ²Ρ€Π΅ΠΌΠ΅Π½Π½ΠΎ ΠΌΠ½ΠΎΠ³ΠΎ ΠΌΠΎΡ‰Π½Ρ‹Ρ… ΠΏΡ€ΠΈΠ±ΠΎΡ€ΠΎΠ². Она просто Π½Π΅ Π²Ρ‹Π΄Π΅Ρ€ΠΆΠΈΡ‚ Ρ‚Π°ΠΊΠΎΠΉ Π½Π°Π³Ρ€ΡƒΠ·ΠΊΠΈ.

Как влияСт соСдинСниС ΠΏΡ€ΠΎΠ²ΠΎΠ΄Π½ΠΈΠΊΠΎΠ² Π½Π° Ρ€Π΅ΠΌΠΎΠ½Ρ‚ Π½ΠΎΠ²ΠΎΠ³ΠΎΠ΄Π½Π΅ΠΉ гирлянды?

Π‘Ρ€Π°Π·Ρƒ ΠΆΠ΅ послС Ρ‚ΠΎΠ³ΠΎ, ΠΊΠ°ΠΊ ΠΏΠ΅Ρ€Π΅Π³ΠΎΡ€ΠΈΡ‚ ΠΎΠ΄Π½Π° ΠΈΠ· Π»Π°ΠΌΠΏΠΎΡ‡Π΅ΠΊ, станСт ясно, ΠΊΠ°ΠΊ ΠΎΠ½ΠΈ Π±Ρ‹Π»ΠΈ соСдинСны. ΠŸΡ€ΠΈ ΠΏΠΎΡΠ»Π΅Π΄ΠΎΠ²Π°Ρ‚Π΅Π»ΡŒΠ½ΠΎΠΌ соСдинСнии Π½Π΅ Π±ΡƒΠ΄Π΅Ρ‚ ΡΠ²Π΅Ρ‚ΠΈΡ‚ΡŒΡΡ Π½ΠΈ ΠΎΠ΄Π½Π° ΠΈΠ· Π½ΠΈΡ…. Π­Ρ‚ΠΎ ΠΎΠ±ΡŠΡΡΠ½ΡΠ΅Ρ‚ΡΡ Ρ‚Π΅ΠΌ, Ρ‡Ρ‚ΠΎ ΠΏΡ€ΠΈΡˆΠ΅Π΄ΡˆΠ°Ρ Π² Π½Π΅Π³ΠΎΠ΄Π½ΠΎΡΡ‚ΡŒ Π»Π°ΠΌΠΏΠ° создаСт Ρ€Π°Π·Ρ€Ρ‹Π² Π² Ρ†Π΅ΠΏΠΈ. ΠŸΠΎΡΡ‚ΠΎΠΌΡƒ Π½ΡƒΠΆΠ½ΠΎ ΠΏΡ€ΠΎΠ²Π΅Ρ€ΠΈΡ‚ΡŒ всС, Ρ‡Ρ‚ΠΎΠ±Ρ‹ ΠΎΠΏΡ€Π΅Π΄Π΅Π»ΠΈΡ‚ΡŒ, какая ΠΏΠ΅Ρ€Π΅Π³ΠΎΡ€Π΅Π»Π°, Π·Π°ΠΌΠ΅Π½ΠΈΡ‚ΡŒ Π΅Π΅ — ΠΈ гирлянда станСт Ρ€Π°Π±ΠΎΡ‚Π°Ρ‚ΡŒ.

Если Π² Π½Π΅ΠΉ ΠΈΡΠΏΠΎΠ»ΡŒΠ·ΡƒΠ΅Ρ‚ΡΡ ΠΏΠ°Ρ€Π°Π»Π»Π΅Π»ΡŒΠ½ΠΎΠ΅ соСдинСниС, Ρ‚ΠΎ ΠΎΠ½Π° Π½Π΅ пСрСстаСт Ρ€Π°Π±ΠΎΡ‚Π°Ρ‚ΡŒ ΠΏΡ€ΠΈ нСисправности ΠΎΠ΄Π½ΠΎΠΉ ΠΈΠ· Π»Π°ΠΌΠΏΠΎΡ‡Π΅ΠΊ. Π’Π΅Π΄ΡŒ Ρ†Π΅ΠΏΡŒ Π½Π΅ Π±ΡƒΠ΄Π΅Ρ‚ ΠΏΠΎΠ»Π½ΠΎΡΡ‚ΡŒΡŽ Ρ€Π°Π·ΠΎΡ€Π²Π°Π½Π°, Π° Ρ‚ΠΎΠ»ΡŒΠΊΠΎ ΠΎΠ΄Π½Π° ΠΏΠ°Ρ€Π°Π»Π»Π΅Π»ΡŒΠ½Π°Ρ Ρ‡Π°ΡΡ‚ΡŒ. Π§Ρ‚ΠΎΠ±Ρ‹ ΠΎΡ‚Ρ€Π΅ΠΌΠΎΠ½Ρ‚ΠΈΡ€ΠΎΠ²Π°Ρ‚ΡŒ Ρ‚Π°ΠΊΡƒΡŽ гирлянду, Π½Π΅ Π½ΡƒΠΆΠ½ΠΎ ΠΏΡ€ΠΎΠ²Π΅Ρ€ΡΡ‚ΡŒ всС элСмСнты Ρ†Π΅ΠΏΠΈ, Π° Ρ‚ΠΎΠ»ΡŒΠΊΠΎ Ρ‚Π΅, ΠΊΠΎΡ‚ΠΎΡ€Ρ‹Π΅ Π½Π΅ свСтятся.

Π§Ρ‚ΠΎ происходит с Ρ†Π΅ΠΏΡŒΡŽ, Ссли Π² Π½Π΅Π΅ Π²ΠΊΠ»ΡŽΡ‡Π΅Π½Ρ‹ Π½Π΅ рСзисторы, Π° кондСнсаторы?

ΠŸΡ€ΠΈ ΠΈΡ… ΠΏΠΎΡΠ»Π΅Π΄ΠΎΠ²Π°Ρ‚Π΅Π»ΡŒΠ½ΠΎΠΌ соСдинСнии Π½Π°Π±Π»ΡŽΠ΄Π°Π΅Ρ‚ΡΡ такая ситуация: заряды ΠΎΡ‚ плюсов источника питания ΠΏΠΎΡΡ‚ΡƒΠΏΠ°ΡŽΡ‚ Ρ‚ΠΎΠ»ΡŒΠΊΠΎ Π½Π° внСшниС ΠΎΠ±ΠΊΠ»Π°Π΄ΠΊΠΈ ΠΊΡ€Π°ΠΉΠ½ΠΈΡ… кондСнсаторов. Π’Π΅, Ρ‡Ρ‚ΠΎ находятся ΠΌΠ΅ΠΆΠ΄Ρƒ Π½ΠΈΠΌΠΈ, просто ΠΏΠ΅Ρ€Π΅Π΄Π°ΡŽΡ‚ этот заряд ΠΏΠΎ Ρ†Π΅ΠΏΠΎΡ‡ΠΊΠ΅. Π­Ρ‚ΠΈΠΌ ΠΎΠ±ΡŠΡΡΠ½ΡΠ΅Ρ‚ΡΡ Ρ‚ΠΎ, Ρ‡Ρ‚ΠΎ Π½Π° всСх ΠΎΠ±ΠΊΠ»Π°Π΄ΠΊΠ°Ρ… ΠΏΠΎΡΠ²Π»ΡΡŽΡ‚ΡΡ ΠΎΠ΄ΠΈΠ½Π°ΠΊΠΎΠ²Ρ‹Π΅ заряды, Π½ΠΎ ΠΈΠΌΠ΅ΡŽΡ‰ΠΈΠ΅ Ρ€Π°Π·Π½Ρ‹Π΅ Π·Π½Π°ΠΊΠΈ. ΠŸΠΎΡΡ‚ΠΎΠΌΡƒ элСктричСский заряд ΠΊΠ°ΠΆΠ΄ΠΎΠ³ΠΎ кондСнсатора, соСдинСнного ΠΏΠΎΡΠ»Π΅Π΄ΠΎΠ²Π°Ρ‚Π΅Π»ΡŒΠ½ΠΎ, ΠΌΠΎΠΆΠ½ΠΎ Π·Π°ΠΏΠΈΡΠ°Ρ‚ΡŒ Ρ‚Π°ΠΊΠΎΠΉ Ρ„ΠΎΡ€ΠΌΡƒΠ»ΠΎΠΉ:

q ΠΎΠ±Ρ‰ = q 1 = q 2 .

Для Ρ‚ΠΎΠ³ΠΎ Ρ‡Ρ‚ΠΎΠ±Ρ‹ ΠΎΠΏΡ€Π΅Π΄Π΅Π»ΠΈΡ‚ΡŒ напряТСниС Π½Π° ΠΊΠ°ΠΆΠ΄ΠΎΠΌ кондСнсаторС, потрСбуСтся Π·Π½Π°Π½ΠΈΠ΅ Ρ„ΠΎΡ€ΠΌΡƒΠ»Ρ‹: U = q / Π‘. Π’ Π½Π΅ΠΉ Π‘ β€” Π΅ΠΌΠΊΠΎΡΡ‚ΡŒ кондСнсатора.

ΠžΠ±Ρ‰Π΅Π΅ напряТСниС подчиняСтся Ρ‚ΠΎΠΌΡƒ ΠΆΠ΅ Π·Π°ΠΊΠΎΠ½Ρƒ, ΠΊΠΎΡ‚ΠΎΡ€Ρ‹ΠΉ справСдлив для рСзисторов. ΠŸΠΎΡΡ‚ΠΎΠΌΡƒ, Π·Π°ΠΌΠ΅Π½ΠΈΠ² Π² Ρ„ΠΎΡ€ΠΌΡƒΠ»Π΅ Смкости напряТСниС Π½Π° сумму, ΠΌΡ‹ ΠΏΠΎΠ»ΡƒΡ‡ΠΈΠΌ, Ρ‡Ρ‚ΠΎ ΠΎΠ±Ρ‰ΡƒΡŽ Π΅ΠΌΠΊΠΎΡΡ‚ΡŒ ΠΏΡ€ΠΈΠ±ΠΎΡ€ΠΎΠ² Π½ΡƒΠΆΠ½ΠΎ Π²Ρ‹Ρ‡ΠΈΡΠ»ΡΡ‚ΡŒ ΠΏΠΎ Ρ„ΠΎΡ€ΠΌΡƒΠ»Π΅:

Π‘ = q / (U 1 + U 2).

Π£ΠΏΡ€ΠΎΡΡ‚ΠΈΡ‚ΡŒ эту Ρ„ΠΎΡ€ΠΌΡƒΠ»Ρƒ ΠΌΠΎΠΆΠ½ΠΎ, ΠΏΠ΅Ρ€Π΅Π²Π΅Ρ€Π½ΡƒΠ² Π΄Ρ€ΠΎΠ±ΠΈ ΠΈ Π·Π°ΠΌΠ΅Π½ΠΈΠ² ΠΎΡ‚Π½ΠΎΡˆΠ΅Π½ΠΈΠ΅ напряТСния ΠΊ заряду Π΅ΠΌΠΊΠΎΡΡ‚ΡŒΡŽ. ΠŸΠΎΠ»ΡƒΡ‡Π°Π΅Ρ‚ΡΡ Ρ‚Π°ΠΊΠΎΠ΅ равСнство: 1 / Π‘ = 1 / Π‘ 1 + 1 / Π‘ 2 .

НСсколько ΠΏΠΎ-Π΄Ρ€ΡƒΠ³ΠΎΠΌΡƒ выглядит ситуация, ΠΊΠΎΠ³Π΄Π° соСдинСниС кондСнсаторов β€” ΠΏΠ°Ρ€Π°Π»Π»Π΅Π»ΡŒΠ½ΠΎΠ΅. Π’ΠΎΠ³Π΄Π° ΠΎΠ±Ρ‰ΠΈΠΉ заряд опрСдСляСтся суммой всСх зарядов, ΠΊΠΎΡ‚ΠΎΡ€Ρ‹Π΅ Π½Π°ΠΊΠ°ΠΏΠ»ΠΈΠ²Π°ΡŽΡ‚ΡΡ Π½Π° ΠΎΠ±ΠΊΠ»Π°Π΄ΠΊΠ°Ρ… всСх ΠΏΡ€ΠΈΠ±ΠΎΡ€ΠΎΠ². А Π·Π½Π°Ρ‡Π΅Π½ΠΈΠ΅ напряТСния ΠΏΠΎ-ΠΏΡ€Π΅ΠΆΠ½Π΅ΠΌΡƒ опрСдСляСтся ΠΏΠΎ ΠΎΠ±Ρ‰ΠΈΠΌ Π·Π°ΠΊΠΎΠ½Π°ΠΌ. ΠŸΠΎΡΡ‚ΠΎΠΌΡƒ Ρ„ΠΎΡ€ΠΌΡƒΠ»Π° для ΠΎΠ±Ρ‰Π΅ΠΉ Смкости ΠΏΠ°Ρ€Π°Π»Π»Π΅Π»ΡŒΠ½ΠΎ соСдинСнных кондСнсаторов выглядит Ρ‚Π°ΠΊ:

Π‘ = (q 1 + q 2) / U.

Π’ΠΎ Π΅ΡΡ‚ΡŒ эта Π²Π΅Π»ΠΈΡ‡ΠΈΠ½Π° считаСтся, ΠΊΠ°ΠΊ сумма ΠΊΠ°ΠΆΠ΄ΠΎΠ³ΠΎ ΠΈΠ· ΠΈΡΠΏΠΎΠ»ΡŒΠ·ΠΎΠ²Π°Π½Π½Ρ‹Ρ… Π² соСдинСнии ΠΏΡ€ΠΈΠ±ΠΎΡ€ΠΎΠ²:

Π‘ = Π‘ 1 + Π‘ 2.

Как ΠΎΠΏΡ€Π΅Π΄Π΅Π»ΠΈΡ‚ΡŒ ΠΎΠ±Ρ‰Π΅Π΅ сопротивлСниС ΠΏΡ€ΠΎΠΈΠ·Π²ΠΎΠ»ΡŒΠ½ΠΎΠ³ΠΎ соСдинСния ΠΏΡ€ΠΎΠ²ΠΎΠ΄Π½ΠΈΠΊΠΎΠ²?

Π’ΠΎ Π΅ΡΡ‚ΡŒ Ρ‚Π°ΠΊΠΎΠ³ΠΎ, Π² ΠΊΠΎΡ‚ΠΎΡ€ΠΎΠΌ ΠΏΠΎΡΠ»Π΅Π΄ΠΎΠ²Π°Ρ‚Π΅Π»ΡŒΠ½Ρ‹Π΅ участки ΡΠΌΠ΅Π½ΡΡŽΡ‚ ΠΏΠ°Ρ€Π°Π»Π»Π΅Π»ΡŒΠ½Ρ‹Π΅, ΠΈ Π½Π°ΠΎΠ±ΠΎΡ€ΠΎΡ‚. Для Π½ΠΈΡ… ΠΏΠΎ-ΠΏΡ€Π΅ΠΆΠ½Π΅ΠΌΡƒ справСдливы всС описанныС Π·Π°ΠΊΠΎΠ½Ρ‹. Волько ΠΏΡ€ΠΈΠΌΠ΅Π½ΡΡ‚ΡŒ ΠΈΡ… Π½ΡƒΠΆΠ½ΠΎ поэтапно.

Π‘ΠΏΠ΅Ρ€Π²Π° полагаСтся мыслСнно Ρ€Π°Π·Π²Π΅Ρ€Π½ΡƒΡ‚ΡŒ схСму. Если ΠΏΡ€Π΅Π΄ΡΡ‚Π°Π²ΠΈΡ‚ΡŒ Π΅Π΅ слоТно, Ρ‚ΠΎ Π½ΡƒΠΆΠ½ΠΎ Π½Π°Ρ€ΠΈΡΠΎΠ²Π°Ρ‚ΡŒ Ρ‚ΠΎ, Ρ‡Ρ‚ΠΎ получаСтся. ОбъяснСниС станСт понятнСС, Ссли Ρ€Π°ΡΡΠΌΠΎΡ‚Ρ€Π΅Ρ‚ΡŒ Π΅Π³ΠΎ Π½Π° ΠΊΠΎΠ½ΠΊΡ€Π΅Ρ‚Π½ΠΎΠΌ ΠΏΡ€ΠΈΠΌΠ΅Ρ€Π΅ (см. рисунок).

Π•Π΅ ΡƒΠ΄ΠΎΠ±Π½ΠΎ Π½Π°Ρ‡Π°Ρ‚ΡŒ Ρ€ΠΈΡΠΎΠ²Π°Ρ‚ΡŒ с Ρ‚ΠΎΡ‡Π΅ΠΊ Π‘ ΠΈ Π’. Π˜Ρ… Π½Π΅ΠΎΠ±Ρ…ΠΎΠ΄ΠΈΠΌΠΎ ΠΏΠΎΡΡ‚Π°Π²ΠΈΡ‚ΡŒ Π½Π° Π½Π΅ΠΊΠΎΡ‚ΠΎΡ€ΠΎΠΌ ΡƒΠ΄Π°Π»Π΅Π½ΠΈΠΈ Π΄Ρ€ΡƒΠ³ ΠΎΡ‚ Π΄Ρ€ΡƒΠ³Π° ΠΈ ΠΎΡ‚ ΠΊΡ€Π°Π΅Π² листа. Π‘Π»Π΅Π²Π° ΠΊ Ρ‚ΠΎΡ‡ΠΊΠ΅ Π‘ ΠΏΠΎΠ΄Ρ…ΠΎΠ΄ΠΈΡ‚ ΠΎΠ΄ΠΈΠ½ ΠΏΡ€ΠΎΠ²ΠΎΠ΄, Π° Π²ΠΏΡ€Π°Π²ΠΎ Π½Π°ΠΏΡ€Π°Π²Π»Π΅Π½Ρ‹ ΡƒΠΆΠ΅ Π΄Π²Π°. Π’ΠΎΡ‡ΠΊΠ° Π’, Π½Π°ΠΏΡ€ΠΎΡ‚ΠΈΠ², слСва ΠΈΠΌΠ΅Π΅Ρ‚ Π΄Π²Π° отвСтвлСния, Π° послС Π½Π΅Π΅ располоТСн ΠΎΠ΄ΠΈΠ½ ΠΏΡ€ΠΎΠ²ΠΎΠ΄.

Π’Π΅ΠΏΠ΅Ρ€ΡŒ Π½Π΅ΠΎΠ±Ρ…ΠΎΠ΄ΠΈΠΌΠΎ Π·Π°ΠΏΠΎΠ»Π½ΠΈΡ‚ΡŒ пространство ΠΌΠ΅ΠΆΠ΄Ρƒ этими Ρ‚ΠΎΡ‡ΠΊΠ°ΠΌΠΈ. По Π²Π΅Ρ€Ρ…Π½Π΅ΠΌΡƒ ΠΏΡ€ΠΎΠ²ΠΎΠ΄Ρƒ Π½ΡƒΠΆΠ½ΠΎ Ρ€Π°ΡΠΏΠΎΠ»ΠΎΠΆΠΈΡ‚ΡŒ Ρ‚Ρ€ΠΈ рСзистора с коэффициСнтами 2, 3 ΠΈ 4, Π° снизу ΠΏΠΎΠΉΠ΄Π΅Ρ‚ Ρ‚ΠΎΡ‚, Ρƒ ΠΊΠΎΡ‚ΠΎΡ€ΠΎΠ³ΠΎ индСкс Ρ€Π°Π²Π΅Π½ 5. ΠŸΠ΅Ρ€Π²Ρ‹Π΅ Ρ‚Ρ€ΠΈ соСдинСны ΠΏΠΎΡΠ»Π΅Π΄ΠΎΠ²Π°Ρ‚Π΅Π»ΡŒΠ½ΠΎ. Π‘ пятым рСзистором ΠΎΠ½ΠΈ ΠΏΠ°Ρ€Π°Π»Π»Π΅Π»ΡŒΠ½Ρ‹.

ΠžΡΡ‚Π°Π²ΡˆΠΈΠ΅ΡΡ Π΄Π²Π° рСзистора (ΠΏΠ΅Ρ€Π²Ρ‹ΠΉ ΠΈ ΡˆΠ΅ΡΡ‚ΠΎΠΉ) Π²ΠΊΠ»ΡŽΡ‡Π΅Π½Ρ‹ ΠΏΠΎΡΠ»Π΅Π΄ΠΎΠ²Π°Ρ‚Π΅Π»ΡŒΠ½ΠΎ с рассмотрСнным участком Π‘Π’. ΠŸΠΎΡΡ‚ΠΎΠΌΡƒ рисунок ΠΌΠΎΠΆΠ½ΠΎ просто Π΄ΠΎΠΏΠΎΠ»Π½ΠΈΡ‚ΡŒ двумя ΠΏΡ€ΡΠΌΠΎΡƒΠ³ΠΎΠ»ΡŒΠ½ΠΈΠΊΠ°ΠΌΠΈ ΠΏΠΎ ΠΎΠ±Π΅ стороны ΠΎΡ‚ Π²Ρ‹Π±Ρ€Π°Π½Π½Ρ‹Ρ… Ρ‚ΠΎΡ‡Π΅ΠΊ. ΠžΡΡ‚Π°Π»ΠΎΡΡŒ ΠΏΡ€ΠΈΠΌΠ΅Π½ΠΈΡ‚ΡŒ Ρ„ΠΎΡ€ΠΌΡƒΠ»Ρ‹ для расчСта сопротивлСния:

  • сначала Ρ‚Ρƒ, которая ΠΏΡ€ΠΈΠ²Π΅Π΄Π΅Π½Π° для ΠΏΠΎΡΠ»Π΅Π΄ΠΎΠ²Π°Ρ‚Π΅Π»ΡŒΠ½ΠΎΠ³ΠΎ соСдинСния;
  • ΠΏΠΎΡ‚ΠΎΠΌ для ΠΏΠ°Ρ€Π°Π»Π»Π΅Π»ΡŒΠ½ΠΎΠ³ΠΎ;
  • ΠΈ снова для ΠΏΠΎΡΠ»Π΅Π΄ΠΎΠ²Π°Ρ‚Π΅Π»ΡŒΠ½ΠΎΠ³ΠΎ.

ΠŸΠΎΠ΄ΠΎΠ±Π½Ρ‹ΠΌ ΠΎΠ±Ρ€Π°Π·ΠΎΠΌ ΠΌΠΎΠΆΠ½ΠΎ Ρ€Π°Π·Π²Π΅Ρ€Π½ΡƒΡ‚ΡŒ Π»ΡŽΠ±ΡƒΡŽ, Π΄Π°ΠΆΠ΅ ΠΎΡ‡Π΅Π½ΡŒ ΡΠ»ΠΎΠΆΠ½ΡƒΡŽ схСму.

Π—Π°Π΄Π°Ρ‡Π° Π½Π° ΠΏΠΎΡΠ»Π΅Π΄ΠΎΠ²Π°Ρ‚Π΅Π»ΡŒΠ½ΠΎΠ΅ соСдинСниС ΠΏΡ€ΠΎΠ²ΠΎΠ΄Π½ΠΈΠΊΠΎΠ²

УсловиС. Π’ Ρ†Π΅ΠΏΠΈ Π΄Ρ€ΡƒΠ³ Π·Π° Π΄Ρ€ΡƒΠ³ΠΎΠΌ подсоСдинСны Π΄Π²Π΅ Π»Π°ΠΌΠΏΡ‹ ΠΈ рСзистор. ΠžΠ±Ρ‰Π΅Π΅ напряТСниС Ρ€Π°Π²Π½ΠΎ 110 Π’, Π° сила Ρ‚ΠΎΠΊΠ° 12 А. Π§Π΅ΠΌΡƒ Ρ€Π°Π²Π½ΠΎ сопротивлСниС рСзистора, Ссли каТдая Π»Π°ΠΌΠΏΠ° рассчитана Π½Π° напряТСниС Π² 40 Π’?

РСшСниС. ΠŸΠΎΡΠΊΠΎΠ»ΡŒΠΊΡƒ рассматриваСтся ΠΏΠΎΡΠ»Π΅Π΄ΠΎΠ²Π°Ρ‚Π΅Π»ΡŒΠ½ΠΎΠ΅ соСдинСниС, Ρ„ΠΎΡ€ΠΌΡƒΠ»Ρ‹ Π΅Π³ΠΎ Π·Π°ΠΊΠΎΠ½ΠΎΠ² извСстны. НуТно Ρ‚ΠΎΠ»ΡŒΠΊΠΎ ΠΏΡ€Π°Π²ΠΈΠ»ΡŒΠ½ΠΎ ΠΈΡ… ΠΏΡ€ΠΈΠΌΠ΅Π½ΠΈΡ‚ΡŒ. ΠΠ°Ρ‡Π°Ρ‚ΡŒ с Ρ‚ΠΎΠ³ΠΎ, Ρ‡Ρ‚ΠΎΠ±Ρ‹ Π²Ρ‹ΡΡΠ½ΠΈΡ‚ΡŒ Π·Π½Π°Ρ‡Π΅Π½ΠΈΠ΅ напряТСния, ΠΊΠΎΡ‚ΠΎΡ€ΠΎΠ΅ приходится Π½Π° рСзистор. Для этого ΠΈΠ· ΠΎΠ±Ρ‰Π΅Π³ΠΎ Π½ΡƒΠΆΠ½ΠΎ Π²Ρ‹Ρ‡Π΅ΡΡ‚ΡŒ Π΄Π²Π° Ρ€Π°Π·Π° напряТСниС ΠΎΠ΄Π½ΠΎΠΉ Π»Π°ΠΌΠΏΡ‹. ΠŸΠΎΠ»ΡƒΡ‡Π°Π΅Ρ‚ΡΡ 30 Π’.

Π’Π΅ΠΏΠ΅Ρ€ΡŒ, ΠΊΠΎΠ³Π΄Π° извСстны Π΄Π²Π΅ Π²Π΅Π»ΠΈΡ‡ΠΈΠ½Ρ‹, U ΠΈ I (вторая ΠΈΠ· Π½ΠΈΡ… Π΄Π°Π½Π° Π² условии, Ρ‚Π°ΠΊ ΠΊΠ°ΠΊ ΠΎΠ±Ρ‰ΠΈΠΉ Ρ‚ΠΎΠΊ Ρ€Π°Π²Π΅Π½ Ρ‚ΠΎΠΊΡƒ Π² ΠΊΠ°ΠΆΠ΄ΠΎΠΌ ΠΏΠΎΡΠ»Π΅Π΄ΠΎΠ²Π°Ρ‚Π΅Π»ΡŒΠ½ΠΎΠΌ ΠΏΠΎΡ‚Ρ€Π΅Π±ΠΈΡ‚Π΅Π»Π΅), ΠΌΠΎΠΆΠ½ΠΎ ΡΠΎΡΡ‡ΠΈΡ‚Π°Ρ‚ΡŒ сопротивлСниС рСзистора ΠΏΠΎ Π·Π°ΠΊΠΎΠ½Ρƒ Ома. Оно оказываСтся Ρ€Π°Π²Π½Ρ‹ΠΌ 2,5 Ом.

ΠžΡ‚Π²Π΅Ρ‚. Π‘ΠΎΠΏΡ€ΠΎΡ‚ΠΈΠ²Π»Π΅Π½ΠΈΠ΅ рСзистора Ρ€Π°Π²Π½ΠΎ 2,5 Ом.

Π—Π°Π΄Π°Ρ‡Π° Π½Π° ΠΏΠ°Ρ€Π°Π»Π»Π΅Π»ΡŒΠ½ΠΎΠ΅ ΠΈ ΠΏΠΎΡΠ»Π΅Π΄ΠΎΠ²Π°Ρ‚Π΅Π»ΡŒΠ½ΠΎΠ΅

УсловиС. Π˜ΠΌΠ΅ΡŽΡ‚ΡΡ Ρ‚Ρ€ΠΈ кондСнсатора с Смкостями 20, 25 ΠΈ 30 ΠΌΠΊΠ€. ΠžΠΏΡ€Π΅Π΄Π΅Π»ΠΈΡ‚Π΅ ΠΈΡ… ΠΎΠ±Ρ‰ΡƒΡŽ Π΅ΠΌΠΊΠΎΡΡ‚ΡŒ ΠΏΡ€ΠΈ ΠΏΠΎΡΠ»Π΅Π΄ΠΎΠ²Π°Ρ‚Π΅Π»ΡŒΠ½ΠΎΠΌ ΠΈ ΠΏΠ°Ρ€Π°Π»Π»Π΅Π»ΡŒΠ½ΠΎΠΌ соСдинСнии.

РСшСниС. ΠŸΡ€ΠΎΡ‰Π΅ Π½Π°Ρ‡Π°Ρ‚ΡŒ с Π’ этой ситуации всС Ρ‚Ρ€ΠΈ значСния Π½ΡƒΠΆΠ½ΠΎ просто ΡΠ»ΠΎΠΆΠΈΡ‚ΡŒ. Π’Π°ΠΊΠΈΠΌ ΠΎΠ±Ρ€Π°Π·ΠΎΠΌ, общая Π΅ΠΌΠΊΠΎΡΡ‚ΡŒ оказываСтся Ρ€Π°Π²Π½ΠΎΠΉ 75 ΠΌΠΊΠ€.

НСсколько слоТнСС расчСты Π±ΡƒΠ΄ΡƒΡ‚ ΠΏΡ€ΠΈ ΠΏΠΎΡΠ»Π΅Π΄ΠΎΠ²Π°Ρ‚Π΅Π»ΡŒΠ½ΠΎΠΌ соСдинСнии этих кондСнсаторов. Π’Π΅Π΄ΡŒ сначала Π½ΡƒΠΆΠ½ΠΎ Π½Π°ΠΉΡ‚ΠΈ ΠΎΡ‚Π½ΠΎΡˆΠ΅Π½ΠΈΡ Π΅Π΄ΠΈΠ½ΠΈΡ†Ρ‹ ΠΊ ΠΊΠ°ΠΆΠ΄ΠΎΠΉ ΠΈΠ· этих СмкостСй, Π° ΠΏΠΎΡ‚ΠΎΠΌ ΡΠ»ΠΎΠΆΠΈΡ‚ΡŒ ΠΈΡ… Π΄Ρ€ΡƒΠ³ с Π΄Ρ€ΡƒΠ³ΠΎΠΌ. ΠŸΠΎΠ»ΡƒΡ‡Π°Π΅Ρ‚ΡΡ, Ρ‡Ρ‚ΠΎ Π΅Π΄ΠΈΠ½ΠΈΡ†Π°, дСлСнная Π½Π° ΠΎΠ±Ρ‰ΡƒΡŽ Π΅ΠΌΠΊΠΎΡΡ‚ΡŒ, Ρ€Π°Π²Π½Π° 37/300. Π’ΠΎΠ³Π΄Π° искомая Π²Π΅Π»ΠΈΡ‡ΠΈΠ½Π° получаСтся ΠΏΡ€ΠΈΠ±Π»ΠΈΠ·ΠΈΡ‚Π΅Π»ΡŒΠ½ΠΎ 8 ΠΌΠΊΠ€.

ΠžΡ‚Π²Π΅Ρ‚. ΠžΠ±Ρ‰Π°Ρ Π΅ΠΌΠΊΠΎΡΡ‚ΡŒ ΠΏΡ€ΠΈ ΠΏΠΎΡΠ»Π΅Π΄ΠΎΠ²Π°Ρ‚Π΅Π»ΡŒΠ½ΠΎΠΌ соСдинСнии 8 ΠΌΠΊΠ€, ΠΏΡ€ΠΈ ΠΏΠ°Ρ€Π°Π»Π»Π΅Π»ΡŒΠ½ΠΎΠΌ β€” 75 ΠΌΠΊΠ€.

ΠŸΡ€ΠΈ Ρ€Π΅ΡˆΠ΅Π½ΠΈΠΈ Π·Π°Π΄Π°Ρ‡ принято ΠΏΡ€Π΅ΠΎΠ±Ρ€Π°Π·ΠΎΠ²Ρ‹Π²Π°Ρ‚ΡŒ схСму, Ρ‚Π°ΠΊ, Ρ‡Ρ‚ΠΎΠ±Ρ‹ ΠΎΠ½Π° Π±Ρ‹Π»Π° ΠΊΠ°ΠΊ ΠΌΠΎΠΆΠ½ΠΎ ΠΏΡ€ΠΎΡ‰Π΅. Для этого ΠΏΡ€ΠΈΠΌΠ΅Π½ΡΡŽΡ‚ эквивалСнтныС прСобразования. Π­ΠΊΠ²ΠΈΠ²Π°Π»Π΅Π½Ρ‚Π½Ρ‹ΠΌΠΈ Π½Π°Π·Ρ‹Π²Π°ΡŽΡ‚ Ρ‚Π°ΠΊΠΈΠ΅ прСобразования части схСмы элСктричСской Ρ†Π΅ΠΏΠΈ, ΠΏΡ€ΠΈ ΠΊΠΎΡ‚ΠΎΡ€Ρ‹Ρ… Ρ‚ΠΎΠΊΠΈ ΠΈ напряТСния Π² Π½Π΅ ΠΏΡ€Π΅ΠΎΠ±Ρ€Π°Π·ΠΎΠ²Π°Π½Π½ΠΎΠΉ Π΅Ρ‘ части ΠΎΡΡ‚Π°ΡŽΡ‚ΡΡ Π½Π΅ΠΈΠ·ΠΌΠ΅Π½Π½Ρ‹ΠΌΠΈ.

БущСствуСт Ρ‡Π΅Ρ‚Ρ‹Ρ€Π΅ основных Π²ΠΈΠ΄Π° соСдинСния ΠΏΡ€ΠΎΠ²ΠΎΠ΄Π½ΠΈΠΊΠΎΠ²: ΠΏΠΎΡΠ»Π΅Π΄ΠΎΠ²Π°Ρ‚Π΅Π»ΡŒΠ½ΠΎΠ΅, ΠΏΠ°Ρ€Π°Π»Π»Π΅Π»ΡŒΠ½ΠΎΠ΅, смСшанноС ΠΈ мостовоС.

ΠŸΠΎΡΠ»Π΅Π΄ΠΎΠ²Π°Ρ‚Π΅Π»ΡŒΠ½ΠΎΠ΅ соСдинСниС

ΠŸΠΎΡΠ»Π΅Π΄ΠΎΠ²Π°Ρ‚Π΅Π»ΡŒΠ½ΠΎΠ΅ соСдинСниС – это Ρ‚Π°ΠΊΠΎΠ΅ соСдинСниС, ΠΏΡ€ΠΈ ΠΊΠΎΡ‚ΠΎΡ€ΠΎΠΌ сила Ρ‚ΠΎΠΊΠ° Π½Π° всСм участкС Ρ†Π΅ΠΏΠΈ ΠΎΠ΄ΠΈΠ½Π°ΠΊΠΎΠ²Π°. Π―Ρ€ΠΊΠΈΠΌ ΠΏΡ€ΠΈΠΌΠ΅Ρ€ΠΎΠΌ ΠΏΠΎΡΠ»Π΅Π΄ΠΎΠ²Π°Ρ‚Π΅Π»ΡŒΠ½ΠΎΠ³ΠΎ соСдинСния являСтся старая Слочная гирлянда. Π’Π°ΠΌ Π»Π°ΠΌΠΏΠΎΡ‡ΠΊΠΈ ΠΏΠΎΠ΄ΠΊΠ»ΡŽΡ‡Π΅Π½Ρ‹ ΠΏΠΎΡΠ»Π΅Π΄ΠΎΠ²Π°Ρ‚Π΅Π»ΡŒΠ½ΠΎ, Π΄Ρ€ΡƒΠ³ Π·Π° Π΄Ρ€ΡƒΠ³ΠΎΠΌ. Π’Π΅ΠΏΠ΅Ρ€ΡŒ ΠΏΡ€Π΅Π΄ΡΡ‚Π°Π²ΡŒΡ‚Π΅, ΠΎΠ΄Π½Π° Π»Π°ΠΌΠΏΠΎΡ‡ΠΊΠ° ΠΏΠ΅Ρ€Π΅Π³ΠΎΡ€Π°Π΅Ρ‚, Ρ†Π΅ΠΏΡŒ Π½Π°Ρ€ΡƒΡˆΠ΅Π½Π° ΠΈ ΠΎΡΡ‚Π°Π»ΡŒΠ½Ρ‹Π΅ Π»Π°ΠΌΠΏΠΎΡ‡ΠΊΠΈ гаснут. Π’Ρ‹Ρ…ΠΎΠ΄ ΠΈΠ· строя ΠΎΠ΄Π½ΠΎΠ³ΠΎ элСмСнта, Π²Π΅Π΄Π΅Ρ‚ Π·Π° собой ΠΎΡ‚ΠΊΠ»ΡŽΡ‡Π΅Π½ΠΈΠ΅ всСх ΠΎΡΡ‚Π°Π»ΡŒΠ½Ρ‹Ρ…, это являСтся сущСствСнным нСдостатком ΠΏΠΎΡΠ»Π΅Π΄ΠΎΠ²Π°Ρ‚Π΅Π»ΡŒΠ½ΠΎΠ³ΠΎ соСдинСния.

ΠŸΡ€ΠΈ ΠΏΠΎΡΠ»Π΅Π΄ΠΎΠ²Π°Ρ‚Π΅Π»ΡŒΠ½ΠΎΠΌ соСдинСнии сопротивлСния элСмСнтов ΡΡƒΠΌΠΌΠΈΡ€ΡƒΡŽΡ‚ΡΡ.

ΠŸΠ°Ρ€Π°Π»Π»Π΅Π»ΡŒΠ½ΠΎΠ΅ соСдинСниС

ΠŸΠ°Ρ€Π°Π»Π»Π΅Π»ΡŒΠ½ΠΎΠ΅ соСдинСниС – это соСдинСниС, ΠΏΡ€ΠΈ ΠΊΠΎΡ‚ΠΎΡ€ΠΎΠΌ напряТСниС Π½Π° ΠΊΠΎΠ½Ρ†Π°Ρ… участка Ρ†Π΅ΠΏΠΈ ΠΎΠ΄ΠΈΠ½Π°ΠΊΠΎΠ²ΠΎ. ΠŸΠ°Ρ€Π°Π»Π»Π΅Π»ΡŒΠ½ΠΎΠ΅ соСдинСниС Π½Π°ΠΈΠ±ΠΎΠ»Π΅Π΅ распространСно, Π² основном ΠΏΠΎΡ‚ΠΎΠΌΡƒ, Ρ‡Ρ‚ΠΎ всС элСмСнты находятся ΠΏΠΎΠ΄ ΠΎΠ΄Π½ΠΈΠΌ напряТСниСм, сила Ρ‚ΠΎΠΊΠ° распрСдСлСна ΠΏΠΎ-Ρ€Π°Π·Π½ΠΎΠΌΡƒ ΠΈ ΠΏΡ€ΠΈ Π²Ρ‹Ρ…ΠΎΠ΄Π΅ ΠΎΠ΄Π½ΠΎΠ³ΠΎ ΠΈΠ· элСмСнтов всС ΠΎΡΡ‚Π°Π»ΡŒΠ½Ρ‹Π΅ ΠΏΡ€ΠΎΠ΄ΠΎΠ»ΠΆΠ°ΡŽΡ‚ свою Ρ€Π°Π±ΠΎΡ‚Ρƒ.

ΠŸΡ€ΠΈ ΠΏΠ°Ρ€Π°Π»Π»Π΅Π»ΡŒΠ½ΠΎΠΌ соСдинСнии эквивалСнтноС сопротивлСниС находится ΠΊΠ°ΠΊ:

Π’ случаС Π΄Π²ΡƒΡ… ΠΏΠ°Ρ€Π°Π»Π»Π΅Π»ΡŒΠ½ΠΎ соСдинСнных рСзисторов

Π’ случаС Ρ‚Ρ€Π΅Ρ… ΠΏΠ°Ρ€Π°Π»Π»Π΅Π»ΡŒΠ½ΠΎ ΠΏΠΎΠ΄ΠΊΠ»ΡŽΡ‡Π΅Π½Π½Ρ‹Ρ… рСзисторов:

БмСшанноС соСдинСниС

БмСшанноС соСдинСниС – соСдинСниС, ΠΊΠΎΡ‚ΠΎΡ€ΠΎΠ΅ являСтся ΡΠΎΠ²ΠΎΠΊΡƒΠΏΠ½ΠΎΡΡ‚ΡŒΡŽ ΠΏΠΎΡΠ»Π΅Π΄ΠΎΠ²Π°Ρ‚Π΅Π»ΡŒΠ½Ρ‹Ρ… ΠΈ ΠΏΠ°Ρ€Π°Π»Π»Π΅Π»ΡŒΠ½Ρ‹Ρ… соСдинСний. Для нахоТдСния эквивалСнтного сопротивлСния Π½ΡƒΠΆΠ½ΠΎ, β€œΡΠ²Π΅Ρ€Π½ΡƒΡ‚ΡŒβ€ схСму ΠΏΠΎΠΎΡ‡Π΅Ρ€Π΅Π΄Π½Ρ‹ΠΌ ΠΏΡ€Π΅ΠΎΠ±Ρ€Π°Π·ΠΎΠ²Π°Π½ΠΈΠ΅ΠΌ ΠΏΠ°Ρ€Π°Π»Π»Π΅Π»ΡŒΠ½Ρ‹Ρ… ΠΈ ΠΏΠΎΡΠ»Π΅Π΄ΠΎΠ²Π°Ρ‚Π΅Π»ΡŒΠ½Ρ‹Ρ… участков Ρ†Π΅ΠΏΠΈ.


Π‘Π½Π°Ρ‡Π°Π»Π° Π½Π°ΠΉΠ΄Π΅ΠΌ эквивалСнтноС сопротивлСниС для ΠΏΠ°Ρ€Π°Π»Π»Π΅Π»ΡŒΠ½ΠΎΠ³ΠΎ участка Ρ†Π΅ΠΏΠΈ, Π° Π·Π°Ρ‚Π΅ΠΌ ΠΏΡ€ΠΈΠ±Π°Π²ΠΈΠΌ ΠΊ Π½Π΅ΠΌΡƒ ΠΎΡΡ‚Π°Π²ΡˆΠ΅Π΅ΡΡ сопротивлСниС R 3 . Π‘Π»Π΅Π΄ΡƒΠ΅Ρ‚ ΠΏΠΎΠ½ΠΈΠΌΠ°Ρ‚ΡŒ, Ρ‡Ρ‚ΠΎ послС прСобразования эквивалСнтноС сопротивлСниС R 1 R 2 ΠΈ рСзистор R 3 , соСдинСны ΠΏΠΎΡΠ»Π΅Π΄ΠΎΠ²Π°Ρ‚Π΅Π»ΡŒΠ½ΠΎ.

Π˜Ρ‚Π°ΠΊ, остаСтся самоС интСрСсноС ΠΈ самоС слоТноС соСдинСниС ΠΏΡ€ΠΎΠ²ΠΎΠ΄Π½ΠΈΠΊΠΎΠ².

ΠœΠΎΡΡ‚ΠΎΠ²Π°Ρ схСма

ΠœΠΎΡΡ‚ΠΎΠ²Π°Ρ схСма соСдинСния прСдставлСна Π½Π° рисункС Π½ΠΈΠΆΠ΅.



Для Ρ‚ΠΎΠ³ΠΎ Ρ‡Ρ‚ΠΎΠ±Ρ‹ ΡΠ²Π΅Ρ€Π½ΡƒΡ‚ΡŒ ΠΌΠΎΡΡ‚ΠΎΠ²ΡƒΡŽ схСму, ΠΎΠ΄ΠΈΠ½ ΠΈΠ· Ρ‚Ρ€Π΅ΡƒΠ³ΠΎΠ»ΡŒΠ½ΠΈΠΊΠΎΠ² моста, Π·Π°ΠΌΠ΅Π½ΡΡŽΡ‚ эквивалСнтной Π·Π²Π΅Π·Π΄ΠΎΠΉ.

И находят сопротивлСния R 1 , R 2 ΠΈ R 3 .

ΠŸΠΎΡΠ»Π΅Π΄ΠΎΠ²Π°Ρ‚Π΅Π»ΡŒΠ½ΠΎΠ΅ соСдинСниС ΠΏΡ€ΠΎΠ²ΠΎΠ΄Π½ΠΈΠΊΠΎΠ² | Π€ΠΈΠ·ΠΈΠΊΠ°

ЭлСктричСскиС Ρ†Π΅ΠΏΠΈ, ΠΈΡΠΏΠΎΠ»ΡŒΠ·ΡƒΠ΅ΠΌΡ‹Π΅ Π½Π° ΠΏΡ€Π°ΠΊΡ‚ΠΈΠΊΠ΅, содСрТат, ΠΊΠ°ΠΊ ΠΏΡ€Π°Π²ΠΈΠ»ΠΎ, нСсколько ΠΏΠΎΡ‚Ρ€Π΅Π±ΠΈΡ‚Π΅Π»Π΅ΠΉ элСктроэнСргии. Π­Ρ‚ΠΈ ΠΏΠΎΡ‚Ρ€Π΅Π±ΠΈΡ‚Π΅Π»ΠΈ ΠΌΠΎΠ³ΡƒΡ‚ Π±Ρ‹Ρ‚ΡŒ ΠΏΠΎ-Ρ€Π°Π·Π½ΠΎΠΌΡƒ соСдинСны Π΄Ρ€ΡƒΠ³ с Π΄Ρ€ΡƒΠ³ΠΎΠΌ, Π½Π°ΠΏΡ€ΠΈΠΌΠ΅Ρ€ ΠΏΠΎΡΠ»Π΅Π΄ΠΎΠ²Π°Ρ‚Π΅Π»ΡŒΠ½ΠΎ ΠΈΠ»ΠΈ ΠΏΠ°Ρ€Π°Π»Π»Π΅Π»ΡŒΠ½ΠΎ.

ΠŸΡ€ΠΈ ΠΏΠΎΡΠ»Π΅Π΄ΠΎΠ²Π°Ρ‚Π΅Π»ΡŒΠ½ΠΎΠΌ соСдинСнии ΠΏΠΎΡ‚Ρ€Π΅Π±ΠΈΡ‚Π΅Π»Π΅ΠΉ ΠΎΠ½ΠΈ Π²ΠΊΠ»ΡŽΡ‡Π°ΡŽΡ‚ΡΡ Π² Ρ†Π΅ΠΏΡŒ ΠΏΠΎΠΎΡ‡Π΅Ρ€Π΅Π΄Π½ΠΎ Π΄Ρ€ΡƒΠ³ Π·Π° Π΄Ρ€ΡƒΠ³ΠΎΠΌ Π±Π΅Π· Ρ€Π°Π·Π²Π΅Ρ‚Π²Π»Π΅Π½ΠΈΠΉ ΠΏΡ€ΠΎΠ²ΠΎΠ΄ΠΎΠ² ΠΌΠ΅ΠΆΠ΄Ρƒ Π½ΠΈΠΌΠΈ. ИмСнно Ρ‚Π°ΠΊ соСдинСны рСзисторы, ΠΈΠ·ΠΎΠ±Ρ€Π°ΠΆΠ΅Π½Π½Ρ‹Π΅ Π½Π° рисункС 41. Π€ΠΎΡ€ΠΌΠ° Π»ΠΈΠ½ΠΈΠΉ, ΠΎΠ±ΠΎΠ·Π½Π°Ρ‡Π°ΡŽΡ‰ΠΈΡ… ΠΏΡ€ΠΈ этом ΡΠΎΠ΅Π΄ΠΈΠ½ΠΈΡ‚Π΅Π»ΡŒΠ½Ρ‹Π΅ ΠΏΡ€ΠΎΠ²ΠΎΠ΄Π°, Π½Π΅ ΠΈΠ³Ρ€Π°Π΅Ρ‚ Ρ€ΠΎΠ»ΠΈ, ΠΈ ΠΏΠΎΡ‚ΠΎΠΌΡƒ схСма Ρ†Π΅ΠΏΠΈ ΠΏΡ€ΠΈ ΠΎΠ΄Π½ΠΎΠΌ ΠΈ Ρ‚ΠΎΠΌ ΠΆΠ΅ Ρ‚ΠΈΠΏΠ΅ соСдинСния ΠΌΠΎΠΆΠ΅Ρ‚ Π²Ρ‹Π³Π»ΡΠ΄Π΅Ρ‚ΡŒ ΠΏΠΎ-Ρ€Π°Π·Π½ΠΎΠΌΡƒ.

ΠžΠ±ΠΎΠ·Π½Π°Ρ‡ΠΈΠΌ Ρ‡Π΅Ρ€Π΅Π· I1, U1 ΠΈ R1 силу Ρ‚ΠΎΠΊΠ°, напряТСниС ΠΈ сопротивлСниС Π½Π° ΠΏΠ΅Ρ€Π²ΠΎΠΌ участкС Ρ†Π΅ΠΏΠΈ (ΠΏΠ΅Ρ€Π²ΠΎΠΌ рСзисторС Π½Π° рисункС 41, Π°), Π° Ρ‡Π΅Ρ€Π΅Π· I2, U2 ΠΈ R2 силу Ρ‚ΠΎΠΊΠ°, напряТСниС ΠΈ сопротивлСниС Π½Π° Π²Ρ‚ΠΎΡ€ΠΎΠΌ участкС Ρ†Π΅ΠΏΠΈ (Π²Ρ‚ΠΎΡ€ΠΎΠΌ рСзисторС Π½Π° рисункС 41, Π°). ΠžΠ±Ρ‰Π΅Π΅ сопротивлСниС ΠΎΠ±ΠΎΠΈΡ… участков ΠΎΠ±ΠΎΠ·Π½Π°Ρ‡ΠΈΠΌ Ρ‡Π΅Ρ€Π΅Π· R, ΠΎΠ±Ρ‰Π΅Π΅ напряТСниС Π½Π° Π½ΠΈΡ… β€” Ρ‡Π΅Ρ€Π΅Π· U, Π° ΠΎΠ±Ρ‰ΡƒΡŽ силу Ρ‚ΠΎΠΊΠ°, которая совпадаСт с силой Ρ‚ΠΎΠΊΠ° Π²Π½ΡƒΡ‚Ρ€ΠΈ источника, β€” Ρ‡Π΅Ρ€Π΅Π· I. Π’ΠΎΠ³Π΄Π° связь ΠΌΠ΅ΠΆΠ΄Ρƒ ΠΎΠ±Ρ‰ΠΈΠΌΠΈ значСниями силы Ρ‚ΠΎΠΊΠ°, напряТСния ΠΈ сопротивлСния с ΠΈΡ… значСниями Π½Π° ΠΎΡ‚Π΄Π΅Π»ΡŒΠ½Ρ‹Ρ… участках Ρ†Π΅ΠΏΠΈ ΠΌΠΎΠΆΠ΅Ρ‚ Π±Ρ‹Ρ‚ΡŒ Π²Ρ‹Ρ€Π°ΠΆΠ΅Π½Π° Π² Π²ΠΈΠ΄Π΅ ΡΠ»Π΅Π΄ΡƒΡŽΡ‰ΠΈΡ… ΡΠΎΠΎΡ‚Π½ΠΎΡˆΠ΅Π½ΠΈΠΉ:

I = I1 = I2, (16.1) U = U1 + U2, (16.2) R = R1 + R2. (16.3)

Π§Ρ‚ΠΎΠ±Ρ‹ ΡƒΠ±Π΅Π΄ΠΈΡ‚ΡŒΡΡ Π² справСдливости этих ΡΠΎΠΎΡ‚Π½ΠΎΡˆΠ΅Π½ΠΈΠΉ, слСдуСт ΡΠΎΠ±Ρ€Π°Ρ‚ΡŒ ΡΠΎΠΎΡ‚Π²Π΅Ρ‚ΡΡ‚Π²ΡƒΡŽΡ‰ΡƒΡŽ Ρ†Π΅ΠΏΡŒ ΠΈ с ΠΏΠΎΠΌΠΎΡ‰ΡŒΡŽ Π°ΠΌΠΏΠ΅Ρ€ΠΌΠ΅Ρ‚Ρ€Π° ΠΈ Π²ΠΎΠ»ΡŒΡ‚ΠΌΠ΅Ρ‚Ρ€Π° произвСсти Π½Π΅ΠΎΠ±Ρ…ΠΎΠ΄ΠΈΠΌΡ‹Π΅ измСрСния.

Π˜Ρ‚Π°ΠΊ, ΠΏΡ€ΠΈ ΠΏΠΎΡΠ»Π΅Π΄ΠΎΠ²Π°Ρ‚Π΅Π»ΡŒΠ½ΠΎΠΌ соСдинСнии ΠΏΡ€ΠΎΠ²ΠΎΠ΄Π½ΠΈΠΊΠΎΠ² сила Ρ‚ΠΎΠΊΠ° Π²Π΅Π·Π΄Π΅ ΠΎΠ΄ΠΈΠ½Π°ΠΊΠΎΠ²Π°, напряТСниС Π² Ρ†Π΅ΠΏΠΈ Ρ€Π°Π²Π½ΠΎ суммС напряТСний Π½Π° ΠΎΡ‚Π΄Π΅Π»ΡŒΠ½Ρ‹Ρ… участках, Π° ΠΎΠ±Ρ‰Π΅Π΅ сопротивлСниС складываСтся ΠΈΠ· сопротивлСний ΠΎΡ‚Π΄Π΅Π»ΡŒΠ½Ρ‹Ρ… ΠΏΡ€ΠΎΠ²ΠΎΠ΄Π½ΠΈΠΊΠΎΠ².

Π‘ΠΎΠΎΡ‚Π½ΠΎΡˆΠ΅Π½ΠΈΡ (16.1)-(16.3) Π΄ΠΎΠΏΡƒΡΠΊΠ°ΡŽΡ‚ ΠΎΠ±ΠΎΠ±Ρ‰Π΅Π½ΠΈΠ΅: всС ΠΏΡ€ΠΈΠ²Π΅Π΄Π΅Π½Π½Ρ‹Π΅ закономСрности справСдливы для любого числа ΠΏΠΎΡΠ»Π΅Π΄ΠΎΠ²Π°Ρ‚Π΅Π»ΡŒΠ½ΠΎ соСдинСнных ΠΏΡ€ΠΎΠ²ΠΎΠ΄Π½ΠΈΠΊΠΎΠ².

Из равСнства (16.3) слСдуСт, Ρ‡Ρ‚ΠΎ ΠΎΠ±Ρ‰Π΅Π΅ сопротивлСниС ΠΏΠΎΡΠ»Π΅Π΄ΠΎΠ²Π°Ρ‚Π΅Π»ΡŒΠ½ΠΎ соСдинСнных ΠΏΡ€ΠΎΠ²ΠΎΠ΄Π½ΠΈΠΊΠΎΠ² всСгда ΠΏΡ€Π΅Π²Ρ‹ΡˆΠ°Π΅Ρ‚ сопротивлСниС любого ΠΈΠ· Π½ΠΈΡ…. Π­Ρ‚ΠΎ ΠΈ понятно: вСдь, соСдиняя ΠΏΡ€ΠΎΠ²ΠΎΠ΄Π½ΠΈΠΊΠΈ ΠΏΠΎΡΠ»Π΅Π΄ΠΎΠ²Π°Ρ‚Π΅Π»ΡŒΠ½ΠΎ, ΠΌΡ‹ ΠΊΠ°ΠΊ Π±Ρ‹ ΡƒΠ²Π΅Π»ΠΈΡ‡ΠΈΠ²Π°Π΅ΠΌ ΠΈΡ… ΠΎΠ±Ρ‰ΡƒΡŽ Π΄Π»ΠΈΠ½Ρƒ, Π° с ΡƒΠ²Π΅Π»ΠΈΡ‡Π΅Π½ΠΈΠ΅ΠΌ Π΄Π»ΠΈΠ½Ρ‹ возрастаСт ΠΈ сопротивлСниС.

ΠŸΡ€ΠΈ ΠΏΠΎΡΠ»Π΅Π΄ΠΎΠ²Π°Ρ‚Π΅Π»ΡŒΠ½ΠΎΠΌ соСдинСнии n ΠΎΠ΄ΠΈΠ½Π°ΠΊΠΎΠ²Ρ‹Ρ… элСмСнтов (рСзисторов, Π»Π°ΠΌΠΏ ΠΈ Ρ‚. Π΄.) ΠΈΡ… ΠΎΠ±Ρ‰Π΅Π΅ сопротивлСниС R ΠΏΡ€Π΅Π²Ρ‹ΡˆΠ°Π΅Ρ‚ сопротивлСниС R1 ΠΎΠ΄Π½ΠΎΠ³ΠΎ ΠΈΠ· Π½ΠΈΡ… Π² n Ρ€Π°Π·:

R = nR1

ΠžΠ±Ρ‰Π΅Π΅ напряТСниС U ΠΏΡ€ΠΈ этом дСлится Π½Π° n Ρ€Π°Π²Π½Ρ‹Ρ… частСй, Ρ‚Π°ΠΊ Ρ‡Ρ‚ΠΎ ΠΊΠ°ΠΆΠ΄Ρ‹ΠΉ ΠΈΠ· элСмСнтов Ρ†Π΅ΠΏΠΈ оказываСтся ΠΏΠΎΠ΄ напряТСниСм U1 Π² n Ρ€Π°Π· мСньшим ΠΎΠ±Ρ‰Π΅Π³ΠΎ значСния. НапримСр, ΠΏΡ€ΠΈ Π²ΠΊΠ»ΡŽΡ‡Π΅Π½ΠΈΠΈ Π² ΡΠ΅Ρ‚ΡŒ с напряТСниСм U = 220 Π’ дСсяти ΠΏΠΎΡΠ»Π΅Π΄ΠΎΠ²Π°Ρ‚Π΅Π»ΡŒΠ½ΠΎ соСдинСнных ΠΎΠ΄ΠΈΠ½Π°ΠΊΠΎΠ²Ρ‹Ρ… Π»Π°ΠΌΠΏ каТдая ΠΈΠ· Π½ΠΈΡ… оказываСтся ΠΏΠΎΠ΄ напряТСниСм U1 = U/10 = 22 Π’.

ΠžΡ‚Π»ΠΈΡ‡ΠΈΡ‚Π΅Π»ΡŒΠ½ΠΎΠΉ ΠΎΡΠΎΠ±Π΅Π½Π½ΠΎΡΡ‚ΡŒΡŽ ΠΏΠΎΡΠ»Π΅Π΄ΠΎΠ²Π°Ρ‚Π΅Π»ΡŒΠ½ΠΎΠ³ΠΎ соСдинСния ΠΏΡ€ΠΎΠ²ΠΎΠ΄Π½ΠΈΠΊΠΎΠ² являСтся Ρ‚ΠΎ, Ρ‡Ρ‚ΠΎ ΠΏΡ€ΠΈ ΠΎΡ‚ΠΊΠ°Π·Π΅ Π² Ρ€Π°Π±ΠΎΡ‚Π΅ хотя Π±Ρ‹ ΠΎΠ΄Π½ΠΎΠ³ΠΎ ΠΈΠ· Π½ΠΈΡ… Ρ‚ΠΎΠΊ прСкращаСтся сразу Π²ΠΎ всСй Ρ†Π΅ΠΏΠΈ. Π’Ρ‹Π²Π΅Ρ€Π½ΡƒΠ², Π½Π°ΠΏΡ€ΠΈΠΌΠ΅Ρ€, ΠΎΠ΄Π½Ρƒ ΠΈΠ· Π»Π°ΠΌΠΏ, ΠΈΠ·ΠΎΠ±Ρ€Π°ΠΆΠ΅Π½Π½Ρ‹Ρ… Π½Π° рисункС 42, ΠΌΡ‹ ΡƒΠ²ΠΈΠ΄ΠΈΠΌ, ΠΊΠ°ΠΊ Ρ‚ΡƒΡ‚ ΠΆΠ΅ пСрСстанСт Π³ΠΎΡ€Π΅Ρ‚ΡŒ ΠΈ другая (ΠΎΡΡ‚Π°Π²ΡˆΠ°ΡΡΡ) Π»Π°ΠΌΠΏΠ°. Π’Π°ΠΊ Ρ‡Ρ‚ΠΎ, Ссли Π²Ρ‹ украситС новогоднюю Π΅Π»ΠΊΡƒ гирляндой ΠΈΠ· ΠΏΠΎΡΠ»Π΅Π΄ΠΎΠ²Π°Ρ‚Π΅Π»ΡŒΠ½ΠΎ соСдинСнных Π»Π°ΠΌΠΏΠΎΡ‡Π΅ΠΊ ΠΈ какая-Ρ‚ΠΎ ΠΈΠ· Π½ΠΈΡ… ΠΏΠ΅Ρ€Π΅Π³ΠΎΡ€ΠΈΡ‚, Ρ‚ΠΎ погаснСт Π½Π΅ Ρ‚ΠΎΠ»ΡŒΠΊΠΎ ΠΎΠ½Π°, Π½ΠΎ ΠΈ всС ΠΎΡΡ‚Π°Π»ΡŒΠ½Ρ‹Π΅ Ρ‚ΠΎΠΆΠ΅. ΠŸΠΎΡΡ‚ΠΎΠΌΡƒ, Ρ‡Ρ‚ΠΎΠ±Ρ‹ ΠΎΠΏΡ€Π΅Π΄Π΅Π»ΠΈΡ‚ΡŒ, какая ΠΈΠ· Π»Π°ΠΌΠΏΠΎΡ‡Π΅ΠΊ ΠΏΠ΅Ρ€Π΅Π³ΠΎΡ€Π΅Π»Π°, Π²Π°ΠΌ придСтся ΠΏΡ€ΠΎΠ²Π΅Ρ€ΠΈΡ‚ΡŒ всю гирлянду.

1. КакоС соСдинСниС ΠΏΡ€ΠΎΠ²ΠΎΠ΄Π½ΠΈΠΊΠΎΠ² Π½Π°Π·Ρ‹Π²Π°ΡŽΡ‚ ΠΏΠΎΡΠ»Π΅Π΄ΠΎΠ²Π°Ρ‚Π΅Π»ΡŒΠ½Ρ‹ΠΌ? 2. НачСртитС схСму Ρ†Π΅ΠΏΠΈ, ΠΈΠ·ΠΎΠ±Ρ€Π°ΠΆΠ΅Π½Π½ΠΎΠΉ Π½Π° рисункС 42. 3. КакиС Ρ‚Ρ€ΠΈ закономСрности справСдливы для ΠΏΠΎΡΠ»Π΅Π΄ΠΎΠ²Π°Ρ‚Π΅Π»ΡŒΠ½ΠΎΠ³ΠΎ соСдинСния ΠΏΡ€ΠΎΠ²ΠΎΠ΄Π½ΠΈΠΊΠΎΠ²? 4. Как находится ΠΎΠ±Ρ‰Π΅Π΅ сопротивлСниС ΠΏΠΎΡΠ»Π΅Π΄ΠΎΠ²Π°Ρ‚Π΅Π»ΡŒΠ½ΠΎ соСдинСнных ΠΏΡ€ΠΎΠ²ΠΎΠ΄Π½ΠΈΠΊΠΎΠ² Π² случаС, ΠΊΠΎΠ³Π΄Π° ΠΎΠ½ΠΈ ΠΎΠ΄ΠΈΠ½Π°ΠΊΠΎΠ²Ρ‹Π΅? Как Π² этом случаС распрСдСляСтся ΠΌΠ΅ΠΆΠ΄Ρƒ ΠΏΡ€ΠΎΠ²ΠΎΠ΄Π½ΠΈΠΊΠ°ΠΌΠΈ ΠΎΠ±Ρ‰Π΅Π΅ напряТСниС?

ΠŸΠΎΡΠ»Π΅Π΄ΠΎΠ²Π°Ρ‚Π΅Π»ΡŒΠ½ΠΎΠ΅ соСдинСниС рСзисторов. Π‘Ρ…Π΅ΠΌΠ° соСдинСния ΠΈ ΠΏΡ€ΠΈΠΌΠ΅Ρ€Ρ‹ расчСта

Π’ΠΎ ΠΌΠ½ΠΎΠ³ΠΈΡ… элСктричСских схСмах ΠΌΡ‹ ΠΌΠΎΠΆΠ΅ΠΌ ΠΎΠ±Π½Π°Ρ€ΡƒΠΆΠΈΡ‚ΡŒ ΠΏΠΎΡΠ»Π΅Π΄ΠΎΠ²Π°Ρ‚Π΅Π»ΡŒΠ½ΠΎΠ΅ ΠΈ ΠΏΠ°Ρ€Π°Π»Π»Π΅Π»ΡŒΠ½ΠΎΠ΅ соСдинСниС рСзисторов. Π Π°Π·Ρ€Π°Π±ΠΎΡ‚Ρ‡ΠΈΠΊ схСм ΠΌΠΎΠΆΠ΅Ρ‚, Π½Π°ΠΏΡ€ΠΈΠΌΠ΅Ρ€, ΠΎΠ±ΡŠΠ΅Π΄ΠΈΠ½ΠΈΡ‚ΡŒ нСсколько рСзисторов со стандартными значСниями (E-сСрии), Ρ‡Ρ‚ΠΎΠ±Ρ‹ ΠΏΠΎΠ»ΡƒΡ‡ΠΈΡ‚ΡŒ Π½Π΅ΠΎΠ±Ρ…ΠΎΠ΄ΠΈΠΌΠΎΠ΅ сопротивлСниС.

ΠŸΠΎΡΠ»Π΅Π΄ΠΎΠ²Π°Ρ‚Π΅Π»ΡŒΠ½ΠΎΠ΅ соСдинСнии рСзисторов β€” это Ρ‚Π°ΠΊΠΎΠ΅ соСдинСниС, ΠΏΡ€ΠΈ ΠΊΠΎΡ‚ΠΎΡ€ΠΎΠΌ Ρ‚ΠΎΠΊ, ΠΏΡ€ΠΎΡ‚Π΅ΠΊΠ°ΡŽΡ‰ΠΈΠΉ Ρ‡Π΅Ρ€Π΅Π· ΠΊΠ°ΠΆΠ΄Ρ‹ΠΉ рСзистор ΠΎΠ΄ΠΈΠ½Π°ΠΊΠΎΠ², ΠΏΠΎΡΠΊΠΎΠ»ΡŒΠΊΡƒ имССтся Ρ‚ΠΎΠ»ΡŒΠΊΠΎ ΠΎΠ΄Π½ΠΎ Π½Π°ΠΏΡ€Π°Π²Π»Π΅Π½ΠΈΠ΅ для протСкания Ρ‚ΠΎΠΊΠ°.Β Π’ Ρ‚ΠΎΠΆΠ΅ врСмя ΠΏΠ°Π΄Π΅Π½ΠΈΠ΅ напряТСния Π±ΡƒΠ΄Π΅Ρ‚ ΠΏΡ€ΠΎΠΏΠΎΡ€Ρ†ΠΈΠΎΠ½Π°Π»ΡŒΠ½ΠΎ ΡΠΎΠΏΡ€ΠΎΡ‚ΠΈΠ²Π»Π΅Π½ΠΈΡŽ ΠΊΠ°ΠΆΠ΄ΠΎΠ³ΠΎ рСзистора Π² ΠΏΠΎΡΠ»Π΅Π΄ΠΎΠ²Π°Ρ‚Π΅Π»ΡŒΠ½ΠΎΠΉ Ρ†Π΅ΠΏΠΈ.

ΠŸΡ€ΠΎΡ„Π΅ΡΡΠΈΠΎΠ½Π°Π»ΡŒΠ½Ρ‹ΠΉ Ρ†ΠΈΡ„Ρ€ΠΎΠ²ΠΎΠΉ осциллограф

ΠšΠΎΠ»ΠΈΡ‡Π΅ΡΡ‚Π²ΠΎ ΠΊΠ°Π½Π°Π»ΠΎΠ²: 1, Ρ€Π°Π·ΠΌΠ΅Ρ€ экрана: 2,4 дюйма, Ρ€Π°Π·Ρ€Π΅ΡˆΠ΅Π½…

ΠŸΠΎΡΠ»Π΅Π΄ΠΎΠ²Π°Ρ‚Π΅Π»ΡŒΠ½ΠΎΠ΅ соСдинСниС рСзисторов

На рисункС Π½ΠΈΠΆΠ΅, рСзисторы R1, R2 ΠΈ R3 связаны Π΄Ρ€ΡƒΠ³ с Π΄Ρ€ΡƒΠ³ΠΎΠΌ ΠΏΠΎΡΠ»Π΅Π΄ΠΎΠ²Π°Ρ‚Π΅Π»ΡŒΠ½ΠΎ ΠΌΠ΅ΠΆΠ΄Ρƒ Ρ‚ΠΎΡ‡ΠΊΠ°ΠΌΠΈ А ΠΈ Π’ с ΠΎΠ±Ρ‰ΠΈΠΌ Ρ‚ΠΎΠΊΠΎΠΌ I, ΠΊΠΎΡ‚ΠΎΡ€Ρ‹ΠΉ ΠΏΡ€ΠΎΡ‚Π΅ΠΊΠ°Π΅Ρ‚ Ρ‡Π΅Ρ€Π΅Π· Π½ΠΈΡ….

Π­ΠΊΠ²ΠΈΠ²Π°Π»Π΅Π½Ρ‚Π½ΠΎΠ΅ сопротивлСниС Π½Π΅ΡΠΊΠΎΠ»ΡŒΠΊΠΈΡ… ΠΏΠΎΡΠ»Π΅Π΄ΠΎΠ²Π°Ρ‚Π΅Π»ΡŒΠ½ΠΎ соСдинСнных рСзисторов ΠΌΠΎΠΆΠ½ΠΎ ΠΎΠΏΡ€Π΅Π΄Π΅Π»ΠΈΡ‚ΡŒ ΠΏΠΎ ΡΠ»Π΅Π΄ΡƒΡŽΡ‰Π΅ΠΉ Ρ„ΠΎΡ€ΠΌΡƒΠ»Π΅:

R = R1 + R2 + R3

Π’ΠΎ Π΅ΡΡ‚ΡŒ, Π² нашСм случаС ΠΎΠ±Ρ‰Π΅Π΅ сопротивлСниС Ρ†Π΅ΠΏΠΈ Π±ΡƒΠ΄Π΅Ρ‚ Ρ€Π°Π²Π½ΠΎ:

R = R1 + R2 + R3 = 1 кОм + 2 кОм + 6 кОм = 9 кОм

Π’Π°ΠΊΠΈΠΌ ΠΎΠ±Ρ€Π°Π·ΠΎΠΌ, ΠΌΡ‹ ΠΌΠΎΠΆΠ΅ΠΌ Π·Π°ΠΌΠ΅Π½ΠΈΡ‚ΡŒ эти Ρ‚Ρ€ΠΈ рСзистора всСго лишь ΠΎΠ΄Π½ΠΈΠΌ «эквивалСнтным» рСзистором, ΠΊΠΎΡ‚ΠΎΡ€Ρ‹ΠΉ Π±ΡƒΠ΄Π΅Ρ‚ ΠΈΠΌΠ΅Ρ‚ΡŒ Π·Π½Π°Ρ‡Π΅Π½ΠΈΠ΅ 9 кОм.

ВСстСр транзисторов / ESR-ΠΌΠ΅Ρ‚Ρ€ / Π³Π΅Π½Π΅Ρ€Π°Ρ‚ΠΎΡ€

ΠœΠ½ΠΎΠ³ΠΎΡ„ΡƒΠ½ΠΊΡ†ΠΈΠΎΠ½Π°Π»ΡŒΠ½Ρ‹ΠΉ ΠΏΡ€ΠΈΠ±ΠΎΡ€ для ΠΏΡ€ΠΎΠ²Π΅Ρ€ΠΊΠΈ транзисторов, Π΄ΠΈΠΎΠ΄ΠΎΠ², тиристоров…

Π’Π°ΠΌ, Π³Π΄Π΅ Ρ‡Π΅Ρ‚Ρ‹Ρ€Π΅, ΠΏΡΡ‚ΡŒ ΠΈΠ»ΠΈ Π±ΠΎΠ»Π΅Π΅ рСзисторов связаны вмСстС Π² ΠΏΠΎΡΠ»Π΅Π΄ΠΎΠ²Π°Ρ‚Π΅Π»ΡŒΠ½ΡƒΡŽ Ρ†Π΅ΠΏΡŒ, ΠΎΠ±Ρ‰Π΅Π΅ ΠΈΠ»ΠΈ эквивалСнтноС сопротивлСниС всСй Ρ†Π΅ΠΏΠΈ Ρ‚Π°ΠΊ ΠΆΠ΅ Π±ΡƒΠ΄Π΅Ρ‚ Ρ€Π°Π²Π½ΠΎ суммС сопротивлСний ΠΎΡ‚Π΄Π΅Π»ΡŒΠ½Ρ‹Ρ… рСзисторов.

Π‘Π»Π΅Π΄ΡƒΠ΅Ρ‚ ΠΎΡ‚ΠΌΠ΅Ρ‚ΠΈΡ‚ΡŒ, Ρ‡Ρ‚ΠΎ ΠΎΠ±Ρ‰Π΅Π΅ сопротивлСниС Π»ΡŽΠ±Ρ‹Ρ… Π΄Π²ΡƒΡ… ΠΈΠ»ΠΈ Π±ΠΎΠ»Π΅Π΅ рСзисторов, соСдинСнных ΠΏΠΎΡΠ»Π΅Π΄ΠΎΠ²Π°Ρ‚Π΅Π»ΡŒΠ½ΠΎ всСгда Π±ΡƒΠ΄Π΅Ρ‚ большС, Ρ‡Π΅ΠΌ самоС большоС сопротивлСниС рСзистора входящСго Π² эту Ρ†Π΅ΠΏΡŒ. Π’ ΠΏΡ€ΠΈΠ²Π΅Π΄Π΅Π½Π½ΠΎΠΌ Π²Ρ‹ΡˆΠ΅ ΠΏΡ€ΠΈΠΌΠ΅Ρ€Π΅ R = 9 кОм, Ρ‚ΠΎΠ³Π΄Π° ΠΊΠ°ΠΊ наибольшСС Π·Π½Π°Ρ‡Π΅Π½ΠΈΠ΅ рСзистора Ρ‚ΠΎΠ»ΡŒΠΊΠΎ 6 кОм (R3).

НапряТСниС Π½Π° ΠΊΠ°ΠΆΠ΄ΠΎΠΌ ΠΈΠ· рСзисторов, соСдинСнных ΠΏΠΎΡΠ»Π΅Π΄ΠΎΠ²Π°Ρ‚Π΅Π»ΡŒΠ½ΠΎ, ΠΏΠΎΠ΄Ρ‡ΠΈΠ½Π΅Π½ΠΎ Π΄Ρ€ΡƒΠ³ΠΎΠΌΡƒ ΠΏΡ€Π°Π²ΠΈΠ»Ρƒ, Π½Π΅ΠΆΠ΅Π»ΠΈ ΠΏΡ€ΠΎΡ‚Π΅ΠΊΠ°ΡŽΡ‰ΠΈΠΉ Ρ‚ΠΎΠΊ. Как извСстно, ΠΈΠ· ΠΏΡ€ΠΈΠ²Π΅Π΄Π΅Π½Π½ΠΎΠΉ Π²Ρ‹ΡˆΠ΅ схСмы, Ρ‡Ρ‚ΠΎ ΠΎΠ±Ρ‰Π΅Π΅ напряТСниС питания Π½Π° рСзисторах Ρ€Π°Π²Π½ΠΎ суммС разности ΠΏΠΎΡ‚Π΅Π½Ρ†ΠΈΠ°Π»Π° Π½Π° ΠΊΠ°ΠΆΠ΄ΠΎΠΌ ΠΈΠ· Π½ΠΈΡ…:

Π˜ΡΠΏΠΎΠ»ΡŒΠ·ΡƒΡ Π·Π°ΠΊΠΎΠ½ Ома , напряТСниС Π½Π° ΠΎΡ‚Π΄Π΅Π»ΡŒΠ½Ρ‹Ρ… рСзисторов ΠΌΠΎΠΆΠ΅Ρ‚ Π±Ρ‹Ρ‚ΡŒ вычислСна ΡΠ»Π΅Π΄ΡƒΡŽΡ‰ΠΈΠΌ ΠΎΠ±Ρ€Π°Π·ΠΎΠΌ:

Π’ ΠΈΡ‚ΠΎΠ³Π΅ сумма разностСй ΠΏΠΎΡ‚Π΅Π½Ρ†ΠΈΠ°Π»ΠΎΠ² Π½Π° рСзисторах Ρ€Π°Π²Π½Π° ΠΎΠ±Ρ‰Π΅ΠΉ разности ΠΏΠΎΡ‚Π΅Π½Ρ†ΠΈΠ°Π»ΠΎΠ² всСй Ρ†Π΅ΠΏΠΈ, нашСм ΠΏΡ€ΠΈΠΌΠ΅Ρ€Π΅ это 9Π’.

Π’ частности, ряд рСзисторов, соСдинСнных ΠΏΠΎΡΠ»Π΅Π΄ΠΎΠ²Π°Ρ‚Π΅Π»ΡŒΠ½ΠΎ, ΠΌΠΎΠΆΠ½ΠΎ Ρ€Π°ΡΡΠΌΠ°Ρ‚Ρ€ΠΈΠ²Π°Ρ‚ΡŒ ΠΊΠ°ΠΊ Π΄Π΅Π»ΠΈΡ‚Π΅Π»ΡŒ напряТСния:

ΠŸΡ€ΠΈΠΌΠ΅Ρ€ β„– 1

Π˜ΡΠΏΠΎΠ»ΡŒΠ·ΡƒΡ Π·Π°ΠΊΠΎΠ½ Ома, Π½Π΅ΠΎΠ±Ρ…ΠΎΠ΄ΠΈΠΌΠΎ Π²Ρ‹Ρ‡ΠΈΡΠ»ΠΈΡ‚ΡŒ эквивалСнтноС сопротивлСниС сСрии ΠΏΠΎΡΠ»Π΅Π΄ΠΎΠ²Π°Ρ‚Π΅Π»ΡŒΠ½ΠΎ соСдинСнных рСзисторов (R1. R2, R3), Π° Ρ‚Π°ΠΊ ΠΆΠ΅ ΠΏΠ°Π΄Π΅Π½ΠΈΠ΅ напряТСния ΠΈ ΠΌΠΎΡ‰Π½ΠΎΡΡ‚ΡŒ для ΠΊΠ°ΠΆΠ΄ΠΎΠ³ΠΎ рСзистора:

ВсС Π΄Π°Π½Π½Ρ‹Π΅ ΠΌΠΎΠ³ΡƒΡ‚ Π±Ρ‹Ρ‚ΡŒ ΠΏΠΎΠ»ΡƒΡ‡Π΅Π½Ρ‹ с ΠΏΠΎΠΌΠΎΡ‰ΡŒΡŽ Π·Π°ΠΊΠΎΠ½Π° Ома ΠΈ для Π»ΡƒΡ‡ΡˆΠ΅Π³ΠΎ понимания прСдставлСны Π² Π²ΠΈΠ΄Π΅ ΡΠ»Π΅Π΄ΡƒΡŽΡ‰Π΅ΠΉ Ρ‚Π°Π±Π»ΠΈΡ†Ρ‹:

ΠŸΡ€ΠΈΠΌΠ΅Ρ€ β„– 2

НСобходимо Ρ€Π°ΡΡΡ‡ΠΈΡ‚Π°Ρ‚ΡŒ ΠΏΠ°Π΄Π΅Π½ΠΈΠ΅ напряТСния Π½Π° Π²Ρ‹Π²ΠΎΠ΄Π°Ρ… «А» ΠΈ Β«Π’Β»:

Π°) Π±Π΅Π· ΠΏΠΎΠ΄ΠΊΠ»ΡŽΡ‡Π΅Π½Π½ΠΎΠ³ΠΎ рСзистора R3

Π±) с ΠΏΠΎΠ΄ΠΊΠ»ΡŽΡ‡Π΅Π½Π½Ρ‹ΠΌ рСзистором R3

Как Π²Ρ‹ ΠΌΠΎΠΆΠ΅Ρ‚Π΅ Π²ΠΈΠ΄Π΅Ρ‚ΡŒ, Π²Ρ‹Ρ…ΠΎΠ΄Π½ΠΎΠ΅ напряТСниС U Π±Π΅Π· Π½Π°Π³Ρ€ΡƒΠ·ΠΎΡ‡Π½ΠΎΠ³ΠΎ рСзистора R3, составляСт 6 Π²ΠΎΠ»ΡŒΡ‚, Π½ΠΎ Ρ‚ΠΎ ΠΆΠ΅ Π²Ρ‹Ρ…ΠΎΠ΄Π½ΠΎΠ΅ напряТСниС ΠΏΡ€ΠΈ ΠΏΠΎΠ΄ΠΊΠ»ΡŽΡ‡Π΅Π½ΠΈΠΈ R3 становится всСго лишь 4 Π’.Β Π’Π°ΠΊΠΈΠΌ ΠΎΠ±Ρ€Π°Π·ΠΎΠΌ, Π½Π°Π³Ρ€ΡƒΠ·ΠΊΠ°, ΠΏΠΎΠ΄ΠΊΠ»ΡŽΡ‡Π΅Π½Π½Π°Ρ ΠΊ Π΄Π΅Π»ΠΈΡ‚Π΅Π»ΡŽ напряТСния, ΠΏΡ€ΠΎΠ²ΠΎΡ†ΠΈΡ€ΡƒΠ΅Ρ‚ Π΄ΠΎΠΏΠΎΠ»Π½ΠΈΡ‚Π΅Π»ΡŒΠ½ΠΎΠ΅ ΠΏΠ°Π΄Π΅Π½ΠΈΠ΅ напряТСниС.Β Π”Π°Π½Π½Ρ‹ΠΉ эффСкт сниТСния напряТСния ΠΌΠΎΠΆΠ΅Ρ‚ Π±Ρ‹Ρ‚ΡŒ компСнсирован с ΠΏΠΎΠΌΠΎΡ‰ΡŒΡŽ ΠΏΠΎΡ‚Π΅Π½Ρ†ΠΈΠΎΠΌΠ΅Ρ‚Ρ€Π° установлСнного вмСсто постоянного рСзистора, с ΠΏΠΎΠΌΠΎΡ‰ΡŒΡŽ ΠΊΠΎΡ‚ΠΎΡ€ΠΎΠ³ΠΎ ΠΌΠΎΠΆΠ½ΠΎ ΡΠΊΠΎΡ€Ρ€Π΅ΠΊΡ‚ΠΈΡ€ΠΎΠ²Π°Ρ‚ΡŒ напряТСниС Π½Π° Π½Π°Π³Ρ€ΡƒΠ·ΠΊΠ΅.

Онлайн ΠΊΠ°Π»ΡŒΠΊΡƒΠ»ΡΡ‚ΠΎΡ€ расчСта сопротивлСния ΠΏΠΎΡΠ»Π΅Π΄ΠΎΠ²Π°Ρ‚Π΅Π»ΡŒΠ½ΠΎ соСдинСнных рСзисторов

Π§Ρ‚ΠΎΠ±Ρ‹ быстро Π²Ρ‹Ρ‡ΠΈΡΠ»ΠΈΡ‚ΡŒ ΠΎΠ±Ρ‰Π΅Π΅ сопротивлСниС Π΄Π²ΡƒΡ… ΠΈ Π±ΠΎΠ»Π΅Π΅ рСзисторов, соСдинСнных ΠΏΠΎΡΠ»Π΅Π΄ΠΎΠ²Π°Ρ‚Π΅Π»ΡŒΠ½ΠΎ, Π²Ρ‹ ΠΌΠΎΠΆΠ΅Ρ‚Π΅ Π²ΠΎΡΠΏΠΎΠ»ΡŒΠ·ΠΎΠ²Π°Ρ‚ΡŒΡΡ ΡΠ»Π΅Π΄ΡƒΡŽΡ‰ΠΈΠΌ ΠΎΠ½Π»Π°ΠΉΠ½ ΠΊΠ°Π»ΡŒΠΊΡƒΠ»ΡΡ‚ΠΎΡ€ΠΎΠΌ:

ПодвСдСм ΠΈΡ‚ΠΎΠ³

Когда Π΄Π²Π° ΠΈΠ»ΠΈ нСсколько рСзисторов соСдинСны вмСстС (Π²Ρ‹Π²ΠΎΠ΄ ΠΎΠ΄Π½ΠΎΠ³ΠΎ соСдиняСтся с Π²Ρ‹Π²ΠΎΠ΄ΠΎΠΌ Π΄Ρ€ΡƒΠ³ΠΎΠ³ΠΎ рСзистора) β€” Ρ‚ΠΎ это ΠΏΠΎΡΠ»Π΅Π΄ΠΎΠ²Π°Ρ‚Π΅Π»ΡŒΠ½ΠΎΠ΅ соСдинСниС рСзисторов. Π’ΠΎΠΊ, ΠΏΡ€ΠΎΡ‚Π΅ΠΊΠ°ΡŽΡ‰ΠΈΠΉ Ρ‡Π΅Ρ€Π΅Π· рСзисторы ΠΈΠΌΠ΅Π΅Ρ‚ ΠΎΠ΄Π½ΠΎ ΠΈ Ρ‚ΠΎΠΆΠ΅ Π·Π½Π°Ρ‡Π΅Π½ΠΈΠ΅, Π½ΠΎ ΠΏΠ°Π΄Π΅Π½ΠΈΠ΅ напряТСния Π½Π° Π½ΠΈΡ… Π½Π΅ ΠΎΠ΄Π½ΠΎ ΠΈ Ρ‚ΠΎ ΠΆΠ΅. Оно опрСдСляСтся сопротивлСниСм ΠΊΠ°ΠΆΠ΄ΠΎΠ³ΠΎ рСзистора, ΠΊΠΎΡ‚ΠΎΡ€ΠΎΠ΅ рассчитываСтся ΠΏΠΎ Π·Π°ΠΊΠΎΠ½Ρƒ Ома (U = I * R).

Π£Ρ‡Π΅Π±Π½ΠΎΠ΅ пособиС ΠΏΠΎ Ρ„ΠΈΠ·ΠΈΠΊΠ΅

: схСмы сСрии

Как ΡƒΠΏΠΎΠΌΠΈΠ½Π°Π»ΠΎΡΡŒ Π² ΠΏΡ€Π΅Π΄Ρ‹Π΄ΡƒΡ‰Π΅ΠΌ Ρ€Π°Π·Π΄Π΅Π»Π΅ Π£Ρ€ΠΎΠΊΠ° 4, Π΄Π²Π° ΠΈΠ»ΠΈ Π±ΠΎΠ»Π΅Π΅ элСктричСских устройства Π² Ρ†Π΅ΠΏΠΈ ΠΌΠΎΠ³ΡƒΡ‚ Π±Ρ‹Ρ‚ΡŒ соСдинСны ΠΏΠΎΡΠ»Π΅Π΄ΠΎΠ²Π°Ρ‚Π΅Π»ΡŒΠ½Ρ‹ΠΌ ΠΈΠ»ΠΈ ΠΏΠ°Ρ€Π°Π»Π»Π΅Π»ΡŒΠ½Ρ‹ΠΌ соСдинСниСм. Когда всС устройства соСдинСны ΠΏΠΎΡΠ»Π΅Π΄ΠΎΠ²Π°Ρ‚Π΅Π»ΡŒΠ½Ρ‹ΠΌ соСдинСниСм, схСма называСтся ΠΏΠΎΡΠ»Π΅Π΄ΠΎΠ²Π°Ρ‚Π΅Π»ΡŒΠ½ΠΎΠΉ схСмой . Π’ ΠΏΠΎΡΠ»Π΅Π΄ΠΎΠ²Π°Ρ‚Π΅Π»ΡŒΠ½ΠΎΠΉ Ρ†Π΅ΠΏΠΈ ΠΊΠ°ΠΆΠ΄ΠΎΠ΅ устройство ΠΏΠΎΠ΄ΠΊΠ»ΡŽΡ‡Π°Π΅Ρ‚ΡΡ Ρ‚Π°ΠΊΠΈΠΌ ΠΎΠ±Ρ€Π°Π·ΠΎΠΌ, Ρ‡Ρ‚ΠΎ сущСствуСт Ρ‚ΠΎΠ»ΡŒΠΊΠΎ ΠΎΠ΄ΠΈΠ½ ΠΏΡƒΡ‚ΡŒ, ΠΏΠΎ ΠΊΠΎΡ‚ΠΎΡ€ΠΎΠΌΡƒ заряд ΠΌΠΎΠΆΠ΅Ρ‚ ΠΏΡ€ΠΎΡ…ΠΎΠ΄ΠΈΡ‚ΡŒ Ρ‡Π΅Ρ€Π΅Π· внСшнюю Ρ†Π΅ΠΏΡŒ. ΠšΠ°ΠΆΠ΄Ρ‹ΠΉ заряд, проходящий Ρ‡Π΅Ρ€Π΅Π· ΠΊΠΎΠ½Ρ‚ΡƒΡ€ внСшнСй Ρ†Π΅ΠΏΠΈ, Π±ΡƒΠ΄Π΅Ρ‚ ΠΏΠΎΡΠ»Π΅Π΄ΠΎΠ²Π°Ρ‚Π΅Π»ΡŒΠ½ΠΎ ΠΏΡ€ΠΎΡ…ΠΎΠ΄ΠΈΡ‚ΡŒ Ρ‡Π΅Ρ€Π΅Π· ΠΊΠ°ΠΆΠ΄Ρ‹ΠΉ рСзистор.

ΠšΡ€Π°Ρ‚ΠΊΠΎΠ΅ сравнСниС ΠΈ контраст ΠΌΠ΅ΠΆΠ΄Ρƒ ΠΏΠΎΡΠ»Π΅Π΄ΠΎΠ²Π°Ρ‚Π΅Π»ΡŒΠ½Ρ‹ΠΌΠΈ ΠΈ ΠΏΠ°Ρ€Π°Π»Π»Π΅Π»ΡŒΠ½Ρ‹ΠΌΠΈ цСпями Π±Ρ‹Π»ΠΎ сдСлано Π² ΠΏΡ€Π΅Π΄Ρ‹Π΄ΡƒΡ‰Π΅ΠΌ Ρ€Π°Π·Π΄Π΅Π»Π΅ Π£Ρ€ΠΎΠΊΠ° 4. Π’ этом Ρ€Π°Π·Π΄Π΅Π»Π΅ Π±Ρ‹Π»ΠΎ ΠΏΠΎΠ΄Ρ‡Π΅Ρ€ΠΊΠ½ΡƒΡ‚ΠΎ, Ρ‡Ρ‚ΠΎ Π΄ΠΎΠ±Π°Π²Π»Π΅Π½ΠΈΠ΅ большСго количСства рСзисторов ΠΊ ΠΏΠΎΡΠ»Π΅Π΄ΠΎΠ²Π°Ρ‚Π΅Π»ΡŒΠ½ΠΎΠΉ Ρ†Π΅ΠΏΠΈ ΠΏΡ€ΠΈΠ²ΠΎΠ΄ΠΈΡ‚ ΠΊ довольно ΠΎΠΆΠΈΠ΄Π°Π΅ΠΌΠΎΠΌΡƒ Ρ€Π΅Π·ΡƒΠ»ΡŒΡ‚Π°Ρ‚Ρƒ — ΡƒΠ²Π΅Π»ΠΈΡ‡Π΅Π½ΠΈΡŽ ΠΎΠ±Ρ‰Π΅Π³ΠΎ сопротивлСния. . ΠŸΠΎΡΠΊΠΎΠ»ΡŒΠΊΡƒ Π² Ρ†Π΅ΠΏΠΈ Π΅ΡΡ‚ΡŒ Ρ‚ΠΎΠ»ΡŒΠΊΠΎ ΠΎΠ΄ΠΈΠ½ ΠΏΡƒΡ‚ΡŒ, ΠΊΠ°ΠΆΠ΄Ρ‹ΠΉ заряд встрСчаСт сопротивлСниС ΠΊΠ°ΠΆΠ΄ΠΎΠ³ΠΎ устройства; поэтому Π΄ΠΎΠ±Π°Π²Π»Π΅Π½ΠΈΠ΅ большСго количСства устройств ΠΏΡ€ΠΈΠ²ΠΎΠ΄ΠΈΡ‚ ΠΊ ΡƒΠ²Π΅Π»ΠΈΡ‡Π΅Π½ΠΈΡŽ ΠΎΠ±Ρ‰Π΅Π³ΠΎ сопротивлСния. Π­Ρ‚ΠΎ ΡƒΠ²Π΅Π»ΠΈΡ‡Π΅Π½Π½ΠΎΠ΅ сопротивлСниС слуТит для ΡƒΠΌΠ΅Π½ΡŒΡˆΠ΅Π½ΠΈΡ скорости протСкания заряда (Ρ‚Π°ΠΊΠΆΠ΅ извСстной ΠΊΠ°ΠΊ Ρ‚ΠΎΠΊ).

Π­ΠΊΠ²ΠΈΠ²Π°Π»Π΅Π½Ρ‚Π½ΠΎΠ΅ сопротивлСниС ΠΈ Ρ‚ΠΎΠΊ

Заряды проходят Ρ‡Π΅Ρ€Π΅Π· внСшнюю Ρ†Π΅ΠΏΡŒ со ΡΠΊΠΎΡ€ΠΎΡΡ‚ΡŒΡŽ, которая Π²Π΅Π·Π΄Π΅ ΠΎΠ΄ΠΈΠ½Π°ΠΊΠΎΠ²Π°. Π’ ΠΎΠ΄Π½ΠΎΠΌ мСстС Ρ‚ΠΎΠΊ Π½Π΅ большС, Ρ‡Π΅ΠΌ Π² Π΄Ρ€ΡƒΠ³ΠΎΠΌ. ЀактичСскоС количСство Ρ‚ΠΎΠΊΠ° ΠΎΠ±Ρ€Π°Ρ‚Π½ΠΎ ΠΏΡ€ΠΎΠΏΠΎΡ€Ρ†ΠΈΠΎΠ½Π°Π»ΡŒΠ½ΠΎ ΠΎΠ±Ρ‰Π΅ΠΌΡƒ ΡΠΎΠΏΡ€ΠΎΡ‚ΠΈΠ²Π»Π΅Π½ΠΈΡŽ. БущСствуСт чСткая взаимосвязь ΠΌΠ΅ΠΆΠ΄Ρƒ сопротивлСниСм ΠΎΡ‚Π΄Π΅Π»ΡŒΠ½Ρ‹Ρ… рСзисторов ΠΈ ΠΎΠ±Ρ‰ΠΈΠΌ сопротивлСниСм Π½Π°Π±ΠΎΡ€Π° рСзисторов.Π§Ρ‚ΠΎ касаСтся Π±Π°Ρ‚Π°Ρ€Π΅ΠΈ, которая Π½Π°Π³Π½Π΅Ρ‚Π°Π΅Ρ‚ заряд, Π½Π°Π»ΠΈΡ‡ΠΈΠ΅ Π΄Π²ΡƒΡ… ΠΏΠΎΡΠ»Π΅Π΄ΠΎΠ²Π°Ρ‚Π΅Π»ΡŒΠ½ΠΎ соСдинСнных рСзисторов сопротивлСниСм 6 Ом Π±Ρ‹Π»ΠΎ Π±Ρ‹ эквивалСнтно Π½Π°Π»ΠΈΡ‡ΠΈΡŽ Π² Ρ†Π΅ΠΏΠΈ ΠΎΠ΄Π½ΠΎΠ³ΠΎ рСзистора сопротивлСниСм 12 Ом. НаличиС Ρ‚Ρ€Π΅Ρ… ΠΏΠΎΡΠ»Π΅Π΄ΠΎΠ²Π°Ρ‚Π΅Π»ΡŒΠ½ΠΎ соСдинСнных рСзисторов сопротивлСниСм 6 Ом Π±Ρ‹Π»ΠΎ Π±Ρ‹ эквивалСнтно Π½Π°Π»ΠΈΡ‡ΠΈΡŽ Π² Ρ†Π΅ΠΏΠΈ ΠΎΠ΄Π½ΠΎΠ³ΠΎ рСзистора сопротивлСниСм 18 Ом. А Π½Π°Π»ΠΈΡ‡ΠΈΠ΅ Ρ‡Π΅Ρ‚Ρ‹Ρ€Π΅Ρ… ΠΏΠΎΡΠ»Π΅Π΄ΠΎΠ²Π°Ρ‚Π΅Π»ΡŒΠ½ΠΎ соСдинСнных рСзисторов 6 Ом Π±Ρ‹Π»ΠΎ Π±Ρ‹ эквивалСнтно Π½Π°Π»ΠΈΡ‡ΠΈΡŽ Π² Ρ†Π΅ΠΏΠΈ ΠΎΠ΄Π½ΠΎΠ³ΠΎ рСзистора 24 Ом.

Π­Ρ‚ΠΎ концСпция эквивалСнтного сопротивлСния. Π­ΠΊΠ²ΠΈΠ²Π°Π»Π΅Π½Ρ‚Π½ΠΎΠ΅ сопротивлСниС Ρ†Π΅ΠΏΠΈ — это Π²Π΅Π»ΠΈΡ‡ΠΈΠ½Π° сопротивлСния, которая потрСбуСтся ΠΎΠ΄Π½ΠΎΠΌΡƒ рСзистору, Ρ‡Ρ‚ΠΎΠ±Ρ‹ ΡΡ€Π°Π²Π½ΡΡ‚ΡŒΡΡ с ΠΎΠ±Ρ‰ΠΈΠΌ эффСктом ΠΎΡ‚ совокупности рСзисторов, ΠΏΡ€ΠΈΡΡƒΡ‚ΡΡ‚Π²ΡƒΡŽΡ‰ΠΈΡ… Π² Ρ†Π΅ΠΏΠΈ.Для ΠΏΠΎΡΠ»Π΅Π΄ΠΎΠ²Π°Ρ‚Π΅Π»ΡŒΠ½Ρ‹Ρ… Ρ†Π΅ΠΏΠ΅ΠΉ матСматичСская Ρ„ΠΎΡ€ΠΌΡƒΠ»Π° для вычислСния эквивалСнтного сопротивлСния (R eq ) составляСт

. рэндов экв. = 1 рэндов + 2 рэндов + 3 + …

, Π³Π΄Π΅ R 1 , R 2 ΠΈ R 3 — значСния сопротивлСния ΠΎΡ‚Π΄Π΅Π»ΡŒΠ½Ρ‹Ρ… рСзисторов, соСдинСнных ΠΏΠΎΡΠ»Π΅Π΄ΠΎΠ²Π°Ρ‚Π΅Π»ΡŒΠ½ΠΎ.

Π‘ΠΎΠ·Π΄Π°Π²Π°ΠΉΡ‚Π΅, Ρ€Π΅ΡˆΠ°ΠΉΡ‚Π΅ ΠΈ провСряйтС свои собствСнныС ΠΏΡ€ΠΎΠ±Π»Π΅ΠΌΡ‹ с ΠΏΠΎΠΌΠΎΡ‰ΡŒΡŽ Π²ΠΈΠ΄ΠΆΠ΅Ρ‚Π° Equivalent Resistance Π½ΠΈΠΆΠ΅.Π‘ΠΎΠ·Π΄Π°ΠΉΡ‚Π΅ сСбС ΠΏΡ€ΠΎΠ±Π»Π΅ΠΌΡƒ с Π»ΡŽΠ±Ρ‹ΠΌ количСством рСзисторов ΠΈ Π»ΡŽΠ±Ρ‹ΠΌΠΈ Π½ΠΎΠΌΠΈΠ½Π°Π»Π°ΠΌΠΈ. Π Π΅ΡˆΠ°Ρ‚ΡŒ ΠΏΡ€ΠΎΠ±Π»Π΅ΠΌΡƒ; Π·Π°Ρ‚Π΅ΠΌ Π½Π°ΠΆΠΌΠΈΡ‚Π΅ ΠΊΠ½ΠΎΠΏΠΊΡƒ Β«ΠžΡ‚ΠΏΡ€Π°Π²ΠΈΡ‚ΡŒΒ», Ρ‡Ρ‚ΠΎΠ±Ρ‹ ΠΏΡ€ΠΎΠ²Π΅Ρ€ΠΈΡ‚ΡŒ свой ΠΎΡ‚Π²Π΅Ρ‚.

Π’ΠΎΠΊ Π² ΠΏΠΎΡΠ»Π΅Π΄ΠΎΠ²Π°Ρ‚Π΅Π»ΡŒΠ½ΠΎΠΉ Ρ†Π΅ΠΏΠΈ Π²Π΅Π·Π΄Π΅ ΠΎΠ΄ΠΈΠ½Π°ΠΊΠΎΠ²Ρ‹ΠΉ. Заряд НЕ накапливаСтся ΠΈ Π½Π΅ Π½Π°Ρ‡ΠΈΠ½Π°Π΅Ρ‚ Π½Π°ΠΊΠ°ΠΏΠ»ΠΈΠ²Π°Ρ‚ΡŒΡΡ Π² любом Π·Π°Π΄Π°Π½Π½ΠΎΠΌ мСстС, Ρ‚Π°ΠΊ Ρ‡Ρ‚ΠΎ Ρ‚ΠΎΠΊ Π² ΠΎΠ΄Π½ΠΎΠΌ мСстС большС, Ρ‡Π΅ΠΌ Π² Π΄Ρ€ΡƒΠ³ΠΈΡ… мСстах. Заряд НЕ расходуСтся рСзисторами, Ρ‚Π°ΠΊ Ρ‡Ρ‚ΠΎ Π² ΠΎΠ΄Π½ΠΎΠΌ мСстС Π΅Π³ΠΎ мСньшС ΠΏΠΎ ΡΡ€Π°Π²Π½Π΅Π½ΠΈΡŽ с Π΄Ρ€ΡƒΠ³ΠΈΠΌ. МоТно ΠΏΡ€Π΅Π΄ΡΡ‚Π°Π²ΠΈΡ‚ΡŒ, Ρ‡Ρ‚ΠΎ заряды двиТутся вмСстС ΠΏΠΎ ΠΏΡ€ΠΎΠ²ΠΎΠ΄Π°ΠΌ элСктричСской Ρ†Π΅ΠΏΠΈ ΠΈ Π²Π΅Π·Π΄Π΅ двиТутся с ΠΎΠ΄ΠΈΠ½Π°ΠΊΠΎΠ²ΠΎΠΉ ΡΠΊΠΎΡ€ΠΎΡΡ‚ΡŒΡŽ.Π’ΠΎΠΊ — ΡΠΊΠΎΡ€ΠΎΡΡ‚ΡŒ, с ΠΊΠΎΡ‚ΠΎΡ€ΠΎΠΉ Ρ‚Π΅Ρ‡Π΅Ρ‚ заряд — Π²Π΅Π·Π΄Π΅ ΠΎΠ΄ΠΈΠ½Π°ΠΊΠΎΠ². Π’ΠΎ ΠΆΠ΅ самоС Π½Π° ΠΏΠ΅Ρ€Π²ΠΎΠΌ рСзисторС, ΠΊΠ°ΠΊ Π½Π° послСднСм рСзисторС, ΠΊΠ°ΠΊ Π² Π±Π°Ρ‚Π°Ρ€Π΅Π΅. ΠœΠ°Ρ‚Π΅ΠΌΠ°Ρ‚ΠΈΡ‡Π΅ΡΠΊΠΈ ΠΌΠΎΠΆΠ½ΠΎ Π½Π°ΠΏΠΈΡΠ°Ρ‚ΡŒ

I аккумулятор = I 1 = I 2 = I 3 = …

, Π³Π΄Π΅ I 1 , I 2 ΠΈ I 3 — значСния Ρ‚ΠΎΠΊΠ° Π² ΠΎΡ‚Π΄Π΅Π»ΡŒΠ½Ρ‹Ρ… мСстах располоТСния рСзисторов.

Π­Ρ‚ΠΈ значСния Ρ‚ΠΎΠΊΠ° Π»Π΅Π³ΠΊΠΎ Π²Ρ‹Ρ‡ΠΈΡΠ»ΠΈΡ‚ΡŒ, Ссли извСстно напряТСниС Π±Π°Ρ‚Π°Ρ€Π΅ΠΈ ΠΈ извСстны ΠΎΡ‚Π΄Π΅Π»ΡŒΠ½Ρ‹Π΅ значСния сопротивлСния.Π˜ΡΠΏΠΎΠ»ΡŒΠ·ΡƒΡ значСния ΠΎΡ‚Π΄Π΅Π»ΡŒΠ½Ρ‹Ρ… рСзисторов ΠΈ ΠΏΡ€ΠΈΠ²Π΅Π΄Π΅Π½Π½ΠΎΠ΅ Π²Ρ‹ΡˆΠ΅ ΡƒΡ€Π°Π²Π½Π΅Π½ΠΈΠ΅, ΠΌΠΎΠΆΠ½ΠΎ Ρ€Π°ΡΡΡ‡ΠΈΡ‚Π°Ρ‚ΡŒ эквивалСнтноС сопротивлСниС. А ΠΈΡΠΏΠΎΠ»ΡŒΠ·ΡƒΡ Π·Π°ΠΊΠΎΠ½ Ома (Ξ”V = I β€’ R), Ρ‚ΠΎΠΊ Π² Π±Π°Ρ‚Π°Ρ€Π΅Π΅ ΠΈ, ΡΠ»Π΅Π΄ΠΎΠ²Π°Ρ‚Π΅Π»ΡŒΠ½ΠΎ, Ρ‡Π΅Ρ€Π΅Π· ΠΊΠ°ΠΆΠ΄Ρ‹ΠΉ рСзистор ΠΌΠΎΠΆΠ½ΠΎ ΠΎΠΏΡ€Π΅Π΄Π΅Π»ΠΈΡ‚ΡŒ, найдя ΡΠΎΠΎΡ‚Π½ΠΎΡˆΠ΅Π½ΠΈΠ΅ напряТСния Π±Π°Ρ‚Π°Ρ€Π΅ΠΈ ΠΈ эквивалСнтного сопротивлСния.

I аккумулятор = I 1 = I 2 = I 3 = Ξ”V аккумулятор / R экв

Π Π°Π·Π½ΠΎΡΡ‚ΡŒ элСктричСских ΠΏΠΎΡ‚Π΅Π½Ρ†ΠΈΠ°Π»ΠΎΠ² ΠΈ падСния напряТСния

Как ΠΎΠ±ΡΡƒΠΆΠ΄Π°Π»ΠΎΡΡŒ Π² Π£Ρ€ΠΎΠΊΠ΅ 1, элСктрохимичСский элСмСнт схСмы ΠΏΠΎΠ΄Π°Π΅Ρ‚ ΡΠ½Π΅Ρ€Π³ΠΈΡŽ Π½Π° заряд, Ρ‡Ρ‚ΠΎΠ±Ρ‹ ΠΏΠ΅Ρ€Π΅ΠΌΠ΅Ρ‰Π°Ρ‚ΡŒ Π΅Π³ΠΎ Ρ‡Π΅Ρ€Π΅Π· элСмСнт ΠΈ ΡƒΡΡ‚Π°Π½Π°Π²Π»ΠΈΠ²Π°Ρ‚ΡŒ Ρ€Π°Π·Π½ΠΎΡΡ‚ΡŒ элСктричСских ΠΏΠΎΡ‚Π΅Π½Ρ†ΠΈΠ°Π»ΠΎΠ² Π½Π° Π΄Π²ΡƒΡ… ΠΊΠΎΠ½Ρ†Π°Ρ… внСшнСй Ρ†Π΅ΠΏΠΈ.Π­Π»Π΅ΠΌΠ΅Π½Ρ‚ с напряТСниСм 1,5 Π’ создаСт Ρ€Π°Π·Π½ΠΎΡΡ‚ΡŒ элСктричСских ΠΏΠΎΡ‚Π΅Π½Ρ†ΠΈΠ°Π»ΠΎΠ² Π²ΠΎ внСшнСй Ρ†Π΅ΠΏΠΈ 1,5 Π’. Π­Ρ‚ΠΎ ΠΎΠ·Π½Π°Ρ‡Π°Π΅Ρ‚, Ρ‡Ρ‚ΠΎ элСктричСский ΠΏΠΎΡ‚Π΅Π½Ρ†ΠΈΠ°Π» Π½Π° ΠΏΠΎΠ»ΠΎΠΆΠΈΡ‚Π΅Π»ΡŒΠ½ΠΎΠΉ ΠΊΠ»Π΅ΠΌΠΌΠ΅ Π½Π° 1,5 Π’ большС, Ρ‡Π΅ΠΌ Π½Π° ΠΎΡ‚Ρ€ΠΈΡ†Π°Ρ‚Π΅Π»ΡŒΠ½ΠΎΠΉ ΠΊΠ»Π΅ΠΌΠΌΠ΅. Когда заряд двиТСтся ΠΏΠΎ внСшнСй Ρ†Π΅ΠΏΠΈ, ΠΎΠ½ тСряСт 1,5 Π²ΠΎΠ»ΡŒΡ‚ элСктричСского ΠΏΠΎΡ‚Π΅Π½Ρ†ΠΈΠ°Π»Π°. Π­Ρ‚Π° потСря элСктричСского ΠΏΠΎΡ‚Π΅Π½Ρ†ΠΈΠ°Π»Π° обозначаСтся ΠΊΠ°ΠΊ ΠΏΠ°Π΄Π΅Π½ΠΈΠ΅ напряТСния . Π­Ρ‚ΠΎ происходит, ΠΊΠΎΠ³Π΄Π° элСктричСская энСргия заряда прСобразуСтся Π² Π΄Ρ€ΡƒΠ³ΠΈΠ΅ Ρ„ΠΎΡ€ΠΌΡ‹ энСргии (Ρ‚Π΅ΠΏΠ»ΠΎΠ²ΡƒΡŽ, ΡΠ²Π΅Ρ‚ΠΎΠ²ΡƒΡŽ, ΠΌΠ΅Ρ…Π°Π½ΠΈΡ‡Π΅ΡΠΊΡƒΡŽ ΠΈ Ρ‚. Π”.).) Π²Π½ΡƒΡ‚Ρ€ΠΈ рСзисторов ΠΈΠ»ΠΈ Π½Π°Π³Ρ€ΡƒΠ·ΠΎΠΊ. Если элСктричСская Ρ†Π΅ΠΏΡŒ, питаСмая элСмСнтом Π½Π° 1,5 Π’, оснащСна Π±ΠΎΠ»Π΅Π΅ Ρ‡Π΅ΠΌ ΠΎΠ΄Π½ΠΈΠΌ рСзистором, Ρ‚ΠΎ совокупная потСря элСктричСского ΠΏΠΎΡ‚Π΅Π½Ρ†ΠΈΠ°Π»Π° составляСт 1,5 Π’. Для ΠΊΠ°ΠΆΠ΄ΠΎΠ³ΠΎ рСзистора сущСствуСт ΠΏΠ°Π΄Π΅Π½ΠΈΠ΅ напряТСния, Π½ΠΎ сумма этих ΠΏΠ°Π΄Π΅Π½ΠΈΠΉ составляСт 1,5 Π’ — Ρ‚ΠΎ ΠΆΠ΅ самоС, Ρ‡Ρ‚ΠΎ ΠΈ номинальноС напряТСниС источника питания. Π­Ρ‚ΠΎ понятиС ΠΌΠΎΠΆΠ΅Ρ‚ Π±Ρ‹Ρ‚ΡŒ Π²Ρ‹Ρ€Π°ΠΆΠ΅Π½ΠΎ матСматичСски ΡΠ»Π΅Π΄ΡƒΡŽΡ‰ΠΈΠΌ ΡƒΡ€Π°Π²Π½Π΅Π½ΠΈΠ΅ΠΌ:

Ξ”V аккумулятор = Ξ”V 1 + Ξ”V 2 + Ξ”V 3 +…

Π§Ρ‚ΠΎΠ±Ρ‹ ΠΏΡ€ΠΎΠΈΠ»Π»ΡŽΡΡ‚Ρ€ΠΈΡ€ΠΎΠ²Π°Ρ‚ΡŒ этот матСматичСский ΠΏΡ€ΠΈΠ½Ρ†ΠΈΠΏ Π² дСйствии, рассмотрим Π΄Π²Π΅ схСмы, ΠΏΠΎΠΊΠ°Π·Π°Π½Π½Ρ‹Π΅ Π½ΠΈΠΆΠ΅ Π½Π° Π΄ΠΈΠ°Π³Ρ€Π°ΠΌΠΌΠ°Ρ… A ΠΈ B. ΠŸΡ€Π΅Π΄ΠΏΠΎΠ»ΠΎΠΆΠΈΠΌ, вас попросили ΠΎΠΏΡ€Π΅Π΄Π΅Π»ΠΈΡ‚ΡŒ Π΄Π²Π° нСизвСстных значСния разности элСктричСских ΠΏΠΎΡ‚Π΅Π½Ρ†ΠΈΠ°Π»ΠΎΠ² ΠΌΠ΅ΠΆΠ΄Ρƒ Π»Π°ΠΌΠΏΠΎΡ‡ΠΊΠ°ΠΌΠΈ Π² ΠΊΠ°ΠΆΠ΄ΠΎΠΉ Ρ†Π΅ΠΏΠΈ. Π§Ρ‚ΠΎΠ±Ρ‹ ΠΎΠΏΡ€Π΅Π΄Π΅Π»ΠΈΡ‚ΡŒ ΠΈΡ… значСния, Π²Π°ΠΌ Π½ΡƒΠΆΠ½ΠΎ Π±ΡƒΠ΄Π΅Ρ‚ ΠΈΡΠΏΠΎΠ»ΡŒΠ·ΠΎΠ²Π°Ρ‚ΡŒ ΠΏΡ€ΠΈΠ²Π΅Π΄Π΅Π½Π½ΠΎΠ΅ Π²Ρ‹ΡˆΠ΅ ΡƒΡ€Π°Π²Π½Π΅Π½ΠΈΠ΅. БатарСя обозначаСтся ΠΎΠ±Ρ‹Ρ‡Π½Ρ‹ΠΌ схСматичСским символом, Π° рядом с Π½Π΅ΠΉ указываСтся Π΅Π΅ напряТСниС. ΠžΠΏΡ€Π΅Π΄Π΅Π»ΠΈΡ‚Π΅ ΠΏΠ°Π΄Π΅Π½ΠΈΠ΅ напряТСния для Π΄Π²ΡƒΡ… Π»Π°ΠΌΠΏΠΎΡ‡Π΅ΠΊ, Π° Π·Π°Ρ‚Π΅ΠΌ Π½Π°ΠΆΠΌΠΈΡ‚Π΅ ΠΊΠ½ΠΎΠΏΠΊΡƒ Β«ΠŸΡ€ΠΎΠ²Π΅Ρ€ΠΈΡ‚ΡŒ ΠΎΡ‚Π²Π΅Ρ‚Ρ‹Β», Ρ‡Ρ‚ΠΎΠ±Ρ‹ ΡƒΠ±Π΅Π΄ΠΈΡ‚ΡŒΡΡ, Ρ‡Ρ‚ΠΎ Π²Ρ‹ ΠΏΡ€Π°Π²Ρ‹.

Π Π°Π½Π΅Π΅ Π² Π£Ρ€ΠΎΠΊΠ΅ 1 ΠΎΠ±ΡΡƒΠΆΠ΄Π°Π»ΠΎΡΡŒ использованиС Π΄ΠΈΠ°Π³Ρ€Π°ΠΌΠΌΡ‹ элСктричСских ΠΏΠΎΡ‚Π΅Π½Ρ†ΠΈΠ°Π»ΠΎΠ². Π”ΠΈΠ°Π³Ρ€Π°ΠΌΠΌΠ° элСктричСского ΠΏΠΎΡ‚Π΅Π½Ρ†ΠΈΠ°Π»Π° — это ΠΊΠΎΠ½Ρ†Π΅ΠΏΡ‚ΡƒΠ°Π»ΡŒΠ½Ρ‹ΠΉ инструмСнт для прСдставлСния разности элСктричСских ΠΏΠΎΡ‚Π΅Π½Ρ†ΠΈΠ°Π»ΠΎΠ² ΠΌΠ΅ΠΆΠ΄Ρƒ нСсколькими Ρ‚ΠΎΡ‡ΠΊΠ°ΠΌΠΈ элСктричСской Ρ†Π΅ΠΏΠΈ. Рассмотрим ΠΏΡ€ΠΈΠ²Π΅Π΄Π΅Π½Π½ΡƒΡŽ Π½ΠΈΠΆΠ΅ ΠΏΡ€ΠΈΠ½Ρ†ΠΈΠΏΠΈΠ°Π»ΡŒΠ½ΡƒΡŽ схСму ΠΈ ΡΠΎΠΎΡ‚Π²Π΅Ρ‚ΡΡ‚Π²ΡƒΡŽΡ‰ΡƒΡŽ Π΄ΠΈΠ°Π³Ρ€Π°ΠΌΠΌΡƒ элСктричСских ΠΏΠΎΡ‚Π΅Π½Ρ†ΠΈΠ°Π»ΠΎΠ².

Π‘Ρ…Π΅ΠΌΠ°, показанная Π½Π° схСмС Π²Ρ‹ΡˆΠ΅, питаСтся ΠΎΡ‚ источника энСргии 12 Π’.Π’ Ρ†Π΅ΠΏΠΈ ΠΏΠΎΡΠ»Π΅Π΄ΠΎΠ²Π°Ρ‚Π΅Π»ΡŒΠ½ΠΎ соСдинСны Ρ‚Ρ€ΠΈ рСзистора, ΠΊΠ°ΠΆΠ΄Ρ‹ΠΉ ΠΈΠ· ΠΊΠΎΡ‚ΠΎΡ€Ρ‹Ρ… ΠΈΠΌΠ΅Π΅Ρ‚ собствСнноС ΠΏΠ°Π΄Π΅Π½ΠΈΠ΅ напряТСния. ΠžΡ‚Ρ€ΠΈΡ†Π°Ρ‚Π΅Π»ΡŒΠ½Ρ‹ΠΉ Π·Π½Π°ΠΊ разности элСктричСских ΠΏΠΎΡ‚Π΅Π½Ρ†ΠΈΠ°Π»ΠΎΠ² просто ΠΎΠ·Π½Π°Ρ‡Π°Π΅Ρ‚ ΠΏΠΎΡ‚Π΅Ρ€ΡŽ элСктричСского ΠΏΠΎΡ‚Π΅Π½Ρ†ΠΈΠ°Π»Π° ΠΏΡ€ΠΈ ΠΏΡ€ΠΎΡ…ΠΎΠΆΠ΄Π΅Π½ΠΈΠΈ Ρ‡Π΅Ρ€Π΅Π· рСзистор. ΠžΠ±Ρ‹Ρ‡Π½Ρ‹ΠΉ Ρ‚ΠΎΠΊ направляСтся ΠΏΠΎ внСшнСй Ρ†Π΅ΠΏΠΈ ΠΎΡ‚ ΠΏΠΎΠ»ΠΎΠΆΠΈΡ‚Π΅Π»ΡŒΠ½ΠΎΠΉ ΠΊΠ»Π΅ΠΌΠΌΡ‹ ΠΊ ΠΎΡ‚Ρ€ΠΈΡ†Π°Ρ‚Π΅Π»ΡŒΠ½ΠΎΠΉ. ΠŸΠΎΡΠΊΠΎΠ»ΡŒΠΊΡƒ схСматичСский символ источника напряТСния ΠΈΡΠΏΠΎΠ»ΡŒΠ·ΡƒΠ΅Ρ‚ Π΄Π»ΠΈΠ½Π½ΡƒΡŽ полосу для обозначСния ΠΏΠΎΠ»ΠΎΠΆΠΈΡ‚Π΅Π»ΡŒΠ½ΠΎΠ³ΠΎ Π²Ρ‹Π²ΠΎΠ΄Π°, Ρ‚ΠΎΡ‡ΠΊΠ° A Π½Π° схСмС находится Π½Π° ΠΏΠΎΠ»ΠΎΠΆΠΈΡ‚Π΅Π»ΡŒΠ½ΠΎΠΌ Π²Ρ‹Π²ΠΎΠ΄Π΅ ΠΈΠ»ΠΈ Π²Ρ‹Π²ΠΎΠ΄Π΅ с высоким ΠΏΠΎΡ‚Π΅Π½Ρ†ΠΈΠ°Π»ΠΎΠΌ.Π’ Ρ‚ΠΎΡ‡ΠΊΠ΅ A элСктричСский ΠΏΠΎΡ‚Π΅Π½Ρ†ΠΈΠ°Π» 12 Π²ΠΎΠ»ΡŒΡ‚, Π° Π² Ρ‚ΠΎΡ‡ΠΊΠ΅ H (ΠΎΡ‚Ρ€ΠΈΡ†Π°Ρ‚Π΅Π»ΡŒΠ½Ρ‹ΠΉ Π²Ρ‹Π²ΠΎΠ΄) — 0 Π²ΠΎΠ»ΡŒΡ‚. ΠŸΡ€ΠΎΡ…ΠΎΠ΄Ρ Ρ‡Π΅Ρ€Π΅Π· Π±Π°Ρ‚Π°Ρ€Π΅ΡŽ, заряд ΠΏΡ€ΠΈΠΎΠ±Ρ€Π΅Ρ‚Π°Π΅Ρ‚ элСктричСский ΠΏΠΎΡ‚Π΅Π½Ρ†ΠΈΠ°Π» 12 Π²ΠΎΠ»ΡŒΡ‚. А ΠΏΡ€ΠΈ ΠΏΡ€ΠΎΡ…ΠΎΠΆΠ΄Π΅Π½ΠΈΠΈ Ρ‡Π΅Ρ€Π΅Π· внСшнюю Ρ†Π΅ΠΏΡŒ заряд тСряСт 12 Π²ΠΎΠ»ΡŒΡ‚ элСктричСского ΠΏΠΎΡ‚Π΅Π½Ρ†ΠΈΠ°Π»Π°, ΠΊΠ°ΠΊ ΠΏΠΎΠΊΠ°Π·Π°Π½ΠΎ Π½Π° Π΄ΠΈΠ°Π³Ρ€Π°ΠΌΠΌΠ΅ элСктричСских ΠΏΠΎΡ‚Π΅Π½Ρ†ΠΈΠ°Π»ΠΎΠ², ΠΏΠΎΠΊΠ°Π·Π°Π½Π½ΠΎΠΉ справа ΠΎΡ‚ ΠΏΡ€ΠΈΠ½Ρ†ΠΈΠΏΠΈΠ°Π»ΡŒΠ½ΠΎΠΉ схСмы. Π­Ρ‚ΠΈ 12 Π²ΠΎΠ»ΡŒΡ‚ элСктричСского ΠΏΠΎΡ‚Π΅Π½Ρ†ΠΈΠ°Π»Π° Ρ‚Π΅Ρ€ΡΡŽΡ‚ΡΡ Π² Ρ‚Ρ€ΠΈ этапа, ΠΊΠ°ΠΆΠ΄Ρ‹ΠΉ ΠΈΠ· ΠΊΠΎΡ‚ΠΎΡ€Ρ‹Ρ… соотвСтствуСт ΠΏΡ€ΠΎΡ…ΠΎΠΆΠ΄Π΅Π½ΠΈΡŽ Ρ‡Π΅Ρ€Π΅Π· рСзистор. ΠŸΡ€ΠΈ ΠΏΡ€ΠΎΡ…ΠΎΠΆΠ΄Π΅Π½ΠΈΠΈ Ρ‡Π΅Ρ€Π΅Π· ΡΠΎΠ΅Π΄ΠΈΠ½ΠΈΡ‚Π΅Π»ΡŒΠ½Ρ‹Π΅ ΠΏΡ€ΠΎΠ²ΠΎΠ΄Π° ΠΌΠ΅ΠΆΠ΄Ρƒ рСзисторами происходит нСбольшая потСря элСктричСского ΠΏΠΎΡ‚Π΅Π½Ρ†ΠΈΠ°Π»Π° ΠΈΠ·-Π·Π° Ρ‚ΠΎΠ³ΠΎ, Ρ‡Ρ‚ΠΎ ΠΏΡ€ΠΎΠ²ΠΎΠ΄ ΠΎΠΊΠ°Π·Ρ‹Π²Π°Π΅Ρ‚ ΠΎΡ‚Π½ΠΎΡΠΈΡ‚Π΅Π»ΡŒΠ½ΠΎ нСбольшоС сопротивлСниС ΠΏΠΎΡ‚ΠΎΠΊΡƒ заряда.ΠŸΠΎΡΠΊΠΎΠ»ΡŒΠΊΡƒ Ρ‚ΠΎΡ‡ΠΊΠΈ A ΠΈ B Ρ€Π°Π·Π΄Π΅Π»Π΅Π½Ρ‹ ΠΏΡ€ΠΎΠ²ΠΎΠ΄ΠΎΠΌ, ΠΎΠ½ΠΈ ΠΈΠΌΠ΅ΡŽΡ‚ практичСски ΠΎΠ΄ΠΈΠ½Π°ΠΊΠΎΠ²Ρ‹ΠΉ элСктричСский ΠΏΠΎΡ‚Π΅Π½Ρ†ΠΈΠ°Π» 12 Π’. Когда заряд ΠΏΡ€ΠΎΡ…ΠΎΠ΄ΠΈΡ‚ Ρ‡Π΅Ρ€Π΅Π· свой ΠΏΠ΅Ρ€Π²Ρ‹ΠΉ рСзистор, ΠΎΠ½ тСряСт 3 Π’ элСктричСского ΠΏΠΎΡ‚Π΅Π½Ρ†ΠΈΠ°Π»Π° ΠΈ ΠΏΠ°Π΄Π°Π΅Ρ‚ Π΄ΠΎ 9 Π’ Π² Ρ‚ΠΎΡ‡ΠΊΠ΅ C. Ρ‚ΠΎΡ‡ΠΊΠ° D ΠΎΡ‚Π΄Π΅Π»Π΅Π½Π° ΠΎΡ‚ Ρ‚ΠΎΡ‡ΠΊΠΈ C простым ΠΏΡ€ΠΎΠ²ΠΎΠ΄ΠΎΠΌ, ΠΎΠ½Π° ΠΈΠΌΠ΅Π΅Ρ‚ практичСски Ρ‚ΠΎΡ‚ ΠΆΠ΅ элСктричСский ΠΏΠΎΡ‚Π΅Π½Ρ†ΠΈΠ°Π» 9 Π’, Ρ‡Ρ‚ΠΎ ΠΈ C. Когда заряд ΠΏΡ€ΠΎΡ…ΠΎΠ΄ΠΈΡ‚ Ρ‡Π΅Ρ€Π΅Π· Π²Ρ‚ΠΎΡ€ΠΎΠΉ рСзистор, ΠΎΠ½ тСряСт 7 Π’ элСктричСского ΠΏΠΎΡ‚Π΅Π½Ρ†ΠΈΠ°Π»Π° ΠΈ ΠΏΠ°Π΄Π°Π΅Ρ‚ Π΄ΠΎ 2 Π’ Π² Ρ‚ΠΎΡ‡ΠΊΠ΅ E. ΠŸΠΎΡΠΊΠΎΠ»ΡŒΠΊΡƒ Ρ‚ΠΎΡ‡ΠΊΠ° F ΠΎΡ‚Π΄Π΅Π»Π΅Π½Π° ΠΎΡ‚ Ρ‚ΠΎΡ‡ΠΊΠΈ E простым ΠΏΡ€ΠΎΠ²ΠΎΠ΄ΠΎΠΌ, ΠΎΠ½Π° ΠΈΠΌΠ΅Π΅Ρ‚ практичСски Ρ‚ΠΎΡ‚ ΠΆΠ΅ элСктричСский ΠΏΠΎΡ‚Π΅Π½Ρ†ΠΈΠ°Π» 2 Π’, Ρ‡Ρ‚ΠΎ ΠΈ E.НаконСц, ΠΊΠΎΠ³Π΄Π° заряд ΠΏΡ€ΠΎΡ…ΠΎΠ΄ΠΈΡ‚ Ρ‡Π΅Ρ€Π΅Π· свой послСдний рСзистор, ΠΎΠ½ тСряСт 2 Π’ элСктричСского ΠΏΠΎΡ‚Π΅Π½Ρ†ΠΈΠ°Π»Π° ΠΈ ΠΏΠ°Π΄Π°Π΅Ρ‚ Π΄ΠΎ 0 Π’ Π² Ρ‚ΠΎΡ‡ΠΊΠ΅ G. схСма снова. ΠŸΡ€ΠΈΡ€ΠΎΡΡ‚ энСргии обСспСчиваСтся аккумулятором ΠΏΡ€ΠΈ ΠΏΠ΅Ρ€Π΅ΠΌΠ΅Ρ‰Π΅Π½ΠΈΠΈ заряда с H Π½Π° A.

Π’ Π£Ρ€ΠΎΠΊΠ΅ 3 Π·Π°ΠΊΠΎΠ½ Ома (Ξ”V = I β€’ R) Π±Ρ‹Π» Π²Π²Π΅Π΄Π΅Π½ ΠΊΠ°ΠΊ ΡƒΡ€Π°Π²Π½Π΅Π½ΠΈΠ΅, ΠΊΠΎΡ‚ΠΎΡ€ΠΎΠ΅ связываСт ΠΏΠ°Π΄Π΅Π½ΠΈΠ΅ напряТСния Π½Π° рСзисторС с сопротивлСниСм рСзистора ΠΈ Ρ‚ΠΎΠΊΠΎΠΌ Π½Π° рСзисторС.Π£Ρ€Π°Π²Π½Π΅Π½ΠΈΠ΅ Π·Π°ΠΊΠΎΠ½Π° Ома ΠΌΠΎΠΆΠ½ΠΎ ΠΈΡΠΏΠΎΠ»ΡŒΠ·ΠΎΠ²Π°Ρ‚ΡŒ для любого ΠΎΡ‚Π΄Π΅Π»ΡŒΠ½ΠΎΠ³ΠΎ рСзистора Π² ΠΏΠΎΡΠ»Π΅Π΄ΠΎΠ²Π°Ρ‚Π΅Π»ΡŒΠ½ΠΎΠΉ Ρ†Π΅ΠΏΠΈ. ΠŸΡ€ΠΈ объСдинСнии Π·Π°ΠΊΠΎΠ½Π° Ома с Π½Π΅ΠΊΠΎΡ‚ΠΎΡ€Ρ‹ΠΌΠΈ ΠΏΡ€ΠΈΠ½Ρ†ΠΈΠΏΠ°ΠΌΠΈ, ΡƒΠΆΠ΅ обсуТдСнными Π½Π° этой страницС, Π²ΠΎΠ·Π½ΠΈΠΊΠ°Π΅Ρ‚ большая идСя.

Π’ ΠΏΠΎΡΠ»Π΅Π΄ΠΎΠ²Π°Ρ‚Π΅Π»ΡŒΠ½Ρ‹Ρ… цСпях рСзистор с наибольшим сопротивлСниСм ΠΈΠΌΠ΅Π΅Ρ‚ наибольшСС ΠΏΠ°Π΄Π΅Π½ΠΈΠ΅ напряТСния.

ΠŸΠΎΡΠΊΠΎΠ»ΡŒΠΊΡƒ Π² ΠΏΠΎΡΠ»Π΅Π΄ΠΎΠ²Π°Ρ‚Π΅Π»ΡŒΠ½ΠΎΠΉ Ρ†Π΅ΠΏΠΈ Ρ‚ΠΎΠΊ Π²Π΅Π·Π΄Π΅ ΠΎΠ΄ΠΈΠ½Π°ΠΊΠΎΠ²Ρ‹ΠΉ, Π·Π½Π°Ρ‡Π΅Π½ΠΈΠ΅ I Ξ”V = I β€’ R ΠΎΠ΄ΠΈΠ½Π°ΠΊΠΎΠ²ΠΎ Π½Π° ΠΊΠ°ΠΆΠ΄ΠΎΠΌ ΠΈΠ· рСзисторов ΠΏΠΎΡΠ»Π΅Π΄ΠΎΠ²Π°Ρ‚Π΅Π»ΡŒΠ½ΠΎΠΉ Ρ†Π΅ΠΏΠΈ. Π’Π°ΠΊΠΈΠΌ ΠΎΠ±Ρ€Π°Π·ΠΎΠΌ, ΠΏΠ°Π΄Π΅Π½ΠΈΠ΅ напряТСния (Ξ”V) Π±ΡƒΠ΄Π΅Ρ‚ ΠΈΠ·ΠΌΠ΅Π½ΡΡ‚ΡŒΡΡ с ΠΈΠ·ΠΌΠ΅Π½Π΅Π½ΠΈΠ΅ΠΌ сопротивлСния.Π“Π΄Π΅ Π±Ρ‹ сопротивлСниС Π½ΠΈ Π±Ρ‹Π»ΠΎ наибольшим, ΠΏΠ°Π΄Π΅Π½ΠΈΠ΅ напряТСния Π±ΡƒΠ΄Π΅Ρ‚ наибольшим Ρƒ этого рСзистора. Π£Ρ€Π°Π²Π½Π΅Π½ΠΈΠ΅ Π·Π°ΠΊΠΎΠ½Π° Ома ΠΌΠΎΠΆΠ½ΠΎ ΠΈΡΠΏΠΎΠ»ΡŒΠ·ΠΎΠ²Π°Ρ‚ΡŒ Π½Π΅ Ρ‚ΠΎΠ»ΡŒΠΊΠΎ для прогнозирования Ρ‚ΠΎΠ³ΠΎ, Ρ‡Ρ‚ΠΎ рСзистор Π² ΠΏΠΎΡΠ»Π΅Π΄ΠΎΠ²Π°Ρ‚Π΅Π»ΡŒΠ½ΠΎΠΉ Ρ†Π΅ΠΏΠΈ Π±ΡƒΠ΄Π΅Ρ‚ ΠΈΠΌΠ΅Ρ‚ΡŒ наибольшСС ΠΏΠ°Π΄Π΅Π½ΠΈΠ΅ напряТСния, Π½ΠΎ ΠΈ для расчСта фактичСских Π·Π½Π°Ρ‡Π΅Π½ΠΈΠΉ падСния напряТСния.

Ξ” Π’ 1 = I β€’ R 1 Ξ” Π’ 2 = I β€’ R 2 Ξ” Π’ 3 = I β€’ R 3

ΠœΠ°Ρ‚Π΅ΠΌΠ°Ρ‚ΠΈΡ‡Π΅ΡΠΊΠΈΠΉ Π°Π½Π°Π»ΠΈΠ· ΠΏΠΎΡΠ»Π΅Π΄ΠΎΠ²Π°Ρ‚Π΅Π»ΡŒΠ½Ρ‹Ρ… Ρ†Π΅ΠΏΠ΅ΠΉ

ΠŸΡ€ΠΈΠ²Π΅Π΄Π΅Π½Π½Ρ‹Π΅ Π²Ρ‹ΡˆΠ΅ ΠΏΡ€ΠΈΠ½Ρ†ΠΈΠΏΡ‹ ΠΈ Ρ„ΠΎΡ€ΠΌΡƒΠ»Ρ‹ ΠΌΠΎΠ³ΡƒΡ‚ Π±Ρ‹Ρ‚ΡŒ ΠΈΡΠΏΠΎΠ»ΡŒΠ·ΠΎΠ²Π°Π½Ρ‹ для Π°Π½Π°Π»ΠΈΠ·Π° ΠΏΠΎΡΠ»Π΅Π΄ΠΎΠ²Π°Ρ‚Π΅Π»ΡŒΠ½ΠΎΠΉ Ρ†Π΅ΠΏΠΈ ΠΈ опрСдСлСния Π·Π½Π°Ρ‡Π΅Π½ΠΈΠΉ Ρ‚ΠΎΠΊΠ° ΠΈ разности элСктричСских ΠΏΠΎΡ‚Π΅Π½Ρ†ΠΈΠ°Π»ΠΎΠ² Π½Π° ΠΊΠ°ΠΆΠ΄ΠΎΠΌ ΠΈΠ· рСзисторов Π² ΠΏΠΎΡΠ»Π΅Π΄ΠΎΠ²Π°Ρ‚Π΅Π»ΡŒΠ½ΠΎΠΉ Ρ†Π΅ΠΏΠΈ.Π˜Ρ… использованиС Π±ΡƒΠ΄Π΅Ρ‚ продСмонстрировано матСматичСским Π°Π½Π°Π»ΠΈΠ·ΠΎΠΌ схСмы, ΠΏΠΎΠΊΠ°Π·Π°Π½Π½ΠΎΠΉ Π½ΠΈΠΆΠ΅. ЦСль состоит Π² Ρ‚ΠΎΠΌ, Ρ‡Ρ‚ΠΎΠ±Ρ‹ ΠΈΡΠΏΠΎΠ»ΡŒΠ·ΠΎΠ²Π°Ρ‚ΡŒ Ρ„ΠΎΡ€ΠΌΡƒΠ»Ρ‹ для опрСдСлСния эквивалСнтного сопротивлСния Ρ†Π΅ΠΏΠΈ (R eq ), Ρ‚ΠΎΠΊΠ° Π² Π±Π°Ρ‚Π°Ρ€Π΅Π΅ (I Π΄ΠΎ ), Π° Ρ‚Π°ΠΊΠΆΠ΅ ΠΏΠ°Π΄Π΅Π½ΠΈΠΉ напряТСния ΠΈ Ρ‚ΠΎΠΊΠ° для ΠΊΠ°ΠΆΠ΄ΠΎΠ³ΠΎ ΠΈΠ· Ρ‚Ρ€Π΅Ρ… рСзисторов.

Анализ начинаСтся с использования Π·Π½Π°Ρ‡Π΅Π½ΠΈΠΉ сопротивлСния ΠΎΡ‚Π΄Π΅Π»ΡŒΠ½Ρ‹Ρ… рСзисторов для опрСдСлСния эквивалСнтного сопротивлСния Ρ†Π΅ΠΏΠΈ.

R экв = R 1 + R 2 + R 3 = 17 Ом + 12 Ом + 11 Ом = 40 Ом

Π’Π΅ΠΏΠ΅Ρ€ΡŒ, ΠΊΠΎΠ³Π΄Π° извСстно эквивалСнтноС сопротивлСниС, Ρ‚ΠΎΠΊ Π² Π±Π°Ρ‚Π°Ρ€Π΅Π΅ ΠΌΠΎΠΆΠ½ΠΎ ΠΎΠΏΡ€Π΅Π΄Π΅Π»ΠΈΡ‚ΡŒ с ΠΏΠΎΠΌΠΎΡ‰ΡŒΡŽ уравнСния Π·Π°ΠΊΠΎΠ½Π° Ома.ΠŸΡ€ΠΈ использовании уравнСния Π·Π°ΠΊΠΎΠ½Π° Ома (Ξ”V = I β€’ R) для опрСдСлСния Ρ‚ΠΎΠΊΠ° Π² Ρ†Π΅ΠΏΠΈ Π²Π°ΠΆΠ½ΠΎ ΠΈΡΠΏΠΎΠ»ΡŒΠ·ΠΎΠ²Π°Ρ‚ΡŒ напряТСниС Π±Π°Ρ‚Π°Ρ€Π΅ΠΈ для Ξ”V ΠΈ эквивалСнтноС сопротивлСниС для R. РасчСт ΠΏΠΎΠΊΠ°Π·Π°Π½ здСсь:

I Π΄ΠΎ = Ξ”V аккумулятор / R eq = (60 Π’) / (40 Ом) = 1,5 А

Π—Π½Π°Ρ‡Π΅Π½ΠΈΠ΅ Ρ‚ΠΎΠΊΠ° 1,5 А — это Ρ‚ΠΎΠΊ Π² мСстС располоТСния Π±Π°Ρ‚Π°Ρ€Π΅ΠΈ. Для ΠΏΠΎΡΠ»Π΅Π΄ΠΎΠ²Π°Ρ‚Π΅Π»ΡŒΠ½ΠΎΠΉ Ρ†Π΅ΠΏΠΈ Π±Π΅Π· Ρ‚ΠΎΡ‡Π΅ΠΊ развСтвлСния Ρ‚ΠΎΠΊ Π²Π΅Π·Π΄Π΅ ΠΎΠ΄ΠΈΠ½Π°ΠΊΠΎΠ²Ρ‹ΠΉ.Π’ΠΎΠΊ Π² мСстС располоТСния Π±Π°Ρ‚Π°Ρ€Π΅ΠΈ Ρ‚Π°ΠΊΠΎΠΉ ΠΆΠ΅, ΠΊΠ°ΠΊ Ρ‚ΠΎΠΊ Π² ΠΊΠ°ΠΆΠ΄ΠΎΠΌ мСстС располоТСния рСзистора. ВпослСдствии 1,5 Π°ΠΌΠΏΠ΅Ρ€ — это Π·Π½Π°Ρ‡Π΅Π½ΠΈΠ΅ I 1 , I 2 ΠΈ I 3 .

I аккумулятор = I 1 = I 2 = I 3 = 1,5 А

ΠžΡΡ‚Π°Π»ΠΎΡΡŒ ΠΎΠΏΡ€Π΅Π΄Π΅Π»ΠΈΡ‚ΡŒ Ρ‚Ρ€ΠΈ значСния — ΠΏΠ°Π΄Π΅Π½ΠΈΠ΅ напряТСния Π½Π° ΠΊΠ°ΠΆΠ΄ΠΎΠΌ ΠΎΡ‚Π΄Π΅Π»ΡŒΠ½ΠΎΠΌ рСзисторС. Π—Π°ΠΊΠΎΠ½ Ома снова ΠΈΡΠΏΠΎΠ»ΡŒΠ·ΡƒΠ΅Ρ‚ΡΡ для опрСдСлСния ΠΏΠ°Π΄Π΅Π½ΠΈΠΉ напряТСния для ΠΊΠ°ΠΆΠ΄ΠΎΠ³ΠΎ рСзистора — это просто ΠΏΡ€ΠΎΠΈΠ·Π²Π΅Π΄Π΅Π½ΠΈΠ΅ Ρ‚ΠΎΠΊΠ° Π½Π° ΠΊΠ°ΠΆΠ΄ΠΎΠΌ рСзисторС (вычислСнноС Π²Ρ‹ΡˆΠ΅ ΠΊΠ°ΠΊ 1.5 Π°ΠΌΠΏΠ΅Ρ€) ΠΈ сопротивлСниС ΠΊΠ°ΠΆΠ΄ΠΎΠ³ΠΎ рСзистора (ΡƒΠΊΠ°Π·Π°Π½ΠΎ Π² постановкС Π·Π°Π΄Π°Ρ‡ΠΈ). РасчСты ΠΏΠΎΠΊΠ°Π·Π°Π½Ρ‹ Π½ΠΈΠΆΠ΅.

Ξ”V 1 = I 1 β€’ R 1

Ξ”V 1 = (1,5 A) β€’ (17 Ом)

Ξ”V 1 = 25,5 Π’

Ξ”V 2 = I 2 β€’ R 2

Ξ”V 2 = (1,5 A) β€’ (12 Ом)

Ξ”V 2 = 18 Π’

Ξ”V 3 = I 3 β€’ R 3

Ξ”V 3 = (1.5 А) β€’ (11 Ом)

Ξ”V 3 = 16,5 Π’

Π’ качСствС ΠΏΡ€ΠΎΠ²Π΅Ρ€ΠΊΠΈ точности Π²Ρ‹ΠΏΠΎΠ»Π½Π΅Π½Π½Ρ‹Ρ… матСматичСских расчСтов цСлСсообразно ΠΏΡ€ΠΎΠ²Π΅Ρ€ΠΈΡ‚ΡŒ, ΡƒΠ΄ΠΎΠ²Π»Π΅Ρ‚Π²ΠΎΡ€ΡΡŽΡ‚ Π»ΠΈ вычислСнныС значСния ΠΏΡ€ΠΈΠ½Ρ†ΠΈΠΏΡƒ, согласно ΠΊΠΎΡ‚ΠΎΡ€ΠΎΠΌΡƒ сумма ΠΏΠ°Π΄Π΅Π½ΠΈΠΉ напряТСния для ΠΊΠ°ΠΆΠ΄ΠΎΠ³ΠΎ ΠΎΡ‚Π΄Π΅Π»ΡŒΠ½ΠΎΠ³ΠΎ рСзистора Ρ€Π°Π²Π½Π° Π½ΠΎΠΌΠΈΠ½Π°Π»ΡŒΠ½ΠΎΠΌΡƒ Π½Π°ΠΏΡ€ΡΠΆΠ΅Π½ΠΈΡŽ Π±Π°Ρ‚Π°Ρ€Π΅ΠΈ. Π”Ρ€ΡƒΠ³ΠΈΠΌΠΈ словами, Ξ”V Π±Π°Ρ‚Π°Ρ€Π΅ΠΈ = Ξ”V 1 + Ξ”V 2 + Ξ”V 3 ?

ЯвляСтся Π»ΠΈ Ξ”V Π±Π°Ρ‚Π°Ρ€Π΅ΠΈ = Ξ”V 1 + Ξ”V 2 + Ξ”V 3 ?

Π­Ρ‚ΠΎ 60 Π’ = 25.5 Π’ + 18 Π’ + 16,5 Π’?

60 Π’ = 60 Π’?

Π”Π° !!

ΠœΠ°Ρ‚Π΅ΠΌΠ°Ρ‚ΠΈΡ‡Π΅ΡΠΊΠΈΠΉ Π°Π½Π°Π»ΠΈΠ· этой ΠΏΠΎΡΠ»Π΅Π΄ΠΎΠ²Π°Ρ‚Π΅Π»ΡŒΠ½ΠΎΠΉ схСмы Π²ΠΊΠ»ΡŽΡ‡Π°Π» смСсь ΠΊΠΎΠ½Ρ†Π΅ΠΏΡ†ΠΈΠΉ ΠΈ ΡƒΡ€Π°Π²Π½Π΅Π½ΠΈΠΉ. Как это часто Π±Ρ‹Π²Π°Π΅Ρ‚ Π² Ρ„ΠΈΠ·ΠΈΠΊΠ΅, ΠΎΡ‚Π΄Π΅Π»Π΅Π½ΠΈΠ΅ понятий ΠΎΡ‚ ΡƒΡ€Π°Π²Π½Π΅Π½ΠΈΠΉ ΠΏΡ€ΠΈ принятии Ρ€Π΅ΡˆΠ΅Π½ΠΈΡ физичСской ΠΏΡ€ΠΎΠ±Π»Π΅ΠΌΡ‹ являСтся опасным Π°ΠΊΡ‚ΠΎΠΌ. Π—Π΄Π΅ΡΡŒ Π½Π΅ΠΎΠ±Ρ…ΠΎΠ΄ΠΈΠΌΠΎ ΡƒΡ‡ΠΈΡ‚Ρ‹Π²Π°Ρ‚ΡŒ ΠΊΠΎΠ½Ρ†Π΅ΠΏΡ†ΠΈΠΈ, согласно ΠΊΠΎΡ‚ΠΎΡ€Ρ‹ΠΌ Ρ‚ΠΎΠΊ Π²Π΅Π·Π΄Π΅ ΠΎΠ΄ΠΈΠ½Π°ΠΊΠΎΠ² ΠΈ Ρ‡Ρ‚ΠΎ напряТСниС Π±Π°Ρ‚Π°Ρ€Π΅ΠΈ эквивалСнтно суммС ΠΏΠ°Π΄Π΅Π½ΠΈΠΉ напряТСния Π½Π° ΠΊΠ°ΠΆΠ΄ΠΎΠΌ рСзисторС, Ρ‡Ρ‚ΠΎΠ±Ρ‹ Π·Π°Π²Π΅Ρ€ΡˆΠΈΡ‚ΡŒ матСматичСский Π°Π½Π°Π»ΠΈΠ·.Π’ ΡΠ»Π΅Π΄ΡƒΡŽΡ‰Π΅ΠΉ части Π£Ρ€ΠΎΠΊΠ° 4 ΠΏΠ°Ρ€Π°Π»Π»Π΅Π»ΡŒΠ½Ρ‹Π΅ Ρ†Π΅ΠΏΠΈ Π±ΡƒΠ΄ΡƒΡ‚ ΠΏΡ€ΠΎΠ°Π½Π°Π»ΠΈΠ·ΠΈΡ€ΠΎΠ²Π°Π½Ρ‹ с использованиСм Π·Π°ΠΊΠΎΠ½Π° Ома ΠΈ ΠΊΠΎΠ½Ρ†Π΅ΠΏΡ†ΠΈΠΉ ΠΏΠ°Ρ€Π°Π»Π»Π΅Π»ΡŒΠ½Ρ‹Ρ… Ρ†Π΅ΠΏΠ΅ΠΉ. ΠœΡ‹ ΡƒΠ²ΠΈΠ΄ΠΈΠΌ, Ρ‡Ρ‚ΠΎ ΠΏΠΎΠ΄Ρ…ΠΎΠ΄ сочСтания ΠΊΠΎΠ½Ρ†Π΅ΠΏΡ†ΠΈΠΉ с уравнСниями Π±ΡƒΠ΄Π΅Ρ‚ Π½Π΅ ΠΌΠ΅Π½Π΅Π΅ Π²Π°ΠΆΠ΅Π½ для этого Π°Π½Π°Π»ΠΈΠ·Π°.

ΠœΡ‹ Ρ…ΠΎΡ‚Π΅Π»ΠΈ Π±Ρ‹ ΠΏΡ€Π΅Π΄Π»ΠΎΠΆΠΈΡ‚ΡŒ … Π—Π°Ρ‡Π΅ΠΌ просто Ρ‡ΠΈΡ‚Π°Ρ‚ΡŒ ΠΎΠ± этом ΠΈ ΠΊΠΎΠ³Π΄Π° ΠΌΠΎΠΆΠ½ΠΎ с этим Π²Π·Π°ΠΈΠΌΠΎΠ΄Π΅ΠΉΡΡ‚Π²ΠΎΠ²Π°Ρ‚ΡŒ? Π’Π·Π°ΠΈΠΌΠΎΠ΄Π΅ΠΉΡΡ‚Π²ΠΎΠ²Π°Ρ‚ΡŒ — это ΠΈΠΌΠ΅Π½Π½ΠΎ Ρ‚ΠΎ, Ρ‡Ρ‚ΠΎ Π²Ρ‹ Π΄Π΅Π»Π°Π΅Ρ‚Π΅, ΠΊΠΎΠ³Π΄Π° ΠΈΡΠΏΠΎΠ»ΡŒΠ·ΡƒΠ΅Ρ‚Π΅ ΠΎΠ΄Π½ΠΎ ΠΈΠ· ΠΈΠ½Ρ‚Π΅Ρ€Π°ΠΊΡ‚ΠΈΠ²Π½Ρ‹Ρ… ΠΌΠ°Ρ‚Π΅Ρ€ΠΈΠ°Π»ΠΎΠ² The Physics Classroom.ΠœΡ‹ Ρ…ΠΎΡ‚Π΅Π»ΠΈ Π±Ρ‹ ΠΏΡ€Π΅Π΄Π»ΠΎΠΆΠΈΡ‚ΡŒ Π²Π°ΠΌ ΡΠΎΠ²ΠΌΠ΅ΡΡ‚ΠΈΡ‚ΡŒ Ρ‡Ρ‚Π΅Π½ΠΈΠ΅ этой страницы с использованиСм нашСго ΠΈΠ½Ρ‚Π΅Ρ€Π°ΠΊΡ‚ΠΈΠ²Π½ΠΎΠ³ΠΎ срСдства построСния Ρ†Π΅ΠΏΠ΅ΠΉ постоянного Ρ‚ΠΎΠΊΠ°. Π’Ρ‹ ΠΌΠΎΠΆΠ΅Ρ‚Π΅ Π½Π°ΠΉΡ‚ΠΈ Π΅Π³ΠΎ Π² Ρ€Π°Π·Π΄Π΅Π»Π΅ Physics Interactives Π½Π° нашСм сайтС. ΠŸΠΎΡΡ‚Ρ€ΠΎΠΈΡ‚Π΅Π»ΡŒ Ρ†Π΅ΠΏΠ΅ΠΉ постоянного Ρ‚ΠΎΠΊΠ° прСдоставляСт учащСмуся Π½Π°Π±ΠΎΡ€ для построСния Π²ΠΈΡ€Ρ‚ΡƒΠ°Π»ΡŒΠ½Ρ‹Ρ… Ρ†Π΅ΠΏΠ΅ΠΉ. Π’Ρ‹ ΠΌΠΎΠΆΠ΅Ρ‚Π΅ Π»Π΅Π³ΠΊΠΎ ΠΏΠ΅Ρ€Π΅Ρ‚Π°Ρ‰ΠΈΡ‚ΡŒ источники напряТСния, рСзисторы ΠΈ ΠΏΡ€ΠΎΠ²ΠΎΠ΄Π° Π½Π° Ρ€Π°Π±ΠΎΡ‡Π΅Π΅ мСсто, Π° Ρ‚Π°ΠΊΠΆΠ΅ Ρ€Π°ΡΠΏΠΎΠ»ΠΎΠΆΠΈΡ‚ΡŒ ΠΈ ΠΏΠΎΠ΄ΠΊΠ»ΡŽΡ‡ΠΈΡ‚ΡŒ ΠΈΡ… Ρ‚Π°ΠΊ, ΠΊΠ°ΠΊ Π·Π°Ρ…ΠΎΡ‚ΠΈΡ‚Π΅. Π’ΠΎΠ»ΡŒΡ‚ΠΌΠ΅Ρ‚Ρ€Ρ‹ ΠΈ Π°ΠΌΠΏΠ΅Ρ€ΠΌΠ΅Ρ‚Ρ€Ρ‹ ΠΏΠΎΠ·Π²ΠΎΠ»ΡΡŽΡ‚ ΠΈΠ·ΠΌΠ΅Ρ€ΡΡ‚ΡŒ ΠΏΠ°Π΄Π΅Π½ΠΈΠ΅ Ρ‚ΠΎΠΊΠ° ΠΈ напряТСния. НаТатиС Π½Π° рСзистор ΠΈΠ»ΠΈ источник напряТСния позволяСт ΠΈΠ·ΠΌΠ΅Π½ΡΡ‚ΡŒ сопротивлСниС ΠΈΠ»ΠΈ Π²Ρ…ΠΎΠ΄Π½ΠΎΠ΅ напряТСниС.Π­Ρ‚ΠΎ просто. Π­Ρ‚ΠΎ вСсСло. И это бСзопасно (Ссли Π²Ρ‹ Π½Π΅ ΠΈΡΠΏΠΎΠ»ΡŒΠ·ΡƒΠ΅Ρ‚Π΅ Π΅Π³ΠΎ Π² Π²Π°Π½Π½Π΅).


ΠŸΡ€ΠΎΠ²Π΅Ρ€ΡŒΡ‚Π΅ своС ΠΏΠΎΠ½ΠΈΠΌΠ°Π½ΠΈΠ΅

1. Π˜ΡΠΏΠΎΠ»ΡŒΠ·ΡƒΠΉΡ‚Π΅ своС ΠΏΠΎΠ½ΠΈΠΌΠ°Π½ΠΈΠ΅ эквивалСнтного сопротивлСния, Ρ‡Ρ‚ΠΎΠ±Ρ‹ Π·Π°ΠΏΠΎΠ»Π½ΠΈΡ‚ΡŒ ΡΠ»Π΅Π΄ΡƒΡŽΡ‰ΠΈΠ΅ утвСрТдСния:

Π°. Π”Π²Π° ΠΏΠΎΡΠ»Π΅Π΄ΠΎΠ²Π°Ρ‚Π΅Π»ΡŒΠ½ΠΎ Π²ΠΊΠ»ΡŽΡ‡Π΅Π½Π½Ρ‹Ρ… рСзистора сопротивлСниСм 3 Ом обСспСчат сопротивлСниС, эквивалСнтноС ΡΠΎΠΏΡ€ΠΎΡ‚ΠΈΠ²Π»Π΅Π½ΠΈΡŽ ΠΎΠ΄Π½ΠΎΠ³ΠΎ рСзистора _____ Ом.

Π³. Π’Ρ€ΠΈ рСзистора сопротивлСниСм 3 Ом, Π²ΠΊΠ»ΡŽΡ‡Π΅Π½Π½Ρ‹Π΅ ΠΏΠΎΡΠ»Π΅Π΄ΠΎΠ²Π°Ρ‚Π΅Π»ΡŒΠ½ΠΎ, обСспСчат сопротивлСниС, эквивалСнтноС ΡΠΎΠΏΡ€ΠΎΡ‚ΠΈΠ²Π»Π΅Π½ΠΈΡŽ ΠΎΠ΄Π½ΠΎΠ³ΠΎ рСзистора _____ Ом.

Π³. Π’Ρ€ΠΈ рСзистора 5 Ом, Π²ΠΊΠ»ΡŽΡ‡Π΅Π½Π½Ρ‹Π΅ ΠΏΠΎΡΠ»Π΅Π΄ΠΎΠ²Π°Ρ‚Π΅Π»ΡŒΠ½ΠΎ, обСспСчат сопротивлСниС, эквивалСнтноС ΠΎΠ΄Π½ΠΎΠΌΡƒ рСзистору _____ Ом.

Π³. Π’Ρ€ΠΈ рСзистора с сопротивлСниСм 2 Ом, 4 Ом ΠΈ 6 Ом Π²ΠΊΠ»ΡŽΡ‡Π΅Π½Ρ‹ ΠΏΠΎΡΠ»Π΅Π΄ΠΎΠ²Π°Ρ‚Π΅Π»ΡŒΠ½ΠΎ. Они обСспСчили Π±Ρ‹ сопротивлСниС, эквивалСнтноС ΠΎΠ΄Π½ΠΎΠΌΡƒ рСзистору _____ Ом.

e. Π’Ρ€ΠΈ рСзистора с сопротивлСниСм 5 Ом, 6 Ом ΠΈ 7 Ом Π²ΠΊΠ»ΡŽΡ‡Π΅Π½Ρ‹ ΠΏΠΎΡΠ»Π΅Π΄ΠΎΠ²Π°Ρ‚Π΅Π»ΡŒΠ½ΠΎ. Они обСспСчили Π±Ρ‹ сопротивлСниС, эквивалСнтноС ΠΎΠ΄Π½ΠΎΠΌΡƒ рСзистору _____ Ом.

Ρ„. Π’Ρ€ΠΈ рСзистора с сопротивлСниСм 12 Ом, 3 Ом ΠΈ 21 Ом Π²ΠΊΠ»ΡŽΡ‡Π΅Π½Ρ‹ ΠΏΠΎΡΠ»Π΅Π΄ΠΎΠ²Π°Ρ‚Π΅Π»ΡŒΠ½ΠΎ. Они обСспСчили Π±Ρ‹ сопротивлСниС, эквивалСнтноС ΠΎΠ΄Π½ΠΎΠΌΡƒ рСзистору _____ Ом.

2. По ΠΌΠ΅Ρ€Π΅ увСличСния количСства рСзисторов Π² ΠΏΠΎΡΠ»Π΅Π΄ΠΎΠ²Π°Ρ‚Π΅Π»ΡŒΠ½ΠΎΠΉ Ρ†Π΅ΠΏΠΈ ΠΎΠ±Ρ‰Π΅Π΅ сопротивлСниС __________ (увСличиваСтся, ΡƒΠΌΠ΅Π½ΡŒΡˆΠ°Π΅Ρ‚ΡΡ, остаСтся ΠΏΡ€Π΅ΠΆΠ½ΠΈΠΌ) ΠΈ Ρ‚ΠΎΠΊ Π² Ρ†Π΅ΠΏΠΈ __________ (увСличиваСтся, ΡƒΠΌΠ΅Π½ΡŒΡˆΠ°Π΅Ρ‚ΡΡ, остаСтся ΠΏΡ€Π΅ΠΆΠ½ΠΈΠΌ).


3. Рассмотрим ΡΠ»Π΅Π΄ΡƒΡŽΡ‰ΠΈΠ΅ Π΄Π²Π΅ схСмы ΠΏΠΎΡΠ»Π΅Π΄ΠΎΠ²Π°Ρ‚Π΅Π»ΡŒΠ½Ρ‹Ρ… Ρ†Π΅ΠΏΠ΅ΠΉ. На ΠΊΠ°ΠΆΠ΄ΠΎΠΉ Π΄ΠΈΠ°Π³Ρ€Π°ΠΌΠΌΠ΅ ΠΈΡΠΏΠΎΠ»ΡŒΠ·ΡƒΠΉΡ‚Π΅ стрСлки, Ρ‡Ρ‚ΠΎΠ±Ρ‹ ΡƒΠΊΠ°Π·Π°Ρ‚ΡŒ Π½Π°ΠΏΡ€Π°Π²Π»Π΅Π½ΠΈΠ΅ ΠΎΠ±Ρ‹Ρ‡Π½ΠΎΠ³ΠΎ Ρ‚ΠΎΠΊΠ°. Π—Π°Ρ‚Π΅ΠΌ сравнитС напряТСниС ΠΈ Ρ‚ΠΎΠΊ Π² ΠΎΠ±ΠΎΠ·Π½Π°Ρ‡Π΅Π½Π½Ρ‹Ρ… Ρ‚ΠΎΡ‡ΠΊΠ°Ρ… для ΠΊΠ°ΠΆΠ΄ΠΎΠΉ Π΄ΠΈΠ°Π³Ρ€Π°ΠΌΠΌΡ‹.


4. Π’Ρ€ΠΈ ΠΎΠ΄ΠΈΠ½Π°ΠΊΠΎΠ²Ρ‹Π΅ Π»Π°ΠΌΠΏΠΎΡ‡ΠΊΠΈ ΠΏΠΎΠ΄ΠΊΠ»ΡŽΡ‡Π΅Π½Ρ‹ ΠΊ D-ячСйкС, ΠΊΠ°ΠΊ ΠΏΠΎΠΊΠ°Π·Π°Π½ΠΎ справа.КакоС ΠΈΠ· ΡΠ»Π΅Π΄ΡƒΡŽΡ‰ΠΈΡ… ΡƒΡ‚Π²Π΅Ρ€ΠΆΠ΄Π΅Π½ΠΈΠΉ Π²Π΅Ρ€Π½ΠΎ?

Π°. ВсС Ρ‚Ρ€ΠΈ Π»Π°ΠΌΠΏΠΎΡ‡ΠΊΠΈ Π±ΡƒΠ΄ΡƒΡ‚ ΠΈΠΌΠ΅Ρ‚ΡŒ ΠΎΠ΄ΠΈΠ½Π°ΠΊΠΎΠ²ΡƒΡŽ ΡΡ€ΠΊΠΎΡΡ‚ΡŒ.

Π³. Π›Π°ΠΌΠΏΠ° ΠΌΠ΅ΠΆΠ΄Ρƒ X ΠΈ Y Π±ΡƒΠ΄Π΅Ρ‚ самой яркой.

Π³. Π›Π°ΠΌΠΏΠ° ΠΌΠ΅ΠΆΠ΄Ρƒ Y ΠΈ Z Π±ΡƒΠ΄Π΅Ρ‚ самой яркой.

Π³. Π›Π°ΠΌΠΏΠΎΡ‡ΠΊΠ° ΠΌΠ΅ΠΆΠ΄Ρƒ Z ΠΈ Π±Π°Ρ‚Π°Ρ€Π΅Π΅ΠΉ Π±ΡƒΠ΄Π΅Ρ‚ самой яркой.

5. Π’Ρ€ΠΈ ΠΎΠ΄ΠΈΠ½Π°ΠΊΠΎΠ²Ρ‹Π΅ Π»Π°ΠΌΠΏΠΎΡ‡ΠΊΠΈ ΠΏΠΎΠ΄ΠΊΠ»ΡŽΡ‡Π΅Π½Ρ‹ ΠΊ Π±Π°Ρ‚Π°Ρ€Π΅Π΅, ΠΊΠ°ΠΊ ΠΏΠΎΠΊΠ°Π·Π°Π½ΠΎ справа.КакиС настройки ΠΌΠΎΠΆΠ½ΠΎ Π±Ρ‹Π»ΠΎ Π±Ρ‹ внСсти Π² схСму, Ρ‡Ρ‚ΠΎΠ±Ρ‹ ΡƒΠ²Π΅Π»ΠΈΡ‡ΠΈΡ‚ΡŒ Ρ‚ΠΎΠΊ, измСряСмый Π² Ρ‚ΠΎΡ‡ΠΊΠ΅ X? ΠŸΠ΅Ρ€Π΅Ρ‡ΠΈΡΠ»ΠΈΡ‚Π΅ всС подходящиС Π²Π°Ρ€ΠΈΠ°Π½Ρ‚Ρ‹.

Π°. Π£Π²Π΅Π»ΠΈΡ‡ΡŒΡ‚Π΅ сопротивлСниС ΠΎΠ΄Π½ΠΎΠΉ ΠΈΠ· Π»Π°ΠΌΠΏΠΎΡ‡Π΅ΠΊ.

Π³. Π£Π²Π΅Π»ΠΈΡ‡ΡŒΡ‚Π΅ сопротивлСниС Π΄Π²ΡƒΡ… Π»Π°ΠΌΠΏΠΎΡ‡Π΅ΠΊ.

Π³. Π£ΠΌΠ΅Π½ΡŒΡˆΠΈΡ‚Π΅ сопротивлСниС Π΄Π²ΡƒΡ… Π»Π°ΠΌΠΏΠΎΡ‡Π΅ΠΊ.

Π³. Π£Π²Π΅Π»ΠΈΡ‡ΡŒΡ‚Π΅ напряТСниС аккумулятора.

e. Π£ΠΌΠ΅Π½ΡŒΡˆΠΈΡ‚Π΅ напряТСниС аккумулятора.

Ρ„. Π£Π΄Π°Π»ΠΈΡ‚Π΅ ΠΎΠ΄Π½Ρƒ ΠΈΠ· Π»ΡƒΠΊΠΎΠ²ΠΈΡ†.


6. Π’Ρ€ΠΈ ΠΎΠ΄ΠΈΠ½Π°ΠΊΠΎΠ²Ρ‹Π΅ Π»Π°ΠΌΠΏΠΎΡ‡ΠΊΠΈ ΠΏΠΎΠ΄ΠΊΠ»ΡŽΡ‡Π΅Π½Ρ‹ ΠΊ Π±Π°Ρ‚Π°Ρ€Π΅Π΅, ΠΊΠ°ΠΊ ΠΏΠΎΠΊΠ°Π·Π°Π½ΠΎ справа. W, X, Y ΠΈ Z ΠΎΠ±ΠΎΠ·Π½Π°Ρ‡Π°ΡŽΡ‚ мСста Π½Π° трассС. КакоС ΠΈΠ· ΡΠ»Π΅Π΄ΡƒΡŽΡ‰ΠΈΡ… ΡƒΡ‚Π²Π΅Ρ€ΠΆΠ΄Π΅Π½ΠΈΠΉ Π²Π΅Ρ€Π½ΠΎ?

Π°. Π Π°Π·Π½ΠΈΡ†Π° ΠΏΠΎΡ‚Π΅Π½Ρ†ΠΈΠ°Π»ΠΎΠ² ΠΌΠ΅ΠΆΠ΄Ρƒ X ΠΈ Y большС, Ρ‡Π΅ΠΌ ΠΌΠ΅ΠΆΠ΄Ρƒ Y ΠΈ Z.

Π³. Π Π°Π·Π½ΠΈΡ†Π° ΠΏΠΎΡ‚Π΅Π½Ρ†ΠΈΠ°Π»ΠΎΠ² ΠΌΠ΅ΠΆΠ΄Ρƒ X ΠΈ Y большС, Ρ‡Π΅ΠΌ ΠΌΠ΅ΠΆΠ΄Ρƒ Y ΠΈ W.

Π³. Π Π°Π·Π½ΠΎΡΡ‚ΡŒ ΠΏΠΎΡ‚Π΅Π½Ρ†ΠΈΠ°Π»ΠΎΠ² ΠΌΠ΅ΠΆΠ΄Ρƒ Y ΠΈ Z большС, Ρ‡Π΅ΠΌ ΠΌΠ΅ΠΆΠ΄Ρƒ Y ΠΈ W.

Π³. Π Π°Π·Π½ΠΎΡΡ‚ΡŒ ΠΏΠΎΡ‚Π΅Π½Ρ†ΠΈΠ°Π»ΠΎΠ² ΠΌΠ΅ΠΆΠ΄Ρƒ X ΠΈ Z большС, Ρ‡Π΅ΠΌ ΠΌΠ΅ΠΆΠ΄Ρƒ Z ΠΈ W.

e. Π Π°Π·Π½ΠΎΡΡ‚ΡŒ ΠΏΠΎΡ‚Π΅Π½Ρ†ΠΈΠ°Π»ΠΎΠ² ΠΌΠ΅ΠΆΠ΄Ρƒ X ΠΈ W большС, Ρ‡Π΅ΠΌ Π½Π° Π±Π°Ρ‚Π°Ρ€Π΅Π΅.

Ρ„. Π Π°Π·Π½ΠΈΡ†Π° ΠΏΠΎΡ‚Π΅Π½Ρ†ΠΈΠ°Π»ΠΎΠ² ΠΌΠ΅ΠΆΠ΄Ρƒ X ΠΈ Y большС, Ρ‡Π΅ΠΌ ΠΌΠ΅ΠΆΠ΄Ρƒ Z ΠΈ W.


7.Π‘Ρ€Π°Π²Π½ΠΈΡ‚Π΅ схСму X ΠΈ Y Π½ΠΈΠΆΠ΅. ΠšΠ°ΠΆΠ΄Ρ‹ΠΉ питаСтся ΠΎΡ‚ 12-Π²ΠΎΠ»ΡŒΡ‚ΠΎΠ²ΠΎΠΉ Π±Π°Ρ‚Π°Ρ€Π΅ΠΈ. ПадСниС напряТСния Π½Π° рСзисторС 12 Ом Π² Ρ†Π΅ΠΏΠΈ Y Ρ€Π°Π²Π½ΠΎ ____ падСнию напряТСния Π½Π° СдинствСнном рСзисторС Π² Ρ†Π΅ΠΏΠΈ X.

Π°. мСньшС Ρ‡Π΅ΠΌ

г. большС

Π³. Ρ‚ΠΎ ΠΆΠ΅, Ρ‡Ρ‚ΠΎ

8. Аккумулятор Π½Π° 12 Π’, рСзистор Π½Π° 12 Ом ΠΈ Π»Π°ΠΌΠΏΠΎΡ‡ΠΊΠ° ΠΏΠΎΠ΄ΠΊΠ»ΡŽΡ‡Π°ΡŽΡ‚ΡΡ, ΠΊΠ°ΠΊ ΠΏΠΎΠΊΠ°Π·Π°Π½ΠΎ Π½Π° схСмС X Π½ΠΈΠΆΠ΅.РСзистор Π½Π° 6 Ом Π΄ΠΎΠ±Π°Π²Π»Π΅Π½ ΠΊ рСзистору Π½Π° 12 Ом ΠΈ Π»Π°ΠΌΠΏΠΎΡ‡ΠΊΠ΅, Ρ‡Ρ‚ΠΎΠ±Ρ‹ ΡΠΎΠ·Π΄Π°Ρ‚ΡŒ Ρ†Π΅ΠΏΡŒ Y, ΠΊΠ°ΠΊ ΠΏΠΎΠΊΠ°Π·Π°Π½ΠΎ. Π›Π°ΠΌΠΏΠΎΡ‡ΠΊΠ° появится ____.

Π°. Π΄ΠΈΠΌΠΌΠ΅Ρ€ Π² ΠΊΠΎΠ½Ρ‚ΡƒΡ€Π΅ X

Π³. Π΄ΠΈΠΌΠΌΠ΅Ρ€ Π² ΠΊΠΎΠ½Ρ‚ΡƒΡ€Π΅ Y

Π³. одинаковая ΡΡ€ΠΊΠΎΡΡ‚ΡŒ Π² ΠΎΠ±Π΅ΠΈΡ… цСпях


9. Π’Ρ€ΠΈ рСзистора Π²ΠΊΠ»ΡŽΡ‡Π΅Π½Ρ‹ ΠΏΠΎΡΠ»Π΅Π΄ΠΎΠ²Π°Ρ‚Π΅Π»ΡŒΠ½ΠΎ. Если ΠΏΠΎΠΌΠ΅ΡΡ‚ΠΈΡ‚ΡŒ Π² Ρ†Π΅ΠΏΡŒ с источником питания 12 Π’.ΠžΠΏΡ€Π΅Π΄Π΅Π»ΠΈΡ‚Π΅ эквивалСнтноС сопротивлСниС, ΠΏΠΎΠ»Π½Ρ‹ΠΉ Ρ‚ΠΎΠΊ Ρ†Π΅ΠΏΠΈ, ΠΏΠ°Π΄Π΅Π½ΠΈΠ΅ напряТСния ΠΈ Ρ‚ΠΎΠΊ Π½Π° ΠΊΠ°ΠΆΠ΄ΠΎΠΌ рСзисторС.

ΠžΡ‚Π΄Π΅Π» Ρ†Π΅ΠΏΠ΅ΠΉ ΠΈ напряТСний сСрии

ΠžΡ‚Π΄Π΅Π» Ρ†Π΅ΠΏΠ΅ΠΉ ΠΈ напряТСний сСрии


ΠŸΡ€ΠΈΠΌΠ΅Ρ€ 1: НайдитС ΠΏΠΎΠ»Π½ΠΎΠ΅ эквивалСнтноС сопротивлСниС Π² ΡΠ»Π΅Π΄ΡƒΡŽΡ‰Π΅ΠΉ Ρ†Π΅ΠΏΠΈ


ΠŸΡ€ΠΈΠΌΠ΅Ρ€ 2: Для ΡΠ»Π΅Π΄ΡƒΡŽΡ‰Π΅ΠΉ Ρ†Π΅ΠΏΠΈ:

  1. НайдитС ΠΏΠΎΠ»Π½ΠΎΠ΅ сопротивлСниС
  2. Найти Ρ‚Π΅ΠΊΡƒΡ‰ΠΈΠΉ i
  3. НайдитС напряТСниС Π½Π° рСзисторС 10 Ом

РСшСниС

  1. ΠžΠ±Ρ‰Π΅Π΅ сопротивлСниС

  1. Π’ΠΎΠΊ ΠΌΠΎΠΆΠ½ΠΎ Ρ€Π°ΡΡΡ‡ΠΈΡ‚Π°Ρ‚ΡŒ ΠΊΠ°ΠΊ

  2. НапряТСниС Π½Π° рСзисторС 10 Ом


    ΠŸΡ€ΠΈΠΌΠ΅Ρ€ 3: Для ΡΠ»Π΅Π΄ΡƒΡŽΡ‰Π΅ΠΉ Ρ†Π΅ΠΏΠΈ:

    1. НайдитС ΠΎΠ±Ρ‰Π΅Π΅ Π·Π½Π°Ρ‡Π΅Π½ΠΈΠ΅ сопротивлСния R T
    2. Найти Ρ‚Π΅ΠΊΡƒΡ‰ΠΈΠΉ i
    3. НайдитС напряТСниС Π½Π° ΠΎΡ‚Π΄Π΅Π»ΡŒΠ½Ρ‹Ρ… рСзисторах
    4. ΠŸΡ€ΠΎΠ²Π΅Ρ€ΠΈΡ‚ΡŒ Π·Π°ΠΊΠΎΠ½ напряТСния ΠšΠΈΡ€Ρ…Π³ΠΎΡ„Π°

    ΠŸΡ€ΠΈΠΌΠ΅Ρ€ 4: Для ΡΠ»Π΅Π΄ΡƒΡŽΡ‰Π΅ΠΉ Ρ†Π΅ΠΏΠΈ:

    1. Найти V 1
    2. Найти V 2
    3. ΠŸΡ€ΠΎΠ²Π΅Ρ€ΠΈΡ‚ΡŒ Π·Π°ΠΊΠΎΠ½ напряТСния ΠšΠΈΡ€Ρ…Π³ΠΎΡ„Π° Π²ΠΎΠΊΡ€ΡƒΠ³ Π·Π°ΠΌΠΊΠ½ΡƒΡ‚ΠΎΠ³ΠΎ ΠΊΠΎΠ½Ρ‚ΡƒΡ€Π°


    ΠžΡ‚Π΄Π΅Π» напряТСния :

    Π’ ΡΠ»Π΅Π΄ΡƒΡŽΡ‰Π΅ΠΉ схСмС Ρ‚ΠΎΠΊ Ρ‡Π΅Ρ€Π΅Π· всС ΠΏΠΎΡΠ»Π΅Π΄ΠΎΠ²Π°Ρ‚Π΅Π»ΡŒΠ½ΠΎ Π²ΠΊΠ»ΡŽΡ‡Π΅Π½Π½Ρ‹Π΅ рСзисторы Ρ€Π°Π²Π΅Π½
    . Π­ΠΊΠ²ΠΈΠ²Π°Π»Π΅Π½Ρ‚Π½Ρ‹ΠΉ рСзистор R eq являСтся суммой Π½ΠΎΠΌΠΈΠ½Π°Π»Π° рСзистора.

    Π§Ρ‚ΠΎΠ±Ρ‹ Π½Π°ΠΉΡ‚ΠΈ ΠΏΠ°Π΄Π΅Π½ΠΈΠ΅ напряТСния v ΠΈ Π½Π° рСзисторС R ΠΈ , ΠΌΡ‹ ΠΈΡΠΏΠΎΠ»ΡŒΠ·ΡƒΠ΅ΠΌ Ρ‚ΠΎΠΊ ΠΈ Π·Π½Π°Ρ‡Π΅Π½ΠΈΠ΅ рСзистора

    .

    ΠŸΡ€Π°Π²Π°Ρ Ρ‡Π°ΡΡ‚ΡŒ уравнСния Π΄Π°Π΅Ρ‚ Π½Π°ΠΌ ΡƒΡ€Π°Π²Π½Π΅Π½ΠΈΠ΅ дСлСния напряТСния.


    ΠŸΡ€ΠΈΠΌΠ΅Ρ€Ρ‹:

    ΠŸΡ€ΠΈΠΌΠ΅Ρ€ 5: Для ΡΠ»Π΅Π΄ΡƒΡŽΡ‰Π΅ΠΉ схСмы

    1. НайдитС ΠΎΠ±Ρ‰Π΅Π΅ Π·Π½Π°Ρ‡Π΅Π½ΠΈΠ΅ сопротивлСния R T
    2. Найти Ρ‚Π΅ΠΊΡƒΡ‰ΠΈΠΉ i
    3. НайдитС напряТСниС Π½Π° рСзисторах
    4. ΠŸΡ€ΠΎΠ²Π΅Ρ€ΠΈΡ‚ΡŒ Π·Π°ΠΊΠΎΠ½ напряТСния ΠšΠΈΡ€Ρ…Π³ΠΎΡ„Π°

    ΠŸΡ€ΠΈΠΌΠ΅Ρ€ 6: НайдитС Ρ‚ΠΎΠΊ для ΡΠ»Π΅Π΄ΡƒΡŽΡ‰Π΅ΠΉ Ρ†Π΅ΠΏΠΈ


    ΠŸΡ€Π°ΠΊΡ‚ΠΈΡ‡Π΅ΡΠΊΠΈΠ΅ Π·Π°Π΄Π°Ρ‡ΠΈ :

    (Π©Π΅Π»ΠΊΠ½ΠΈΡ‚Π΅ ΠΈΠ·ΠΎΠ±Ρ€Π°ΠΆΠ΅Π½ΠΈΠ΅, Ρ‡Ρ‚ΠΎΠ±Ρ‹ ΠΏΡ€ΠΎΡΠΌΠΎΡ‚Ρ€Π΅Ρ‚ΡŒ Ρ€Π΅ΡˆΠ΅Π½ΠΈΠ΅)

    Π—Π°Π΄Π°Ρ‡Π° 1: НайдитС ΡΠ»Π΅Π΄ΡƒΡŽΡ‰ΡƒΡŽ схСму.

    ΠŸΡ€ΠΎΡΠΌΠΎΡ‚Ρ€Π΅Ρ‚ΡŒ Ρ€Π΅ΡˆΠ΅Π½ΠΈΠ΅

    РСшСниС:

    ΠŸΡ€ΠΈΠΌΠ΅Π½ΠΈΡ‚ΡŒ Π·Π°ΠΊΠΎΠ½ ΠšΠΈΡ€Ρ…Π³ΠΎΡ„Π° ΠΎ напряТСнии


      Π—Π°Π΄Π°Ρ‡Π° 2: РассчитайтС напряТСния для ΡΠ»Π΅Π΄ΡƒΡŽΡ‰Π΅ΠΉ Ρ†Π΅ΠΏΠΈ.

      ΠŸΡ€ΠΎΡΠΌΠΎΡ‚Ρ€Π΅Ρ‚ΡŒ Ρ€Π΅ΡˆΠ΅Π½ΠΈΠ΅

      РСшСниС:

      Ρ‚Π΅ΠΊΡƒΡ‰Π΅Π΅,

      ΠŸΡ€ΠΈΠΌΠ΅Π½Π΅Π½ΠΈΠ΅ Π·Π°ΠΊΠΎΠ½Π° ΠšΠΈΡ€Ρ…Π³ΠΎΡ„Π°:


        ΠŸΡ€ΠΎΠ±Π»Π΅ΠΌΠ° 3: НайдитС ΠΏΡ€ΠΈΠ»ΠΎΠΆΠ΅Π½Π½ΠΎΠ΅ напряТСниС ΠΊ ΡΠ»Π΅Π΄ΡƒΡŽΡ‰Π΅ΠΉ Ρ†Π΅ΠΏΠΈ, ΠΈΡΠΏΠΎΠ»ΡŒΠ·ΡƒΡ ΠΏΡ€Π΅Π΄ΠΎΡΡ‚Π°Π²Π»Π΅Π½Π½ΡƒΡŽ ΠΈΠ½Ρ„ΠΎΡ€ΠΌΠ°Ρ†ΠΈΡŽ

        ΠŸΡ€ΠΎΡΠΌΠΎΡ‚Ρ€Π΅Ρ‚ΡŒ Ρ€Π΅ΡˆΠ΅Π½ΠΈΠ΅

        ΠŸΡ€ΠΎΠ±Π»Π΅ΠΌΠ° 4: НайдитС Π·Π½Π°Ρ‡Π΅Π½ΠΈΠ΅ рСзистора R , ΠΈΡΠΏΠΎΠ»ΡŒΠ·ΡƒΡ ΠΏΡ€Π΅Π΄ΠΎΡΡ‚Π°Π²Π»Π΅Π½Π½ΡƒΡŽ ΠΈΠ½Ρ„ΠΎΡ€ΠΌΠ°Ρ†ΠΈΡŽ

        ΠŸΡ€ΠΎΡΠΌΠΎΡ‚Ρ€Π΅Ρ‚ΡŒ Ρ€Π΅ΡˆΠ΅Π½ΠΈΠ΅

        Π—Π°Π΄Π°Ρ‡Π° 5 : Найти V x

        ΠŸΡ€ΠΎΡΠΌΠΎΡ‚Ρ€Π΅Ρ‚ΡŒ Ρ€Π΅ΡˆΠ΅Π½ΠΈΠ΅

        ΠžΡΡƒΡ‰Π΅ΡΡ‚Π²Π»Π΅Π½ΠΈΡ:
          Π‘Π΅Ρ€Ρ‚ΠΈΡ„ΠΈΠΊΠ°Ρ‚ для сСрии

          Β»ΠŸΡ€ΠΈΠΌΠ΅Ρ‡Π°Π½ΠΈΡ ΠΏΠΎ элСктроникС

          ΠŸΠΎΡΠ»Π΅Π΄ΠΎΠ²Π°Ρ‚Π΅Π»ΡŒΠ½Ρ‹ΠΉ рСгулятор ΠΈΠ»ΠΈ рСгулятор ΠΏΠΎΡΠ»Π΅Π΄ΠΎΠ²Π°Ρ‚Π΅Π»ΡŒΠ½ΠΎΠ³ΠΎ ΠΏΡ€ΠΎΡ…ΠΎΠ΄Π° — это Π½Π°ΠΈΠ±ΠΎΠ»Π΅Π΅ ΡˆΠΈΡ€ΠΎΠΊΠΎ распространСнный Π²ΠΈΠ΄ рСгуляторов напряТСния, ΠΈΡΠΏΠΎΠ»ΡŒΠ·ΡƒΠ΅ΠΌΡ‹Ρ… Π² Π»ΠΈΠ½Π΅ΠΉΠ½Ρ‹Ρ… источниках питания.


          ПособиС ΠΏΠΎ цСпям Π»ΠΈΠ½Π΅ΠΉΠ½ΠΎΠ³ΠΎ источника питания ΠΈ руководство Π’ΠΊΠ»ΡŽΡ‡Π°Π΅Ρ‚:
          Π›ΠΈΠ½Π΅ΠΉΠ½Ρ‹ΠΉ источник питания Π¨ΡƒΠ½Ρ‚ΠΈΡ€ΡƒΡŽΡ‰ΠΈΠΉ рСгулятор РСгулятор сСрии ΠžΠ³Ρ€Π°Π½ΠΈΡ‡ΠΈΡ‚Π΅Π»ΡŒ Ρ‚ΠΎΠΊΠ° РСгуляторы сСрий 7805, 7812 ΠΈ 78 **

          Π‘ΠΌ. Π’Π°ΠΊΠΆΠ΅: ΠžΠ±Π·ΠΎΡ€ элСктроники Π±Π»ΠΎΠΊΠ° питания Π˜ΠΌΠΏΡƒΠ»ΡŒΡΠ½Ρ‹ΠΉ источник питания Π—Π°Ρ‰ΠΈΡ‚Π° ΠΎΡ‚ пСрСнапряТСния Π₯арактСристики Π±Π»ΠΎΠΊΠ° питания Цифровая ΠΌΠΎΡ‰Π½ΠΎΡΡ‚ΡŒ Π¨ΠΈΠ½Π° управлСния ΠΏΠΈΡ‚Π°Π½ΠΈΠ΅ΠΌ: PMbus БСспСрСбойный источник питания


          ΠŸΠΎΡΠ»Π΅Π΄ΠΎΠ²Π°Ρ‚Π΅Π»ΡŒΠ½Ρ‹ΠΉ стабилизатор напряТСния ΠΈΠ»ΠΈ, ΠΊΠ°ΠΊ Π΅Π³ΠΎ ΠΈΠ½ΠΎΠ³Π΄Π° Π½Π°Π·Ρ‹Π²Π°ΡŽΡ‚, ΠΏΠΎΡΠ»Π΅Π΄ΠΎΠ²Π°Ρ‚Π΅Π»ΡŒΠ½Ρ‹ΠΉ стабилизатор напряТСния — Π½Π°ΠΈΠ±ΠΎΠ»Π΅Π΅ часто ΠΈΡΠΏΠΎΠ»ΡŒΠ·ΡƒΠ΅ΠΌΡ‹ΠΉ ΠΏΠΎΠ΄Ρ…ΠΎΠ΄ для обСспСчСния ΠΎΠΊΠΎΠ½Ρ‡Π°Ρ‚Π΅Π»ΡŒΠ½ΠΎΠ³ΠΎ рСгулирования напряТСния Π² Π»ΠΈΠ½Π΅ΠΉΠ½ΠΎ Ρ€Π΅Π³ΡƒΠ»ΠΈΡ€ΡƒΠ΅ΠΌΠΎΠΌ источникС питания.

          Π›ΠΈΠ½Π΅ΠΉΠ½Ρ‹ΠΉ стабилизатор сСрии обСспСчиваСт высокий ΡƒΡ€ΠΎΠ²Π΅Π½ΡŒ ΠΏΡ€ΠΎΠΈΠ·Π²ΠΎΠ΄ΠΈΡ‚Π΅Π»ΡŒΠ½ΠΎΡΡ‚ΠΈ, особСнно ΠΊΠΎΠ³Π΄Π° трСбуСтся Π½ΠΈΠ·ΠΊΠΈΠΉ ΡƒΡ€ΠΎΠ²Π΅Π½ΡŒ ΡˆΡƒΠΌΠ°, ΠΏΡƒΠ»ΡŒΡΠ°Ρ†ΠΈΠΉ ΠΈ ΠΏΠ΅Ρ€Π΅Ρ…ΠΎΠ΄Π½Ρ‹Ρ… процСссов Π½Π° Ρ€Π΅Π³ΡƒΠ»ΠΈΡ€ΡƒΠ΅ΠΌΠΎΠΌ Π²Ρ‹Ρ…ΠΎΠ΄Π΅.

          БущСствуСт мноТСство схСм, ΠΈΡΠΏΠΎΠ»ΡŒΠ·ΡƒΡŽΡ‰ΠΈΡ… дискрСтныС элСктронныС ΠΊΠΎΠΌΠΏΠΎΠ½Π΅Π½Ρ‚Ρ‹, ΠΊΠΎΡ‚ΠΎΡ€Ρ‹Π΅ ΠΎΠ±Π΅ΡΠΏΠ΅Ρ‡ΠΈΠ²Π°ΡŽΡ‚ Π»ΠΈΠ½Π΅ΠΉΠ½ΠΎΠ΅ Ρ€Π΅Π³ΡƒΠ»ΠΈΡ€ΠΎΠ²Π°Π½ΠΈΠ΅ с ΠΏΠΎΠΌΠΎΡ‰ΡŒΡŽ ΠΏΠΎΡΠ»Π΅Π΄ΠΎΠ²Π°Ρ‚Π΅Π»ΡŒΠ½ΠΎΠ³ΠΎ элСмСнта, ΠΈ Π² Π΄ΠΎΠΏΠΎΠ»Π½Π΅Π½ΠΈΠ΅ ΠΊ этому практичСски всС ИБ Π»ΠΈΠ½Π΅ΠΉΠ½Ρ‹Ρ… рСгуляторов ΠΈΡΠΏΠΎΠ»ΡŒΠ·ΡƒΡŽΡ‚ этот ΠΏΠΎΠ΄Ρ…ΠΎΠ΄.

          Π­Ρ‚ΠΎ ΠΎΠ·Π½Π°Ρ‡Π°Π΅Ρ‚, Ρ‡Ρ‚ΠΎ сущСствуСт мноТСство Π²Π°Ρ€ΠΈΠ°Π½Ρ‚ΠΎΠ² ΠΏΠΎΡΠ»Π΅Π΄ΠΎΠ²Π°Ρ‚Π΅Π»ΡŒΠ½Ρ‹Ρ… рСгуляторов напряТСния, ΠΊΠΎΡ‚ΠΎΡ€Ρ‹Π΅ ΠΎΡ‚ΠΊΡ€Ρ‹Π²Π°ΡŽΡ‚ΡΡ ΠΏΡ€ΠΈ ΠΏΡ€ΠΎΠ΅ΠΊΡ‚ΠΈΡ€ΠΎΠ²Π°Π½ΠΈΠΈ элСктронной схСмы источника питания.

          ΠžΡΠ½ΠΎΠ²Ρ‹ рСгуляторов напряТСния сСрии

          Π’ ΠΏΠΎΡΠ»Π΅Π΄ΠΎΠ²Π°Ρ‚Π΅Π»ΡŒΠ½ΠΎΠΌ рСгуляторС напряТСния ΠΈΠ»ΠΈ ΠΏΠΎΡΠ»Π΅Π΄ΠΎΠ²Π°Ρ‚Π΅Π»ΡŒΠ½ΠΎΠΌ рСгуляторС напряТСния ΠΈΡΠΏΠΎΠ»ΡŒΠ·ΡƒΠ΅Ρ‚ΡΡ Ρ€Π΅Π³ΡƒΠ»ΠΈΡ€ΡƒΠ΅ΠΌΡ‹ΠΉ элСмСнт, Π²ΠΊΠ»ΡŽΡ‡Π΅Π½Π½Ρ‹ΠΉ ΠΏΠΎΡΠ»Π΅Π΄ΠΎΠ²Π°Ρ‚Π΅Π»ΡŒΠ½ΠΎ с Π½Π°Π³Ρ€ΡƒΠ·ΠΊΠΎΠΉ. ИзмСняя сопротивлСниС ΠΏΠΎΡΠ»Π΅Π΄ΠΎΠ²Π°Ρ‚Π΅Π»ΡŒΠ½ΠΎΠ³ΠΎ элСмСнта, ΠΌΠΎΠΆΠ½ΠΎ ΠΈΠ·ΠΌΠ΅Π½ΡΡ‚ΡŒ ΠΏΠ°Π΄Π΅Π½ΠΈΠ΅ напряТСния Π½Π° Π½Π΅ΠΌ, Ρ‡Ρ‚ΠΎΠ±Ρ‹ напряТСниС Π½Π° Π½Π°Π³Ρ€ΡƒΠ·ΠΊΠ΅ ΠΎΡΡ‚Π°Π²Π°Π»ΠΎΡΡŒ постоянным.

          Π‘Π»ΠΎΠΊ-схСма ΠΏΠΎΡΠ»Π΅Π΄ΠΎΠ²Π°Ρ‚Π΅Π»ΡŒΠ½ΠΎΠ³ΠΎ рСгулятора напряТСния

          ΠŸΡ€Π΅ΠΈΠΌΡƒΡ‰Π΅ΡΡ‚Π²ΠΎ ΠΏΠΎΡΠ»Π΅Π΄ΠΎΠ²Π°Ρ‚Π΅Π»ΡŒΠ½ΠΎΠ³ΠΎ рСгулятора напряТСния состоит Π² Ρ‚ΠΎΠΌ, Ρ‡Ρ‚ΠΎ Π²Π΅Π»ΠΈΡ‡ΠΈΠ½Π° потрСбляСмого Ρ‚ΠΎΠΊΠ° фактичСски Ρ€Π°Π²Π½Π° Π²Π΅Π»ΠΈΡ‡ΠΈΠ½Π΅ потрСбляСмого Π½Π°Π³Ρ€ΡƒΠ·ΠΊΠΎΠΉ, хотя нСкоторая Ρ‡Π°ΡΡ‚ΡŒ Π±ΡƒΠ΄Π΅Ρ‚ ΠΏΠΎΡ‚Ρ€Π΅Π±Π»ΡΡ‚ΡŒΡΡ любой схСмой, связанной с рСгулятором.Π’ ΠΎΡ‚Π»ΠΈΡ‡ΠΈΠ΅ ΠΎΡ‚ ΡˆΡƒΠ½Ρ‚ΠΈΡ€ΡƒΡŽΡ‰Π΅Π³ΠΎ рСгулятора напряТСния, ΠΏΠΎΡΠ»Π΅Π΄ΠΎΠ²Π°Ρ‚Π΅Π»ΡŒΠ½Ρ‹ΠΉ рСгулятор Π½Π΅ потрСбляСт ΠΏΠΎΠ»Π½Ρ‹ΠΉ Ρ‚ΠΎΠΊ, Π΄Π°ΠΆΠ΅ Ссли Π½Π°Π³Ρ€ΡƒΠ·ΠΊΠ° Π½Π΅ Ρ‚Ρ€Π΅Π±ΡƒΠ΅Ρ‚ Π½ΠΈΠΊΠ°ΠΊΠΎΠ³ΠΎ Ρ‚ΠΎΠΊΠ°. Π’ Ρ€Π΅Π·ΡƒΠ»ΡŒΡ‚Π°Ρ‚Π΅ ΠΏΠΎΡΠ»Π΅Π΄ΠΎΠ²Π°Ρ‚Π΅Π»ΡŒΠ½Ρ‹ΠΉ стабилизатор напряТСния Π·Π½Π°Ρ‡ΠΈΡ‚Π΅Π»ΡŒΠ½ΠΎ Π±ΠΎΠ»Π΅Π΅ эффСктивСн.

          ВмСсто Ρ‚ΠΎΠ³ΠΎ, Ρ‡Ρ‚ΠΎΠ±Ρ‹ ΠΏΠΎΡ‚Ρ€Π΅Π±Π»ΡΡ‚ΡŒ Ρ‚ΠΎΠΊ, ΠΊΠΎΡ‚ΠΎΡ€Ρ‹ΠΉ Π½Π΅ трСбуСтся Π½Π°Π³Ρ€ΡƒΠ·ΠΊΠ΅ для поддСрТания напряТСния, ΠΎΠ½ сниТаСт Ρ€Π°Π·Π½ΠΈΡ†Ρƒ напряТСний ΠΌΠ΅ΠΆΠ΄Ρƒ Π²Ρ…ΠΎΠ΄Π½Ρ‹ΠΌ напряТСниСм ΠΈ Ρ‚Ρ€Π΅Π±ΡƒΠ΅ΠΌΡ‹ΠΌ стабилизированным напряТСниСм.

          Для поддСрТания достаточного уровня рСгулирования ΠΈ подавлСния ΡˆΡƒΠΌΠΎΠ² ΠΈ ΠΏΠ΅Ρ€Π΅Ρ…ΠΎΠ΄Π½Ρ‹Ρ… процСссов, ΠΊΠΎΡ‚ΠΎΡ€Ρ‹Π΅ ΠΌΠΎΠ³ΡƒΡ‚ Π²ΠΎΠ·Π½ΠΈΠΊΠ°Ρ‚ΡŒ Π½Π° входящСм напряТСнии, ΠΏΠΎΡΠ»Π΅Π΄ΠΎΠ²Π°Ρ‚Π΅Π»ΡŒΠ½Ρ‹Π΅ Π»ΠΈΠ½Π΅ΠΉΠ½Ρ‹Π΅ рСгуляторы напряТСния Π΄ΠΎΠ»ΠΆΠ½Ρ‹ Π·Π½Π°Ρ‡ΠΈΡ‚Π΅Π»ΡŒΠ½ΠΎ ΡΠ½ΠΈΠΆΠ°Ρ‚ΡŒ напряТСниС.Многим высококачСствСнным стабилизаторам напряТСния с Π½ΠΈΠ·ΠΊΠΈΠΌ ΡƒΡ€ΠΎΠ²Π½Π΅ΠΌ ΡˆΡƒΠΌΠ° ΠΈ ΠΏΡƒΠ»ΡŒΡΠ°Ρ†ΠΈΠΈ трСбуСтся нСсколько Π²ΠΎΠ»ΡŒΡ‚ Π½Π° ΠΏΠΎΡΠ»Π΅Π΄ΠΎΠ²Π°Ρ‚Π΅Π»ΡŒΠ½ΠΎΠΌ Ρ€Π΅Π³ΡƒΠ»ΠΈΡ€ΡƒΡŽΡ‰Π΅ΠΌ элСмСнтС. Π­Ρ‚ΠΎ ΠΎΠ·Π½Π°Ρ‡Π°Π΅Ρ‚, Ρ‡Ρ‚ΠΎ Π² этом ΠΊΠΎΠΌΠΏΠΎΠ½Π΅Π½Ρ‚Π΅ рассСиваСтся Π·Π½Π°Ρ‡ΠΈΡ‚Π΅Π»ΡŒΠ½Π°Ρ ΠΌΠΎΡ‰Π½ΠΎΡΡ‚ΡŒ, ΠΈ для устройства ΠΏΠΎΡΠ»Π΅Π΄ΠΎΠ²Π°Ρ‚Π΅Π»ΡŒΠ½ΠΎΠ³ΠΎ рСгулятора, Π° Ρ‚Π°ΠΊΠΆΠ΅ для источника питания Π² Ρ†Π΅Π»ΠΎΠΌ трСбуСтся Ρ…ΠΎΡ€ΠΎΡˆΠΈΠΉ Ρ‚Π΅ΠΏΠ»ΠΎΠΎΡ‚Π²ΠΎΠ΄ ΠΈ ΠΎΡ‚Π²ΠΎΠ΄ Ρ‚Π΅ΠΏΠ»Π°.

          Π₯отя ΠΏΠΎΡΠ»Π΅Π΄ΠΎΠ²Π°Ρ‚Π΅Π»ΡŒΠ½Ρ‹ΠΉ рСгулятор Π·Π½Π°Ρ‡ΠΈΡ‚Π΅Π»ΡŒΠ½ΠΎ Π±ΠΎΠ»Π΅Π΅ эффСктивСн, Ρ‡Π΅ΠΌ ΡˆΡƒΠ½Ρ‚ΠΈΡ€ΡƒΡŽΡ‰ΠΈΠΉ рСгулятор, ΠΎΠ½ Π·Π½Π°Ρ‡ΠΈΡ‚Π΅Π»ΡŒΠ½ΠΎ ΠΌΠ΅Π½Π΅Π΅ эффСктивСн, Ρ‡Π΅ΠΌ ΠΈΠΌΠΏΡƒΠ»ΡŒΡΠ½Ρ‹ΠΉ источник питания. Π­Ρ„Ρ„Π΅ΠΊΡ‚ΠΈΠ²Π½ΠΎΡΡ‚ΡŒ ΠΏΠΎΡΠ»Π΅Π΄ΠΎΠ²Π°Ρ‚Π΅Π»ΡŒΠ½ΠΎΠ³ΠΎ рСгулятора напряТСния ΠΈ Π»ΡŽΠ±Ρ‹Ρ… Π»ΠΈΠ½Π΅ΠΉΠ½Ρ‹Ρ… источников питания, ΠΈΡΠΏΠΎΠ»ΡŒΠ·ΡƒΡŽΡ‰ΠΈΡ… ΠΈΡ…, Π±ΡƒΠ΄Π΅Ρ‚ Π·Π°Π²ΠΈΡΠ΅Ρ‚ΡŒ ΠΎΡ‚ Π½Π°Π³Ρ€ΡƒΠ·ΠΊΠΈ ΠΈ Ρ‚. Π”., Но часто Π΄ΠΎΡΡ‚ΠΈΠ³Π°ΡŽΡ‚ΡΡ ΡƒΡ€ΠΎΠ²Π½ΠΈ эффСктивности ΠΌΠ΅Π½Π΅Π΅ 50%, Ρ‚ΠΎΠ³Π΄Π° ΠΊΠ°ΠΊ источники питания с ΠΈΠΌΠΏΡƒΠ»ΡŒΡΠ½Ρ‹ΠΌ Ρ€Π΅ΠΆΠΈΠΌΠΎΠΌ ΠΌΠΎΠ³ΡƒΡ‚ Π΄ΠΎΡΡ‚ΠΈΠ³Π°Ρ‚ΡŒ ΡƒΡ€ΠΎΠ²Π½Π΅ΠΉ Π±ΠΎΠ»Π΅Π΅ 90%.

          Π‘Ρ‚Π°Π±ΠΈΠ»ΠΈΠ·Π°Ρ‚ΠΎΡ€Ρ‹ напряТСния сСрии

          ΠΈΠΌΠ΅ΡŽΡ‚ ΠΎΡ‚Π½ΠΎΡΠΈΡ‚Π΅Π»ΡŒΠ½ΠΎ Π½ΠΈΠ·ΠΊΠΈΠΉ ΡƒΡ€ΠΎΠ²Π΅Π½ΡŒ эффСктивности ΠΏΠΎ ΡΡ€Π°Π²Π½Π΅Π½ΠΈΡŽ с ΠΈΠΌΠΏΡƒΠ»ΡŒΡΠ½Ρ‹ΠΌ источником питания, Π½ΠΎ Ρƒ Π½ΠΈΡ… Π΅ΡΡ‚ΡŒ прСимущСства простоты, Π° Ρ‚Π°ΠΊΠΆΠ΅ Π½Π° ΠΈΡ… Π²Ρ‹Ρ…ΠΎΠ΄Π΅ ΠΎΡ‚ΡΡƒΡ‚ΡΡ‚Π²ΡƒΡŽΡ‚ всплСски ΠΏΠ΅Ρ€Π΅ΠΊΠ»ΡŽΡ‡Π΅Π½ΠΈΡ, Π½Π°Π±Π»ΡŽΠ΄Π°Π΅ΠΌΡ‹Π΅ Π½Π° Π½Π΅ΠΊΠΎΡ‚ΠΎΡ€Ρ‹Ρ… ΠΈΠΌΠΏΡƒΠ»ΡŒΡΠ½Ρ‹Ρ… источниках питания, хотя SMPS ΡƒΠ»ΡƒΡ‡ΡˆΠ°ΡŽΡ‚ΡΡ, Π° ΠΏΡ€ΠΎΠΈΠ·Π²ΠΎΠ΄ΠΈΡ‚Π΅Π»ΡŒΠ½ΠΎΡΡ‚ΡŒ ΠΌΠ½ΠΎΠ³ΠΈΡ… сСйчас ΠΈΡΠΊΠ»ΡŽΡ‡ΠΈΡ‚Π΅Π»ΡŒΠ½ΠΎ Ρ…ΠΎΡ€ΠΎΡˆΠΎ.

          РСгулятор напряТСния простой эмиттСрный ΠΏΠΎΠ²Ρ‚ΠΎΡ€ΠΈΡ‚Π΅Π»ΡŒ

          ΠšΠΎΠ½ΡΡ‚Ρ€ΡƒΠΊΡ†ΠΈΡ элСктронной схСмы простого транзисторного рСгулятора напряТСния с эмиттСрным ΠΏΠΎΠ²Ρ‚ΠΎΡ€ΠΈΡ‚Π΅Π»Π΅ΠΌ ΠΎΡ‡Π΅Π½ΡŒ проста.Π‘Π°ΠΌΠ° ΠΏΠΎ сСбС эта схСма Π½Π΅ ΠΈΡΠΏΠΎΠ»ΡŒΠ·ΡƒΠ΅Ρ‚ΡΡ ΡˆΠΈΡ€ΠΎΠΊΠΎ Π² Π»ΠΈΠ½Π΅ΠΉΠ½ΠΎΠΌ источникС питания, Π½ΠΎ ΠΌΠΎΠΆΠ΅Ρ‚ ΠΈΡΠΏΠΎΠ»ΡŒΠ·ΠΎΠ²Π°Ρ‚ΡŒΡΡ Π² Π΄Ρ€ΡƒΠ³ΠΎΠΌ ΠΎΠ±ΠΎΡ€ΡƒΠ΄ΠΎΠ²Π°Π½ΠΈΠΈ для обСспСчСния ΠΏΠΎΠ½ΠΈΠΆΠ°ΡŽΡ‰Π΅Π³ΠΎ напряТСния ΠΈ Ρ‚. Π”. ΠžΡ‚ ΡˆΠΈΠ½Ρ‹ с Π±ΠΎΠ»Π΅Π΅ высоким напряТСниСм.

          Π‘Π°Π·ΠΎΠ²Ρ‹ΠΉ ΠΏΠΎΡΠ»Π΅Π΄ΠΎΠ²Π°Ρ‚Π΅Π»ΡŒΠ½Ρ‹ΠΉ стабилизатор с использованиСм стабилитрона ΠΈ эмиттСрного повторитСля

          Π’ схСмС ΠΈΡΠΏΠΎΠ»ΡŒΠ·ΡƒΠ΅Ρ‚ΡΡ ΠΎΠ΄Π½ΠΎΠΏΡ€ΠΎΡ…ΠΎΠ΄Π½Ρ‹ΠΉ транзистор Π² Π²ΠΈΠ΄Π΅ ΠΊΠΎΠ½Ρ„ΠΈΠ³ΡƒΡ€Π°Ρ†ΠΈΠΈ эмиттСрного повторитСля ΠΈ ΠΎΠ΄ΠΈΠ½ΠΎΡ‡Π½Ρ‹ΠΉ стабилитрон ΠΈΠ»ΠΈ Π΄Ρ€ΡƒΠ³ΠΎΠΉ Π΄ΠΈΠΎΠ΄ рСгулятора напряТСния, управляСмый рСзистором ΠΎΡ‚ Π½Π΅Ρ€Π΅Π³ΡƒΠ»ΠΈΡ€ΡƒΠ΅ΠΌΠΎΠ³ΠΎ источника питания.

          Π­Ρ‚ΠΎ обСспСчиваСт ΠΏΡ€ΠΎΡΡ‚ΡƒΡŽ Ρ„ΠΎΡ€ΠΌΡƒ систСмы ΠΎΠ±Ρ€Π°Ρ‚Π½ΠΎΠΉ связи, ΠΎΠ±Π΅ΡΠΏΠ΅Ρ‡ΠΈΠ²Π°ΡŽΡ‰ΡƒΡŽ ΠΏΠΎΠ΄Π΄Π΅Ρ€ΠΆΠ°Π½ΠΈΠ΅ напряТСния стабилитрона Π½Π° Π²Ρ‹Ρ…ΠΎΠ΄Π΅, хотя ΠΈ со сниТСниСм напряТСния, Ρ€Π°Π²Π½Ρ‹ΠΌ Π½Π°ΠΏΡ€ΡΠΆΠ΅Π½ΠΈΡŽ ΠΏΠ΅Ρ€Π΅Ρ…ΠΎΠ΄Π° Π±Π°Π·Π°-эмиттСр — 0.6 Π²ΠΎΠ»ΡŒΡ‚ для ΠΊΡ€Π΅ΠΌΠ½ΠΈΠ΅Π²ΠΎΠ³ΠΎ транзистора.

          Π‘ΠΏΡ€ΠΎΠ΅ΠΊΡ‚ΠΈΡ€ΠΎΠ²Π°Ρ‚ΡŒ Ρ‚Π°ΠΊΡƒΡŽ ​​схСму ΠΏΠΎΡΠ»Π΅Π΄ΠΎΠ²Π°Ρ‚Π΅Π»ΡŒΠ½ΠΎΠ³ΠΎ рСгулятора напряТСния нСслоТно. Зная ΠΌΠ°ΠΊΡΠΈΠΌΠ°Π»ΡŒΠ½Ρ‹ΠΉ Ρ‚ΠΎΠΊ, Ρ‚Ρ€Π΅Π±ΡƒΠ΅ΠΌΡ‹ΠΉ Π½Π°Π³Ρ€ΡƒΠ·ΠΊΠΎΠΉ, ΠΌΠΎΠΆΠ½ΠΎ Ρ€Π°ΡΡΡ‡ΠΈΡ‚Π°Ρ‚ΡŒ ΠΌΠ°ΠΊΡΠΈΠΌΠ°Π»ΡŒΠ½Ρ‹ΠΉ Ρ‚ΠΎΠΊ эмиттСра. Π­Ρ‚ΠΎ достигаСтся Π΄Π΅Π»Π΅Π½ΠΈΠ΅ΠΌ Ρ‚ΠΎΠΊΠ° Π½Π°Π³Ρ€ΡƒΠ·ΠΊΠΈ, Ρ‚ΠΎ Π΅ΡΡ‚ΡŒ Ρ‚ΠΎΠΊΠ° эмиттСра транзистора, Π½Π° Ξ’ ΠΈΠ»ΠΈ hfe транзистора.

          Π‘Ρ‚Π°Π±ΠΈΠ»ΠΈΡ‚Ρ€ΠΎΠ½Ρƒ ΠΎΠ±Ρ‹Ρ‡Π½ΠΎ трСбуСтся ΠΌΠΈΠ½ΠΈΠΌΡƒΠΌ ΠΎΠΊΠΎΠ»ΠΎ 10 мА, Ρ‡Ρ‚ΠΎΠ±Ρ‹ малСнький стабилитрон ΠΌΠΎΠ³ ΠΏΠΎΠ΄Π΄Π΅Ρ€ΠΆΠΈΠ²Π°Ρ‚ΡŒ своС Ρ€Π΅Π³ΡƒΠ»ΠΈΡ€ΡƒΠ΅ΠΌΠΎΠ΅ напряТСниС.Π—Π°Ρ‚Π΅ΠΌ слСдуСт Ρ€Π°ΡΡΡ‡ΠΈΡ‚Π°Ρ‚ΡŒ рСзистор, Ρ‡Ρ‚ΠΎΠ±Ρ‹ ΠΎΠ±Π΅ΡΠΏΠ΅Ρ‡ΠΈΡ‚ΡŒ Π±Π°Π·ΠΎΠ²Ρ‹ΠΉ Ρ‚ΠΎΠΊ возбуТдСния ΠΈ ΠΌΠΈΠ½ΠΈΠΌΠ°Π»ΡŒΠ½Ρ‹ΠΉ Ρ‚ΠΎΠΊ Π—Π΅Π½Π΅Ρ€Π°, исходя ΠΈΠ· Π΄Π°Π½Π½Ρ‹Ρ… ΠΎ Π½Π΅Ρ€Π΅Π³ΡƒΠ»ΠΈΡ€ΡƒΠ΅ΠΌΠΎΠΌ напряТСнии, напряТСнии Π—Π΅Π½Π΅Ρ€Π° ΠΈ Ρ‚Ρ€Π΅Π±ΡƒΠ΅ΠΌΠΎΠΌ Ρ‚ΠΎΠΊΠ΅. [(НСрСгулируСмоС напряТСниС — напряТСниС стабилитрона) / Ρ‚ΠΎΠΊ]. К Ρ‚ΠΎΠΊΡƒ слСдуСт Π΄ΠΎΠ±Π°Π²ΠΈΡ‚ΡŒ нСбольшой запас, Ρ‡Ρ‚ΠΎΠ±Ρ‹ ΠΎΠ±Π΅ΡΠΏΠ΅Ρ‡ΠΈΡ‚ΡŒ достаточно мСста для запаса ΠΏΡ€ΠΈ Π½Π°Π³Ρ€ΡƒΠ·ΠΊΠ΅, ΠΈ, ΡΠ»Π΅Π΄ΠΎΠ²Π°Ρ‚Π΅Π»ΡŒΠ½ΠΎ, Π±Π°Π·Π° транзистора ΠΏΡ€ΠΈΠ½ΠΈΠΌΠ°Π΅Ρ‚ ΠΏΠΎΠ»Π½Ρ‹ΠΉ Ρ‚ΠΎΠΊ.

          РассСиваСмая ΠΌΠΎΡ‰Π½ΠΎΡΡ‚ΡŒ стабилитрона Π΄ΠΎΠ»ΠΆΠ½Π° Π±Ρ‹Ρ‚ΡŒ рассчитана для случая, ΠΊΠΎΠ³Π΄Π° Ρ‚ΠΎΠΊ Π½Π°Π³Ρ€ΡƒΠ·ΠΊΠΈ ΠΈ, ΡΠ»Π΅Π΄ΠΎΠ²Π°Ρ‚Π΅Π»ΡŒΠ½ΠΎ, Ρ‚ΠΎΠΊ Π±Π°Π·Ρ‹ Ρ€Π°Π²Π΅Π½ Π½ΡƒΠ»ΡŽ.Π’ этом случаС стабилитрон Π΄ΠΎΠ»ΠΆΠ΅Π½ Π±ΡƒΠ΄Π΅Ρ‚ ΠΏΡ€ΠΈΠ½ΠΈΠΌΠ°Ρ‚ΡŒ ΠΏΠΎΠ»Π½Ρ‹ΠΉ Ρ‚ΠΎΠΊ, проходящий Ρ‡Π΅Ρ€Π΅Π· ΠΏΠΎΡΠ»Π΅Π΄ΠΎΠ²Π°Ρ‚Π΅Π»ΡŒΠ½Ρ‹ΠΉ рСзистор.

          Иногда Ρ‡Π΅Ρ€Π΅Π· стабилитрон ΠΈΠ»ΠΈ ΠΎΠΏΠΎΡ€Π½Ρ‹ΠΉ Π΄ΠΈΠΎΠ΄ напряТСния ΠΌΠΎΠΆΠ΅Ρ‚ Π±Ρ‹Ρ‚ΡŒ ΠΏΠΎΠΌΠ΅Ρ‰Π΅Π½ кондСнсатор, Ρ‡Ρ‚ΠΎΠ±Ρ‹ ΠΏΠΎΠΌΠΎΡ‡ΡŒ ΡƒΡΡ‚Ρ€Π°Π½ΠΈΡ‚ΡŒ ΡˆΡƒΠΌ ΠΈ Π»ΡŽΠ±Ρ‹Π΅ ΠΏΠ΅Ρ€Π΅Ρ…ΠΎΠ΄Π½Ρ‹Π΅ процСссы напряТСния, ΠΊΠΎΡ‚ΠΎΡ€Ρ‹Π΅ ΠΌΠΎΠ³ΡƒΡ‚ Π²ΠΎΠ·Π½ΠΈΠΊΠ½ΡƒΡ‚ΡŒ.

          Π’Ρ‹Π±ΠΎΡ€ΠΊΠ° Π²Ρ‹Ρ…ΠΎΠ΄Π½ΠΎΠ³ΠΎ сигнала

          ΠŸΡ€ΠΎΡΡ‚Π°Ρ схСма ΠΏΠΎΡΠ»Π΅Π΄ΠΎΠ²Π°Ρ‚Π΅Π»ΡŒΠ½ΠΎΠ³ΠΎ рСгулятора напряТСния с эмиттСрным ΠΏΠΎΠ²Ρ‚ΠΎΡ€ΠΈΡ‚Π΅Π»Π΅ΠΌ Π½Π°ΠΏΡ€ΡΠΌΡƒΡŽ сравниваСт Π²Ρ‹Ρ…ΠΎΠ΄Π½ΠΎΠΉ сигнал с ΠΎΠΏΠΎΡ€Π½Ρ‹ΠΌ напряТСниСм. Π’Π°ΠΊΠΈΠΌ ΠΎΠ±Ρ€Π°Π·ΠΎΠΌ, Π²Ρ‹Ρ…ΠΎΠ΄Π½ΠΎΠ΅ напряТСниС Π±Ρ‹Π»ΠΎ Ρ€Π°Π²Π½ΠΎ ΠΎΠΏΠΎΡ€Π½ΠΎΠΌΡƒ, Π±Π΅Π· ΡƒΡ‡Π΅Ρ‚Π° падСния напряТСния Π½Π° Π±Π°Π·Π΅ эмиттСра.

          Однако ΠΌΠΎΠΆΠ½ΠΎ ΡƒΠ»ΡƒΡ‡ΡˆΠΈΡ‚ΡŒ характСристики рСгулятора напряТСния, Π²Ρ‹Π±Ρ€Π°Π² Ρ‡Π°ΡΡ‚ΡŒ Π²Ρ‹Ρ…ΠΎΠ΄Π½ΠΎΠ³ΠΎ напряТСния ΠΈ сравнив Π΅Π΅ с эталонным. Для этой Ρ„ΡƒΠ½ΠΊΡ†ΠΈΠΈ ΠΌΠΎΠΆΠ½ΠΎ ΠΈΡΠΏΠΎΠ»ΡŒΠ·ΠΎΠ²Π°Ρ‚ΡŒ Π΄ΠΈΡ„Ρ„Π΅Ρ€Π΅Π½Ρ†ΠΈΠ°Π»ΡŒΠ½Ρ‹ΠΉ ΡƒΡΠΈΠ»ΠΈΡ‚Π΅Π»ΡŒ, Π½Π°ΠΏΡ€ΠΈΠΌΠ΅Ρ€ ΠΎΠΏΠ΅Ρ€Π°Ρ†ΠΈΠΎΠ½Π½Ρ‹ΠΉ ΡƒΡΠΈΠ»ΠΈΡ‚Π΅Π»ΡŒ. Если это Π±ΡƒΠ΄Π΅Ρ‚ сдСлано, Ρ‚ΠΎ Π²Ρ‹Ρ…ΠΎΠ΄Π½ΠΎΠ΅ напряТСниС станСт большС, Ρ‡Π΅ΠΌ ΠΎΠΏΠΎΡ€Π½ΠΎΠ΅ напряТСниС, ΠΏΠΎΡΠΊΠΎΠ»ΡŒΠΊΡƒ ΠΎΡ‚Ρ€ΠΈΡ†Π°Ρ‚Π΅Π»ΡŒΠ½Π°Ρ обратная связь Π² Ρ†Π΅ΠΏΠΈ борСтся Π·Π° сохранСниС Π΄Π²ΡƒΡ… сравниваСмых напряТСний ΠΎΠ΄ΠΈΠ½Π°ΠΊΠΎΠ²Ρ‹ΠΌΠΈ.

          Если, Π½Π°ΠΏΡ€ΠΈΠΌΠ΅Ρ€, ΠΎΠΏΠΎΡ€Π½ΠΎΠ΅ напряТСниС составляСт 5 Π²ΠΎΠ»ΡŒΡ‚, Π° дискрСтизатор ΠΈΠ»ΠΈ Π΄Π΅Π»ΠΈΡ‚Π΅Π»ΡŒ ΠΏΠΎΡ‚Π΅Π½Ρ†ΠΈΠ°Π»Π° обСспСчиваСт 50% Π²Ρ‹Ρ…ΠΎΠ΄Π½ΠΎΠ³ΠΎ напряТСния, Ρ‚ΠΎ Π²Ρ‹Ρ…ΠΎΠ΄Π½ΠΎΠ΅ напряТСниС Π±ΡƒΠ΄Π΅Ρ‚ ΠΏΠΎΠ΄Π΄Π΅Ρ€ΠΆΠΈΠ²Π°Ρ‚ΡŒΡΡ Π½Π° ΡƒΡ€ΠΎΠ²Π½Π΅ 10 Π²ΠΎΠ»ΡŒΡ‚.

          ΠŸΠΎΡΠ»Π΅Π΄ΠΎΠ²Π°Ρ‚Π΅Π»ΡŒΠ½Ρ‹ΠΉ рСгулятор напряТСния с дискрСтным Π²Ρ‹Ρ…ΠΎΠ΄ΠΎΠΌ / figcaption>

          Π”Π΅Π»Π΅Π½ΠΈΠ΅ ΠΏΠΎΡ‚Π΅Π½Ρ†ΠΈΠ°Π»Π° ΠΈΠ»ΠΈ Π²Ρ‹Π±ΠΎΡ€ΠΊΡƒ ΠΌΠΎΠΆΠ½ΠΎ ΡΠ΄Π΅Π»Π°Ρ‚ΡŒ ΠΏΠ΅Ρ€Π΅ΠΌΠ΅Π½Π½Ρ‹ΠΌΠΈ, ΠΈ, Ρ‚Π°ΠΊΠΈΠΌ ΠΎΠ±Ρ€Π°Π·ΠΎΠΌ, Π²Ρ‹Ρ…ΠΎΠ΄Π½ΠΎΠ΅ напряТСниС ΠΌΠΎΠΆΠ½ΠΎ ΠΎΡ‚Ρ€Π΅Π³ΡƒΠ»ΠΈΡ€ΠΎΠ²Π°Ρ‚ΡŒ Π΄ΠΎ Ρ‚Ρ€Π΅Π±ΡƒΠ΅ΠΌΠΎΠ³ΠΎ значСния. ΠžΠ±Ρ‹Ρ‡Π½ΠΎ этот ΠΌΠ΅Ρ‚ΠΎΠ΄ ΠΈΡΠΏΠΎΠ»ΡŒΠ·ΡƒΠ΅Ρ‚ΡΡ Ρ‚ΠΎΠ»ΡŒΠΊΠΎ для Π½Π΅Π±ΠΎΠ»ΡŒΡˆΠΈΡ… настроСк, ΠΏΠΎΡΠΊΠΎΠ»ΡŒΠΊΡƒ ΠΌΠΈΠ½ΠΈΠΌΠ°Π»ΡŒΠ½Ρ‹ΠΉ Π²Ρ‹Ρ…ΠΎΠ΄Π½ΠΎΠΉ ΡƒΡ€ΠΎΠ²Π΅Π½ΡŒ, ΠΏΠΎΠ»ΡƒΡ‡Π΅Π½Π½Ρ‹ΠΉ этим ΠΌΠ΅Ρ‚ΠΎΠ΄ΠΎΠΌ, Ρ€Π°Π²Π΅Π½ Π²Ρ‹Ρ…ΠΎΠ΄Π½ΠΎΠΌΡƒ сигналу, Ρ€Π°Π²Π½ΠΎΠΌΡƒ ΠΎΠΏΠΎΡ€Π½ΠΎΠΌΡƒ Π½Π°ΠΏΡ€ΡΠΆΠ΅Π½ΠΈΡŽ.

          Π‘Π»Π΅Π΄ΡƒΠ΅Ρ‚ ΠΏΠΎΠΌΠ½ΠΈΡ‚ΡŒ, Ρ‡Ρ‚ΠΎ использованиС дСлитСля ΠΏΠΎΡ‚Π΅Π½Ρ†ΠΈΠ°Π»Π° сниТаСт коэффициСнт усилСния ΠΊΠΎΠ½Ρ‚ΡƒΡ€Π° ΠΎΠ±Ρ€Π°Ρ‚Π½ΠΎΠΉ связи. Π­Ρ‚ΠΎ сниТаСт коэффициСнт усилСния ΠΊΠΎΠ½Ρ‚ΡƒΡ€Π° ΠΈ Ρ‚Π΅ΠΌ самым сниТаСт характСристики рСгулирования.ΠžΠ±Ρ‹Ρ‡Π½ΠΎ сущСствуСт достаточноС усилСниС ΠΊΠΎΠ½Ρ‚ΡƒΡ€Π°, Ρ‡Ρ‚ΠΎΠ±Ρ‹ это Π½Π΅ Π±Ρ‹Π»ΠΎ большой ΠΏΡ€ΠΎΠ±Π»Π΅ΠΌΠΎΠΉ, Π·Π° ΠΈΡΠΊΠ»ΡŽΡ‡Π΅Π½ΠΈΠ΅ΠΌ случаСв, ΠΊΠΎΠ³Π΄Π° дискрСтизируСтся Ρ‚ΠΎΠ»ΡŒΠΊΠΎ ΠΎΡ‡Π΅Π½ΡŒ нСбольшая Ρ‡Π°ΡΡ‚ΡŒ Π²Ρ‹Ρ…ΠΎΠ΄Π½ΠΎΠ³ΠΎ сигнала.

          Π’Π°ΠΊΠΆΠ΅ слСдуСт ΠΏΡ€ΠΎΡΠ²Π»ΡΡ‚ΡŒ ΠΎΡΡ‚ΠΎΡ€ΠΎΠΆΠ½ΠΎΡΡ‚ΡŒ, Ρ‡Ρ‚ΠΎΠ±Ρ‹ Π½Π΅ ΡƒΠ²Π΅Π»ΠΈΡ‡ΠΈΠ²Π°Ρ‚ΡŒ Π²Ρ‹Ρ…ΠΎΠ΄Π½ΠΎΠ΅ напряТСниС Π΄ΠΎ Ρ‚ΠΎΡ‡ΠΊΠΈ, ΠΏΡ€ΠΈ ΠΊΠΎΡ‚ΠΎΡ€ΠΎΠΉ Π½Π° рСгуляторС Π½Π΅ Π±ΡƒΠ΄Π΅Ρ‚ достаточного падСния напряТСния для достаточного рСгулирования Π²Ρ‹Ρ…ΠΎΠ΄Π½ΠΎΠ³ΠΎ напряТСния.

          РСгулятор ΠΏΡ€ΠΎΡ…ΠΎΠ΄Π° сСрии

          с ΠΎΠ±Ρ€Π°Ρ‚Π½ΠΎΠΉ связью

          Для обСспСчСния ΡƒΠ»ΡƒΡ‡ΡˆΠ΅Π½Π½Ρ‹Ρ… ΡƒΡ€ΠΎΠ²Π½Π΅ΠΉ ΠΏΡ€ΠΎΠΈΠ·Π²ΠΎΠ΄ΠΈΡ‚Π΅Π»ΡŒΠ½ΠΎΡΡ‚ΠΈ ΠΏΠΎ ΡΡ€Π°Π²Π½Π΅Π½ΠΈΡŽ с Ρ‚Π΅ΠΌ, Ρ‡Ρ‚ΠΎ обСспСчиваСтся простым эмиттСрным ΠΏΠΎΠ²Ρ‚ΠΎΡ€ΠΈΡ‚Π΅Π»Π΅ΠΌ, ΠΌΠΎΠΆΠ½ΠΎ Π΄ΠΎΠ±Π°Π²ΠΈΡ‚ΡŒ Π±ΠΎΠ»Π΅Π΅ ΡΠ»ΠΎΠΆΠ½ΡƒΡŽ ΡΠ΅Ρ‚ΡŒ ΠΎΠ±Ρ€Π°Ρ‚Π½ΠΎΠΉ связи Π² схСму рСгулятора напряТСния.Π­Ρ‚ΠΎ достигаСтся ΠΏΡƒΡ‚Π΅ΠΌ дискрСтизации Π²Ρ‹Ρ…ΠΎΠ΄Π½ΠΎΠ³ΠΎ сигнала, сравнСния Π΅Π³ΠΎ с эталоном, Π° Π·Π°Ρ‚Π΅ΠΌ использования Π½Π΅ΠΊΠΎΡ‚ΠΎΡ€ΠΎΠ³ΠΎ Π²ΠΈΠ΄Π° Π΄ΠΈΡ„Ρ„Π΅Ρ€Π΅Π½Ρ†ΠΈΠ°Π»ΡŒΠ½ΠΎΠ³ΠΎ усилитСля для ΠΎΠ±Ρ€Π°Ρ‚Π½ΠΎΠΉ связи ΠΏΠΎ Ρ€Π°Π·Π½ΠΈΡ†Π΅ с Ρ†Π΅Π»ΡŒΡŽ исправлСния ошибок.

          МоТно ΠΈΡΠΏΠΎΠ»ΡŒΠ·ΠΎΠ²Π°Ρ‚ΡŒ ΠΏΡ€ΠΎΡΡ‚ΡƒΡŽ Π΄Π²ΡƒΡ…Ρ‚Ρ€Π°Π½Π·ΠΈΡΡ‚ΠΎΡ€Π½ΡƒΡŽ схСму для ΠΏΠΎΡΠ»Π΅Π΄ΠΎΠ²Π°Ρ‚Π΅Π»ΡŒΠ½ΠΎΠ³ΠΎ рСгулятора с ΠΈΠ·ΠΌΠ΅Ρ€Π΅Π½ΠΈΠ΅ΠΌ напряТСния ΠΈ ΠΎΠ±Ρ€Π°Ρ‚Π½ΠΎΠΉ связью. Π₯отя довольно просто ΠΈΡΠΏΠΎΠ»ΡŒΠ·ΠΎΠ²Π°Ρ‚ΡŒ ΠΎΠΏΠ΅Ρ€Π°Ρ†ΠΈΠΎΠ½Π½Ρ‹ΠΉ ΡƒΡΠΈΠ»ΠΈΡ‚Π΅Π»ΡŒ, ΠΊΠΎΡ‚ΠΎΡ€Ρ‹ΠΉ обСспСчит Π±ΠΎΠ»Π΅Π΅ высокий ΡƒΡ€ΠΎΠ²Π΅Π½ΡŒ ΠΎΠ±Ρ€Π°Ρ‚Π½ΠΎΠΉ связи ΠΈ, ΡΠ»Π΅Π΄ΠΎΠ²Π°Ρ‚Π΅Π»ΡŒΠ½ΠΎ, Π»ΡƒΡ‡ΡˆΠ΅Π΅ Ρ€Π΅Π³ΡƒΠ»ΠΈΡ€ΠΎΠ²Π°Π½ΠΈΠ΅, эта двухтранзисторная схСма Ρ…ΠΎΡ€ΠΎΡˆΠΎ ΠΈΠ»Π»ΡŽΡΡ‚Ρ€ΠΈΡ€ΡƒΠ΅Ρ‚ ΠΏΡ€ΠΈΠ½Ρ†ΠΈΠΏΡ‹.

          ΠŸΡ€ΠΎΡΡ‚Π°Ρ схСма ΠΏΠΎΡΠ»Π΅Π΄ΠΎΠ²Π°Ρ‚Π΅Π»ΡŒΠ½ΠΎΠ³ΠΎ рСгулятора с двумя транзисторами

          Π’ этой схСмС TR1 ΠΎΠ±Ρ€Π°Π·ΡƒΠ΅Ρ‚ ΠΏΠΎΡΠ»Π΅Π΄ΠΎΠ²Π°Ρ‚Π΅Π»ΡŒΠ½Ρ‹ΠΉ транзистор. Π’Ρ‚ΠΎΡ€ΠΎΠΉ транзистор, TR2, дСйствуСт ΠΊΠ°ΠΊ Π΄ΠΈΡ„Ρ„Π΅Ρ€Π΅Π½Ρ†ΠΈΠ°Π»ΡŒΠ½Ρ‹ΠΉ ΡƒΡΠΈΠ»ΠΈΡ‚Π΅Π»ΡŒ, подавая напряТСниС ошибки ΠΌΠ΅ΠΆΠ΄Ρƒ ΠΎΠΏΠΎΡ€Π½Ρ‹ΠΌ Π΄ΠΈΠΎΠ΄ΠΎΠΌ ΠΈ ΠΈΠ·ΠΌΠ΅Ρ€Π΅Π½Π½Ρ‹ΠΌ Π²Ρ‹Ρ…ΠΎΠ΄Π½Ρ‹ΠΌ напряТСниСм, ΠΊΠΎΡ‚ΠΎΡ€ΠΎΠ΅ являСтся ΠΏΡ€ΠΎΠΏΠΎΡ€Ρ†ΠΈΠ΅ΠΉ Π²Ρ‹Ρ…ΠΎΠ΄Π½ΠΎΠ³ΠΎ напряТСния, установлСнного ΠΏΠΎΡ‚Π΅Π½Ρ†ΠΈΠΎΠΌΠ΅Ρ‚Ρ€ΠΎΠΌ. РСзистор R1 обСспСчиваСт Ρ‚ΠΎΠΊ для ΠΊΠΎΠ»Π»Π΅ΠΊΡ‚ΠΎΡ€Π° TR2 ΠΈ Π΄ΠΈΠΎΠ΄Π° ΠΎΠΏΠΎΡ€Π½ΠΎΠ³ΠΎ напряТСния ZD1.

          ΠžΠΏΠΎΡ€Π½ΠΎΠ΅ напряТСниС

          Π›ΡŽΠ±ΠΎΠΉ Π»ΠΈΠ½Π΅ΠΉΠ½Ρ‹ΠΉ рСгулятор напряТСния ΠΌΠΎΠΆΠ΅Ρ‚ Π±Ρ‹Ρ‚ΡŒ Π½Π°ΡΡ‚ΠΎΠ»ΡŒΠΊΠΎ Ρ…ΠΎΡ€ΠΎΡˆ, насколько Ρ…ΠΎΡ€ΠΎΡˆ источник ΠΎΠΏΠΎΡ€Π½ΠΎΠ³ΠΎ напряТСния, ΠΊΠΎΡ‚ΠΎΡ€Ρ‹ΠΉ ΠΈΡΠΏΠΎΠ»ΡŒΠ·ΡƒΠ΅Ρ‚ΡΡ Π² качСствС основы для сравнСния Π² систСмС.Π₯отя тСорСтичСски ΠΌΠΎΠΆΠ½ΠΎ ΠΈΡΠΏΠΎΠ»ΡŒΠ·ΠΎΠ²Π°Ρ‚ΡŒ аккумулятор, для Π±ΠΎΠ»ΡŒΡˆΠΈΠ½ΡΡ‚Π²Π° ΠΏΡ€ΠΈΠ»ΠΎΠΆΠ΅Π½ΠΈΠΉ это Π½Π΅ ΠΏΠΎΠ΄Ρ…ΠΎΠ΄ΠΈΡ‚. ВмСсто этого ΠΏΠΎΡ‡Ρ‚ΠΈ повсСмСстно ΠΈΡΠΏΠΎΠ»ΡŒΠ·ΡƒΡŽΡ‚ΡΡ эталоны Π½Π° основС стабилитронов.

          Π˜Π½Ρ‚Π΅Π³Ρ€Π°Π»ΡŒΠ½Ρ‹Π΅ стабилизаторы ΠΈ эталоны

          ΠΈΡΠΏΠΎΠ»ΡŒΠ·ΡƒΡŽΡ‚ слоТныС ΠΊΠΎΠΌΠ±ΠΈΠ½Π°Ρ†ΠΈΠΈ транзисторов ΠΈ рСзисторов Π½Π° кристаллС для получСния Ρ‚ΠΎΡ‡Π½Ρ‹Ρ… источников ΠΎΠΏΠΎΡ€Π½ΠΎΠ³ΠΎ напряТСния с Ρ‚Π΅ΠΌΠΏΠ΅Ρ€Π°Ρ‚ΡƒΡ€Π½ΠΎΠΉ компСнсациСй.

          ΠžΠΏΠΎΡ€Π½ΠΎΠ΅ напряТСниС Π΄ΠΎΠ»ΠΆΠ½ΠΎ ΠΏΠΎΠ΄Π°Π²Π°Ρ‚ΡŒΡΡ ΠΎΡ‚ Π½Π΅Ρ€Π΅Π³ΡƒΠ»ΠΈΡ€ΡƒΠ΅ΠΌΠΎΠ³ΠΎ источника питания. Π•Π³ΠΎ нСльзя Π²Π·ΡΡ‚ΡŒ ΠΈΠ· Ρ€Π΅Π³ΡƒΠ»ΠΈΡ€ΡƒΠ΅ΠΌΠΎΠΉ мощности, Ρ‚Π°ΠΊ ΠΊΠ°ΠΊ Π΅ΡΡ‚ΡŒ ΠΏΡ€ΠΎΠ±Π»Π΅ΠΌΡ‹ с запуском.ΠŸΡ€ΠΈ запускС Π½Π΅Ρ‚ Π²Ρ‹Ρ…ΠΎΠ΄Π°, поэтому Π²Ρ‹Ρ…ΠΎΠ΄ задания Π±ΡƒΠ΄Π΅Ρ‚ Π½ΡƒΠ»Π΅Π²Ρ‹ΠΌ, ΠΈ ΠΎΠ½ Π±ΡƒΠ΄Π΅Ρ‚ ΠΏΠΎΠ΄Π΄Π΅Ρ€ΠΆΠΈΠ²Π°Ρ‚ΡŒΡΡ Π΄ΠΎ запуска задания.

          Π£ΠΏΡ€ΠΎΡ‰Π΅Π½Π½Ρ‹ΠΉ источник ΠΎΠΏΠΎΡ€Π½ΠΎΠ³ΠΎ напряТСния для ΠΏΠΎΡΠ»Π΅Π΄ΠΎΠ²Π°Ρ‚Π΅Π»ΡŒΠ½ΠΎΠ³ΠΎ рСгулятора напряТСния

          Часто Π²Ρ‹Ρ…ΠΎΠ΄ ΠΎΠΏΠΎΡ€Π½ΠΎΠ³ΠΎ источника подаСтся Ρ‡Π΅Ρ€Π΅Π· Π΄Π΅Π»ΠΈΡ‚Π΅Π»ΡŒ ΠΏΠΎΡ‚Π΅Π½Ρ†ΠΈΠ°Π»Π°. Π­Ρ‚ΠΎ Π½Π΅ Ρ‚ΠΎΠ»ΡŒΠΊΠΎ сниТаСт Π²Ρ‹Ρ…ΠΎΠ΄Π½ΠΎΠ΅ напряТСниС, ΠΊΠΎΡ‚ΠΎΡ€ΠΎΠ΅ ΠΎΠ±Ρ‹Ρ‡Π½ΠΎ ΠΎΡ‡Π΅Π½ΡŒ ΠΏΠΎΠ»Π΅Π·Π½ΠΎ, Π½ΠΎ Ρ‚Π°ΠΊΠΆΠ΅ позволяСт Π΄ΠΎΠ±Π°Π²ΠΈΡ‚ΡŒ ΠΊ Π²Ρ‹Ρ…ΠΎΠ΄Ρƒ кондСнсатор, Ρ‡Ρ‚ΠΎΠ±Ρ‹ ΠΏΠΎΠΌΠΎΡ‡ΡŒ ΡƒΡΡ‚Ρ€Π°Π½ΠΈΡ‚ΡŒ Π»ΡŽΠ±ΡƒΡŽ ΠΏΡƒΠ»ΡŒΡΠ°Ρ†ΠΈΡŽ ΠΈΠ»ΠΈ ΡˆΡƒΠΌ, ΠΊΠΎΡ‚ΠΎΡ€Ρ‹Π΅ ΠΌΠΎΠ³ΡƒΡ‚ ΠΏΡ€ΠΈΡΡƒΡ‚ΡΡ‚Π²ΠΎΠ²Π°Ρ‚ΡŒ. ПониТСнноС напряТСниС Ρ‚Π°ΠΊΠΆΠ΅ ΠΏΠΎΠ»Π΅Π·Π½ΠΎ, ΠΏΠΎΡ‚ΠΎΠΌΡƒ Ρ‡Ρ‚ΠΎ минимальноС Π²Ρ‹Ρ…ΠΎΠ΄Π½ΠΎΠ΅ напряТСниС опрСдСляСтся ΠΎΠΏΠΎΡ€Π½Ρ‹ΠΌ напряТСниСм.

          Π‘Ρ‚Π°Π±ΠΈΠ»ΠΈΠ·Π°Ρ‚ΠΎΡ€Ρ‹ напряТСния сСрии с ΠΌΠ°Π»Ρ‹ΠΌ ΠΏΠ°Π΄Π΅Π½ΠΈΠ΅ΠΌ напряТСния

          Одним ΠΈΠ· Ρ„Π°ΠΊΡ‚ΠΎΡ€ΠΎΠ², ΠΊΠΎΡ‚ΠΎΡ€Ρ‹Π΅ Π½Π΅ΠΎΠ±Ρ…ΠΎΠ΄ΠΈΠΌΠΎ ΡƒΡ‡ΠΈΡ‚Ρ‹Π²Π°Ρ‚ΡŒ ΠΏΡ€ΠΈ Π²Ρ‹Π±ΠΎΡ€Π΅ любого рСгулятора, являСтся напряТСниС, ΠΊΠΎΡ‚ΠΎΡ€ΠΎΠ΅ Π΄ΠΎΠ»ΠΆΠ½ΠΎ ΠΏΠΎΠ΄Π°Π²Π°Ρ‚ΡŒΡΡ Π½Π° элСмСнт ΠΏΠΎΡΠ»Π΅Π΄ΠΎΠ²Π°Ρ‚Π΅Π»ΡŒΠ½ΠΎΠ³ΠΎ ΠΏΡ€ΠΎΡ…ΠΎΠ΄Π°. Часто для Π»ΠΈΠ½Π΅ΠΉΠ½Ρ‹Ρ… рСгуляторов трСбуСтся Π·Π½Π°Ρ‡ΠΈΡ‚Π΅Π»ΡŒΠ½ΠΎΠ΅ ΠΏΠ°Π΄Π΅Π½ΠΈΠ΅ ΠΏΠΎΠΏΠ΅Ρ€Π΅Ρ‡Π½ΠΎΠ³ΠΎ сСчСния элСмСнта ΠΏΠΎΡΠ»Π΅Π΄ΠΎΠ²Π°Ρ‚Π΅Π»ΡŒΠ½ΠΎΠ³ΠΎ ΠΏΡ€ΠΎΡ…ΠΎΠ΄Π° для достиТСния Π½Π°ΠΈΠ»ΡƒΡ‡ΡˆΠ΅Π³ΠΎ рСгулирования ΠΈ подавлСния ΡˆΡƒΠΌΠ°. НапримСр, Π»ΠΈΠ½Π΅ΠΉΠ½Ρ‹ΠΉ рСгулятор с Π²Ρ‹Ρ…ΠΎΠ΄Π½Ρ‹ΠΌ напряТСниСм 12 Π²ΠΎΠ»ΡŒΡ‚ ΠΌΠΎΠΆΠ΅Ρ‚ Π±Ρ‹Ρ‚ΡŒ рассчитан Π½Π° Π²Ρ…ΠΎΠ΄Π½ΠΎΠ΅ напряТСниС 18 Π²ΠΎΠ»ΡŒΡ‚ ΠΈΠ»ΠΈ Π±ΠΎΠ»Π΅Π΅.

          Для любого Π»ΠΈΠ½Π΅ΠΉΠ½ΠΎΠ³ΠΎ рСгулятора сущСствуСт минимальноС напряТСниС, ΠΊΠΎΡ‚ΠΎΡ€ΠΎΠ΅ трСбуСтся Π½Π° ΠΏΠΎΡΠ»Π΅Π΄ΠΎΠ²Π°Ρ‚Π΅Π»ΡŒΠ½ΠΎΠΌ элСмСнтС, ΠΏΡ€Π΅ΠΆΠ΄Π΅ Ρ‡Π΅ΠΌ рСгулятор Β«ΠΎΡ‚ΠΊΠ»ΡŽΡ‡ΠΈΡ‚ΡΡΒ».Β«Π­Ρ‚ΠΎ ΠΏΠ°Π΄Π΅Π½ΠΈΠ΅ напряТСния ΠΌΠΎΠΆΠ½ΠΎ ΡƒΠ²ΠΈΠ΄Π΅Ρ‚ΡŒ Π²ΠΎ ΠΌΠ½ΠΎΠ³ΠΈΡ… ΠΈΠ½Ρ‚Π΅Π³Ρ€Π°Π»ΡŒΠ½Ρ‹Ρ… схСмах Π»ΠΈΠ½Π΅ΠΉΠ½Ρ‹Ρ… рСгуляторов.

          Π’ Π½Π΅ΠΊΠΎΡ‚ΠΎΡ€Ρ‹Ρ… схСмах Π²Π°ΠΆΠ½ΠΎ ΠΈΠΌΠ΅Ρ‚ΡŒ рСгулятор с Π½ΠΈΠ·ΠΊΠΈΠΌ ΠΏΠ°Π΄Π΅Π½ΠΈΠ΅ΠΌ напряТСния. Если доступноС Π²Ρ…ΠΎΠ΄Π½ΠΎΠ΅ напряТСниС Π½Π΅ ΠΎΡ‡Π΅Π½ΡŒ высокоС, Π²Π°ΠΆΠ½ΠΎ ΠΈΠΌΠ΅Ρ‚ΡŒ Π»ΠΈΠ½Π΅ΠΉΠ½Ρ‹ΠΉ стабилизатор с Π½ΠΈΠ·ΠΊΠΈΠΌ ΠΏΠ°Π΄Π΅Π½ΠΈΠ΅ΠΌ напряТСния. Он Π΄ΠΎΠ»ΠΆΠ΅Π½ Ρ…ΠΎΡ€ΠΎΡˆΠΎ Ρ€Π΅Π³ΡƒΠ»ΠΈΡ€ΠΎΠ²Π°Ρ‚ΡŒΡΡ, нСсмотря Π½Π° ΠΎΠ³Ρ€Π°Π½ΠΈΡ‡Π΅Π½Π½ΠΎΠ΅ напряТСниС Π½Π° Π½Π΅ΠΌ.

          Π₯отя схСмы, ΠΏΠΎΠΊΠ°Π·Π°Π½Π½Ρ‹Π΅ здСсь, ΠΏΡ€Π΅Π΄ΡΡ‚Π°Π²Π»ΡΡŽΡ‚ собой простыС транзисторныС схСмы, Ρ‚Π΅ ΠΆΠ΅ ΠΏΡ€ΠΈΠ½Ρ†ΠΈΠΏΡ‹ ΠΈΡΠΏΠΎΠ»ΡŒΠ·ΡƒΡŽΡ‚ΡΡ Π² Π±ΠΎΠ»Π΅Π΅ ΠΊΡ€ΡƒΠΏΠ½Ρ‹Ρ… схСмах, Π° Ρ‚Π°ΠΊΠΆΠ΅ Π² ΠΈΠ½Ρ‚Π΅Π³Ρ€Π°Π»ΡŒΠ½Ρ‹Ρ… схСмах.Π’ ΠΎΠ΄Π½ΠΈΡ… ΠΈ Ρ‚Π΅Ρ… ΠΆΠ΅ концСпциях рСгуляторов сСрии, Π° Ρ‚Π°ΠΊΠΆΠ΅ Π² схСмах ΠΎΠΏΠΎΡ€Π½Ρ‹Ρ… Π΄ΠΈΠΎΠ΄ΠΎΠ², Π²Ρ‹Π±ΠΎΡ€ΠΊΠΈ ΠΈ Π΄Ρ€ΡƒΠ³ΠΈΡ… областях ΠΈΡΠΏΠΎΠ»ΡŒΠ·ΡƒΡŽΡ‚ΡΡ ΠΎΠ΄Π½ΠΈ ΠΈ Ρ‚Π΅ ΠΆΠ΅ элСмСнты.

          Π˜ΡΠΏΠΎΠ»ΡŒΠ·ΡƒΠ΅ΠΌΡ‹Π΅ здСсь ΠΊΠΎΠ½Ρ†Π΅ΠΏΡ†ΠΈΠΈ ΠΈΡΠΏΠΎΠ»ΡŒΠ·ΡƒΡŽΡ‚ΡΡ практичСски Π² Π»ΠΈΠ½Π΅ΠΉΠ½Ρ‹Ρ… Ρ€Π΅Π³ΡƒΠ»ΠΈΡ€ΡƒΠ΅ΠΌΡ‹Ρ… источниках питания, ΠΊΠΎΡ‚ΠΎΡ€Ρ‹Π΅ ΠΌΠΎΠ³ΡƒΡ‚ ΠΏΡ€Π΅Π΄Π»ΠΎΠΆΠΈΡ‚ΡŒ ΠΎΡ‡Π΅Π½ΡŒ Ρ…ΠΎΡ€ΠΎΡˆΠΈΠ΅ ΡƒΡ€ΠΎΠ²Π½ΠΈ ΠΏΡ€ΠΎΠΈΠ·Π²ΠΎΠ΄ΠΈΡ‚Π΅Π»ΡŒΠ½ΠΎΡΡ‚ΠΈ. Π˜ΡΡ‚ΠΎΡ‡Π½ΠΈΠΊΠΈ питания с Π»ΠΈΠ½Π΅ΠΉΠ½Ρ‹ΠΌ Ρ€Π΅Π³ΡƒΠ»ΠΈΡ€ΠΎΠ²Π°Π½ΠΈΠ΅ΠΌ большС ΠΈ тяТСлСС, Ρ‡Π΅ΠΌ Π±Π»ΠΎΠΊΠΈ питания с ΠΈΠΌΠΏΡƒΠ»ΡŒΡΠ½Ρ‹ΠΌ Ρ€Π΅ΠΆΠΈΠΌΠΎΠΌ, ΠΎΠ΄Π½Π°ΠΊΠΎ ΠΎΠ½ΠΈ ΠΏΠΎΠ»ΡƒΡ‡ΠΈΠ»ΠΈ Π½Π°Π·Π²Π°Π½ΠΈΠ΅ Π·Π° Π½ΠΈΠ·ΠΊΠΈΠΉ ΡƒΡ€ΠΎΠ²Π΅Π½ΡŒ ΡˆΡƒΠΌΠ° ΠΈ Ρ…ΠΎΡ€ΠΎΡˆΠ΅Π΅ Ρ€Π΅Π³ΡƒΠ»ΠΈΡ€ΠΎΠ²Π°Π½ΠΈΠ΅ Π½Π° Π²Ρ‹Ρ…ΠΎΠ΄Π΅, Π±Π΅Π· Ρ€Π΅Π·ΠΊΠΈΡ… скачков, ΠΊΠΎΡ‚ΠΎΡ€Ρ‹Π΅ Π΅ΡΡ‚ΡŒ Ρƒ Π½Π΅ΠΊΠΎΡ‚ΠΎΡ€Ρ‹Ρ… Π±Π»ΠΎΠΊΠΎΠ² питания с ΠΈΠΌΠΏΡƒΠ»ΡŒΡΠ½Ρ‹ΠΌ Ρ€Π΅ΠΆΠΈΠΌΠΎΠΌ.

          Π”Ρ€ΡƒΠ³ΠΈΠ΅ схСмы ΠΈ схСмотСхника:
          ΠžΡΠ½ΠΎΠ²Ρ‹ ΠΎΠΏΠ΅Ρ€Π°Ρ†ΠΈΠΎΠ½Π½Ρ‹Ρ… усилитСлСй Π‘Ρ…Π΅ΠΌΡ‹ ΠΎΠΏΠ΅Ρ€Π°Ρ†ΠΈΠΎΠ½Π½Ρ‹Ρ… усилитСлСй Π¦Π΅ΠΏΠΈ питания ΠšΠΎΠ½ΡΡ‚Ρ€ΡƒΠΊΡ†ΠΈΡ транзистора Вранзистор Π”Π°Ρ€Π»ΠΈΠ½Π³Ρ‚ΠΎΠ½Π° ВранзисторныС схСмы Π‘Ρ…Π΅ΠΌΡ‹ Π½Π° ΠΏΠΎΠ»Π΅Π²Ρ‹Ρ… транзисторах УсловныС обозначСния схСм
          Π’Π΅Ρ€Π½ΡƒΡ‚ΡŒΡΡ Π² мСню Β«ΠšΠΎΠ½ΡΡ‚Ρ€ΡƒΠΊΡ†ΠΈΡ схСмы». . .

          БСрия

          AG — EMCO, высокоС напряТСниС

          Π˜Π·ΠΎΠ»ΠΈΡ€ΠΎΠ²Π°Π½Π½Ρ‹Π΅, ΠΏΡ€ΠΎΠΏΠΎΡ€Ρ†ΠΈΠΎΠ½Π°Π»ΡŒΠ½Ρ‹Π΅ ΠΏΡ€Π΅ΠΎΠ±Ρ€Π°Π·ΠΎΠ²Π°Ρ‚Π΅Π»ΠΈ постоянного Ρ‚ΠΎΠΊΠ° Π² постоянный Ρ‚ΠΎΠΊ высокого напряТСния
          ΠžΡ‚ 100 Π’ Π΄ΠΎ 6 ΠΊΠ’ ΠΏΡ€ΠΈ 1 Π’Ρ‚ ΠΈ 1,5 Π’Ρ‚
          Π’Π΅ΠΏΠ΅Ρ€ΡŒ ΠΏΡ€ΠΈΠ·Π½Π°Π½ΠΎ UL

          AG Series — это новая Π»ΠΈΠ½Π΅ΠΉΠΊΠ° ΡΠ²Π΅Ρ€Ρ…ΠΌΠΈΠ½ΠΈΠ°Ρ‚ΡŽΡ€Π½Ρ‹Ρ… ΠΏΡ€Π΅ΠΎΠ±Ρ€Π°Π·ΠΎΠ²Π°Ρ‚Π΅Π»Π΅ΠΉ постоянного Ρ‚ΠΎΠΊΠ° Π² Π²Ρ‹ΡΠΎΠΊΠΎΠ²ΠΎΠ»ΡŒΡ‚Π½Ρ‹Π΅, ΠΊΠΎΡ‚ΠΎΡ€Ρ‹Π΅ ΡƒΡΡ‚Π°Π½Π°Π²Π»ΠΈΠ²Π°ΡŽΡ‚ отраслСвой стандарт ΠΌΠΈΠ½ΠΈΠ°Ρ‚ΡŽΡ€ΠΈΠ·Π°Ρ†ΠΈΠΈ высокого напряТСния.Π­Ρ‚ΠΈ ΠΏΡ€Π΅ΠΎΠ±Ρ€Π°Π·ΠΎΠ²Π°Ρ‚Π΅Π»ΠΈ Ρ€Π°Π·ΠΌΠ΅Ρ€ΠΎΠΌ с ΠΊΠΎΠΌΠΏΠΎΠ½Π΅Π½Ρ‚ идСально подходят для ΠΏΡ€ΠΈΠ»ΠΎΠΆΠ΅Π½ΠΈΠΉ, Ρ‚Ρ€Π΅Π±ΡƒΡŽΡ‰ΠΈΡ… минимального Ρ€Π°Π·ΠΌΠ΅Ρ€Π° ΠΈ вСса.

          Занимая ΠΌΠ΅Π½Π΅Π΅ ΠΎΠ΄Π½ΠΎΠΉ дСсятой кубичСского дюйма объСма, этот ΡƒΠ½ΠΈΠΊΠ°Π»ΡŒΠ½Ρ‹ΠΉ корпус отличаСтся Ρ‡Ρ€Π΅Π·Π²Ρ‹Ρ‡Π°ΠΉΠ½ΠΎ Π½ΠΈΠ·ΠΊΠΈΠΌ ΠΏΡ€ΠΎΡ„ΠΈΠ»Π΅ΠΌ — всСго 0,128 дюйма (3,25 ΠΌΠΌ) ΠΏΡ€ΠΈ установкС свСрху ΠΈΠ»ΠΈ 0,152 дюйма (3,86 ΠΌΠΌ) ΠΏΡ€ΠΈ установкС снизу. ΠŸΠ΅Ρ‡Π°Ρ‚Π½Π°Ρ ΠΏΠ»Π°Ρ‚Π°. ΠŸΡ€Π΅ΠΎΠ±Ρ€Π°Π·ΠΎΠ²Π°Ρ‚Π΅Π»ΡŒ сСрии AG Ρ‚Π°ΠΊΠΆΠ΅ ΠΌΠΎΠΆΠ½ΠΎ ΡƒΡΡ‚Π°Π½ΠΎΠ²ΠΈΡ‚ΡŒ Π½Π° ΠΏΠ΅Ρ‡Π°Ρ‚Π½ΠΎΠΉ ΠΏΠ»Π°Ρ‚Π΅ с ΠΏΠΎΠΌΠΎΡ‰ΡŒΡŽ ΡˆΡ‚Ρ‹Ρ€Π΅ΠΉ Π΄ΠΈΠ°ΠΌΠ΅Ρ‚Ρ€ΠΎΠΌ 0,030 дюйма.

          Π˜Π»Π»ΡŽΡΡ‚Ρ€Π°Ρ†ΠΈΠΈ, см. На страницС 9 Ρ‚Π°Π±Π»ΠΈΡ†Ρ‹ Π΄Π°Π½Π½Ρ‹Ρ… сСрии AG.Π”ΠΈΠ°ΠΏΠ°Π·ΠΎΠ½ ΠΊΠΎΠ½Ρ‚Ρ€ΠΎΠ»ΠΈΡ€ΡƒΠ΅ΠΌΡ‹Ρ… Π²Ρ‹Ρ…ΠΎΠ΄Π½Ρ‹Ρ… напряТСний ΠΎΡ‚ 100 Π’ Π΄ΠΎ 6 ΠΊΠ’. ΠŸΠΎΠΆΠ°Π»ΡƒΠΉΡΡ‚Π°, ΠΎΠ±Ρ€Π°Ρ‚ΠΈΡ‚Π΅ΡΡŒ ΠΊ Ρ‚Π°Π±Π»ΠΈΡ†Π΅ Π΄Π°Π½Π½Ρ‹Ρ… для ΠΌΠΎΠ½Ρ‚Π°ΠΆΠ° Π½Π° ΠΏΠ΅Ρ‡Π°Ρ‚Π½ΠΎΠΉ ΠΏΠ»Π°Ρ‚Π΅.

          ΠŸΡ€ΠΈΠ»ΠΎΠΆΠ΅Π½ΠΈΡ

          • Π›Π°Π²ΠΈΠ½Π½Ρ‹Π΅ Ρ„ΠΎΡ‚ΠΎΠ΄ΠΈΠΎΠ΄Ρ‹
          • Зарядка кондСнсатора
          • Π­Π»Π΅ΠΊΡ‚Ρ€ΠΎΡ„ΠΎΡ€Π΅Π·
          • ЀотоэлСктронныС ΡƒΠΌΠ½ΠΎΠΆΠΈΡ‚Π΅Π»ΠΈ
          • ΠŸΡŒΠ΅Π·ΠΎΡƒΡΡ‚Ρ€ΠΎΠΉΡΡ‚Π²Π°
          • Масс-спСктромСтрия
          • ΠŸΠΎΠ΄Π΄Π΅Ρ€ΠΆΠΈΠ²Π°ΡŽΡ‰ΠΈΠ΅ ΠΈΠΎΠ½Π½Ρ‹Π΅ насосы

          Π₯арактСристики

          • ΠŸΡ€ΠΎΠΏΠΎΡ€Ρ†ΠΈΠΎΠ½Π°Π»ΡŒΠ½Ρ‹ΠΉ Π²Π²ΠΎΠ΄ / Π²Ρ‹Π²ΠΎΠ΄
          • ΠœΠ°Π»ΠΎΡˆΡƒΠΌΡΡ‰ΠΈΠΉ ΠΊΠ²Π°Π·ΠΈΡΠΈΠ½ΡƒΡΠΎΠΈΠ΄Π°Π»ΡŒΠ½Ρ‹ΠΉ осциллятор
          • ΠšΠΎΠ½Ρ‚Π°ΠΊΡ‚ управлСния (ΠΌΠΎΠΆΠ΅Ρ‚ ΠΈΡΠΏΠΎΠ»ΡŒΠ·ΠΎΠ²Π°Ρ‚ΡŒΡΡ для Π²ΠΊΠ»ΡŽΡ‡Π΅Π½ΠΈΡ / Π²Ρ‹ΠΊΠ»ΡŽΡ‡Π΅Π½ΠΈΡ)
          • Низкий Ρ‚ΠΎΠΊ ΡƒΡ‚Π΅Ρ‡ΠΊΠΈ
          • Низкая входная / выходная Π΅ΠΌΠΊΠΎΡΡ‚ΡŒ
          • Π“Π°Π»ΡŒΠ²Π°Π½ΠΈΡ‡Π΅ΡΠΊΠ°Ρ развязка Π²Ρ…ΠΎΠ΄Π° ΠΈ Π²Ρ‹Ρ…ΠΎΠ΄Π°
          • Π—Π°Ρ‰ΠΈΡ‚Π° ΠΎΡ‚ ΠΊΠΎΡ€ΠΎΡ‚ΠΊΠΎΠ³ΠΎ замыкания: ΠΌΠΈΠ½ΠΈΠΌΡƒΠΌ 1 ΠΌΠΈΠ½ΡƒΡ‚Π°
          • Минимальная Π½Π°Π³Ρ€ΡƒΠ·ΠΊΠ° Π½Π΅ трСбуСтся
          • MTBF> 1,862,000 часов Π½Π° Bellcore TR 332
          • ΠšΠΎΠΌΠΏΠ»Π΅ΠΊΡ‚ для повСрхностного ΠΌΠΎΠ½Ρ‚Π°ΠΆΠ°
          • Π’Π½Π΅ΡˆΠ½ΠΈΠ΅ ΠΊΠΎΠΌΠΏΠΎΠ½Π΅Π½Ρ‚Ρ‹ Π½Π΅ Ρ‚Ρ€Π΅Π±ΡƒΡŽΡ‚ΡΡ
          • БоотвСтствуСт RoHS
          • Π‘Π΅Ρ€Ρ‚ΠΈΡ„ΠΈΠΊΠ°Ρ‚ UL

          ΠžΠΏΡ†ΠΈΠΈ

          • 1 Π’Ρ‚ ΠΈ 1.Доступны вСрсии 5 Π’Ρ‚ (AG / AGH)
          • ДоступСн Π² Ρ‚Ρ€Π΅Ρ… стандартных Π΄ΠΈΠ°ΠΏΠ°Π·ΠΎΠ½Π°Ρ… Π²Ρ…ΠΎΠ΄Π½ΠΎΠ³ΠΎ напряТСния: ΠΎΡ‚ 0 Π΄ΠΎ 5, 12 ΠΈΠ»ΠΈ 24 Π’ (Π΄ΠΎ 3 ΠΊΠ’)
          • ΠŸΠΎΠ»ΡΡ€Π½ΠΎΡΡ‚ΡŒ: Π²Ρ‹Π±Π΅Ρ€ΠΈΡ‚Π΅ ΠΏΠΎΠ»ΠΎΠΆΠΈΡ‚Π΅Π»ΡŒΠ½Ρ‹ΠΉ ΠΈΠ»ΠΈ ΠΎΡ‚Ρ€ΠΈΡ†Π°Ρ‚Π΅Π»ΡŒΠ½Ρ‹ΠΉ Π²Ρ‹Ρ…ΠΎΠ΄ (P / N)
          • Π Π°ΡΡˆΠΈΡ€Π΅Π½Π½Π°Ρ рабочая Ρ‚Π΅ΠΌΠΏΠ΅Ρ€Π°Ρ‚ΡƒΡ€Π° (ΠΌΠΎΠ΄Π΅Π»ΠΈ AG / 1 Π’Ρ‚) (суффикс T)
          • ΠΠ»ΡŒΡ‚Π΅Ρ€Π½Π°Ρ‚ΠΈΠ²Π½Ρ‹Π΅ Π²Ρ…ΠΎΠ΄Π½Ρ‹Π΅ / Π²Ρ‹Ρ…ΠΎΠ΄Π½Ρ‹Π΅ напряТСния (ΠΎΠ±Ρ€Π°Ρ‚ΠΈΡ‚Π΅ΡΡŒ Π½Π° Π·Π°Π²ΠΎΠ΄-ΠΈΠ·Π³ΠΎΡ‚ΠΎΠ²ΠΈΡ‚Π΅Π»ΡŒ)
          • Π‘ΠΌ. Π˜Π½Ρ„ΠΎΡ€ΠΌΠ°Ρ†ΠΈΡŽ для Π·Π°ΠΊΠ°Π·Π°: страница 12 ВСхничСского описания сСрии AG

          элСктричСских Ρ†Π΅ΠΏΠ΅ΠΉ — Как происходит ΠΏΠ°Π΄Π΅Π½ΠΈΠ΅ напряТСния ΠΏΡ€ΠΈ ΠΏΠΎΡΠ»Π΅Π΄ΠΎΠ²Π°Ρ‚Π΅Π»ΡŒΠ½ΠΎΠΌ Π²ΠΊΠ»ΡŽΡ‡Π΅Π½ΠΈΠΈ?

          Π§Π°ΡΡ‚ΡŒ 0 — рСзисторы

          Π”Π°, рСзисторы Π²Ρ‹Π·Ρ‹Π²Π°ΡŽΡ‚ ΠΏΠ°Π΄Π΅Π½ΠΈΠ΅ напряТСния, «локально» замСдляя элСктроны, поэтому ΠΈΠΌ приходится Β«Ρ‚Ρ€Π°Ρ‚ΠΈΡ‚ΡŒΒ» Ρ‡Π°ΡΡ‚ΡŒ энСргии Π½Π° ΠΏΡ€Π΅ΠΎΠ΄ΠΎΠ»Π΅Π½ΠΈΠ΅ прСпятствия, ΠΊΠΎΡ‚ΠΎΡ€ΠΎΠ΅ Π²Ρ‹Π·Ρ‹Π²Π°Π΅Ρ‚ ΠΏΠ°Π΄Π΅Π½ΠΈΠ΅ напряТСния.Π’Ρ‹ ΠΌΠΎΠΆΠ΅Ρ‚Π΅ Π΄ΡƒΠΌΠ°Ρ‚ΡŒ ΠΎ Π½ΠΈΡ… ΠΊΠ°ΠΊ ΠΎ «ступСнях» ΠΏΠΎ Π½Π°ΠΏΡ€ΡΠΆΠ΅Π½ΠΈΡŽ.

          Π§Π°ΡΡ‚ΡŒ пСрвая — ΠΏΠ°Π΄Π΅Π½ΠΈΠ΅ напряТСния — ΡƒΠ΄Π΅Π»ΡŒΠ½ΠΎΠ΅ сопротивлСниС — ΠΈΠ΄Π΅Π°Π»ΡŒΠ½Ρ‹Π΅ схСмы

          ΠŸΡ€ΠΈΡ‡ΠΈΠ½Π° Π² Ρ‚ΠΎΠΌ, Ρ‡Ρ‚ΠΎ ΠΌΡ‹ ΠΈΠ΄Π΅Π°Π»ΠΈΠ·ΠΈΡ€ΡƒΠ΅ΠΌ «соСдинСния» ΠΊΠ°ΠΊ ΠΈΠΌΠ΅ΡŽΡ‰ΠΈΠ΅ Π½ΡƒΠ»Π΅Π²ΠΎΠ΅ сопротивлСниС, Ρ‚ΠΎ Π΅ΡΡ‚ΡŒ Π΄Π²Π΅ Ρ‚ΠΎΡ‡ΠΊΠΈ Π² Ρ†Π΅ΠΏΠΈ Π±Π΅Π· сопротивлСния ΠΌΠ΅ΠΆΠ΄Ρƒ Π½ΠΈΠΌΠΈ Π±ΡƒΠ΄ΡƒΡ‚ ΠΈΠΌΠ΅Ρ‚ΡŒ ΠΎΠ΄ΠΈΠ½Π°ΠΊΠΎΠ²ΠΎΠ΅ напряТСниС. ΠšΡ€ΠΎΠΌΠ΅ Ρ‚ΠΎΠ³ΠΎ, ΠΌΡ‹ ΠΏΡ€Π΅Π΄ΠΏΠΎΠ»Π°Π³Π°Π΅ΠΌ, Ρ‡Ρ‚ΠΎ сущСствуСт устойчивоС состояниС, Ρ‚Π°ΠΊ Ρ‡Ρ‚ΠΎ Β«Π½Π°Ρ‡Π°Π»ΡŒΠ½Ρ‹ΠΌΒ» ΠΏΠΎΡ‚ΠΎΠΊΠΎΠΌ элСктронов Π²Π½ΡƒΡ‚Ρ€ΠΈ Ρ†Π΅ΠΏΠΈ ΠΏΡ€Π΅Π½Π΅Π±Ρ€Π΅Π³Π°Π΅ΠΌ, ΠΈ ΠΌΡ‹ сосрСдотачиваСмся Π½Π° Ρ‚ΠΎΠΌ, ΠΊΠΎΠ³Π΄Π° систСма Ρ‚Π΅Ρ‡Π΅Ρ‚, Π½Π΅ мСняясь с Ρ‚Π΅Ρ‡Π΅Π½ΠΈΠ΅ΠΌ Π²Ρ€Π΅ΠΌΠ΅Π½ΠΈ.Π­Ρ‚ΠΎ идСализация.

          Если ΠΏΠΎΠ΄ΡƒΠΌΠ°Ρ‚ΡŒ, Ρ‚ΠΎΡ‚ Ρ„Π°ΠΊΡ‚, Ρ‡Ρ‚ΠΎ ΠΊΠ°Π±Π΅Π»ΠΈ Π½Π΅ ΠΈΠΌΠ΅ΡŽΡ‚ сопротивлСния, Π΄ΠΎΠ»ΠΆΠ΅Π½ Π±Ρ‹Ρ‚ΡŒ Π»ΠΎΠΆΠ½Ρ‹ΠΌ, ΠΏΠΎΡ‚ΠΎΠΌΡƒ Ρ‡Ρ‚ΠΎ Π² ΠΏΡ€ΠΎΡ‚ΠΈΠ²Π½ΠΎΠΌ случаС ΠΊΠΎΡ€ΠΎΡ‚ΠΊΠΎΠ΅ Π·Π°ΠΌΡ‹ΠΊΠ°Π½ΠΈΠ΅ Ρ†Π΅ΠΏΠΈ (Ρ‚ΠΎ Π΅ΡΡ‚ΡŒ соСдинСниС Π΄Π²ΡƒΡ… полюсов Π±Π°Ρ‚Π°Ρ€Π΅ΠΈ с напряТСниСм $ V $ Π±Π΅Π· ΠΊΠ°ΠΊΠΎΠ³ΠΎ-Π»ΠΈΠ±ΠΎ сопротивлСния ΠΌΠ΅ΠΆΠ΄Ρƒ Π½ΠΈΠΌΠΈ) ΠΏΡ€ΠΈΠ²Π΅Π»ΠΎ Π±Ρ‹ ΠΊ Ρ‚ΠΎΠΊ $ I = V / R $, Π³Π΄Π΅ сопротивлСниС $ R = 0 $, Ρ‡Ρ‚ΠΎ ΠΏΡ€ΠΈΠ²Π΅Π΄Π΅Ρ‚ ΠΊ бСсконСчному Ρ‚ΠΎΠΊΡƒ $ I = \ infty $.

          На самом Π΄Π΅Π»Π΅ ΠΊΠ°Π±Π΅Π»ΠΈ ΠΈΠΌΠ΅ΡŽΡ‚ сопротивлСниС $ r $. ΠœΡ‹ ΠΌΠΎΠΆΠ΅ΠΌ ΡΠΌΠΎΠ΄Π΅Π»ΠΈΡ€ΠΎΠ²Π°Ρ‚ΡŒ это ΠΊΠ°ΠΊ Ρ„Ρ€Π°Π³ΠΌΠ΅Π½Ρ‚ кабСля Π΄Π»ΠΈΠ½ΠΎΠΉ $ dx $, ΠΈΠΌΠ΅ΡŽΡ‰ΠΈΠΉ сопротивлСниС $$ r = \ rho dx $$, Ρ‚ΠΎ Π΅ΡΡ‚ΡŒ $ \ rho $ — это сопротивлСниС Π½Π° Π΅Π΄ΠΈΠ½ΠΈΡ†Ρƒ Π΄Π»ΠΈΠ½Ρ‹.ΠœΡ‹ ΠΏΡ€Π΅Π΄ΠΏΠΎΠ»Π°Π³Π°Π΅ΠΌ, Ρ‡Ρ‚ΠΎ ΠΎΠ½ постоянный.

          Π’ этом случаС, Ссли Π²Ρ‹ измСряСтС напряТСниС ΠΌΠ΅ΠΆΠ΄Ρƒ двумя Ρ‚ΠΎΡ‡ΠΊΠ°ΠΌΠΈ Ρ†Π΅ΠΏΠΈ Π½Π° расстоянии $ L $ Π±Π΅Π· ΠΊΠ°ΠΊΠΎΠ³ΠΎ-Π»ΠΈΠ±ΠΎ сопротивлСния ΠΌΠ΅ΠΆΠ΄Ρƒ Π½ΠΈΠΌΠΈ, Π²Ρ‹, , Π±ΡƒΠ΄Π΅Ρ‚Π΅ ΠΈΠ·ΠΌΠ΅Ρ€ΡΡ‚ΡŒ зависящСС ΠΎΡ‚ расстояния ΠΏΠ°Π΄Π΅Π½ΠΈΠ΅ напряТСния $$ \ Delta V = — \ rho LI $$ Π³Π΄Π΅ $ I $ — ΠΈΡΠΏΡ‹Ρ‚Π°Ρ‚Π΅Π»ΡŒΠ½Ρ‹ΠΉ Ρ‚ΠΎΠΊ.

          Π­Ρ‚ΠΎ ΠΏΠ°Π΄Π΅Π½ΠΈΠ΅, ΠΊΠ°ΠΊ ΠΏΡ€Π°Π²ΠΈΠ»ΠΎ, ΠΎΡ‡Π΅Π½ΡŒ ΠΌΠ°Π»ΠΎ ΠΏΠΎ ΡΡ€Π°Π²Π½Π΅Π½ΠΈΡŽ с напряТСниСм $ V $ Π±Π°Ρ‚Π°Ρ€Π΅ΠΈ, ΠΈ ΠΈΠΌ ΠΎΠ±Ρ‹Ρ‡Π½ΠΎ ΠΏΡ€Π΅Π½Π΅Π±Ρ€Π΅Π³Π°ΡŽΡ‚. Однако ΠΈΠ½ΠΎΠ³Π΄Π° это Π½Π΅ΠΎΠ±Ρ…ΠΎΠ΄ΠΈΠΌΠΎ ΡƒΡ‡ΠΈΡ‚Ρ‹Π²Π°Ρ‚ΡŒ!

          Π˜Ρ‚Π°ΠΊ, Ссли Ρƒ вас Π΅ΡΡ‚ΡŒ Ρ†Π΅ΠΏΡŒ Π΄Π»ΠΈΠ½ΠΎΠΉ $ L_ {tot} $ с Π±Π°Ρ‚Π°Ρ€Π΅Π΅ΠΉ Π² $ \ ell = 0 $ (Π³Π΄Π΅ $ \ ell $ — Π΄Π»ΠΈΠ½Π° Ρ†Π΅ΠΏΠΈ), (Ρ‚ΠΎΡ‡Π΅Ρ‡Π½ΠΎΠ΅) «Π½ΠΎΡ€ΠΌΠ°Π»ΡŒΠ½ΠΎΠ΅» сопротивлСниС $ R $ Π² $ \ ell = \ ell_0 $ ΠΈ снова батарСя Π² $ \ ell = L_ {tot} $, Ρƒ вас Π±ΡƒΠ΄Π΅Ρ‚ ΠΏΠ°Π΄Π΅Π½ΠΈΠ΅ напряТСния $ V (\ ell) $

          $$ V (\ ell) = \ begin {case} -I \ rho \ ell, & \ text {if} \ \ ell <\ ell_0 \\ -I \ rho \ ell-RI, & \ ell> = \ ell_0 \ end {case}

          $

          , ΠΊΠΎΡ‚ΠΎΡ€Ρ‹ΠΉ зависит ΠΎΡ‚ Π΄Π»ΠΈΠ½Ρ‹.

          ΠžΠ±Ρ€Π°Ρ‚ΠΈΡ‚Π΅ Π²Π½ΠΈΠΌΠ°Π½ΠΈΠ΅, Ρ‡Ρ‚ΠΎ Π²Ρ‹ всСгда ΠΌΠΎΠΆΠ΅Ρ‚Π΅ Π°ΠΏΠΏΡ€ΠΎΠΊΡΠΈΠΌΠΈΡ€ΠΎΠ²Π°Ρ‚ΡŒ это идСальной схСмой с сопротивлСниСм $ r = \ rho L $ ΠΏΠΎΡΠ»Π΅Π΄ΠΎΠ²Π°Ρ‚Π΅Π»ΡŒΠ½ΠΎ с Β«Π½ΠΎΡ€ΠΌΠ°Π»ΡŒΠ½Ρ‹ΠΌΒ» сопротивлСниСм $ R $ — это, ΠΊΠΎΠ½Π΅Ρ‡Π½ΠΎ, Ссли Π²Ρ‹ Π΄Π΅ΠΉΡΡ‚Π²ΠΈΡ‚Π΅Π»ΡŒΠ½ΠΎ Π½Π΅ заинтСрСсованы Π² пространствСнном Π·Π½Π°Ρ‡Π΅Π½ΠΈΠΈ напряТСниС $ V (\ ell) $.

          Π˜ΡΠΏΠΎΠ»ΡŒΠ·ΡƒΡ Ρ‚Π΅ ΠΆΠ΅ рассуТдСния, Π²Ρ‹ Ρ‚Π°ΠΊΠΆΠ΅ ΠΌΠΎΠΆΠ΅Ρ‚Π΅, вмСсто модСлирования вашСго сопротивлСния $ R $ ΠΊΠ°ΠΊ Ρ‚ΠΎΡ‡Π΅Ρ‡Π½ΠΎΠ³ΠΎ ΠΎΠ±ΡŠΠ΅ΠΊΡ‚Π°, ΠΏΡ€Π΅Π΄ΠΏΠΎΠ»ΠΎΠΆΠΈΡ‚ΡŒ, Ρ‡Ρ‚ΠΎ Ρƒ Π½Π΅Π³ΠΎ Π΅ΡΡ‚ΡŒ Ρ€Π°ΡΡˆΠΈΡ€Π΅Π½ΠΈΠ΅ $ L_0 $ с сопротивлСниСм Π½Π° Π΅Π΄ΠΈΠ½ΠΈΡ†Ρƒ Π΄Π»ΠΈΠ½Ρ‹ $ r_0 = R / L_0 $, Ρ‚Π°ΠΊ Ρ‡Ρ‚ΠΎ Ρ‚Π΅ΠΏΠ΅Ρ€ΡŒ напряТСниС зависит ΠΎΡ‚ Ρ‚ΠΎΠ³ΠΎ, Π³Π΄Π΅ Π²Ρ‹ измСряСтС сопротивлСниС, с $ \ Delta V = -r_0 x I $ с ΠΏΠΎΠ»ΠΎΠΆΠ΅Π½ΠΈΠ΅ΠΌ $ x $ ΠΏΠΎ ΡΠΎΠΏΡ€ΠΎΡ‚ΠΈΠ²Π»Π΅Π½ΠΈΡŽ.

          Π’ схСмС, ΠΊΠΎΡ‚ΠΎΡ€ΡƒΡŽ Π²Ρ‹ нарисовали, это ΠΏΠΎΡ…ΠΎΠΆΠ΅ Π½Π° Π½Π°Π»ΠΈΡ‡ΠΈΠ΅ $ \ rho = 0 $, Π° Π·Π°Ρ‚Π΅ΠΌ Π΄Π²ΡƒΡ… Ρ‚ΠΎΡ‡Π΅Ρ‡Π½Ρ‹Ρ… рСзисторов со Π·Π½Π°Ρ‡Π΅Π½ΠΈΠ΅ΠΌ $ R $, ΠΊΠΎΡ‚ΠΎΡ€Ρ‹Π΅ Π΄ΠΎΠ»ΠΆΠ½Ρ‹ ΠΎΠ±Π΅ΡΠΏΠ΅Ρ‡ΠΈΠ²Π°Ρ‚ΡŒ Π½Π΅ΠΎΠ±Ρ…ΠΎΠ΄ΠΈΠΌΠΎΠ΅ ΠΏΠ°Π΄Π΅Π½ΠΈΠ΅ напряТСния, ΠΈ ΠΎΠ½ΠΈ Β«Ρ€Π°Π·Π΄Π΅Π»ΡΡŽΡ‚Β» Π΅Π³ΠΎ, ΠΏΠΎΡΠΊΠΎΠ»ΡŒΠΊΡƒ ΠΎΠ½ΠΈ Ρ€Π°Π²Π½Ρ‹. Но Π²Ρ‹ Ρ‚Π°ΠΊΠΆΠ΅ ΠΌΠΎΠΆΠ΅Ρ‚Π΅ ΡƒΠ²ΠΈΠ΄Π΅Ρ‚ΡŒ Π΅Π³ΠΎ ΠΊΠ°ΠΊ ΠΎΠ΄ΠΈΠ½ большой кондСнсатор с ΠΎΠ±Ρ‰ΠΈΠΌ сопротивлСниСм $ 2R $ ΠΈ Π΄Π»ΠΈΠ½ΠΎΠΉ $ 2L $, Ρ‚Π°ΠΊ Ρ‡Ρ‚ΠΎ ΠΎΠ½ ΠΈΠΌΠ΅Π΅Ρ‚ $ \ rho_0 = (2R) / (2L) $, Ρ‚Π°ΠΊ Ρ‡Ρ‚ΠΎ послС Π½Π΅ΠΊΠΎΡ‚ΠΎΡ€ΠΎΠ³ΠΎ расстояния $ L $ (эквивалСнтно ΠΎΠ΄Π½ΠΎΠΌΡƒ рСзистору ) ΠΏΠ°Π΄Π΅Π½ΠΈΠ΅ напряТСния составляСт $ \ Delta V_1 = (2R / 2L) * L * I = R * I $, Ρ‡Ρ‚ΠΎ эквивалСнтно ΠΎΠ΄Π½ΠΎΠΌΡƒ ΠΏΠΎΠ»Π½ΠΎΠΌΡƒ рСзистору. ΠŸΠΎΡ‚ΠΎΠΌΡƒ Ρ‡Ρ‚ΠΎ Π² вашСм случаС Ρ‚ΠΎΠΊ I составляСт 100 Π’ / 2R $ (ΠΎΠ±Ρ‰Π΅Π΅ напряТСниС, Π΄Π΅Π»Π΅Π½Π½ΠΎΠ΅ Π½Π° ΠΎΠ±Ρ‰Π΅Π΅ сопротивлСниС), ΠΈ Π²Ρ‹ ΠΏΠΎΠ»ΡƒΡ‡Π°Π΅Ρ‚Π΅ $$ \ Delta V_1 = R * 100V / (2R) = 100V / 2 = 50V $$

          Π’Ρ‹ Ρ‚Π°ΠΊΠΆΠ΅ ΠΌΠΎΠΆΠ΅Ρ‚Π΅ ΠΌΠΎΠ΄Π΅Π»ΠΈΡ€ΠΎΠ²Π°Ρ‚ΡŒ ΠΊΠ°Π±Π΅Π»ΠΈ Π² Π²ΠΈΠ΄Π΅ Ρ‚Ρ€Π΅Ρ…ΠΌΠ΅Ρ€Π½Ρ‹Ρ… Ρ†ΠΈΠ»ΠΈΠ½Π΄Ρ€ΠΎΠ² с сопротивлСниСм Π½Π° Π΅Π΄ΠΈΠ½ΠΈΡ†Ρƒ объСма ΠΈ Ρ‚. Π”. — Ρ‡Ρ‚ΠΎΠ±Ρ‹ ΡƒΠ·Π½Π°Ρ‚ΡŒ большС: ΡƒΠ΄Π΅Π»ΡŒΠ½ΠΎΠ΅ сопротивлСниС

          Π§Π°ΡΡ‚ΡŒ вторая — Π½Π°Ρ‡Π°Π»ΡŒΠ½Ρ‹Π΅ Π²Ρ€Π΅ΠΌΠ΅Π½Π° — сорт

          Π’Π΅ΠΏΠ΅Ρ€ΡŒ ΠΌΠΎΠ΄Π΅Π»ΠΈΡ€ΡƒΠ΅ΠΌ схСму Π² Π½Π°Ρ‡Π°Π»ΡŒΠ½Ρ‹Π΅ ΠΌΠΎΠΌΠ΅Π½Ρ‚Ρ‹ Π²Ρ€Π΅ΠΌΠ΅Π½ΠΈ.Π§Ρ‚ΠΎΠ±Ρ‹ ΡΠ΄Π΅Π»Π°Ρ‚ΡŒ это ΠΏΡ€Π°Π²ΠΈΠ»ΡŒΠ½ΠΎ, Π½Π°ΠΌ понадобится кондСнсатор ΠΈ Π΄ΠΈΡ„Ρ„Π΅Ρ€Π΅Π½Ρ†ΠΈΠ°Π»ΡŒΠ½ΠΎΠ΅ ΡƒΡ€Π°Π²Π½Π΅Π½ΠΈΠ΅, Π½ΠΎ я Π΄ΡƒΠΌΠ°ΡŽ, Ρ‡Ρ‚ΠΎ сСйчас это Π±Ρ‹Π»ΠΎ Π±Ρ‹ слишком слоТно. Π― ΠΏΠΎΠΏΡ€ΠΎΠ±ΡƒΡŽ Π΄Ρ€ΡƒΠ³ΠΎΠΉ ΠΏΠΎΠ΄Ρ…ΠΎΠ΄.

          ΠŸΡ€Π΅Π΄ΠΏΠΎΠ»ΠΎΠΆΠΈΠΌ, Ρƒ нас Π΅ΡΡ‚ΡŒ кабСль с ΠΎΠ±Ρ‰ΠΈΠΌ сопротивлСниСм $ R $, ΠΈ ΠΌΡ‹ Ρ€Π°Π·Π΄Π΅Π»ΠΈΠ»ΠΈ Π΅Π³ΠΎ Π½Π° $ N $ частСй сопротивлСния $ r = R / N $. Π£ нас Π΅ΡΡ‚ΡŒ аккумулятор, ΠΈ ΠΊΠ°Π±Π΅Π»ΠΈ ΡΠΎΠ΅Π΄ΠΈΠ½ΡΡŽΡ‚ Π΄Π²Π΅ части аккумулятора. Π’ Π½Π°Ρ‡Π°Π»ΡŒΠ½Ρ‹ΠΉ ΠΌΠΎΠΌΠ΅Π½Ρ‚ всС находится ΠΏΠΎΠ΄ напряТСниСм $ V = 0 $, ΠΊΡ€ΠΎΠΌΠ΅ Ρ‚ΠΎΡ‡ΠΊΠΈ, Π³Π΄Π΅ находится батарСя, которая ΠΈΠΌΠ΅Π΅Ρ‚ напряТСниС $ V $. Π’Π½Π°Ρ‡Π°Π»Π΅ элСктроны видят нСбольшой кусок Ρ†Π΅ΠΏΠΈ с сопротивлСниСм $ r $, Ρ€Π°Π·Π½ΠΎΡΡ‚ΡŒΡŽ напряТСний $ V $ ΠΈ ΠΈΠ·Π»ΡƒΡ‡Π°ΡŽΡ‚ Ρ‚ΠΎΠΊ $ i_1 = V / r $.Π’Π΅ΠΏΠ΅Ρ€ΡŒ ΠΎΠ½ΠΈ видят Π΄Ρ€ΡƒΠ³ΠΎΠΉ рСзистор ΠΈ ΠΈΠ·Π»ΡƒΡ‡Π°ΡŽΡ‚ Ρ‚ΠΎΠΊ $ i_2 = V / (2r) $. (Π­Ρ‚ΠΎ ΠΏΠΎΡ‚ΠΎΠΌΡƒ, Ρ‡Ρ‚ΠΎ ΠΎΠ½ΠΈ видят рСзистор $ r $ ΠΏΡ€ΠΈ напряТСнии $ V $, Π° Π·Π°Ρ‚Π΅ΠΌ ΡΠ»Π΅Π΄ΡƒΡŽΡ‚ Π·Π°ΠΊΠΎΠ½Ρƒ Ома).

          Если Ρ€Π°Π½ΡŒΡˆΠ΅ ΠΏΠ°Π΄Π΅Π½ΠΈΠ΅ напряТСния Π½Π° ΠΏΠ΅Ρ€Π²ΠΎΠΌ рСзисторС Π±Ρ‹Π»ΠΎ $ V_1 (0) = i_2 * r = V $, Ρ‚Π°ΠΊ ΠΊΠ°ΠΊ ΠΏΠΎΠ»Π½ΠΎΠ΅ «напряТСниС» Π΄ΠΎΠ»ΠΆΠ½ΠΎ Π±Ρ‹Π»ΠΎ ΡƒΠΌΠ΅Π½ΡŒΡˆΠΈΡ‚ΡŒΡΡ Ρ‡Π΅Ρ€Π΅Π· ΠΏΠ΅Ρ€Π²Ρ‹ΠΉ рСзистор, Ρ‚ΠΎ Ρ‚Π΅ΠΏΠ΅Ρ€ΡŒ напряТСниС дСлится ΠΌΠ΅ΠΆΠ΄Ρƒ двумя рСзисторами. , Ρ‚Π°ΠΊ Ρ‡Ρ‚ΠΎ ΠΏΠ°Π΄Π΅Π½ΠΈΠ΅ напряТСния Π½Π° ΠΏΠ΅Ρ€Π²ΠΎΠΌ составляСт $ V_1 (1) = i_2 r = V / (2r) * r = V / 2 $.

          И Ρ‚Π°ΠΊ Π΄Π°Π»Π΅Π΅, ΠΊΠΎΠ³Π΄Π° элСктроны ΠΏΠ΅Ρ€Π΅ΡΠ΅ΠΊΠ°ΡŽΡ‚ $ n $ рСзисторов, ΠΏΠ°Π΄Π΅Π½ΠΈΠ΅ напряТСния Π½Π° ΠΏΠ΅Ρ€Π²ΠΎΠΌ ΠΈΠ· Π½ΠΈΡ… составляСт $$ V_1 (n) = V / n $$

          Если врСмя, Π½Π΅ΠΎΠ±Ρ…ΠΎΠ΄ΠΈΠΌΠΎΠ΅ элСктронам, Ρ‡Ρ‚ΠΎΠ±Ρ‹ ΠΏΠ΅Ρ€Π΅ΡΠ΅Ρ‡ΡŒ рСзистор, Ρ€Π°Π²Π½ΠΎ $ \ tau $, Ρ‚Π΅ΠΏΠ΅Ρ€ΡŒ ΠΌΡ‹ ΠΈΠΌΠ΅Π΅ΠΌ, Ρ‡Ρ‚ΠΎ $ n = t / \ tau $ (Ρ‚.Π΅. Ρ‡Π΅Ρ€Π΅Π· врСмя $ 10 \ tau $ ΠΌΡ‹ пСрСсСкли $ n = 10 $ рСзисторов) Ρ‚Π°ΠΊ Ρ‡Ρ‚ΠΎ

          $$ V_1 (t) = V / n = {V \ over (t / \ tau)} = {V \ tau \ over t} $$ Π΄Π΅ΠΉΡΡ‚Π²ΠΈΡ‚Π΅Π»ΡŒΠ½ΠΎ для $ t .

          Π’ Ρ‚ΠΎ ΠΆΠ΅ врСмя Ρ‚ΠΎΠΊ ΠΈΠ΄Π΅Ρ‚ ΠΎΡ‚ 0 (ΠΊΠΎΠ³Π΄Π° ΠΌΡ‹ ΠΏΠΎΠ΄ΠΊΠ»ΡŽΡ‡Π°Π΅ΠΌ Ρ†Π΅ΠΏΡŒ, Ρ‚ΠΎΠΊ отсутствуСт) Π΄ΠΎ $$ i_n = V / (nr) = {V \ over (t / \ tau) r} = {V \ tau \ over nt} $$, ΠΊΠΎΡ‚ΠΎΡ€Ρ‹ΠΉ Ρ‚Π°ΠΊΠΆΠ΅ ΡƒΠΌΠ΅Π½ΡŒΡˆΠ°Π΅Ρ‚ΡΡ Π΄ΠΎ ΠΊΠΎΠ½Π΅Ρ‡Π½ΠΎΠ³ΠΎ значСния $ I = V / (Nr) = V / R $.

          Π˜Ρ‚Π°ΠΊ, Π² ΠΌΠΎΠΌΠ΅Π½Ρ‚ Π²Ρ€Π΅ΠΌΠ΅Π½ΠΈ $ t = 0 $, Π³Π΄Π΅ — всплСск Ρ‚ΠΎΠΊΠ° Π½Π° ΠΏΠ΅Ρ€Π²ΠΎΠΌ рСзисторС, ΠΊΠΎΡ‚ΠΎΡ€Ρ‹ΠΉ Π·Π°Ρ‚Π΅ΠΌ ΡƒΠΌΠ΅Π½ΡŒΡˆΠ°Π΅Ρ‚ΡΡ Π΄ΠΎ ΠΊΠΎΠ½Π΅Ρ‡Π½ΠΎΠ³ΠΎ значСния, ΠΏΠΎΡΠΊΠΎΠ»ΡŒΠΊΡƒ элСктроны Β«ΠΈΡΡΠ»Π΅Π΄ΡƒΡŽΡ‚Β» Π½Π°Π»ΠΈΡ‡ΠΈΠ΅ Π½ΠΎΠ²Ρ‹Ρ… рСзисторов ΠΈ ΡƒΡ‚ΠΎΡ‡Π½ΡΡŽΡ‚ ΠΈΡ… срСдний ΠΊΠΎΠ½Π΅Ρ‡Π½Ρ‹ΠΉ Ρ‚ΠΎΠΊ. Π’ΠΎ ΠΆΠ΅ самоС ΠΈ с напряТСниСм.

          Π˜ΡΡ‚ΠΎΡ‡Π½ΠΈΠΊΠΈ ΠΎΠΏΠΎΡ€Π½ΠΎΠ³ΠΎ напряТСния сСрии

          | Analog Devices

          НСкоторыС Ρ„Π°ΠΉΠ»Ρ‹ cookie Π½Π΅ΠΎΠ±Ρ…ΠΎΠ΄ΠΈΠΌΡ‹ для бСзопасного Π²Ρ…ΠΎΠ΄Π° Π² систСму, Π½ΠΎ Π΄Ρ€ΡƒΠ³ΠΈΠ΅ Π½Π΅ΠΎΠ±ΡΠ·Π°Ρ‚Π΅Π»ΡŒΠ½Ρ‹ для Ρ„ΡƒΠ½ΠΊΡ†ΠΈΠΎΠ½Π°Π»ΡŒΠ½ΠΎΠΉ Π΄Π΅ΡΡ‚Π΅Π»ΡŒΠ½ΠΎΡΡ‚ΠΈ. Π‘Π±ΠΎΡ€ Π½Π°ΡˆΠΈΡ… Π΄Π°Π½Π½Ρ‹Ρ… ΠΈΡΠΏΠΎΠ»ΡŒΠ·ΡƒΠ΅Ρ‚ΡΡ для ΡƒΠ»ΡƒΡ‡ΡˆΠ΅Π½ΠΈΡ Π½Π°ΡˆΠΈΡ… ΠΏΡ€ΠΎΠ΄ΡƒΠΊΡ‚ΠΎΠ² ΠΈ услуг. ΠœΡ‹ Ρ€Π΅ΠΊΠΎΠΌΠ΅Π½Π΄ΡƒΠ΅ΠΌ Π²Π°ΠΌ ΠΏΡ€ΠΈΠ½ΡΡ‚ΡŒ наши Ρ„Π°ΠΉΠ»Ρ‹ cookie, Ρ‡Ρ‚ΠΎΠ±Ρ‹ ΠΎΠ±Π΅ΡΠΏΠ΅Ρ‡ΠΈΡ‚ΡŒ ΠΌΠ°ΠΊΡΠΈΠΌΠ°Π»ΡŒΠ½ΡƒΡŽ ΠΏΡ€ΠΎΠΈΠ·Π²ΠΎΠ΄ΠΈΡ‚Π΅Π»ΡŒΠ½ΠΎΡΡ‚ΡŒ ΠΈ Ρ„ΡƒΠ½ΠΊΡ†ΠΈΠΎΠ½Π°Π»ΡŒΠ½ΠΎΡΡ‚ΡŒ нашСго сайта.Для получСния Π΄ΠΎΠΏΠΎΠ»Π½ΠΈΡ‚Π΅Π»ΡŒΠ½ΠΎΠΉ ΠΈΠ½Ρ„ΠΎΡ€ΠΌΠ°Ρ†ΠΈΠΈ Π²Ρ‹ ΠΌΠΎΠΆΠ΅Ρ‚Π΅ ΠΏΡ€ΠΎΡΠΌΠΎΡ‚Ρ€Π΅Ρ‚ΡŒ свСдСния ΠΎ Ρ„Π°ΠΉΠ»Π°Ρ… cookie. Π£Π·Π½Π°ΠΉΡ‚Π΅ большС ΠΎ нашСй ΠΏΠΎΠ»ΠΈΡ‚ΠΈΠΊΠ΅ ΠΊΠΎΠ½Ρ„ΠΈΠ΄Π΅Π½Ρ†ΠΈΠ°Π»ΡŒΠ½ΠΎΡΡ‚ΠΈ.

          ΠŸΡ€ΠΈΠ½ΡΡ‚ΡŒ ΠΈ ΠΏΡ€ΠΎΠ΄ΠΎΠ»ΠΆΠΈΡ‚ΡŒ ΠŸΡ€ΠΈΠ½ΡΡ‚ΡŒ ΠΈ ΠΏΡ€ΠΎΠ΄ΠΎΠ»ΠΆΠΈΡ‚ΡŒ

          Π€Π°ΠΉΠ»Ρ‹ cookie, ΠΊΠΎΡ‚ΠΎΡ€Ρ‹Π΅ ΠΌΡ‹ ΠΈΡΠΏΠΎΠ»ΡŒΠ·ΡƒΠ΅ΠΌ, ΠΌΠΎΠΆΠ½ΠΎ Ρ€Π°Π·Π΄Π΅Π»ΠΈΡ‚ΡŒ Π½Π° ΡΠ»Π΅Π΄ΡƒΡŽΡ‰ΠΈΠ΅ ΠΊΠ°Ρ‚Π΅Π³ΠΎΡ€ΠΈΠΈ:

          Π‘Ρ‚Ρ€ΠΎΠ³ΠΎ Π½Π΅ΠΎΠ±Ρ…ΠΎΠ΄ΠΈΠΌΡ‹Π΅ Ρ„Π°ΠΉΠ»Ρ‹ cookie:
          Π­Ρ‚ΠΎ Ρ„Π°ΠΉΠ»Ρ‹ cookie, ΠΊΠΎΡ‚ΠΎΡ€Ρ‹Π΅ Π½Π΅ΠΎΠ±Ρ…ΠΎΠ΄ΠΈΠΌΡ‹ для Ρ€Π°Π±ΠΎΡ‚Ρ‹ analog.com ΠΈΠ»ΠΈ ΠΎΠΏΡ€Π΅Π΄Π΅Π»Π΅Π½Π½Ρ‹Ρ… ΠΏΡ€Π΅Π΄Π»Π°Π³Π°Π΅ΠΌΡ‹Ρ… Ρ„ΡƒΠ½ΠΊΡ†ΠΈΠΉ. Они Π»ΠΈΠ±ΠΎ слуТат СдинствСнной Ρ†Π΅Π»ΠΈ ΠΏΠ΅Ρ€Π΅Π΄Π°Ρ‡ΠΈ Π΄Π°Π½Π½Ρ‹Ρ… ΠΏΠΎ сСти, Π»ΠΈΠ±ΠΎ строго Π½Π΅ΠΎΠ±Ρ…ΠΎΠ΄ΠΈΠΌΡ‹ для прСдоставлСния ΠΎΠ½Π»Π°ΠΉΠ½-услуг, явно Π·Π°ΠΏΡ€ΠΎΡˆΠ΅Π½Π½Ρ‹Ρ… Π²Π°ΠΌΠΈ.
          АналитичСскиС / Ρ€Π°Π±ΠΎΡ‡ΠΈΠ΅ Ρ„Π°ΠΉΠ»Ρ‹ cookie:
          Π­Ρ‚ΠΈ Ρ„Π°ΠΉΠ»Ρ‹ cookie ΠΏΠΎΠ·Π²ΠΎΠ»ΡΡŽΡ‚ Π½Π°ΠΌ Π²Ρ‹ΠΏΠΎΠ»Π½ΡΡ‚ΡŒ Π²Π΅Π±-Π°Π½Π°Π»ΠΈΡ‚ΠΈΠΊΡƒ ΠΈΠ»ΠΈ Π΄Ρ€ΡƒΠ³ΠΈΠ΅ Ρ„ΠΎΡ€ΠΌΡ‹ измСрСния Π°ΡƒΠ΄ΠΈΡ‚ΠΎΡ€ΠΈΠΈ, Ρ‚Π°ΠΊΠΈΠ΅ ΠΊΠ°ΠΊ распознаваниС ΠΈ подсчСт количСства посСтитСлСй ΠΈ наблюдСниС Π·Π° Ρ‚Π΅ΠΌ, ΠΊΠ°ΠΊ посСтитСли ΠΏΠ΅Ρ€Π΅ΠΌΠ΅Ρ‰Π°ΡŽΡ‚ΡΡ ΠΏΠΎ Π½Π°ΡˆΠ΅ΠΌΡƒ Π²Π΅Π±-сайту. Π­Ρ‚ΠΎ ΠΏΠΎΠΌΠΎΠ³Π°Π΅Ρ‚ Π½Π°ΠΌ ΡƒΠ»ΡƒΡ‡ΡˆΠΈΡ‚ΡŒ Ρ€Π°Π±ΠΎΡ‚Ρƒ Π²Π΅Π±-сайта, Π½Π°ΠΏΡ€ΠΈΠΌΠ΅Ρ€, Π·Π° счСт Ρ‚ΠΎΠ³ΠΎ, Ρ‡Ρ‚ΠΎ ΠΏΠΎΠ»ΡŒΠ·ΠΎΠ²Π°Ρ‚Π΅Π»ΠΈ Π»Π΅Π³ΠΊΠΎ находят Ρ‚ΠΎ, Ρ‡Ρ‚ΠΎ ΠΈΡ‰ΡƒΡ‚.
          Π€ΡƒΠ½ΠΊΡ†ΠΈΠΎΠ½Π°Π»ΡŒΠ½Ρ‹Π΅ Ρ„Π°ΠΉΠ»Ρ‹ cookie:
          Π­Ρ‚ΠΈ Ρ„Π°ΠΉΠ»Ρ‹ cookie ΠΈΡΠΏΠΎΠ»ΡŒΠ·ΡƒΡŽΡ‚ΡΡ для распознавания вас, ΠΊΠΎΠ³Π΄Π° Π²Ρ‹ Π²ΠΎΠ·Π²Ρ€Π°Ρ‰Π°Π΅Ρ‚Π΅ΡΡŒ Π½Π° наш Π²Π΅Π±-сайт.Π­Ρ‚ΠΎ позволяСт Π½Π°ΠΌ ΠΏΠ΅Ρ€ΡΠΎΠ½Π°Π»ΠΈΠ·ΠΈΡ€ΠΎΠ²Π°Ρ‚ΡŒ наш ΠΊΠΎΠ½Ρ‚Π΅Π½Ρ‚ для вас, ΠΏΡ€ΠΈΠ²Π΅Ρ‚ΡΡ‚Π²ΠΎΠ²Π°Ρ‚ΡŒ вас ΠΏΠΎ ΠΈΠΌΠ΅Π½ΠΈ ΠΈ Π·Π°ΠΏΠΎΠΌΠΈΠ½Π°Ρ‚ΡŒ ваши прСдпочтСния (Π½Π°ΠΏΡ€ΠΈΠΌΠ΅Ρ€, ваш Π²Ρ‹Π±ΠΎΡ€ языка ΠΈΠ»ΠΈ Ρ€Π΅Π³ΠΈΠΎΠ½Π°). ΠŸΠΎΡ‚Π΅Ρ€Ρ ΠΈΠ½Ρ„ΠΎΡ€ΠΌΠ°Ρ†ΠΈΠΈ Π² этих Ρ„Π°ΠΉΠ»Π°Ρ… cookie ΠΌΠΎΠΆΠ΅Ρ‚ ΡΠ΄Π΅Π»Π°Ρ‚ΡŒ наши слуТбы ΠΌΠ΅Π½Π΅Π΅ Ρ„ΡƒΠ½ΠΊΡ†ΠΈΠΎΠ½Π°Π»ΡŒΠ½Ρ‹ΠΌΠΈ, Π½ΠΎ Π½Π΅ ΠΏΠΎΠΌΠ΅ΡˆΠ°Π΅Ρ‚ Ρ€Π°Π±ΠΎΡ‚Π΅ Π²Π΅Π±-сайта.
          Π¦Π΅Π»Π΅Π²Ρ‹Π΅ / ΠΏΡ€ΠΎΡ„ΠΈΠ»ΠΈΡ€ΡƒΡŽΡ‰ΠΈΠ΅ Ρ„Π°ΠΉΠ»Ρ‹ cookie:
          Π­Ρ‚ΠΈ Ρ„Π°ΠΉΠ»Ρ‹ cookie Π·Π°ΠΏΠΈΡΡ‹Π²Π°ΡŽΡ‚ вашС посСщСниС нашСго Π²Π΅Π±-сайта ΠΈ / ΠΈΠ»ΠΈ использованиС Π²Π°ΠΌΠΈ услуг, страницы, ΠΊΠΎΡ‚ΠΎΡ€Ρ‹Π΅ Π²Ρ‹ посСтили, ΠΈ ссылки, ΠΏΠΎ ΠΊΠΎΡ‚ΠΎΡ€Ρ‹ΠΌ Π²Ρ‹ ΠΏΠ΅Ρ€Π΅Ρ…ΠΎΠ΄ΠΈΠ»ΠΈ. ΠœΡ‹ Π±ΡƒΠ΄Π΅ΠΌ ΠΈΡΠΏΠΎΠ»ΡŒΠ·ΠΎΠ²Π°Ρ‚ΡŒ эту ΠΈΠ½Ρ„ΠΎΡ€ΠΌΠ°Ρ†ΠΈΡŽ, Ρ‡Ρ‚ΠΎΠ±Ρ‹ ΡΠ΄Π΅Π»Π°Ρ‚ΡŒ Π²Π΅Π±-сайт ΠΈ ΠΎΡ‚ΠΎΠ±Ρ€Π°ΠΆΠ°Π΅ΠΌΡƒΡŽ Π½Π° Π½Π΅ΠΌ Ρ€Π΅ΠΊΠ»Π°ΠΌΡƒ Π±ΠΎΠ»Π΅Π΅ ΡΠΎΠΎΡ‚Π²Π΅Ρ‚ΡΡ‚Π²ΡƒΡŽΡ‰ΠΈΠΌΠΈ вашим интСрСсам.ΠœΡ‹ Ρ‚Π°ΠΊΠΆΠ΅ ΠΌΠΎΠΆΠ΅ΠΌ ΠΏΠ΅Ρ€Π΅Π΄Π°Π²Π°Ρ‚ΡŒ эту ΠΈΠ½Ρ„ΠΎΡ€ΠΌΠ°Ρ†ΠΈΡŽ Ρ‚Ρ€Π΅Ρ‚ΡŒΠΈΠΌ Π»ΠΈΡ†Π°ΠΌ с этой Ρ†Π΅Π»ΡŒΡŽ.
          ΠŸΠ΅Ρ‡Π΅Π½ΡŒΠ΅ отклонСния Π¦Π΅ΠΏΠΈ постоянного Ρ‚ΠΎΠΊΠ° сСрии

          ΠŸΠΎΡΠ»Π΅Π΄ΠΎΠ²Π°Ρ‚Π΅Π»ΡŒΠ½Π°Ρ Ρ†Π΅ΠΏΡŒ — это самая простая элСктричСская Ρ†Π΅ΠΏΡŒ, которая Π΄Π°Π΅Ρ‚ Ρ…ΠΎΡ€ΠΎΡˆΠ΅Π΅ Π²Π²Π΅Π΄Π΅Π½ΠΈΠ΅ Π² Π±Π°Π·ΠΎΠ²Ρ‹ΠΉ Π°Π½Π°Π»ΠΈΠ· Ρ†Π΅ΠΏΠ΅ΠΉ. ΠŸΠΎΡΠ»Π΅Π΄ΠΎΠ²Π°Ρ‚Π΅Π»ΡŒΠ½Π°Ρ схСма прСдставляСт собой ΠΏΠ΅Ρ€Π²Ρ‹ΠΉ ΡΡ‚Ρ€ΠΎΠΈΡ‚Π΅Π»ΡŒΠ½Ρ‹ΠΉ Π±Π»ΠΎΠΊ для всСх схСм, ΠΊΠΎΡ‚ΠΎΡ€Ρ‹Π΅ Π½Π΅ΠΎΠ±Ρ…ΠΎΠ΄ΠΈΠΌΠΎ ΠΈΠ·ΡƒΡ‡ΠΈΡ‚ΡŒ ΠΈ ΠΏΡ€ΠΎΠ°Π½Π°Π»ΠΈΠ·ΠΈΡ€ΠΎΠ²Π°Ρ‚ΡŒ. На рис. 12-81 ΠΏΠΎΠΊΠ°Π·Π°Π½Π° эта простая схСма, Π² ΠΊΠΎΡ‚ΠΎΡ€ΠΎΠΉ Π½Π΅ Π±ΠΎΠ»Π΅Π΅ Ρ‡Π΅ΠΌ источника напряТСния ΠΈΠ»ΠΈ Π±Π°Ρ‚Π°Ρ€Π΅ΠΈ, ΠΏΡ€ΠΎΠ²ΠΎΠ΄Π½ΠΈΠΊΠ° ΠΈ рСзистора. Π­Ρ‚ΠΎ классифицируСтся ΠΊΠ°ΠΊ ΠΏΠΎΡΠ»Π΅Π΄ΠΎΠ²Π°Ρ‚Π΅Π»ΡŒΠ½Π°Ρ Ρ†Π΅ΠΏΡŒ, ΠΏΠΎΡ‚ΠΎΠΌΡƒ Ρ‡Ρ‚ΠΎ ΠΊΠΎΠΌΠΏΠΎΠ½Π΅Π½Ρ‚Ρ‹ соСдинСны встык, Ρ‚Π°ΠΊ Ρ‡Ρ‚ΠΎ ΠΎΠ΄ΠΈΠ½Π°ΠΊΠΎΠ²Ρ‹ΠΉ Ρ‚ΠΎΠΊ Ρ‚Π΅Ρ‡Π΅Ρ‚ Ρ‡Π΅Ρ€Π΅Π· ΠΊΠ°ΠΆΠ΄Ρ‹ΠΉ ΠΊΠΎΠΌΠΏΠΎΠ½Π΅Π½Ρ‚ ΠΎΠ΄ΠΈΠ½Π°ΠΊΠΎΠ²ΠΎ.Π•ΡΡ‚ΡŒ Ρ‚ΠΎΠ»ΡŒΠΊΠΎ ΠΎΠ΄ΠΈΠ½ ΠΏΡƒΡ‚ΡŒ для прохоТдСния Ρ‚ΠΎΠΊΠ°, Π° батарСя ΠΈ рСзистор Π²ΠΊΠ»ΡŽΡ‡Π΅Π½Ρ‹ ΠΏΠΎΡΠ»Π΅Π΄ΠΎΠ²Π°Ρ‚Π΅Π»ΡŒΠ½ΠΎ Π΄Ρ€ΡƒΠ³ с Π΄Ρ€ΡƒΠ³ΠΎΠΌ. Π”Π°Π»Π΅Π΅ слСдуСт внСсти нСсколько Π΄ΠΎΠΏΠΎΠ»Π½Π΅Π½ΠΈΠΉ Π² ΠΏΡ€ΠΎΡΡ‚ΡƒΡŽ схСму, ΠΏΠΎΠΊΠ°Π·Π°Π½Π½ΡƒΡŽ Π½Π° рис. 12-81.

          Рисунок 12-81. ΠŸΡ€ΠΎΡΡ‚Π°Ρ Ρ†Π΅ΠΏΡŒ постоянного Ρ‚ΠΎΠΊΠ°.

          На РисункС 12-82 ΠΏΠΎΠΊΠ°Π·Π°Π½ Π΄ΠΎΠΏΠΎΠ»Π½ΠΈΡ‚Π΅Π»ΡŒΠ½Ρ‹ΠΉ рСзистор ΠΈ Π½Π΅ΠΌΠ½ΠΎΠ³ΠΎ ΠΏΠΎΠ΄Ρ€ΠΎΠ±Π½Π΅Π΅ ΠΎ Π΅Π³ΠΎ значСниях. Π‘ этими значСниями ΠΌΡ‹ Ρ‚Π΅ΠΏΠ΅Ρ€ΡŒ ΠΌΠΎΠΆΠ΅ΠΌ Π½Π°Ρ‡Π°Ρ‚ΡŒ большС ΡƒΠ·Π½Π°Π²Π°Ρ‚ΡŒ ΠΎ ΠΏΡ€ΠΈΡ€ΠΎΠ΄Π΅ схСмы. Π’ этой ΠΊΠΎΠ½Ρ„ΠΈΠ³ΡƒΡ€Π°Ρ†ΠΈΠΈ имССтся 12-Π²ΠΎΠ»ΡŒΡ‚ΠΎΠ²Ρ‹ΠΉ источник постоянного Ρ‚ΠΎΠΊΠ°, соСдинСнный ΠΏΠΎΡΠ»Π΅Π΄ΠΎΠ²Π°Ρ‚Π΅Π»ΡŒΠ½ΠΎ с двумя рСзисторами: R 1 = 10 Ом ΠΈ R 2 = 30 Ом.Для рСзисторов Π² ΠΏΠΎΡΠ»Π΅Π΄ΠΎΠ²Π°Ρ‚Π΅Π»ΡŒΠ½ΠΎΠΉ ΠΊΠΎΠ½Ρ„ΠΈΠ³ΡƒΡ€Π°Ρ†ΠΈΠΈ ΠΎΠ±Ρ‰Π΅Π΅ сопротивлСниС Ρ†Π΅ΠΏΠΈ Ρ€Π°Π²Π½ΠΎ суммС ΠΎΡ‚Π΄Π΅Π»ΡŒΠ½Ρ‹Ρ… рСзисторов.

          Рисунок 12-82. ΠŸΡ€ΠΎΡΡ‚Π°Ρ схСма постоянного Ρ‚ΠΎΠΊΠ° с Π΄ΠΎΠΏΠΎΠ»Π½ΠΈΡ‚Π΅Π»ΡŒΠ½Ρ‹ΠΌ рСзистором.

          Основная Ρ„ΠΎΡ€ΠΌΡƒΠ»Π°:

          Для рисунка 12-82 это Π±ΡƒΠ΄Π΅Ρ‚:

          Π’Π΅ΠΏΠ΅Ρ€ΡŒ, ΠΊΠΎΠ³Π΄Π° извСстно ΠΏΠΎΠ»Π½ΠΎΠ΅ сопротивлСниС Ρ†Π΅ΠΏΠΈ, ΠΌΠΎΠΆΠ½ΠΎ ΠΎΠΏΡ€Π΅Π΄Π΅Π»ΠΈΡ‚ΡŒ Ρ‚ΠΎΠΊ Ρ†Π΅ΠΏΠΈ. Π’ ΠΏΠΎΡΠ»Π΅Π΄ΠΎΠ²Π°Ρ‚Π΅Π»ΡŒΠ½ΠΎΠΉ Ρ†Π΅ΠΏΠΈ Ρ‚ΠΎΠΊ Π½Π΅ ΠΌΠΎΠΆΠ΅Ρ‚ Π±Ρ‹Ρ‚ΡŒ Ρ€Π°Π·Π½Ρ‹ΠΌ Π² Ρ€Π°Π·Π½Ρ‹Ρ… Ρ‚ΠΎΡ‡ΠΊΠ°Ρ… Ρ†Π΅ΠΏΠΈ. Π’ΠΎΠΊ Π² ΠΏΠΎΡΠ»Π΅Π΄ΠΎΠ²Π°Ρ‚Π΅Π»ΡŒΠ½ΠΎΠΉ Ρ†Π΅ΠΏΠΈ всСгда ΠΎΠ΄ΠΈΠ½Π°ΠΊΠΎΠ² Π² ΠΊΠ°ΠΆΠ΄ΠΎΠΌ элСмСнтС ΠΈ Π² любой Ρ‚ΠΎΡ‡ΠΊΠ΅.Π‘Π»Π΅Π΄ΠΎΠ²Π°Ρ‚Π΅Π»ΡŒΠ½ΠΎ, Ρ‚ΠΎΠΊ Π² простой Ρ†Π΅ΠΏΠΈ Ρ‚Π΅ΠΏΠ΅Ρ€ΡŒ ΠΌΠΎΠΆΠ½ΠΎ ΠΎΠΏΡ€Π΅Π΄Π΅Π»ΠΈΡ‚ΡŒ с ΠΏΠΎΠΌΠΎΡ‰ΡŒΡŽ Π·Π°ΠΊΠΎΠ½Π° Ома:

          Π—Π°ΠΊΠΎΠ½ Ома описываСт Π»ΠΈΠ½Π΅ΠΉΠ½ΡƒΡŽ взаимосвязь ΠΌΠ΅ΠΆΠ΄Ρƒ ΠΏΠ΅Ρ€Π΅ΠΌΠ΅Π½Π½Ρ‹ΠΌΠΈ напряТСния, Ρ‚ΠΎΠΊΠ° ΠΈ сопротивлСния, ΠΊΠΎΡ‚ΠΎΡ€ΡƒΡŽ Π»Π΅Π³ΠΊΠΎ ΠΏΡ€ΠΎΠΈΠ»Π»ΡŽΡΡ‚Ρ€ΠΈΡ€ΠΎΠ²Π°Ρ‚ΡŒ с ΠΏΠΎΠΌΠΎΡ‰ΡŒΡŽ Π½Π΅ΡΠΊΠΎΠ»ΡŒΠΊΠΈΡ… Π΄ΠΎΠΏΠΎΠ»Π½ΠΈΡ‚Π΅Π»ΡŒΠ½Ρ‹Ρ… вычислСний. ΠŸΠ΅Ρ€Π²Ρ‹ΠΉ — это ΠΈΠ·ΠΌΠ΅Π½Π΅Π½ΠΈΠ΅ ΠΎΠ±Ρ‰Π΅Π³ΠΎ сопротивлСния Ρ†Π΅ΠΏΠΈ, Π² Ρ‚ΠΎ врСмя ΠΊΠ°ΠΊ Π΄Π²Π° Π΄Ρ€ΡƒΠ³ΠΈΡ… ΠΎΡΡ‚Π°ΡŽΡ‚ΡΡ постоянными. Π’ этом ΠΏΡ€ΠΈΠΌΠ΅Ρ€Π΅ R T схСмы Π½Π° РисункС 12-82 удваиваСтся.

          Рисунок 12-82. ΠŸΡ€ΠΎΡΡ‚Π°Ρ схСма постоянного Ρ‚ΠΎΠΊΠ° с Π΄ΠΎΠΏΠΎΠ»Π½ΠΈΡ‚Π΅Π»ΡŒΠ½Ρ‹ΠΌ рСзистором.

          ВлияниС Π½Π° ΠΎΠ±Ρ‰ΠΈΠΉ Ρ‚ΠΎΠΊ Π² Ρ†Π΅ΠΏΠΈ ΡΠ»Π΅Π΄ΡƒΡŽΡ‰Π΅Π΅:

          ΠšΠΎΠ»ΠΈΡ‡Π΅ΡΡ‚Π²Π΅Π½Π½ΠΎ ΠΈ ΠΈΠ½Ρ‚ΡƒΠΈΡ‚ΠΈΠ²Π½ΠΎ Π²ΠΈΠ΄Π½ΠΎ, Ρ‡Ρ‚ΠΎ ΠΊΠΎΠ³Π΄Π° сопротивлСниС Ρ†Π΅ΠΏΠΈ удваиваСтся, Ρ‚ΠΎΠΊ ΡƒΠΌΠ΅Π½ΡŒΡˆΠ°Π΅Ρ‚ΡΡ Π²Π΄Π²ΠΎΠ΅ ΠΏΠΎ ΡΡ€Π°Π²Π½Π΅Π½ΠΈΡŽ с исходным Π·Π½Π°Ρ‡Π΅Π½ΠΈΠ΅ΠΌ.

          Π—Π°Ρ‚Π΅ΠΌ ΡƒΠΌΠ΅Π½ΡŒΡˆΠΈΡ‚Π΅ R T схСмы Π½Π° РисункС 12-82 Π΄ΠΎ ΠΏΠΎΠ»ΠΎΠ²ΠΈΠ½Ρ‹ Π΅Π³ΠΎ ΠΏΠ΅Ρ€Π²ΠΎΠ½Π°Ρ‡Π°Π»ΡŒΠ½ΠΎΠ³ΠΎ значСния. ВлияниС Π½Π° ΠΎΠ±Ρ‰ΠΈΠΉ Ρ‚ΠΎΠΊ:

          Рисунок 12-82. ΠŸΡ€ΠΎΡΡ‚Π°Ρ схСма постоянного Ρ‚ΠΎΠΊΠ° с Π΄ΠΎΠΏΠΎΠ»Π½ΠΈΡ‚Π΅Π»ΡŒΠ½Ρ‹ΠΌ рСзистором.

          ПадСния напряТСния ΠΈ дальнСйшСС ΠΏΡ€ΠΈΠΌΠ΅Π½Π΅Π½ΠΈΠ΅ Π·Π°ΠΊΠΎΠ½Π° Ома

          ΠŸΡ€ΠΈΠΌΠ΅Ρ€ схСмы Π½Π° рисункС 12-83 ΠΈΡΠΏΠΎΠ»ΡŒΠ·ΡƒΠ΅Ρ‚ΡΡ для ΠΈΠ»Π»ΡŽΡΡ‚Ρ€Π°Ρ†ΠΈΠΈ ΠΈΠ΄Π΅ΠΈ падСния напряТСния.ΠŸΡ€ΠΈ обсуТдСнии ΠΏΠΎΡΠ»Π΅Π΄ΠΎΠ²Π°Ρ‚Π΅Π»ΡŒΠ½Ρ‹Ρ… Ρ†Π΅ΠΏΠ΅ΠΉ Π²Π°ΠΆΠ½ΠΎ Ρ€Π°Π·Π»ΠΈΡ‡Π°Ρ‚ΡŒ напряТСниС ΠΈ ΠΏΠ°Π΄Π΅Π½ΠΈΠ΅ напряТСния. ПадСниС напряТСния относится ΠΊ ΠΏΠΎΡ‚Π΅Ρ€Π΅ элСктричСского давлСния ΠΈΠ»ΠΈ Π­Π”Π‘, Π²Ρ‹Π·Π²Π°Π½Π½ΠΎΠΉ ΠΏΡ€ΠΎΡ‚Π°Π»ΠΊΠΈΠ²Π°Π½ΠΈΠ΅ΠΌ элСктронов Ρ‡Π΅Ρ€Π΅Π· рСзистор. ΠŸΠΎΡΠΊΠΎΠ»ΡŒΠΊΡƒ Π² ΠΏΡ€ΠΈΠΌΠ΅Ρ€Π΅ ΠΈΡΠΏΠΎΠ»ΡŒΠ·ΡƒΡŽΡ‚ΡΡ Π΄Π²Π° рСзистора, Π²ΠΎΠ·Π½ΠΈΠΊΠ°ΡŽΡ‚ ΠΎΡ‚Π΄Π΅Π»ΡŒΠ½Ρ‹Π΅ падСния напряТСния. КаТдая капля связана с ΠΊΠ°ΠΆΠ΄Ρ‹ΠΌ ΠΎΡ‚Π΄Π΅Π»ΡŒΠ½Ρ‹ΠΌ рСзистором. Π’Π΅Π»ΠΈΡ‡ΠΈΠ½Π° элСктричСского давлСния, Π½Π΅ΠΎΠ±Ρ…ΠΎΠ΄ΠΈΠΌΠΎΠ³ΠΎ для Ρ‚ΠΎΠ³ΠΎ, Ρ‡Ρ‚ΠΎΠ±Ρ‹ Π·Π°ΡΡ‚Π°Π²ΠΈΡ‚ΡŒ ΠΎΠΏΡ€Π΅Π΄Π΅Π»Π΅Π½Π½ΠΎΠ΅ количСство элСктронов ΠΏΡ€ΠΎΠΉΡ‚ΠΈ Ρ‡Π΅Ρ€Π΅Π· сопротивлСниС, ΠΏΡ€ΠΎΠΏΠΎΡ€Ρ†ΠΈΠΎΠ½Π°Π»ΡŒΠ½Π° Ρ€Π°Π·ΠΌΠ΅Ρ€Ρƒ рСзистора.

          Рисунок 12-83.ΠŸΡ€ΠΈΠΌΠ΅Ρ€ Ρ‚Ρ€Π΅Ρ… ΠΏΠΎΡΠ»Π΅Π΄ΠΎΠ²Π°Ρ‚Π΅Π»ΡŒΠ½ΠΎ соСдинСнных рСзисторов.

          На рис. 12-83 для ΠΈΠ»Π»ΡŽΡΡ‚Ρ€Π°Ρ†ΠΈΠΈ падСния напряТСния ΠΈΡΠΏΠΎΠ»ΡŒΠ·ΡƒΡŽΡ‚ΡΡ ΡΠ»Π΅Π΄ΡƒΡŽΡ‰ΠΈΠ΅ значСния:

          ПадСниС напряТСния Π½Π° ΠΊΠ°ΠΆΠ΄ΠΎΠΌ рСзисторС рассчитываСтся с использованиСм Π·Π°ΠΊΠΎΠ½Π° Ома. ПадСниС для ΠΊΠ°ΠΆΠ΄ΠΎΠ³ΠΎ рСзистора являСтся ΠΏΡ€ΠΎΠΈΠ·Π²Π΅Π΄Π΅Π½ΠΈΠ΅ΠΌ ΠΊΠ°ΠΆΠ΄ΠΎΠ³ΠΎ сопротивлСния ΠΈ ΠΎΠ±Ρ‰Π΅Π³ΠΎ Ρ‚ΠΎΠΊΠ° Π² Ρ†Π΅ΠΏΠΈ. Π˜ΠΌΠ΅ΠΉΡ‚Π΅ Π² Π²ΠΈΠ΄Ρƒ, Ρ‡Ρ‚ΠΎ Ρ‡Π΅Ρ€Π΅Π· ΠΏΠΎΡΠ»Π΅Π΄ΠΎΠ²Π°Ρ‚Π΅Π»ΡŒΠ½Ρ‹ΠΉ рСзистор ΠΏΡ€ΠΎΡ‚Π΅ΠΊΠ°Π΅Ρ‚ Ρ‚ΠΎΡ‚ ΠΆΠ΅ Ρ‚ΠΎΠΊ.

          Π’Π΅ΠΏΠ΅Ρ€ΡŒ ΠΌΠΎΠΆΠ½ΠΎ ΠΎΠΏΡ€Π΅Π΄Π΅Π»ΠΈΡ‚ΡŒ напряТСниС источника, ΠΊΠΎΡ‚ΠΎΡ€ΠΎΠ΅ Π·Π°Ρ‚Π΅ΠΌ ΠΌΠΎΠΆΠ½ΠΎ ΠΈΡΠΏΠΎΠ»ΡŒΠ·ΠΎΠ²Π°Ρ‚ΡŒ для подтвСрТдСния расчСтов для ΠΊΠ°ΠΆΠ΄ΠΎΠ³ΠΎ падСния напряТСния. ИспользованиС Π·Π°ΠΊΠΎΠ½Π° Ома:

          ΠŸΡ€ΠΎΡΡ‚Ρ‹Π΅ ΠΏΡ€ΠΎΠ²Π΅Ρ€ΠΊΠΈ для подтвСрТдСния расчСта ΠΈ ΠΈΠ»Π»ΡŽΡΡ‚Ρ€Π°Ρ†ΠΈΠΈ ΠΊΠΎΠ½Ρ†Π΅ΠΏΡ†ΠΈΠΈ падСния напряТСния ΡΠΊΠ»Π°Π΄Ρ‹Π²Π°ΡŽΡ‚ ΠΎΡ‚Π΄Π΅Π»ΡŒΠ½Ρ‹Π΅ значСния ΠΏΠ°Π΄Π΅Π½ΠΈΠΉ напряТСния ΠΈ ΡΡ€Π°Π²Π½ΠΈΠ²Π°ΡŽΡ‚ ΠΈΡ… с Ρ€Π΅Π·ΡƒΠ»ΡŒΡ‚Π°Ρ‚Π°ΠΌΠΈ Π²Ρ‹ΡˆΠ΅ΡƒΠΊΠ°Π·Π°Π½Π½ΠΎΠ³ΠΎ расчСта.

          1 Π’ + 3 Π’ + 5 Π’ = 9 Π’

          Π˜ΡΡ‚ΠΎΡ‡Π½ΠΈΠΊΠΈ напряТСния сСрии

          Π˜ΡΡ‚ΠΎΡ‡Π½ΠΈΠΊ напряТСния — это источник энСргии, ΠΊΠΎΡ‚ΠΎΡ€Ρ‹ΠΉ обСспСчиваСт постоянноС напряТСниС Π½Π° Π½Π°Π³Ρ€ΡƒΠ·ΠΊΠ΅. Π”Π²Π° ΠΈΠ»ΠΈ Π±ΠΎΠ»Π΅Π΅ ΠΈΠ· этих источников, соСдинСнных ΠΏΠΎΡΠ»Π΅Π΄ΠΎΠ²Π°Ρ‚Π΅Π»ΡŒΠ½ΠΎ, Ρ€Π°Π²Π½Ρ‹ алгСбраичСской суммС всСх ΠΏΠΎΡΠ»Π΅Π΄ΠΎΠ²Π°Ρ‚Π΅Π»ΡŒΠ½ΠΎ соСдинСнных источников. Π—Π½Π°Ρ‡Π΅Π½ΠΈΠ΅ указания алгСбраичСской суммы состоит Π² Ρ‚ΠΎΠΌ, Ρ‡Ρ‚ΠΎΠ±Ρ‹ ΡƒΠΊΠ°Π·Π°Ρ‚ΡŒ, Ρ‡Ρ‚ΠΎ ΠΏΠΎΠ»ΡΡ€Π½ΠΎΡΡ‚ΡŒ источников Π΄ΠΎΠ»ΠΆΠ½Π° ΡƒΡ‡ΠΈΡ‚Ρ‹Π²Π°Ρ‚ΡŒΡΡ ΠΏΡ€ΠΈ суммировании источников. ΠŸΠΎΠ»ΡΡ€Π½ΠΎΡΡ‚ΡŒ обозначаСтся Π·Π½Π°ΠΊΠΎΠΌ плюс ΠΈΠ»ΠΈ минус Π² зависимости ΠΎΡ‚ полоТСния источника Π² Ρ†Π΅ΠΏΠΈ.

          На РисункС 12-84 всС источники располоТСны Π² ΠΎΠ΄Π½ΠΎΠΌ Π½Π°ΠΏΡ€Π°Π²Π»Π΅Π½ΠΈΠΈ с Ρ‚ΠΎΡ‡ΠΊΠΈ зрСния ΠΈΡ… полярности. ΠŸΡ€ΠΈ слоТСнии всС напряТСния ΠΈΠΌΠ΅ΡŽΡ‚ ΠΎΠ΄ΠΈΠ½ ΠΈ Ρ‚ΠΎΡ‚ ΠΆΠ΅ Π·Π½Π°ΠΊ. Π’ случаС, ΠΏΠΎΠΊΠ°Π·Π°Π½Π½ΠΎΠΌ Π½Π° РисункС 12-84, Ρ‚Ρ€ΠΈ ячСйки напряТСниСм 1,5 Π’ Π²ΠΊΠ»ΡŽΡ‡Π΅Π½Ρ‹ ΠΏΠΎΡΠ»Π΅Π΄ΠΎΠ²Π°Ρ‚Π΅Π»ΡŒΠ½ΠΎ с ΠΏΠΎΠ»ΡΡ€Π½ΠΎΡΡ‚ΡŒΡŽ Π² ΠΎΠ΄Π½ΠΎΠΌ Π½Π°ΠΏΡ€Π°Π²Π»Π΅Π½ΠΈΠΈ.

          Рисунок 12-84. ΠŸΠΎΡΠ»Π΅Π΄ΠΎΠ²Π°Ρ‚Π΅Π»ΡŒΠ½Ρ‹Π΅ источники напряТСния ΡΠΊΠ»Π°Π΄Ρ‹Π²Π°ΡŽΡ‚ΡΡ алгСбраичСски.

          Π”ΠΎΠ±Π°Π²ΠΈΡ‚ΡŒ достаточно просто:

          Однако, Π½Π° рис. 12-85, ΠΎΠ΄ΠΈΠ½ ΠΈΠ· Ρ‚Ρ€Π΅Ρ… источников ΠΏΠ΅Ρ€Π΅Π²Π΅Ρ€Π½ΡƒΡ‚, ΠΈ ΠΏΠΎΠ»ΡΡ€Π½ΠΎΡΡ‚ΡŒ ΠΏΡ€ΠΎΡ‚ΠΈΠ²ΠΎΠΏΠΎΠ»ΠΎΠΆΠ½Π° Π΄Π²ΡƒΠΌ Π΄Ρ€ΡƒΠ³ΠΈΠΌ источникам.

          Рисунок 12-85. Π˜ΡΡ‚ΠΎΡ‡Π½ΠΈΠΊΠΈ напряТСния ΡΠΊΠ»Π°Π΄Ρ‹Π²Π°ΡŽΡ‚ΡΡ алгСбраичСски; ΠΎΠ΄ΠΈΠ½ источник ΠΏΠ΅Ρ€Π΅Π²Π΅Ρ€Π½ΡƒΡ‚.

          ΠžΠΏΡΡ‚ΡŒ ΠΆΠ΅, слоТСниС простоС:

          Π—Π°ΠΊΠΎΠ½ напряТСния ΠšΠΈΡ€Ρ…Π³ΠΎΡ„Π°

          Π—Π°ΠΊΠΎΠ½ напряТСния ΠšΠΈΡ€Ρ…Π³ΠΎΡ„Π°, ΠΈΠΌΠ΅ΡŽΡ‰ΠΈΠΉ ΠΏΡ€ΠΈΠ½Ρ†ΠΈΠΏΠΈΠ°Π»ΡŒΠ½ΠΎΠ΅ Π·Π½Π°Ρ‡Π΅Π½ΠΈΠ΅ для Π°Π½Π°Π»ΠΈΠ·Π° элСктричСской Ρ†Π΅ΠΏΠΈ, являСтся. Π­Ρ‚ΠΎΡ‚ Π·Π°ΠΊΠΎΠ½ просто ΡƒΡ‚Π²Π΅Ρ€ΠΆΠ΄Π°Π΅Ρ‚, Ρ‡Ρ‚ΠΎ алгСбраичСская сумма всСх напряТСний Π½Π° Π·Π°ΠΌΠΊΠ½ΡƒΡ‚ΠΎΠΌ ΠΏΡƒΡ‚ΠΈ ΠΈΠ»ΠΈ ΠΊΠΎΠ½Ρ‚ΡƒΡ€Π΅ Ρ€Π°Π²Π½Π° Π½ΡƒΠ»ΡŽ. Π”Ρ€ΡƒΠ³ΠΈΠΌΠΈ словами: сумма всСх ΠΏΠ°Π΄Π΅Π½ΠΈΠΉ напряТСния Ρ€Π°Π²Π½Π° ΠΎΠ±Ρ‰Π΅ΠΌΡƒ Π½Π°ΠΏΡ€ΡΠΆΠ΅Π½ΠΈΡŽ источника. УпрощСнная Ρ„ΠΎΡ€ΠΌΡƒΠ»Π°, ΠΏΠΎΠΊΠ°Π·Ρ‹Π²Π°ΡŽΡ‰Π°Ρ этот Π·Π°ΠΊΠΎΠ½, ΠΏΠΎΠΊΠ°Π·Π°Π½Π° Π½ΠΈΠΆΠ΅:

          ΠžΠ±Ρ€Π°Ρ‚ΠΈΡ‚Π΅ Π²Π½ΠΈΠΌΠ°Π½ΠΈΠ΅, Ρ‡Ρ‚ΠΎ Π·Π½Π°ΠΊ источника ΠΏΡ€ΠΎΡ‚ΠΈΠ²ΠΎΠΏΠΎΠ»ΠΎΠΆΠ΅Π½ Π·Π½Π°ΠΊΡƒ ΠΎΡ‚Π΄Π΅Π»ΡŒΠ½Ρ‹Ρ… ΠΏΠ°Π΄Π΅Π½ΠΈΠΉ напряТСния.Π‘Π»Π΅Π΄ΠΎΠ²Π°Ρ‚Π΅Π»ΡŒΠ½ΠΎ, алгСбраичСская сумма Ρ€Π°Π²Π½Π° Π½ΡƒΠ»ΡŽ. Написано ΠΏΠΎ-Π΄Ρ€ΡƒΠ³ΠΎΠΌΡƒ:

          НапряТСниС источника Ρ€Π°Π²Π½ΠΎ суммС ΠΏΠ°Π΄Π΅Π½ΠΈΠΉ напряТСния. ΠŸΠΎΠ»ΡΡ€Π½ΠΎΡΡ‚ΡŒ падСния напряТСния опрСдСляСтся Π½Π°ΠΏΡ€Π°Π²Π»Π΅Π½ΠΈΠ΅ΠΌ Ρ‚ΠΎΠΊΠ°. ΠžΠ±Ρ…ΠΎΠ΄Ρ Ρ†Π΅ΠΏΡŒ, ΠΎΠ±Ρ€Π°Ρ‚ΠΈΡ‚Π΅ Π²Π½ΠΈΠΌΠ°Π½ΠΈΠ΅, Ρ‡Ρ‚ΠΎ ΠΏΠΎΠ»ΡΡ€Π½ΠΎΡΡ‚ΡŒ рСзистора ΠΏΡ€ΠΎΡ‚ΠΈΠ²ΠΎΠΏΠΎΠ»ΠΎΠΆΠ½Π° полярности напряТСния источника. ΠŸΠΎΠ»ΠΎΠΆΠΈΡ‚Π΅Π»ΡŒΠ½Ρ‹ΠΉ полюс рСзистора ΠΎΠ±Ρ€Π°Ρ‰Π΅Π½ ΠΊ ΠΏΠΎΠ»ΠΎΠΆΠΈΡ‚Π΅Π»ΡŒΠ½ΠΎΠΌΡƒ ΠΏΠΎΠ»ΡŽΡΡƒ источника, Π° ΠΎΡ‚Ρ€ΠΈΡ†Π°Ρ‚Π΅Π»ΡŒΠ½Ρ‹ΠΉ полюс рСзистора ΠΎΠ±Ρ€Π°Ρ‰Π΅Π½ ΠΊ ΠΎΡ‚Ρ€ΠΈΡ†Π°Ρ‚Π΅Π»ΡŒΠ½ΠΎΠΌΡƒ ΠΏΠΎΠ»ΡŽΡΡƒ источника.

          Рисунок 12-86 ΠΈΠ»Π»ΡŽΡΡ‚Ρ€ΠΈΡ€ΡƒΠ΅Ρ‚ ΡΠ°ΠΌΡƒΡŽ ΠΎΡΠ½ΠΎΠ²Π½ΡƒΡŽ идСю Π·Π°ΠΊΠΎΠ½Π° ΠšΠΈΡ€Ρ…Π³ΠΎΡ„Π° для напряТСния.Π’ этом ΠΏΡ€ΠΈΠΌΠ΅Ρ€Π΅ Π΄Π²Π° рСзистора. Π£ ΠΎΠ΄Π½ΠΎΠ³ΠΎ ΠΏΠ°Π΄Π΅Π½ΠΈΠ΅ составляСт 14 Π²ΠΎΠ»ΡŒΡ‚, Π° Ρƒ Π΄Ρ€ΡƒΠ³ΠΎΠ³ΠΎ — 10 Π²ΠΎΠ»ΡŒΡ‚. НапряТСниС источника Π΄ΠΎΠ»ΠΆΠ½ΠΎ Ρ€Π°Π²Π½ΡΡ‚ΡŒΡΡ суммС ΠΏΠ°Π΄Π΅Π½ΠΈΠΉ напряТСния Π² Ρ†Π΅ΠΏΠΈ. ΠŸΡ€ΠΈ осмотрС Π»Π΅Π³ΠΊΠΎ ΠΎΠΏΡ€Π΅Π΄Π΅Π»ΠΈΡ‚ΡŒ напряТСниС источника ΠΊΠ°ΠΊ 24 Π²ΠΎΠ»ΡŒΡ‚Π°.

          Рисунок 12-86. Π—Π°ΠΊΠΎΠ½ ΠšΠΈΡ€Ρ…Π³ΠΎΡ„Π° ΠΎ напряТСнии.

          На рис. 12-87 ΠΏΠΎΠΊΠ°Π·Π°Π½Π° ΠΏΠΎΡΠ»Π΅Π΄ΠΎΠ²Π°Ρ‚Π΅Π»ΡŒΠ½Π°Ρ Ρ†Π΅ΠΏΡŒ с трСмя падСниями напряТСния ΠΈ ΠΎΠ΄Π½ΠΈΠΌ источником напряТСния, рассчитанным Π½Π° 24 Π²ΠΎΠ»ΡŒΡ‚. Π˜Π·Π²Π΅ΡΡ‚Π½Ρ‹ Π΄Π²Π° падСния напряТСния. Однако ΠΎ Ρ‚Ρ€Π΅Ρ‚ΡŒΠ΅ΠΌ Π½ΠΈΡ‡Π΅Π³ΠΎ Π½Π΅ извСстно. Π˜ΡΠΏΠΎΠ»ΡŒΠ·ΡƒΡ Π·Π°ΠΊΠΎΠ½ ΠšΠΈΡ€Ρ…Π³ΠΎΡ„Π°, ΠΌΠΎΠΆΠ½ΠΎ ΠΎΠΏΡ€Π΅Π΄Π΅Π»ΠΈΡ‚ΡŒ Ρ‚Ρ€Π΅Ρ‚ΡŒΠ΅ ΠΏΠ°Π΄Π΅Π½ΠΈΠ΅ напряТСния.

          Рисунок 12-87. ΠžΠΏΡ€Π΅Π΄Π΅Π»ΠΈΡ‚Π΅ нСизвСстноС ΠΏΠ°Π΄Π΅Π½ΠΈΠ΅ напряТСния.

          ΠžΠΏΡ€Π΅Π΄Π΅Π»ΠΈΡ‚Π΅ Π·Π½Π°Ρ‡Π΅Π½ΠΈΠ΅ E 4 Π½Π° РисункС 12-88. Π’ этом ΠΏΡ€ΠΈΠΌΠ΅Ρ€Π΅ I = 200 мА.

          Π‘Π½Π°Ρ‡Π°Π»Π° Π½Π΅ΠΎΠ±Ρ…ΠΎΠ΄ΠΈΠΌΠΎ ΠΎΠΏΡ€Π΅Π΄Π΅Π»ΠΈΡ‚ΡŒ ΠΏΠ°Π΄Π΅Π½ΠΈΠ΅ напряТСния Π½Π° ΠΊΠ°ΠΆΠ΄ΠΎΠΌ ΠΎΡ‚Π΄Π΅Π»ΡŒΠ½ΠΎΠΌ рСзисторС.

          Π—Π°ΠΊΠΎΠ½ ΠšΠΈΡ€Ρ…Π³ΠΎΡ„Π° ΠΏΠΎ Π½Π°ΠΏΡ€ΡΠΆΠ΅Π½ΠΈΡŽ Ρ‚Π΅ΠΏΠ΅Ρ€ΡŒ ΠΈΡΠΏΠΎΠ»ΡŒΠ·ΡƒΠ΅Ρ‚ΡΡ для опрСдСлСния падСния напряТСния Π½Π° E 4 .

          Π˜ΡΠΏΠΎΠ»ΡŒΠ·ΡƒΡ Π·Π°ΠΊΠΎΠ½ Ома ΠΈ подставив Π² E 4 , Ρ‚Π΅ΠΏΠ΅Ρ€ΡŒ ΠΌΠΎΠΆΠ½ΠΎ ΠΎΠΏΡ€Π΅Π΄Π΅Π»ΠΈΡ‚ΡŒ Π·Π½Π°Ρ‡Π΅Π½ΠΈΠ΅ для R 4 .

          Π”Π΅Π»ΠΈΡ‚Π΅Π»ΠΈ напряТСния

          Π”Π΅Π»ΠΈΡ‚Π΅Π»ΠΈ напряТСния — это устройства, ΠΏΠΎΠ·Π²ΠΎΠ»ΡΡŽΡ‰ΠΈΠ΅ ΠΏΠΎΠ»ΡƒΡ‡Π°Ρ‚ΡŒ Π±ΠΎΠ»Π΅Π΅ ΠΎΠ΄Π½ΠΎΠ³ΠΎ напряТСния ΠΎΡ‚ ΠΎΠ΄Π½ΠΎΠ³ΠΎ источника питания.Π”Π΅Π»ΠΈΡ‚Π΅Π»ΡŒ напряТСния ΠΎΠ±Ρ‹Ρ‡Π½ΠΎ состоит ΠΈΠ· рСзистора ΠΈΠ»ΠΈ рСзисторов, соСдинСнных ΠΏΠΎΡΠ»Π΅Π΄ΠΎΠ²Π°Ρ‚Π΅Π»ΡŒΠ½ΠΎ, с фиксированными ΠΈΠ»ΠΈ ΠΏΠΎΠ΄Π²ΠΈΠΆΠ½Ρ‹ΠΌΠΈ ΠΊΠΎΠ½Ρ‚Π°ΠΊΡ‚Π°ΠΌΠΈ ΠΈ двумя фиксированными ΠΊΠ»Π΅ΠΌΠΌΠ½Ρ‹ΠΌΠΈ ΠΊΠΎΠ½Ρ‚Π°ΠΊΡ‚Π°ΠΌΠΈ. По ΠΌΠ΅Ρ€Π΅ протСкания Ρ‚ΠΎΠΊΠ° Ρ‡Π΅Ρ€Π΅Π· рСзистор ΠΌΠ΅ΠΆΠ΄Ρƒ ΠΊΠΎΠ½Ρ‚Π°ΠΊΡ‚Π°ΠΌΠΈ ΠΌΠΎΠ³ΡƒΡ‚ Π±Ρ‹Ρ‚ΡŒ Ρ€Π°Π·Π½Ρ‹Π΅ напряТСния.

          ΠŸΠΎΡΠ»Π΅Π΄ΠΎΠ²Π°Ρ‚Π΅Π»ΡŒΠ½Ρ‹Π΅ Ρ†Π΅ΠΏΠΈ ΠΈΡΠΏΠΎΠ»ΡŒΠ·ΡƒΡŽΡ‚ΡΡ для Π΄Π΅Π»ΠΈΡ‚Π΅Π»Π΅ΠΉ напряТСния. ΠŸΡ€Π°Π²ΠΈΠ»ΠΎ дСлитСля напряТСния позволяСт Ρ‚Π΅Ρ…Π½ΠΈΠΊΡƒ Ρ€Π°ΡΡΡ‡ΠΈΡ‚Π°Ρ‚ΡŒ напряТСниС Π½Π° ΠΎΠ΄Π½ΠΎΠΌ ΠΈΠ»ΠΈ Π½Π΅ΡΠΊΠΎΠ»ΡŒΠΊΠΈΡ… ΠΏΠΎΡΠ»Π΅Π΄ΠΎΠ²Π°Ρ‚Π΅Π»ΡŒΠ½Ρ‹Ρ… рСзисторах Π±Π΅Π· нСобходимости сначала Ρ€Π°ΡΡΡ‡ΠΈΡ‚Ρ‹Π²Π°Ρ‚ΡŒ Ρ‚ΠΎΠΊ Π² Ρ†Π΅ΠΏΠΈ. [Рисунок 12-89] ΠŸΠΎΡΠΊΠΎΠ»ΡŒΠΊΡƒ Ρ‚ΠΎΠΊ Ρ‚Π΅Ρ‡Π΅Ρ‚ Ρ‡Π΅Ρ€Π΅Π· ΠΊΠ°ΠΆΠ΄Ρ‹ΠΉ рСзистор, ΠΏΠ°Π΄Π΅Π½ΠΈΠ΅ напряТСния ΠΏΡ€ΠΎΠΏΠΎΡ€Ρ†ΠΈΠΎΠ½Π°Π»ΡŒΠ½ΠΎ омичСским значСниям ΡΠΎΡΡ‚Π°Π²Π»ΡΡŽΡ‰ΠΈΡ… рСзисторов.

          Рисунок 12-89. Π‘Ρ…Π΅ΠΌΠ° дСлитСля напряТСния.

          Π§Ρ‚ΠΎΠ±Ρ‹ ΠΏΠΎΠ½ΡΡ‚ΡŒ, ΠΊΠ°ΠΊ Ρ€Π°Π±ΠΎΡ‚Π°Π΅Ρ‚ Π΄Π΅Π»ΠΈΡ‚Π΅Π»ΡŒ напряТСния, Π²Π½ΠΈΠΌΠ°Ρ‚Π΅Π»ΡŒΠ½ΠΎ ΠΈΠ·ΡƒΡ‡ΠΈΡ‚Π΅ рисунок 12-90 ΠΈ ΠΎΠ±Ρ€Π°Ρ‚ΠΈΡ‚Π΅ Π²Π½ΠΈΠΌΠ°Π½ΠΈΠ΅ Π½Π° ΡΠ»Π΅Π΄ΡƒΡŽΡ‰Π΅Π΅:

          КаТдая Π½Π°Π³Ρ€ΡƒΠ·ΠΊΠ° потрСбляСт ΠΎΠΏΡ€Π΅Π΄Π΅Π»Π΅Π½Π½ΡƒΡŽ Π²Π΅Π»ΠΈΡ‡ΠΈΠ½Ρƒ Ρ‚ΠΎΠΊΠ°: I 1 , I 2 , I 3 . Помимо Ρ‚ΠΎΠΊΠΎΠ² Π½Π°Π³Ρ€ΡƒΠ·ΠΊΠΈ, ΠΏΡ€ΠΎΡ‚Π΅ΠΊΠ°Π΅Ρ‚ Π½Π΅ΠΊΠΎΡ‚ΠΎΡ€Ρ‹ΠΉ Ρ‚ΠΎΠΊ ΡƒΡ‚Π΅Ρ‡ΠΊΠΈ (I B ). Π’ΠΎΠΊ (I T ) бСрСтся ΠΈΠ· источника питания ΠΈ Ρ€Π°Π²Π΅Π½ суммС всСх Ρ‚ΠΎΠΊΠΎΠ².

          Рисунок 12-90. Π’ΠΈΠΏΠΎΠ²ΠΎΠΉ Π΄Π΅Π»ΠΈΡ‚Π΅Π»ΡŒ напряТСния.

          НапряТСниС Π² ΠΊΠ°ΠΆΠ΄ΠΎΠΉ Ρ‚ΠΎΡ‡ΠΊΠ΅ измСряСтся ΠΎΡ‚Π½ΠΎΡΠΈΡ‚Π΅Π»ΡŒΠ½ΠΎ ΠΎΠ±Ρ‰Π΅ΠΉ Ρ‚ΠΎΡ‡ΠΊΠΈ.ΠžΠ±Ρ€Π°Ρ‚ΠΈΡ‚Π΅ Π²Π½ΠΈΠΌΠ°Π½ΠΈΠ΅, Ρ‡Ρ‚ΠΎ общая Ρ‚ΠΎΡ‡ΠΊΠ° — это Ρ‚ΠΎΡ‡ΠΊΠ°, Π² ΠΊΠΎΡ‚ΠΎΡ€ΠΎΠΉ ΠΏΠΎΠ»Π½Ρ‹ΠΉ Ρ‚ΠΎΠΊ (I T ) дСлится Π½Π° ΠΎΡ‚Π΄Π΅Π»ΡŒΠ½Ρ‹Π΅ Ρ‚ΠΎΠΊΠΈ (I 1 , I 2 , I 3 ). Π’ ΠΊΠ°ΠΆΠ΄ΠΎΠΉ части дСлитСля напряТСния Ρ‚Π΅Ρ‡Π΅Ρ‚ Ρ€Π°Π·Π½Ρ‹ΠΉ Ρ‚ΠΎΠΊ. РаспрСдСлСниС Ρ‚ΠΎΠΊΠ° выглядит ΡΠ»Π΅Π΄ΡƒΡŽΡ‰ΠΈΠΌ ΠΎΠ±Ρ€Π°Π·ΠΎΠΌ:

          НапряТСниС Π½Π° ΠΊΠ°ΠΆΠ΄ΠΎΠΌ рСзисторС дСлитСля напряТСния составляСт:

          90 Π²ΠΎΠ»ΡŒΡ‚ Π½Π° R 1

          60 Π²ΠΎΠ»ΡŒΡ‚ Π½Π° R 2

          50 Π²ΠΎΠ»ΡŒΡ‚ Π½Π° R 3

          Π‘Ρ…Π΅ΠΌΠ° дСлитСля напряТСния, ΠΎΠ±ΡΡƒΠΆΠ΄Π°Π²ΡˆΠ°ΡΡΡ Π΄ΠΎ этого ΠΌΠΎΠΌΠ΅Π½Ρ‚Π°, ΠΈΠΌΠ΅Π»Π° ΠΎΠ΄Π½Ρƒ сторону источника питания (Π±Π°Ρ‚Π°Ρ€Π΅ΠΈ) ΠΏΠΎΠ΄ ΠΏΠΎΡ‚Π΅Π½Ρ†ΠΈΠ°Π»ΠΎΠΌ Π·Π΅ΠΌΠ»ΠΈ.На РисункС 12-91 общая ΠΊΠΎΠ½Ρ‚Ρ€ΠΎΠ»ΡŒΠ½Π°Ρ Ρ‚ΠΎΡ‡ΠΊΠ° (символ зазСмлСния) ΠΏΠ΅Ρ€Π΅ΠΌΠ΅Ρ‰Π΅Π½Π° Π² Π΄Ρ€ΡƒΠ³ΡƒΡŽ Ρ‚ΠΎΡ‡ΠΊΡƒ дСлитСля напряТСния.

          Рисунок 12-91. ΠŸΠΎΠ»ΠΎΠΆΠΈΡ‚Π΅Π»ΡŒΠ½ΠΎΠ΅ ΠΈ ΠΎΡ‚Ρ€ΠΈΡ†Π°Ρ‚Π΅Π»ΡŒΠ½ΠΎΠ΅ напряТСниС Π½Π° Π΄Π΅Π»ΠΈΡ‚Π΅Π»Π΅ напряТСния.

          ПадСниС напряТСния Π½Π° R1 составляСт 20 Π²ΠΎΠ»ΡŒΡ‚; ΠΎΠ΄Π½Π°ΠΊΠΎ, ΠΏΠΎΡΠΊΠΎΠ»ΡŒΠΊΡƒ ΠΎΡ‚Π²ΠΎΠ΄ A ΠΏΠΎΠ΄ΠΊΠ»ΡŽΡ‡Π΅Π½ ΠΊ Ρ‚ΠΎΡ‡ΠΊΠ΅ Π² Ρ†Π΅ΠΏΠΈ, ΠΈΠΌΠ΅ΡŽΡ‰Π΅ΠΉ Ρ‚ΠΎΡ‚ ΠΆΠ΅ ΠΏΠΎΡ‚Π΅Π½Ρ†ΠΈΠ°Π», Ρ‡Ρ‚ΠΎ ΠΈ ΠΎΡ‚Ρ€ΠΈΡ†Π°Ρ‚Π΅Π»ΡŒΠ½Π°Ρ сторона Π±Π°Ρ‚Π°Ρ€Π΅ΠΈ, напряТСниС ΠΌΠ΅ΠΆΠ΄Ρƒ ΠΎΡ‚Π²ΠΎΠ΄ΠΎΠΌ A ΠΈ ΠΎΠΏΠΎΡ€Π½ΠΎΠΉ Ρ‚ΠΎΡ‡ΠΊΠΎΠΉ составляСт ΠΎΡ‚Ρ€ΠΈΡ†Π°Ρ‚Π΅Π»ΡŒΠ½Ρ‹Π΅ (-) 20 Π²ΠΎΠ»ΡŒΡ‚. ΠŸΠΎΡΠΊΠΎΠ»ΡŒΠΊΡƒ рСзисторы R 2 ΠΈ R 3 ΠΏΠΎΠ΄ΠΊΠ»ΡŽΡ‡Π΅Π½Ρ‹ ΠΊ ΠΏΠΎΠ»ΠΎΠΆΠΈΡ‚Π΅Π»ΡŒΠ½ΠΎΠΉ сторонС Π±Π°Ρ‚Π°Ρ€Π΅ΠΈ, напряТСния ΠΌΠ΅ΠΆΠ΄Ρƒ ΠΎΠΏΠΎΡ€Π½ΠΎΠΉ Ρ‚ΠΎΡ‡ΠΊΠΎΠΉ ΠΈ ΠΎΡ‚Π²ΠΎΠ΄ΠΎΠΌ B ΠΈΠ»ΠΈ C ΡΠ²Π»ΡΡŽΡ‚ΡΡ ΠΏΠΎΠ»ΠΎΠΆΠΈΡ‚Π΅Π»ΡŒΠ½Ρ‹ΠΌΠΈ.

          Π‘Π»Π΅Π΄ΡƒΡŽΡ‰ΠΈΠ΅ ΠΏΡ€Π°Π²ΠΈΠ»Π° ΠΎΠ±Π΅ΡΠΏΠ΅Ρ‡ΠΈΠ²Π°ΡŽΡ‚ простой ΠΌΠ΅Ρ‚ΠΎΠ΄ опрСдСлСния ΠΎΡ‚Ρ€ΠΈΡ†Π°Ρ‚Π΅Π»ΡŒΠ½ΠΎΠ³ΠΎ ΠΈ ΠΏΠΎΠ»ΠΎΠΆΠΈΡ‚Π΅Π»ΡŒΠ½ΠΎΠ³ΠΎ напряТСния: (1) Если Ρ‚ΠΎΠΊ Π²Ρ…ΠΎΠ΄ΠΈΡ‚ Π² сопротивлСниС, Ρ‚Π΅ΠΊΡƒΡ‰Π΅Π΅ ΠΎΡ‚ ΠΊΠΎΠ½Ρ‚Ρ€ΠΎΠ»ΡŒΠ½ΠΎΠΉ Ρ‚ΠΎΡ‡ΠΊΠΈ, ΠΏΠ°Π΄Π΅Π½ΠΈΠ΅ напряТСния Π½Π° этом сопротивлСнии являСтся ΠΏΠΎΠ»ΠΎΠΆΠΈΡ‚Π΅Π»ΡŒΠ½Ρ‹ΠΌ ΠΏΠΎ ΠΎΡ‚Π½ΠΎΡˆΠ΅Π½ΠΈΡŽ ΠΊ ΠΊΠΎΠ½Ρ‚Ρ€ΠΎΠ»ΡŒΠ½ΠΎΠΉ Ρ‚ΠΎΡ‡ΠΊΠ΅; (2) Ссли Ρ‚ΠΎΠΊ Ρ‚Π΅Ρ‡Π΅Ρ‚ Ρ‡Π΅Ρ€Π΅Π· сопротивлСниС Π² Π½Π°ΠΏΡ€Π°Π²Π»Π΅Π½ΠΈΠΈ ΠΊΠΎΠ½Ρ‚Ρ€ΠΎΠ»ΡŒΠ½ΠΎΠΉ Ρ‚ΠΎΡ‡ΠΊΠΈ, ΠΏΠ°Π΄Π΅Π½ΠΈΠ΅ напряТСния Π½Π° этом сопротивлСнии Π±ΡƒΠ΄Π΅Ρ‚ ΠΎΡ‚Ρ€ΠΈΡ†Π°Ρ‚Π΅Π»ΡŒΠ½Ρ‹ΠΌ ΠΏΠΎ ΠΎΡ‚Π½ΠΎΡˆΠ΅Π½ΠΈΡŽ ΠΊ ΠΊΠΎΠ½Ρ‚Ρ€ΠΎΠ»ΡŒΠ½ΠΎΠΉ Ρ‚ΠΎΡ‡ΠΊΠ΅. РасполоТСниС ΠΊΠΎΠ½Ρ‚Ρ€ΠΎΠ»ΡŒΠ½ΠΎΠΉ Ρ‚ΠΎΡ‡ΠΊΠΈ опрСдСляСт, Π±ΡƒΠ΄Π΅Ρ‚ Π»ΠΈ напряТСниС ΠΎΡ‚Ρ€ΠΈΡ†Π°Ρ‚Π΅Π»ΡŒΠ½Ρ‹ΠΌ ΠΈΠ»ΠΈ ΠΏΠΎΠ»ΠΎΠΆΠΈΡ‚Π΅Π»ΡŒΠ½Ρ‹ΠΌ.ΠžΡ‚ΡΠ»Π΅ΠΆΠΈΠ²Π°Π½ΠΈΠ΅ протСкания Ρ‚ΠΎΠΊΠ° позволяСт ΠΎΠΏΡ€Π΅Π΄Π΅Π»ΠΈΡ‚ΡŒ ΠΏΠΎΠ»ΡΡ€Π½ΠΎΡΡ‚ΡŒ напряТСния. На рисункС 12-92 ΠΏΠΎΠΊΠ°Π·Π°Π½Π° Ρ‚Π° ΠΆΠ΅ схСма с ΡƒΠΊΠ°Π·Π°Π½ΠΈΠ΅ΠΌ полярностСй ΠΏΠ°Π΄Π΅Π½ΠΈΠΉ напряТСния ΠΈ направлСния Ρ‚ΠΎΠΊΠ°. Π‘ ΠΎΡ‚Ρ€ΠΈΡ†Π°Ρ‚Π΅Π»ΡŒΠ½ΠΎΠΉ стороны Π±Π°Ρ‚Π°Ρ€Π΅ΠΈ Ρ‚ΠΎΠΊ Ρ‚Π΅Ρ‡Π΅Ρ‚ Π½Π° R 1 . ΠžΡ‚Π²ΠΎΠ΄ A ΠΈΠΌΠ΅Π΅Ρ‚ Ρ‚ΠΎΡ‚ ΠΆΠ΅ ΠΏΠΎΡ‚Π΅Π½Ρ†ΠΈΠ°Π», Ρ‡Ρ‚ΠΎ ΠΈ ΠΎΡ‚Ρ€ΠΈΡ†Π°Ρ‚Π΅Π»ΡŒΠ½Ρ‹ΠΉ Π²Ρ‹Π²ΠΎΠ΄ Π±Π°Ρ‚Π°Ρ€Π΅ΠΈ, ΠΏΠΎΡΠΊΠΎΠ»ΡŒΠΊΡƒ нСбольшоС ΠΏΠ°Π΄Π΅Π½ΠΈΠ΅ напряТСния, Π²Ρ‹Π·Π²Π°Π½Π½ΠΎΠ΅ сопротивлСниСм ΠΏΡ€ΠΎΠ²ΠΎΠ΄Π½ΠΈΠΊΠ°, Π½Π΅ учитываСтся; ΠΎΠ΄Π½Π°ΠΊΠΎ для протСкания Ρ‚ΠΎΠΊΠ° Ρ‡Π΅Ρ€Π΅Π· R 1 трСбуСтся 20 Π²ΠΎΠ»ΡŒΡ‚ напряТСния источника, ΠΈ это ΠΏΠ°Π΄Π΅Π½ΠΈΠ΅ Π½Π° 20 Π²ΠΎΠ»ΡŒΡ‚ ΠΈΠΌΠ΅Π΅Ρ‚ ΡƒΠΊΠ°Π·Π°Π½Π½ΡƒΡŽ ΠΏΠΎΠ»ΡΡ€Π½ΠΎΡΡ‚ΡŒ.Π”Ρ€ΡƒΠ³ΠΈΠΌΠΈ словами, Π² Ρ†Π΅ΠΏΠΈ со стороны зазСмлСния R 1 остаСтся Ρ‚ΠΎΠ»ΡŒΠΊΠΎ 80 Π’ΠΎΠ»ΡŒΡ‚ элСктричСского давлСния.

          Рисунок 12-92. Π’ΠΎΠΊ ΠΏΡ€ΠΎΡ‚Π΅ΠΊΠ°Π΅Ρ‚ Ρ‡Π΅Ρ€Π΅Π· Π΄Π΅Π»ΠΈΡ‚Π΅Π»ΡŒ напряТСния.

          Когда Ρ‚ΠΎΠΊ достигаСт ΠΎΡ‚Π²ΠΎΠ΄Π° B, Π±Ρ‹Π»ΠΎ использовано Π΅Ρ‰Π΅ 30 Π²ΠΎΠ»ΡŒΡ‚ для пСрСмСщСния элСктронов Ρ‡Π΅Ρ€Π΅Π· R 2 , ΠΈ Π°Π½Π°Π»ΠΎΠ³ΠΈΡ‡Π½Ρ‹ΠΌ ΠΎΠ±Ρ€Π°Π·ΠΎΠΌ ΠΎΡΡ‚Π°Π²ΡˆΠΈΠ΅ΡΡ 50 Π²ΠΎΠ»ΡŒΡ‚ ΠΈΡΠΏΠΎΠ»ΡŒΠ·ΡƒΡŽΡ‚ΡΡ для R 3 . Но напряТСния Π½Π° R 2 ΠΈ R 3 ΡΠ²Π»ΡΡŽΡ‚ΡΡ ΠΏΠΎΠ»ΠΎΠΆΠΈΡ‚Π΅Π»ΡŒΠ½Ρ‹ΠΌΠΈ напряТСниями, ΠΏΠΎΡΠΊΠΎΠ»ΡŒΠΊΡƒ ΠΎΠ½ΠΈ Π²Ρ‹ΡˆΠ΅ ΠΏΠΎΡ‚Π΅Π½Ρ†ΠΈΠ°Π»Π° Π·Π΅ΠΌΠ»ΠΈ.

          На рисункС 12-93 ΠΏΠΎΠΊΠ°Π·Π°Π½ Ρ€Π°Π½Π΅Π΅ ΠΈΡΠΏΠΎΠ»ΡŒΠ·ΠΎΠ²Π°Π½Π½Ρ‹ΠΉ Π΄Π΅Π»ΠΈΡ‚Π΅Π»ΡŒ напряТСния. ПадСния напряТСния Π½Π° сопротивлСниях ΠΎΠ΄ΠΈΠ½Π°ΠΊΠΎΠ²Ρ‹; ΠΎΠ΄Π½Π°ΠΊΠΎ ΠΊΠΎΠ½Ρ‚Ρ€ΠΎΠ»ΡŒΠ½Π°Ρ Ρ‚ΠΎΡ‡ΠΊΠ° (зСмля) Π±Ρ‹Π»Π° ΠΈΠ·ΠΌΠ΅Π½Π΅Π½Π°. НапряТСниС ΠΌΠ΅ΠΆΠ΄Ρƒ Π·Π΅ΠΌΠ»Π΅ΠΉ ΠΈ ΠΎΡ‚Π²Π΅Ρ‚Π²Π»Π΅Π½ΠΈΠ΅ΠΌ A Ρ‚Π΅ΠΏΠ΅Ρ€ΡŒ Ρ€Π°Π²Π½ΠΎ ΠΎΡ‚Ρ€ΠΈΡ†Π°Ρ‚Π΅Π»ΡŒΠ½ΠΎΠΌΡƒ Π·Π½Π°Ρ‡Π΅Π½ΠΈΡŽ 100 Π²ΠΎΠ»ΡŒΡ‚ ΠΈΠ»ΠΈ ΠΏΡ€ΠΈΠ»ΠΎΠΆΠ΅Π½Π½ΠΎΠΌΡƒ Π½Π°ΠΏΡ€ΡΠΆΠ΅Π½ΠΈΡŽ.

          Рисунок 12-93. Π”Π΅Π»ΠΈΡ‚Π΅Π»ΡŒ напряТСния с ΠΈΠ·ΠΌΠ΅Π½Π΅Π½Π½ΠΎΠΉ массой.

          НапряТСниС ΠΌΠ΅ΠΆΠ΄Ρƒ Π·Π΅ΠΌΠ»Π΅ΠΉ ΠΈ ΠΎΡ‚Π²Π΅Ρ‚Π²Π»Π΅Π½ΠΈΠ΅ΠΌ B составляСт 80 Π²ΠΎΠ»ΡŒΡ‚, Π° напряТСниС ΠΌΠ΅ΠΆΠ΄Ρƒ Π·Π΅ΠΌΠ»Π΅ΠΉ ΠΈ Π²Ρ‹Π²ΠΎΠ΄ΠΎΠΌ C — 50 Π²ΠΎΠ»ΡŒΡ‚.

          ΠžΠΏΡ€Π΅Π΄Π΅Π»Π΅Π½ΠΈΠ΅ Ρ„ΠΎΡ€ΠΌΡƒΠ»Ρ‹ дСлитСля напряТСния

          На рисункС 12-94 ΠΏΠΎΠΊΠ°Π·Π°Π½ ΠΏΡ€ΠΈΠΌΠ΅Ρ€ сСти ΠΈΠ· Ρ‡Π΅Ρ‚Ρ‹Ρ€Π΅Ρ… рСзисторов ΠΈ источника напряТСния.Π‘ ΠΏΠΎΠΌΠΎΡ‰ΡŒΡŽ Π½Π΅ΡΠΊΠΎΠ»ΡŒΠΊΠΈΡ… простых вычислСний ΠΌΠΎΠΆΠ½ΠΎ ΠΎΠΏΡ€Π΅Π΄Π΅Π»ΠΈΡ‚ΡŒ Ρ„ΠΎΡ€ΠΌΡƒΠ»Ρƒ для опрСдСлСния дСлСния напряТСния Π² ΠΏΠΎΡΠ»Π΅Π΄ΠΎΠ²Π°Ρ‚Π΅Π»ΡŒΠ½ΠΎΠΉ Ρ†Π΅ΠΏΠΈ.

          Рисунок 12-94. ЧСтырСхрСзисторный Π΄Π΅Π»ΠΈΡ‚Π΅Π»ΡŒ напряТСния.

          ПадСниС напряТСния Π½Π° любом ΠΊΠΎΠ½ΠΊΡ€Π΅Ρ‚Π½ΠΎΠΌ рСзисторС Π΄ΠΎΠ»ΠΆΠ½ΠΎ Π½Π°Π·Ρ‹Π²Π°Ρ‚ΡŒΡΡ E X , Π³Π΄Π΅ Π½ΠΈΠΆΠ½ΠΈΠΉ индСкс x — это Π·Π½Π°Ρ‡Π΅Π½ΠΈΠ΅ ΠΊΠΎΠ½ΠΊΡ€Π΅Ρ‚Π½ΠΎΠ³ΠΎ рСзистора (1, 2, 3 ΠΈΠ»ΠΈ 4). Π˜ΡΠΏΠΎΠ»ΡŒΠ·ΡƒΡ Π·Π°ΠΊΠΎΠ½ Ома, ΠΌΠΎΠΆΠ½ΠΎ ΠΎΠΏΡ€Π΅Π΄Π΅Π»ΠΈΡ‚ΡŒ ΠΏΠ°Π΄Π΅Π½ΠΈΠ΅ напряТСния Π½Π° любом рСзисторС.

          Π—Π°ΠΊΠΎΠ½ Ома: E X = I (R X )

          Как Π²ΠΈΠ΄Π½ΠΎ Ρ€Π°Π½Π΅Π΅ Π² тСкстС, Ρ‚ΠΎΠΊ Ρ€Π°Π²Π΅Π½ Π½Π°ΠΏΡ€ΡΠΆΠ΅Π½ΠΈΡŽ источника, Π΄Π΅Π»Π΅Π½Π½ΠΎΠΌΡƒ Π½Π° ΠΎΠ±Ρ‰Π΅Π΅ сопротивлСниС ΠΏΠΎΡΠ»Π΅Π΄ΠΎΠ²Π°Ρ‚Π΅Π»ΡŒΠ½ΠΎΠΉ Ρ†Π΅ΠΏΠΈ.

        alexxlab

        Π”ΠΎΠ±Π°Π²ΠΈΡ‚ΡŒ ΠΊΠΎΠΌΠΌΠ΅Π½Ρ‚Π°Ρ€ΠΈΠΉ

        Π’Π°Ρˆ адрСс email Π½Π΅ Π±ΡƒΠ΄Π΅Ρ‚ ΠΎΠΏΡƒΠ±Π»ΠΈΠΊΠΎΠ²Π°Π½. ΠžΠ±ΡΠ·Π°Ρ‚Π΅Π»ΡŒΠ½Ρ‹Π΅ поля ΠΏΠΎΠΌΠ΅Ρ‡Π΅Π½Ρ‹ *